You are on page 1of 337

अभ्यास

A Monthly Magazine for Bank Exams by Aashish Arora

Topic-wise Quantitative Aptitude MCQs Oct 2021


1000+
Based on Updated Pattern
Exam Strategies and Tips
Recent Memory Based Papers PREMIUM QUESTIONS

Toppers Insight

Partnered with

For SBI, IBPS, RBI & Other Bank Exams


PREFACE
Dear Students,

Quantitative Aptitude is one of the most crucial portions of all the major Bank and Insurance
Exams including IBPS, SBI and RBI. You all know that no preparation is complete unless you
incorporate your learning with practice so to master this section you need to practice standard
quality, updated and exam-specific questions. After interacting with you people, I got to know
that one of the major issues that you face is dearth of sources containing good quality questions
of Quantitative Aptitude. So I came up with “Abhyaas” (that translates to 'Practice') – a free-to-all
monthly magazine of Quantitative Aptitude that targets all the major bank exams of the country.
The main idea behind publishing this monthly magazine was to provide you with a single-stop
source of practice that you can rely on. The content of this practice magazine has been written to
cater the need of lakhs of Banking Aspirants so that they can polish their learnings.

The content of this magazine has been arranged chapter-wise so that you can practice
accordingly. There is a separate section dedicated to Data Interpretation which is the most
important section of the Quantitative Aptitude. The Topper's Interview corner will help you get
insights of the banking exams.

Also, your suggestion for further improvement of the magazine are always welcome.

Aashish Arora
Hisar, Haryana
Follow

Use Invite Code 'TT10'


A letter from Aashish Arora
October, 2021

Dear Students,

Hope you liked the September Edition of Abhyaas and benefitted from it.

I received a lot of messages about how the magazine has proved beneficial in your preparation
for Bank exams and how you are getting better at solving questions. But having said that, I also
know that most of you would have just downloaded it and kept it ‘safely’ without even bothering
to look at it again. My only purpose behind publishing this monthly booklet was to provide you a
single source of quality questions for practice so that you don’t have to look here and there. If
you belong to the group that is trying to better themselves or wanting to do so, the magazine
you’re now reading is for you. For others, this is just a PDF. They will never dare to solve
questions in this module because then they will know that they don’t know how to solve a
question and they don’t want to face this reality. One of the biggest reasons why students fail to
crack any examination is this ‘fear’. Fear of bringing about a change in their routine. Fear of
stepping out of their comfort zone. Fear of failure.

This fear is your dream killer. The failure that you are scared of is nothing but a ‘sculpting tool’.
Michelangelo once said that : “Every block of stone has a statue inside it and it is the task of the
sculptor to discover it.” This simply means that each failure will fuel your curiosity to discover the
statue inside the stone you’re carving. The process of carving this metaphorical stone doesn’t
merely shape the stone; it also shapes the sculptor. Each failure improves your carving skills
and as you are slowly discovering the sculpture inside the stone you’re carving, you are also
uncovering a better sculptor in yourself.

So, drop everything and get to work. Uninstall those addictive apps. Stop wasting time and take
charge of life as a brave human, Students, sacrifice is very necessary. Sacrificing less-
important temptations will give you more power to work on the most crucial objectives. Prioritize
big life improvements like waking up early, studying for atleast five hours a day, eating healthy,
over less significant and time wasting stuffs like social media, gossips, lazying around
purposelessly.
Do not fear failure but rather fear not trying.

Love,
Aashish Arora
Topper’s Talk
HARSHITA KATIYAR
(SBI PO 2020 SELECTED)

1. Congratulations on your success. What was the first reaction when you saw your
result?

HARSHITA : I had slept early that day so I came to know about the result only next morning
around 5 am through messages from friends... i was half-sleepy half unwell, so when I saw that
'congratulations' on the screen, i felt- 'dizzy'... in a good way.. but dizzy.

2. Out of all four subjects in the banking exams, how is the preparation of Quantitative
Aptitude different from the other three and how important Quant is in the exam?

HARSHITA : How prep is different-


For me, i didnt realize how bad I was at quant, until I started getting better.
The other subjects one can probably manage on their own... but quant can be pretty
misleading.. your brain tells you that it's just '10th std maths' and you can get all frustrated when
you can't solve it in 20 min...
What you need to learn is the right way to approach things, not 'tricks', just the right methods, not
just to solve questions but also how to look at and think things correctly.

How imp quant is-

It can be a game-changer bcz it can fetch you 100% marks.

3. How crucial Studified and Aashish Arora's guidance has been in your preparation?

HARSHITA : Ashish Sir's guidance was the one thing that actually gave me my selection.
I had given IBPS PO exam twice earlier without much preparation and both times I could not get
through the mains exam bcz I couldnt clear quant sectional cut off.
I knew where I had to focus. And when I came accross Ashish sir, I knew this was it. If anyone
can help me, it's him.
4. What are the things that students should take care of when preparing for exams
IBPS and SBI (concerning Quantitative Aptitude)?

HARSHITA : Things to take care of-

1. If you are starting as a novice like I did, have a lot of patience esp regarding 'speed'. You need
to first invest a lot of time into yourself through practice questions and mock tests and failures in
mock tests... and gradually you'll see your speed getting better.

2. Have faith in your hard work and your teacher.


Situations may be bitter today for you but sitting idle and complaining about things is not going
to solve anything.
Keep working on yourself. You will attract the right things one day.

5. What are the few mistakes that students commit while preparing for Quantitative
Aptitude?

HARSHITA : Mistakes -

1. Not giving enough mock tests- start giving mock tests as early and as frequently as you can
so you know where you stand.
Get used to seeing bad marks as soon as you can, so that it doesn't bother you anymore and
you can focus on more important things like- the topics that need more practice.

2. Getting impatient with results- things take time to get better and changes start from within.
When your concepts will actually get better it will reflect in your marks too.

3. Trying to accumulate 'better books'- a fancy book will not necessarily give you a fancy job.
There's no point running after the big names and big books just for the sake of telling your
friends that you have solved xyz completely.
Go step-by-step... from zero. You are under no obligation to keep your friends updated about
where you're practicing from. Do your own thing, at your own pace.
You'll get there, have faith.
Unacademy Plus Subscription Features
1. Access to 40k hours of Recorded Content.
2. Access to more than 150 High Quality Test Series.
3. Access to 20k Practice Questions.
4. 70+ Top Educators of the Country.
5. Ask-A-Doubt feature by Unacademy.
6. Live Tests and Quizes.
7. Dedicated Doubt Clearing Sessions.

SUBSCRIBE

Subscribe to Unacademy Plus & Get Unlimited Access of all


Live and Recorded Classes of All Bank Exams Educators Use
Code 'TT10' for 10% Discount in Subscription Fees.
Get Plus Subscription
Use Code TT10 for 10% Discount

STEP-1 STEP-2

TT10
INDEX
Chapter Name Page No.

1. Simplification and Approximation 1 - 18

2. Quadratic Equations 19 - 28

3. Missing Number Series 29 - 42

4. Wrong Number Series 43 - 53

5. Arithmetic Word Problem 54 - 153

6. Data Interpretation (Caselet) 154 - 201

7. Data Interpretation (Graph Based) 201 - 318

8. Memory Based Paper (SBI Clerk Prelims 2021) 319 - 327


अ ास by Aashish Arora
(SBI/IBPS/RBI/LIC/All other banking and insurance Exams)

1 Simplification and Approximation

What value will come in the place of question Mark (?) in the following question?
िन िल खत म वाचक िच (?) के थान पर िकतना मान आयेगा?
1. ( 37 × ? ) ÷ 3.5 + 124 = 420
(A) 22 (B) 28 (C) 19
(D) 23 (E) None of these
3 3 2
2. ( 8 – 7 ) ÷ 13 + ? = 227 × 2
(A) 29 (B) 19 (C) 21
(D) 27 (E) None of these

3. (52)% of 1600 + 420 = ? × 205


(A) 8 (B) 7 (C) 5
(D) 4 (E) None of these

4. { ( 45 × 36 ) – 345 } ÷ 25 = ? - 11
(A) 59 (B) 62 (C) 52
(D) 69 (E) None of these

5. ( 552 – 516 ) ÷ 13 = ? + 83
(A) 100 (B) 80 (C) 120
(D) 90 (E) 110
2
6. (28 × 35) + 73.33% of 360 = ? + 33
(A) 129 (B) 155 (C) 115
(D) 179 (E) None of these

7. (17/23) × 161 – 63.63% of 132 = ? – 45


(A) 90 (B) 70 (C) 80
(D) 60 (E) None of these

8. (352 + 3 × 102 – 252)0.5 × 111 – 1297 = 19 × (?)


(A) 119 (B) 129 (C) 107
(D) 127 (E) None of these
2
9. 34% of (34 + 44) + 182 = (?)
(A) 600 (B) 580 (C) 610
(D) 590 (E) None of these

10. 65% of (83 × 1.25) – 219 = (?)


(A) 197 (B) 187 (C) 181
(D) 191 (E) None of these
3 2
11. 19 × 50 + ( √343 × 17) + 20 = ?
(A) 37 (B) 33 (C) 43
(D) 47 (E) None of these

PAGE h ps://t.me/studified h ps://instagram.com/aashisharorasocial?utm_medium=copy_link


Follow
1 Aashish Arora on: h ps://youtube.com/channel/UCYa4_JrOrf8R5Kz2uOtccXQ https://www.facebook.com/aashisharorasocial/
अ ास by Aashish Arora
(SBI/IBPS/RBI/LIC/All other banking and insurance Exams)

2 2
12. 17.5% of 1240 + 19 = 17 × (?)+ 17
(A) 23 (B) 27 (C) 13
(D) 17 (E) None of these
2
13. (729 + 271) ÷ 2.5 = ?
(A) 25 (B) 30 (C) 15
(D) 20 (E) None of these

14. 27.5% of 1600 + 242 = 14.5 × ? + 282


(A) 16 (B) 26 (C) 14
(D) 24 (E) None of these
3 2
15. 11 + 17 + 400 = (?) × 101
(A) 22 (B) 2 (C) 20
(D) 10 (E) None of these

16. 21/13 of 7/15 of 162.5 = (?)


(A) 122.5 (B) 106 (C) 114.5
(D) 108 (E) None of these
2
17. (36 – 448) ÷ 16 + 398 = ? + 44 × 2.5
(A) 341 (B) 289 (C) 327
(D) 293 (E) None of these

18. (38% of 750 + 227)1/3 + 36 = ? × 4


(A) 21 (B) 11 (C) 13
(D) 23 (E) None of these

19. {(44 × 35 ÷ 5.5) + 110} ÷ 13 = ? – 12


(A) 52 (B) 34 (C) 42
(D) 54 (E) None of these
2
20. 14 × 5 – 170 + 26% of 1750 = ? × 23
(A) 35 (B) 65 (C) 45
(D) 55 (E) None of these

21. {(129 + 297) ÷ 2 + 93} ÷ 18 = ? – 50


(A) 67 (B) 51 (C) 69
(D) 53 (E) None of these

22. 16 × 45 + (?) – 9 × 46 = 17 × 26
(A) 122 (B) 112 (C) 204
(D) 136 (E) None of these

23. 1279 + 2128 + (?) – 297 = 113 × 30


(A) 252 (B) 262 (C) 280
(D) 352 (E) None of these

24. 49 + √1156 × 33 = 20% of (?)


(A) 5150 (B) 5200 (C) 5250
(D) 5855 (E) None of these

PAGE h ps://t.me/studified h ps://instagram.com/aashisharorasocial?utm_medium=copy_link


Follow
2 Aashish Arora on: h ps://youtube.com/channel/UCYa4_JrOrf8R5Kz2uOtccXQ https://www.facebook.com/aashisharorasocial/
अ ास by Aashish Arora
(SBI/IBPS/RBI/LIC/All other banking and insurance Exams)

25. 267 × 3 + 43 × 6 – (?) – 317 = 443 – 237


(A) 536 (B) 406 (C) 336
(D) 506 (E) None of these

26. 18% of 350 – 72% of 150 = (?) % of 240 – 105


(A) 20 (B) 28 (C) 25
(D) 15 (E) None of these

27. (?)% of 2200 – 12 × 20 = 90


(A) 15 (B) 17 (C) 30
(D) 20 (E) 25

28. (?) = 64% of 1850 + 13 × 19 – 16 × 22


(A) 1229 (B) 1129 (C) 1079
(D) 1329 (E) None of these

29. (?)2 = 35.25 × 16 + 162 + √441


(A) 49 (B) 41 (C) 31
(D) 29 (E) None of these

30. 315 + 21 × (?) = 24% of 4550


(A) 47 (B) 33 (C) 37
(D) 43 (E) None of these

31. 62.5% of 632 + 35% of (?) = 675


(A) 700 (B) 800 (C) 850
(D) 750 (E) None of these

32. (?) = 38% of 25% of 83.33% of 1200


(A) 115 (B) 105 (C) 95
(D) 85 (E) None of these
2
33. 23 × 25 + 14 – 36 × (?) = 159
(A) 17 (B) 19 (C) 13
(D) 11 (E) None of these

34. 98/3 % of 769.002 + 24% of 160.89 – 67.9900 = (?)


(A) 226 (B) 224 (C) 228
(D) 208 (E) 222

35. (48% of 12000) ÷ 20 = 18 × ?


(A) 18 (B) 14 (C) 16
(D) 12 (E) 24

36. (√1024 × 40) ÷ 6.4 = (?) + 120


(A) 40 (B) 90 (C) 80
(D) 50 (E) None of these

37. 222 – 48 – 25% of 784 = (?) × 5


(A) 24 (B) 48 (C) 36
(D) 60 (E) None of these

PAGE h ps://t.me/studified h ps://instagram.com/aashisharorasocial?utm_medium=copy_link


Follow
3 Aashish Arora on: h ps://youtube.com/channel/UCYa4_JrOrf8R5Kz2uOtccXQ https://www.facebook.com/aashisharorasocial/
अ ास by Aashish Arora
(SBI/IBPS/RBI/LIC/All other banking and insurance Exams)

38. (8/17) × 221 – 24 = (?) + 15% of 220


(A) 41 (B) 31 (C) 51
(D) 47 (E) None of these

39. 87.5% of 556 – 25% of (?) = 399


(A) 400 (B) 500 (C) 450
(D) 350 (E) None of these

40. 158.33% of 144 + 18.75% of 240 = (?)


(A) 273 (B) 263 (C) 283
(D) 233 (E) None of these
3
41. {(1764 ÷ 7 × 3 – 18 × 7) + 44% of 150} = (?) + 184
(A) 6 (B) 7 (C) 8
(D) 9 (E) None of these

42. 482 – 292 – 49.4 × 15 = (?)% of 380


(A) 180 (B) 160 (C) 170
(D) 190 (E) None of these

43. {(? ÷ 16 + 45% of 1600) ÷ 7} × 4 = 428


(A) 448 (B) 464 (C) 432
(D) 416 (E) None of these

44. 232 + 16 × 32 – 43.75% of 800 = (?)2 – 93


(A) 38 (B) 32 (C) 28
(D) 22 (E) None of these

45. (1039.8 ÷ 11.84) ÷ 19.006 × 1823.85 = (?)


(A) 8320 (B) 4064 (C) 9015
(D) 4514 (E) None of these

46. (?)% of 2479.62 + 14.86 × 33.89 = 1439.94


(A) 75.24 (B) 90.25 (C) 37.5
(D) 29.5 (E) None of these

47. 28.063 × 14.98 + 47.86% of (?) = 780.06


(A) 690 (B) 125 (C) 790
(D) 750 (E) None of these
2.063 1.98
48. 23.96 - 15.9 + 75.06% of 39.8% of 480.062 = (?)
(A) 464 (B) 902 (C) 192
(D) 225 (E) None of these

49. (?) × 17.89 + 3071.94 ÷ 23.96 = 19.8% of 1899


(A) 18 (B) 12 (C) 14
(D) 25 (E) None of these

50. [(6.94 ÷ 11.98) × 960.006 + 139.986] ÷ 34.962 = ? ÷ 5.98


(A) 120 (B) 404 (C) 245
(D) 514 (E) None of these

PAGE h ps://t.me/studified h ps://instagram.com/aashisharorasocial?utm_medium=copy_link


Follow
4 Aashish Arora on: h ps://youtube.com/channel/UCYa4_JrOrf8R5Kz2uOtccXQ https://www.facebook.com/aashisharorasocial/
अ ास by Aashish Arora
(SBI/IBPS/RBI/LIC/All other banking and insurance Exams)

2
51. (? + √1295.98) ÷ 29.89 + (11.92 × 14.68) = 15.96 × 11.98
(A) 75 (B) 36 (C) 37
(D) 18 (E) None of these
2
52. 41.962% of 499 – 11.84 = ? % of 240.062 – 114.08
(A) 33 (B) 25 (C) 75
(D) 74 (E) None of these

53. (√675.962 + 23.94) × 2.4 + 141.98 = ? × 3.92


(A) 45 (B) 79 (C) 54
(D) 78 (E) None of these

54. (3 2/5 + 4.68) ÷ 5.86 ÷ 4.96 = ? ÷ 24.92


(A) 15 (B) 9 (C) 125
(D) 7 (E) None of these

55. 6240.96 + 4114.006 – 6461.846 = (?) + 473.086


(A) 8320 (B) 3420 (C) 2745
(D) 4514 (E) None of these

56. 74.086% of 1700.962 – 149.84 = (?) × 14.006


(A) 75 (B) 36 (C) 37
(D) 18 (E) None of these

57. {(50.062)1.98 ÷ 25.008} + 100.08 = (?)% of 2120.86


(A) 3 (B) 5 (C) 9
(D) 7 (E) None of these

58. (200.24 × 9.086) – 489.942 = (?) × 10.862


(A) 145 (B) 131 (C) 191
(D) 152 (E) None of these
2 1.98 2.08
59. (?) – {(25.086) – (9.082) } = 320.089 + 750.086
(A) 70 (B) 20 (C) 50
(D) 40 (E) None of these

60. (1024.8 ÷ 18.84) ÷ 18.006 × 914.85 = (?)


(A) 8320 (B) 4064 (C) 2745
(D) 4514 (E) None of these

61. (?)% of 2679.8 + 15.86 × 32.89 = 1499.94


(A) 75 (B) 36 (C) 45
(D) 29 (E) None of these

62. 24.063 × 28.98 ÷ 47.86% of 32.09 = (?)


(A) 32 (B) 36 (C) 48
(D) 78 (E) None of these

63. 23.962.063 - 15.91.98 + 75.06% of 39.8% of 480.062 = (?)


(A) 464 (B) 544 (C) 924
(D) 528 (E) None of these

PAGE h ps://t.me/studified h ps://instagram.com/aashisharorasocial?utm_medium=copy_link


Follow
5 Aashish Arora on: h ps://youtube.com/channel/UCYa4_JrOrf8R5Kz2uOtccXQ https://www.facebook.com/aashisharorasocial/
अ ास by Aashish Arora
(SBI/IBPS/RBI/LIC/All other banking and insurance Exams)

64. (?) × 18.98 + 3079.94 ÷ 24.96 = 20.008% of 1999.89


(A) 14.5 (B) 12.5 (C) 18.5
(D) 25.5 (E) None of these

65. √1024 + ? = 3.14 of 77


(A) 242 (B) 210 (C) 190
(D) 288 (E) None of these

66. 53 = 172 – 39 – (?)3


(A) 5 (B) 7 (C) 9
(D) 8 (E) None of these
2
67. 19 + √17 = ? + √840
(A) 362 (B) 398 (C) 336
(D) 292 (E) None of these

68. 33.33% of 99 + 25% of 112 + 16.66% of 54= ?


(A) 86 (B) 70 (C) 99
(D) 103 (E) None of these

69. 20/0.33 + (0.45) × 9 = ? + 40/(0.45) - 35


(A) 45 (B) 19 (C) –14
(D) 10 (E) None of these

70. (31.97)2 – (25.07)2 – (17.95)2 = (?)


(A) 85 (B) 93 (C) 75
(D) 71 (E) 107

71. 513.04 ÷ 26.98 × 29.91 = (?) + 60.08


(A) 510 (B) 441 (C) 487
(D) 520 (E) 498

72. 818.94 ÷ 7.02 – 22.99 × 5.02 + √36.06 = (?)


(A) 18 (B) 8 (C) 25
(D) 13 (E) 2

73. 32.02 × √528.96 = (?) + √255.97


(A) 24 (B) 36 (C) 42
(D) 46 (E) 54

74. 53.03 × 36.98 – 28.08 × 13.98 = (?)


(A) 1551 (B) 1510 (C) 1544
(D) 1569 (E) 1526

75. 16.66 of 341.97 + 94.03 × 85.07 – (?) = 22.22% of 566.95


(A) 7733 (B) 7921 (C) 7291
(D) 7457 (E) None of these

76. 24.96 × (?) × 11.03 = (37.02)2 + 280.93


(A) 5 (B) 7 (C) 6
(D) 8 (E) None of these

PAGE h ps://t.me/studified h ps://instagram.com/aashisharorasocial?utm_medium=copy_link


Follow
6 Aashish Arora on: h ps://youtube.com/channel/UCYa4_JrOrf8R5Kz2uOtccXQ https://www.facebook.com/aashisharorasocial/
अ ास by Aashish Arora
(SBI/IBPS/RBI/LIC/All other banking and insurance Exams)

77. 57.98 × 35.04 + 40.03 × 13.97 ÷ ? = 83.97 × 25.03


(A) 8 (B) 11 (C) 7
(D) 9 (E) None of these

78. (?)/8 of 3√729 = 32/(?) of √81


(A) 18 (B) 16 (C) 15
(D) 12.5 (E) None of these

79. (0.343)58/(0.49)69 = (0.7)?


(A) 62 (B) 75 (C) 36
(D) 24 (E) None of these

80. (√441 × 0.33 + √676 × 4)/(?) = 3√216 × √1156 + 18


(A) 0.5 (B) 1.75 (C) 3
(D) 2.5 (E) None of these

81. 57.98 × 35.04 + (40.03 × 13.97)/(?) = 83.97 × 25.03


(A) 8 (B) 11 (C) 7
(D) 9 (E) None of these
2 2 3
82. 27.03 – 34.99 = 15.97 × 4.02 – 4.03 × (?)
(A) 324 (B) 388 (C) 380
(D) 362 (E) None of these

83. (12.5% of 299.84 – 12.5) * 4.062 = (?) + 79.986


(A) 18 (B) 20 (C) 30
(D) 25 (E) None of these

84. (3.862 ÷ 14.842) × 899 + (199.621 ÷ 1.25) = (?) × 20.082


(A) 18 (B) 20 (C) 28
(D) 25 (E) None of these
2
85. (15.962 × 29.941) + 39.62% of 225.082 = (?) + 85.92
(A) 18 (B) 20 (C) 28
(D) 22 (E) None of these

86. (? × 14.96) + (179.94 ÷ 1.96) = 89.942% of 599.621


(A) 30 (B) 20 (C) 28
(D) 25 (E) None of these

87. (416.062 – 65.942) ÷ 6.942 + (11.942 × 15.062) = (?) × 9.86


(A) 23 (B) 20 (C) 28
(D) 25 (E) None of these

88. (24.08)2 + (28.56)2 - (32.04)2 = ?


(A) 296 (B) –393 (C) –269
(D) 393 (E) None of these

89. (154.6/24.5)+(5.25/6.3)×(9/25)=?
(A) 35 (B) 29.7 (C) 6.6
(D) 32.05 (E) 39

PAGE h ps://t.me/studified h ps://instagram.com/aashisharorasocial?utm_medium=copy_link


Follow
7 Aashish Arora on: h ps://youtube.com/channel/UCYa4_JrOrf8R5Kz2uOtccXQ https://www.facebook.com/aashisharorasocial/
अ ास by Aashish Arora
(SBI/IBPS/RBI/LIC/All other banking and insurance Exams)

90. 35% of 500+25-75 =?% of 250


(A) 65 (B) 55 (C) 45
(D) 52 (E) None of these

91. 135 × 25.6= ? X √2896.89


(A) 46 (B) 64 (C) 54
(D) 72 (E) None of these

92. (322 + 476) ÷ 6 + 224 = ? × 6


(A) 79 (B) 67 (C) 89
(D) 87 (E) None of these

93. 43.75% of 360 – 25% of 789 + 89.7% of 56 = ?


(A) 13 (B) 34 (C) 8
(D) 11 (E) None of these

94. 44.6 × 189.2 ÷ 72.6 + (?)=14.4 × 52.3


(A) 638 (B) 662.9 (C) 750
(D) 569.2 (E) None of these
2 2 2 2
95. (?) + (87.02) - (57.17) - (61.92) = (19.06)
(A) 134 (B) –241 (C) –115
(D) 96 (E) None of these

96. (76.08)2 + (36.13)2- (18.99)2 = (?)2 + (74.90)2


(A) 33 (B) 29 (C) 43
(D) 37 (E) None of these

97. 3015.96 - (25.98 × 12.5)=(?)2


(A) 69 (B) 48 (C) 52
(D) 38 (E) None of these

98. 12.5% of 460 + 35% of 768 – 8.33% of 62 = (?)


(A) 332 (B) 321 (C) 289.09
(D) 324 (E) None of these

99. (25.99)2 + (32.13)2 – (11.99)2 = (?)2 + (34.90)2


(A) 18 (B) 22 (C) 16
(D) 20 (E) None of these

100. 33.08 × 13.97 ÷ 7.09 + (?) = 19.5 × 55.3


(A) 637 (B) 1012 (C) 750.9
(D) 569.2 (E) None of these

101. 22848 ÷ 68 ÷ 8 = (?)


(A) 544 (B) 292 (C) 482
(D) 346 (E) None of these

102. √{2304 – (15.9 × 6)} = (?)


(A) 39 (B) 52 (C) 47
(D) 41 (E) None of these

PAGE h ps://t.me/studified h ps://instagram.com/aashisharorasocial?utm_medium=copy_link


Follow
8 Aashish Arora on: h ps://youtube.com/channel/UCYa4_JrOrf8R5Kz2uOtccXQ https://www.facebook.com/aashisharorasocial/
अ ास by Aashish Arora
(SBI/IBPS/RBI/LIC/All other banking and insurance Exams)

2
103. (12.006 ÷ 24.962) × 375.006 + 219.92 = (?) + (151.96 × 1/2)
(A) 18 (B) 40 (C) 90
(D) 45 (E) None of these

104. 47.96% of 750.062 + √575.962 = (199.94 ÷ 12.5) × (?)


(A) 75 (B) 90 (C) 24
(D) 29 (E) None of these

105. (220.062 ÷ 24.12) × 4.96 + 225.06 = (?) × 8.96


(A) 69 (B) 12 (C) 79
(D) 30 (E) None of these

106. (? × 1.96) + 12.5% of 959.94 = (2699 ÷ 8.96)


(A) 13 (B) 90 (C) 19
(D) 25 (E) None of these

107. 39.94% of 12.5% of 1999.94 + 24.96% of 20.06% of 2499 = ?


(A) 188 (B) 125 (C) 115
(D) 225 (E) None of these

Join
Joinme
meonon

Join me on
Join me on

Join me on
Join me on
Join me on
BY: AASHISH ARORA

PAGE h ps://t.me/studified h ps://instagram.com/aashisharorasocial?utm_medium=copy_link


Follow
9 Aashish Arora on: h ps://youtube.com/channel/UCYa4_JrOrf8R5Kz2uOtccXQ https://www.facebook.com/aashisharorasocial/
अ ास by Aashish Arora
(SBI/IBPS/RBI/LIC/All other banking and insurance Exams)

SOLUTION

1. Ans. (B) 119 - 84 = (?) - 45


(37 × ?) ÷ 3.5 + 124 = 420 (?) = 35 + 45
(37 × ?) ÷ 3.5 = 420 - 124 (?) = 80
(37 × ?) ÷ 3.5 = 296
37 × ? = 296 × 3.5
(?) = 1036/37 = 28 8. Ans. (C)
2 2 2 0.5
(35 + 3 × 10 – 25 ) × 111 – 1297 =
2. Ans. (C) 19 × (?)
(83 - 73) ÷ 13 + (?)2 = 227 × 2 0.5
(1225 + 300 – 625) × 111 – 1297 = 19
169 ÷ 13 + (?)2 = 454 × (?)
(?)2 = 454 - 13 9000.5 × 111 – 1297 = 19 × (?)
(?)2 = 441 30 × 111 – 1297 = 19 × (?)
(?) = 21 3330 – 1297 = 19 × (?)
2033 = 19 × (?)
3. Ans. (D) (?) = 107
2
(5 )% of 1600 + 420 = (?) × 205
0.25 × 1600 + 420 = (?) × 205 9. Ans. (D)
400 + 420 = (?) × 205 34% of (342 + 44) + 182 = (?)
(?) × 205 = 820 0.34 × (1156 + 44) + 182 = (?)
(?) = 4 0.34 × 1200 + 182 = (?)
408 + 182 = (?)
4. Ans. (B) (?) = 590
{(45 × 36) – 345} ÷ 25 = (?) - 11
{1620 – 345} ÷ 25 = (?) - 11 10. Ans. (A)
1275 ÷ 25 = (?) - 11 3
65% of (8 × 1.25) – 219 = (?)
(?) = 51 + 11 0.65 × (512 × 1.25) – 219 = (?)
(?) = 62 0.65 × 640 – 219 = (?)
416 – 219 = (?)
5. Ans. (E) (?) = 197
(552 - 516) ÷ 13 = (?) + 83
(3025 - 516) ÷ 13 = (?) + 83 11. Ans. (B)
2509 ÷ 13 = (?) + 83 19 × 50 + ( 3√343 × 17) + 20 = (?)2
(?) = 193 - 83 950 + (7× 17) + 20 = (?)
2

(?) = 110 950 + 119 + 20 = (?)


2

2
(?) = 1089
6. Ans. (B) (?) = 33
2
(28 × 35) + 73.33% of 360 = (?) + 33
980 + (11/15) × 360 = (?) + 1089 12. Ans. (D)
980 + 264 = (?) + 1089 17.5% of 1240 + 192 = 17 × (?) + 172
(?) = 1244 – 1089 0.175 × 1240 + 361 = 17 × (?) + 289
(?) = 155 217 + 361 – 289 = 17 × (?)
17 × (?) = 289
7. Ans. (C) (?) = 17
(17/23) × 161 - 63.63% of 132 = (?) - 45
119 - (7/11) × 132 = (?) - 45

PAGE h ps://t.me/studified h ps://instagram.com/aashisharorasocial?utm_medium=copy_link


Follow
10 Aashish Arora on: h ps://youtube.com/channel/UCYa4_JrOrf8R5Kz2uOtccXQ https://www.facebook.com/aashisharorasocial/
अ ास by Aashish Arora
(SBI/IBPS/RBI/LIC/All other banking and insurance Exams)

13. Ans. (D) 196 × 5 – 170 + 0.26 × 1750 = (?) × 23


2
(729 + 271) ÷ 2.5 = (?) 980 – 170 + 455 = (?) × 23
2
1000 ÷ 2.5 = (?) (?) × 23 = 1265
2
400 = (?) (?) = 55
(?) = √400
(?) = 20 21. Ans. (A)
{(129 + 297) ÷ 2 + 93} ÷ 18 = (?) – 50
14. Ans. (A) {426 ÷ 2 + 93} ÷ 18 = (?) – 50
2 2
27.5% of 1600 + 24 = 14.5 × (?) + 28 {213 + 93} ÷ 18 = (?) – 50
0.275 × 1600 + 576 = 14.5 × (?) + 784 306 ÷ 18 = (?) – 50
440 + 576 – 784 = 14.5 × (?) 17 = (?) – 50
14.5 × (?) = 232 (?) = 67
(?) = 16
22. Ans. (D)
16 × 45 + (?) – 9 × 46 = 17 × 26
15. Ans. (C) 720 + (?) – 414 = 442
113 + 172 + 400 = (?) × 101 (?) = 442 + 414 – 720
1331 + 289 + 400 = (?) × 101 (?) = 136
(?) × 101 =2020
(?) = 20 23. Ans. (C)
1279 + 2128 + (?) – 297 = 113 × 30
16. Ans. (A) 3407 + (?) – 297 = 3390
21/13 of 7/15 of 162.5 = (?) (?) = 3390 – 3110
21/13 × 7/15 × 162.5 = (?) (?) = 280
(?) = 122.5
24. Ans. (D)
17. Ans. (A) 49 + √1156 × 33 = 20% of (?)
2
(36 – 448) ÷ 16 + 398 = (?) + 44 × 2.5 49 + 34 × 33 = 0.2 × (?)
(1296 – 448) ÷ 16 + 398 = (?) + 110 49 + 1122 = 0.2 × (?)
848 ÷ 16 + 398 = (?) + 110 (?) × 0.2 = 1171
53 + 398 = (?) + 110 (?) = 5855
(?) = 451 – 110
(?) = 341 25. Ans. (A)
267 × 3 + 43 × 6 – (?) – 317 = 443 – 237
18. Ans. (B) 801 + 258 – (?) – 317 = 206
(38% of 750 + 227)1/3 + 36 = (?) × 4 (?) = 742 – 206
(0.38 × 750 + 227)1/3 + 36 = (?) × 4 (?) = 536
1/3
(285 + 227) + 36 = (?) × 4
1/3
512 + 36 = (?) × 4 26. Ans. (C)
8 + 36 = (?) × 4 18% of 350 – 72% of 150 = (?) % of 240 –
44 = (?) × 4 105
(?) = 11 0.18 × 350 – 0.72 × 150 = (?) × 2.4 – 105
63 – 108 = (?) × 2.4 – 105
19. Ans. (C) (?) × 2.4 = 60
{(44 × 35 ÷ 5.5) + 110} ÷ 13 = (?) – 12 (?) = 25
{280 + 110} ÷ 13 = (?) – 12
390 ÷ 13 = (?) – 12 27. Ans. (A)
30 = (?) – 12 (?)% of 2200 – 12 × 20 = 90
(?) = 42 (?) × 22 – 12 × 20 = 90
(?) × 22 = 90 + 240
20. Ans. (D) (?) × 22 = 330
2
14 × 5 – 170 + 26% of 1750 = (?) × 23

PAGE h ps://t.me/studified h ps://instagram.com/aashisharorasocial?utm_medium=copy_link


Follow
11 Aashish Arora on: h ps://youtube.com/channel/UCYa4_JrOrf8R5Kz2uOtccXQ https://www.facebook.com/aashisharorasocial/
अ ास by Aashish Arora
(SBI/IBPS/RBI/LIC/All other banking and insurance Exams)

(?) = 15 (48% of 12000) ÷ 20 = 18 × (?)


(0.48 × 12000) ÷ 20 = 18 × (?)
28. Ans. (C) 5760 ÷ 20 = 18 × (?)
(?) = 64% of 1850 + 13 × 19 – 16 × 22 288 = 18 × (?)
(?) = 0.64 × 1850 + 247 – 352 (?) = 16
(?) = 1184 – 105
(?) = 1079 36. Ans. (C)
(√1024 × 40) ÷ 6.4 = (?) + 120
29. Ans. (D) (32 × 40) ÷ 6.4 = (?) + 120
(?)2 = 35.25 × 16 + 162 + √441 1280 ÷ 6.4 = (?) + 120
(?)2 = 564 + 256 + 21 200 = (?) + 120
(?)2 = 841 (?) = 80
(?) = 29
37. Ans. (B)
30. Ans. (C) 222 – 48 – 25% of 784 = (?) × 5
315 + 21 × (?) = 24% of 4550 484 – 48 – 0.25 × 784 = (?) × 5
21 × (?) = 0.24 × 4550 – 315 436 – 196 = (?) × 5
21 × (?) = 1092 – 315 240 = (?) × 5
21 × (?) = 777 (?) = 48
(?) = 37
38. Ans. (D)
31. Ans. (B) 8/17 × 221 – 24 = (?) + 15% of 220
62.5% of 632 + 35% of (?) = 675 8/17 × 221 – 24 = (?) + 0.15 × 220
0.625 × 632 + 0.35 × (?) = 675 104 – 24 = (?) + 33
395 + 0.35 × (?) = 675 (?) = 80 - 33
0.35 × (?) = 280 (?) = 47
(?) = 800
39. Ans. (D)
32. Ans. (C) 87.5% of 556 – 25% of (?) = 399
(?) = 38% of 25% of 83.33% of 1200 0.875 × 556 – 0.25 × (?) = 399
(?) = 0.38 × 0.25 × 5/6 × 1200 486.5 – 339 = 0.25 × (?)
(?) = 0.095 × 1000 0.25 × (?) = 87.5
(?) = 95 (?) = 350

33. Ans. (A) 40. Ans. (A)


2
23 × 25 + 14 – 36 × (?) = 159 158.33% of 144 + 18.75% of 240 = (?)
575 + 196 – 36 × (?) = 159 19/21 × 144 + 0.1875 × 240 = (?)
36 × (?) = 771 – 159 228 + 45 = (?)
36 × (?) = 612 (?) = 273
(?) = 17
41. Ans. (C)
{(1764 ÷ 7 × 3 – 18 × 7) + 44% of 150} =
34. Ans. (E) 3
(?) + 184
98/3% of 769.002 + 24% of 160.89 – {(252 × 3 – 126) + 0.44 × 150} = (?)3 +
67.9900 = (?) 184
(0.98/3) × 769.002 + 0.24 × 160.89 – {(756 – 126) + 66} = (?)3 + 184
67.99 = (?) {630 + 66} = (?)3 + 184
251.29732 + 38.6136 – 67.99 = (?) 3
(?) = 696 – 184
(?) = 221.92092 = 222 3
(?) = 512
(?) = 8
35. Ans. (C)

PAGE h ps://t.me/studified h ps://instagram.com/aashisharorasocial?utm_medium=copy_link


Follow
12 Aashish Arora on: h ps://youtube.com/channel/UCYa4_JrOrf8R5Kz2uOtccXQ https://www.facebook.com/aashisharorasocial/
अ ास by Aashish Arora
(SBI/IBPS/RBI/LIC/All other banking and insurance Exams)

42. Ans. (D) 480.062 = (?)


482 – 292 – 49.4 × 15 = (?)% of 380 2 2
24 - 16 + 75% of 40% of 480 = (?)
2304 – 841 – 741 = (?) × 3.8 576 - 256 + 0.75 × 0.4 × 480 = (?)
(?) × 3.8 = 722 320 + 144 = (?)
(?) = 190 (?) = 464

43. Ans. (B) 49. Ans. (C)


{(? ÷ 16 + 45% of 1600) ÷ 7} × 4 = 428 (?) × 17.89 + 3071.94 ÷ 23.96 = 19.8% of
{(? ÷ 16 + 0.45 × 1600) ÷ 7} = 428 ÷ 4 1899
{(? ÷ 16 + 720) ÷ 7} = 107 (?) × 18 + 3072 ÷ 24 = 20% of 1900
(? ÷ 16 + 720) = 107 × 7 (?) × 18 + 128 = 0.20 × 1900
(?) ÷ 16 = 749 - 720 (?) × 18 = 380 - 128 = 252
(?) = 29 × 16 (?) = 14
(?) = 464
50. Ans. (A)
44. Ans. (C) [(6.94 ÷ 11.98) × 960.006 + 139.986] ÷
2 2
23 + 16 × 32 – 43.75% of 800 = (?) – 34.962 = (?) ÷ 5.98
93 [(7 ÷ 12) × 960 + 140] ÷ 35 = (?) ÷ 6
529 + 512 – 0.4375 × 800 = (?)2 – 93 [560 + 140] ÷ 35 = (?) ÷ 6
1041 – 350 = (?)2 – 93 700 ÷ 35 = (?) ÷ 6
(?)2 = 691 + 93 20 = (?) ÷ 6
(?)2 = 784 (?) = 120
(?) = 28
51. Ans. (D)
45. Ans. (A) (?2 + √1295.98) ÷ 29.89 + (11.92 ×
(1039.8 ÷ 11.84) ÷ 19.006 × 1823.85 = 14.68) = 15.96 × 11.98
(?) (?2 + √1296) ÷ 30 + (12 × 15) = 16 × 12
(1040 ÷ 12) ÷ 19 × 1824 = (?) 2
(? + 36) ÷ 30 + 180 = 192
1040/(12 × 19) × 1824 = (?) 2
(? + 36) ÷ 30 = 12
(?) = 1040 × 8 2
(?) + 36 = 360
(?) = 8320 2
(?) = 324
(?) = 18
46. Ans. (C)
(?)% of 2479.62 + 14.86 × 33.89 = 52. Ans. (C)
1439.94 41.962% of 499 – 11.842 = (?)% of
(?)% × 2480 + 15 × 34 = 1440 240.062 – 114.08
(?)% × 2480 = 1440 - 510 2
42% of 500 – 12 = (?)% of 240 – 114
(?)% × 2480 = 930 210 – 144 = (?)% of 240 – 114
(?) = 930 × 100 / 2480 66 + 114 = (?)% of 240
(?) = 37.5 180 = (?)% × 240
(?) = 75
47. Ans. (D)
28.063 × 14.98 + 47.86% of (?) = 53. Ans. (B)
780.06 (√675.962 + 23.94) × 2.4 + 141.98 = (?) ×
28 × 15 + 48% of (?) = 780 3.92
0.48 × (?) = 780 - 420 2
(√676 + 24) × 2.4 + 14 = (?) × 4
0.48 × (?) = 360 (26 + 24) × 2.4 + 196 = (?) × 4
(?) = 750 50 × 2.4 + 196 = (?) × 4
120 + 196 = (?) × 4
48. Ans. (A) 316 = (?) × 4
2.063 1.98
23.96 - 15.9 + 75.06% of 39.8% of (?) = 79

PAGE h ps://t.me/studified h ps://instagram.com/aashisharorasocial?utm_medium=copy_link


Follow
13 Aashish Arora on: h ps://youtube.com/channel/UCYa4_JrOrf8R5Kz2uOtccXQ https://www.facebook.com/aashisharorasocial/
अ ास by Aashish Arora
(SBI/IBPS/RBI/LIC/All other banking and insurance Exams)

54. Ans. (D) 3 × 915 = (?)


(3 2/5 + 4.68) ÷ 5.86 ÷ 4.96 = (?) ÷ 24.92 (?) = 2745
(3 2/5 + 5) ÷ 6 ÷ 5 = (?) ÷ 25
8.4 ÷ 6 = (?) ÷ 5 61. Ans. (B)
(?) = 1.4 × 5 (?)% of 2679.8 + 15.86 × 32.89 =
(?) = 7 1499.94
(?)% of 2680 + 16 × 33 = 1500
55. Ans. (B) (?)% of 2680 + 528 = 1500
6240.96 + 4114.006 – 6461.846 = (?) + (?)% of 2680 = 972
473.086 (?) × 26.8 = 972
6241 + 4114 - 6462 = (?) + 473 (?) × 27 = 972
(?) = 3893 – 473 (?) = 36
(?) = 3420
62. Ans. (C)
56. Ans. (C) 24.063 × 28.98 ÷ 47.86% of 32.09 = (?)
74.086% of 1700.962 – 149.84 = (?) × 24 × 29 ÷ 48% of 32 = (?)
14.006 24 × 29 ÷ 0.48 of 32 = (?)
74% of 1700 – 150 = (?) × 14 24 × 29 ÷ 15.36 = (?)
1258 – 150 = (?) × 14 24 × 30 ÷ 15 = (?)
(?) × 14 = 1108 24 × 2 = (?)
(?) = 79 (?) = 48

57. Ans. (C) 63. Ans. (A)


1.98
{(50.062) ÷ 25.008} + 100.08 = (?)% of 23.962.063 - 15.91.98 + 75.06% of 39.8% of
2120.86 480.062 = (?)
{(50)2 ÷ 25} + 100 = (?)% of 2121 242 – 162 + 75% of 40% of 480 = (?)
100 + 100 = (?)% × 2121 576 - 256 + 0.75 × 0.4 × 480 = (?)
200 = (?) × 21.21 320 + 144 = (?)
(?) = 9.42 = 9 (?) = 464

58. Ans. (B) 64. Ans. (A)


(200.24 × 9.086) – 489.942 = (?) × (?) × 18.98 + 3079.94 ÷ 24.96 =
10.082 20.008% of 1999.89
(200 × 9) – 490 = (?) × 10 (?) × 19 + 3080 ÷ 25 = 20% of 2000
1800 – 490 = (?) × 10 = 1310 (?) × 19 + 123.2 = 400
(?) = 131 (?) × 19 = 400 - 123
(?) × 19 = 277
59. Ans. (D) (?) = 14.5
2 1.98 2.08
(?) – {(25.086) – (9.082) } = 320.089
65. Ans. (B)
+ 750.086
2 2 2 √1024 + (?) = 3.14 of 77
(?) – {25 – 9 } = 320 + 750
2 32 + (?) = 241.78
(?) – {625 – 81) = 1070
(?) = 241.78 - 32
(?)2 – 544 = 1070
(?) = 209.78
(?)2 = 1614
(?) = 210
(?) = 40
66. Ans. (A)
60. Ans. (C) 3 2 3
5 = 17 - 39 – (?)
(1024.8 ÷ 18.84) ÷ 18.006 × 914.85 = (?) 3
125 = 289 - 39 - (?)
(1025 ÷ 19) ÷ 18 × 915 = (?) 3
(?) = 250 – 125
54 ÷ 18 × 915 = (?)
(?)3 = 125

PAGE h ps://t.me/studified h ps://instagram.com/aashisharorasocial?utm_medium=copy_link


Follow
14 Aashish Arora on: h ps://youtube.com/channel/UCYa4_JrOrf8R5Kz2uOtccXQ https://www.facebook.com/aashisharorasocial/
अ ास by Aashish Arora
(SBI/IBPS/RBI/LIC/All other banking and insurance Exams)

(?) = 5 53 × 37 – 28 × 14 = (?)
1961 – 392 = (?)
67. Ans. (C) (?) = 1569
192 + √17 = (?) + √840
361 + 4 = (?) + 29
(?) = 365 - 29 75. Ans. (B)
(?) = 336 16.66 of 341.97 + 94.03 × 85.07 – (?) =
22.22% of 566.95
68. Ans. (B) 1/6 × 342 + 94 × 85 – (?) = 2/9 × 567
33.33%of99+25%of112+16.66%of54= 57 + 7990 – (?) = 126
(?) (?) = 8047 – 126
(1/3) × 99 + (1/4) × 112 + (1/6) × 54 = (?) (?) = 7921
(?) = 33 + 28 + 9
(?) = 70
76. Ans. (C)
2
69. Ans. (D) 24.96 × (?) × 11.03 = (37.02) + 280.93
2
20/0.33 + (0.45) × 9 = (?) + 40/(0.45) - 35 25 × (?) × 11 = 37 + 281
60.60 + 4.05 = (?) + 88.89 - 35 275 × (?) = 1369 + 281
60 + 4 = (?) + 89 - 35 275 × (?) = 1650
(?) = 64 - 89 + 35 (?) = 6
(?) = 10

70. Ans. (C) 77. Ans. (A)


(31.97)2 – (25.07)2 – (17.95)2 = (?) 57.98 × 35.04 + 40.03 × 13.97 ÷ (?) =
2 2 2
32 – 25 – 18 = (?) 83.97 × 25.03
1024 – 625 – 324 = (?) 58 × 35 + 40 × 14 ÷ (?) = 84 × 25
(?) = 75 2030 + 40 × 14 ÷ (?) = 2100
40 × 14 ÷ (?) = 70
71. Ans. (A) (?) = 8
513.04 ÷ 26.98 × 29.21 = (?) + 60.08
513 ÷ 27 × 30 = (?) + 60
19 × 30 = (?) + 60 78. Ans. (B)
(?) = 570 – 60 (?)/8 of 3√729 = 32/(?) of √81
(?) = 510 (?)/8 × 3√729 = 32/(?) × √27
2
(?) × 9 = 8 × 32 × 9
72. Ans. (B) 2
(?) = 256
818.94 ÷ 7.02 – 22.99 × 5.02 + √36.06 = (?) = 16
(?)
819 ÷ 7 – 23 × 5 + √36 = (?)
117 – 115 + 6 = (?) 79. Ans. (C)
(?) = 8 (0.343)58/(0.49)69 = (0.7)?
(0.7)3 × 58/(0.7)2 × 69 = (0.7)?
(174 – 138) ?
73. Ans. (D) (0.7) = (0.7)
36 ?
32.02 × √528.96 = (?) + √255.97 (0.7) = (0.7)
32 × √529 = (?) + √256 (?) = 36
32 × 23 = (?) + 16
(?) = 2 × 23 80. Ans. (A)
(?) = 46 441 ´ 0.33 + 676 ´ 4 3
= 216 ´ 1156 + 18
74. Ans. (D) (?)
53.03 × 36.98 – 28.08 × 13.98 = (?)

PAGE h ps://t.me/studified h ps://instagram.com/aashisharorasocial?utm_medium=copy_link


Follow
15 Aashish Arora on: h ps://youtube.com/channel/UCYa4_JrOrf8R5Kz2uOtccXQ https://www.facebook.com/aashisharorasocial/
अ ास by Aashish Arora
(SBI/IBPS/RBI/LIC/All other banking and insurance Exams)

1 86. (? × 14.96) + (179.94 ÷ 1.96) = 89.942%


21 ´ + 26 ´ 4
3 = 6 ´ 34 + 18 of 599.621
(?) (? × 15) + (180 ÷ 2) = 90% of 600
7 + 104 (? × 15) + 90 = 540
= 204 + 18 (? × 15) = 450
(?)
(?) = 30 Ans. (A)
111/(?) = 222
(?) = 0.5 87 (416.062 – 65.942) ÷ 6.942 + (11.942 ×
15.062) = (?) × 9.86
81. Ans. (A) (416 – 66) ÷ 7 + (12 × 15) = (?) × 10
40.03 ´13.97 50 + 180 = (?) × 10
57.98 ´ 35.04 + = 83.97 Ì25.03
(?) (?) = 23 Ans. (A)

40 ´14 2 2
88. (24.08) + (28.56) - (32.04) = (?)
2
58 ´ 35 + = 84 Ì25
(?) (24)2 + (29)2 - (32)2 = (?)
576 + 841 - 1024 = (?)
560 (?) = 393 Ans. (D)
2030 + = 2100
(?)
89. (154.6/24.5) + (5.25/6.3) × (9/25) = (?)
560/(?) = 70 (154.5/24.5) + (5/6) × (9/25) = (?)
(?) = 8 6 + 15/49 + 3/10 = (?)
3237/490 = 6.606 = 6.6 Ans. (C)
82. Ans. (C)
2 2 3
27.03 – 34.99 = 15.97 × 4.02 – 4.03 × 90. 35% of 500 + 25 - 75 = (?)% of 250
(?) (35/100) × 500 - 50 = [(?)/100] × 250
272 – 352 = 16 × 43 – 4 × (?) 35 × 5 - 50 = (?) × 25/10
729 – 1225 = 1024 – 4 × (?) 125 = (?) × 2.5
4 × (?) = 1024 + 496 45 + 225 = (?) = 50
4 × (?) = 1520
(?) = 380
91. 135 × 25.6 = (?) × √2896.89
83. (12.5% of 299.84 – 12.5) × 4.062 = (?) + 3456 = (?) × √2916
79.986 3456 = (?) × 54
(12.5% of 300 – 12.5) × 4 = (?) + 80 (?) = 64 Ans. (B)
(37.5 – 12.5) × 4 = (?) + 80
100 - 80 = (?) 92. (322 + 476) ÷ 6 + 224 = (?) × 6
(?) = 20 Ans. (B) 1500 ÷ 6 + 224 = (?) × 6
(?) = 474/6
84. (3.862 ÷ 14.842) × 899 + (199.621 ÷ (?) = 79 Ans. (A)
1.25) = (?) × 20.082
(4 ÷ 15) × 900 + (200 ÷ 1.25) = (?) × 20 93. Ans. (D)
240 + 160 = (?) × 20 43.75% of 360 – 25% of 789 + 89.7% of
(?) = 400/20 = 20 Ans. (B) 56 = (?)
7/16 × 360 – 1/4 × 789 + 0.897 × 56 = (?)
85. (15.962 × 29.941) + 39.62% of 225.082 157.5 – 197.25 + 50.232 = (?)
2
= (?) + 85.92 10.482 = (?)
2
(16 × 30) + 40% of 225 = (?) + 86 (?) = 11
2
480 + 90 = (?) + 86
(?)
2
94. Ans. (A)
= 484 44.6 × 189.2 ÷ 72.6 + (?) = 14.4 × 52.3
(?) = 22 Ans. (D) 44 × 86 ÷ 33 + (?) = 753.12

PAGE h ps://t.me/studified h ps://instagram.com/aashisharorasocial?utm_medium=copy_link


Follow
16 Aashish Arora on: h ps://youtube.com/channel/UCYa4_JrOrf8R5Kz2uOtccXQ https://www.facebook.com/aashisharorasocial/
अ ास by Aashish Arora
(SBI/IBPS/RBI/LIC/All other banking and insurance Exams)

114.66 + (?) = 753.12 (?) = 1012


(?) = 753 – 115
(?) = 638 101. Ans. (C)
22848 ÷ 68 ÷ 8 = (?)
95. Ans. (C) 336 ÷ 8 = (?)
2 2 2
(?) + (87.02) – (57.17) – (61.92) = (?) = 482
2
(19.06)
(?) + 872 – 572 – 622 = 192 102. Ans. (C)
(?) + 7568 – 3249 – 3844 = 361 √{2304 – (15.9 × 6)} = (?)
(?) = 361 – 476 √{2304 – 95.4) = (?)
(?) = - 115 √2208.6 = (?)
(?) = √2209
96. Ans. (A) (?) = 47
2 2 2 2
(76.08) + (36.13) – (18.99) = (?) +
(74.90)
2 103. Ans. (A)
2 2 2 2
76 + 36 – 19 = (?) + 75
2 (12.006 ÷ 24.962) × 375.006 + 219.92 =
2
2
5776 + 1296 – 361 = (?) + 5625 (?) + (151.96 × ½)
2
(?)2 = 6711 – 5625 = 1086 (12 ÷ 25) × 375 + 220 = (?) + (152 × ½)
2
(?) = 33 180 + 220 = (?) + 76
(?)2 = 324
97. Ans. (C) (?) = 18 (Ans.)
2
3016.96 – (25.98 × 12.5) = (?)
3016 – (26 × 12.5) = (?)
2
104. Ans. (C)
3016 – 325 = (?)
2
47.97% of 750.062 + √575.962 =
2
(?) = 2691 (199.94 ÷ 12.5) × (?)
(?) = 51.87 48% of 750 + √576 = (200 ÷ 12.5) × (?)
(?) = 52 360 + 24 = 16 × (?)
(?) = 384/16 = 24 (Ans.)
98. Ans. (B)
12.5% of 460 + 35% of 768 – 8.33% of 105. Ans. (D)
62 = (?) (220.062 ÷ 24.12) × 4.96 + 225.06 = (?) ×
0.125 × 460 + 0.35 × 768 – 1/12 × 62 8.96
= (?) (220 ÷ 24) × 5 + 225 = (?) × 9
57.5 + 268.8 – 5.16 = (?) 9.16 × 5 + 225 = (?) × 9
321.13 = (?) 9 × 5 + 225 = (?) × 9
(?) = 321 45 + 225 = (?) × 9
(?) = 270/9 = 30 (Ans.)
99. Ans. (A)
(25.99)2 + (32.13)2 – (11.99)2 = (?)2 + 106. Ans. (B)
(34.90)
2 (? × 1.96) + 12.5% of 959.94 = (2699 ÷
2 2 2 2
26 + 32 – 12 = (?) + 35
2
8.96)
2
676 + 1024 – 144 = (?) + 1225 (?) × 2 + 12.5% of 960 = (2700 ÷ 9)
2
(?) = 1556 – 1225 = 331 (?) × 2 + 120 = 300
(?) = 18 (?) × 2 = 180
(?) = 90 (Ans.)
100. Ans. (B)
33.08 × 13.97 ÷ 7.09 + (?) = 19.5 × 55.3 107. Ans. (D)
33 × 14 ÷ 7 + (?) = 1078.35 39.94% of 12.5% of 1999.94 + 24.96%
33 × 2 + (?) = 1078 of 20.06% of 2499 = (?)
(?) = 1078 - 66 40% of 12.5% of 2000 + 25% of 20% of

PAGE h ps://t.me/studified h ps://instagram.com/aashisharorasocial?utm_medium=copy_link


Follow
17 Aashish Arora on: h ps://youtube.com/channel/UCYa4_JrOrf8R5Kz2uOtccXQ https://www.facebook.com/aashisharorasocial/
अ ास by Aashish Arora
(SBI/IBPS/RBI/LIC/All other banking and insurance Exams)

2500 = (?)
(2/5) × (1/8) × 2000 + (1/4) × (1/5) ×
2500 = (?)
100 + 125 = (?)
(?) = 225 (Ans.)

Join
Joinme
meonon

Join me on
Join me on

Join me on
Join me on
Join me on
BY: AASHISH ARORA

PAGE h ps://t.me/studified h ps://instagram.com/aashisharorasocial?utm_medium=copy_link


Follow
18 Aashish Arora on: h ps://youtube.com/channel/UCYa4_JrOrf8R5Kz2uOtccXQ https://www.facebook.com/aashisharorasocial/
अ ास by Aashish Arora
(SBI/IBPS/RBI/LIC/All other banking and insurance Exams)

2 Quadratic Equations

In the given question, two equations numbered I and II are given. Solve both the equations and
mark the appropriate answer.
िदए गए म दो समीकरण I और II िदए गए ह। दोनो ं समीकरणो ं को हल कर और उिचत उ र को िचि त कर।
2
1. 6x + 17x – 3= 0
2
6y – 31y + 5= 0
(A) x > y (B) x<y
(C) x = y or no relation can be established (D) x≤y
(E) x ≥ y

2. 9x + 7y – 112 = 0
7x – 9y + 18 = 0
(A) x > y (B) x<y
(C) x = y or no relation can be established (D) x≤y
(E) x ≥ y

3. 2x2 = 450
y2 + 32y + 255 = 0
(A) x > y (B) x<y
(C) x = y or no relation can be established (D) x≤y
(E) x ≥ y
2
4. x – 19x + 90 = 0
2
y + y – 72 = 0
(A) x > y (B) x<y
(C) x = y or no relation can be established (D) x≤y
(E) x ≥ y

5. 8x2 + 94x + 105 = 0


15y2 – 41y – 34 = 0
(A) x > y (B) x<y
(C) x = y or no relation can be established (D) x≤y
(E) x ≥ y
2
6. x – 256 = 0
y2 – 33y + 272 = 0
(A) x > y (B) x<y
(C) x = y or no relation can be established (D) x≤y
(E) x ≥ y
2
7. 3x – 22x + 40 = 0
2
3y + 11y – 70 = 0
(A) x > y (B) x<y
(C) x = y or no relation can be established (D) x≤y
(E) x ≥ y

PAGE h ps://t.me/studified h ps://instagram.com/aashisharorasocial?utm_medium=copy_link


Follow
19 Aashish Arora on: h ps://youtube.com/channel/UCYa4_JrOrf8R5Kz2uOtccXQ https://www.facebook.com/aashisharorasocial/
अ ास by Aashish Arora
(SBI/IBPS/RBI/LIC/All other banking and insurance Exams)

2
8. 3x = 147
3
5y = 1080
(A) x > y (B) x<y
(C) x = y or no relation can be established (D) x≤y
(E) x ≥ y

9. 2x2 – 5x – 33 = 0
2
3y + 25y + 48 = 0
(A) x > y (B) x<y
(C) x = y or no relation can be established (D) x≤y
(E) x ≥ y

10. 4x2 + 23x – 6 = 0


4y2 – 29y + 7 = 0
(A) x > y (B) x<y
(C) x = y or no relation can be established (D) x≤y
(E) x ≥ y
2
11. 2x + 5x – 18 = 0
2
y – 19y – 42 = 0
(A) x > y (B) x<y
(C) x = y or no relation can be established (D) x≤y
(E) x ≥ y

12. 21x + 8y – 95 = 0
14x + 5y – 62 = 0
(A) x > y (B) x<y
(C) x = y or no relation can be established (D) x≤y
(E) x ≥ y
2
13. 2x = 128
3
y + 816 = 87
(A) x > y (B) x<y
(C) x = y or no relation can be established (D) x≤y
(E) x ≥ y

14. x2 – 19x + 78 = 0
2
y – 16y + 48 = 0
(A) x > y (B) x<y
(C) x = y or no relation can be established (D) x≤y
(E) x ≥ y

15. 11x – 5.5y = 0


4x + 2.5y + 5 = 0
(A) x > y (B) x<y
(C) x = y or no relation can be established (D) x≤y
(E) x ≥ y
2 2
16. 5x – 20x + 100 = 4x + 5x – 36
2
y + 18y= 0
(A) x > y (B) x<y
(C) x = y or no relation can be established (D) x≤y

PAGE h ps://t.me/studified h ps://instagram.com/aashisharorasocial?utm_medium=copy_link


Follow
20 Aashish Arora on: h ps://youtube.com/channel/UCYa4_JrOrf8R5Kz2uOtccXQ https://www.facebook.com/aashisharorasocial/
अ ास by Aashish Arora
(SBI/IBPS/RBI/LIC/All other banking and insurance Exams)

(E) x ≥ y

17. x2 – 21x + 108= 0


y2 = 100
(A) x > y (B) x<y
(C) x = y or no relation can be established (D) x≤y
(E) x ≥ y

18. 7x – 5y = 25
4x + 3y = 26
(A) x > y (B) x<y
(C) x = y or no relation can be established (D) x≤y
(E) x ≥ y

19. 5x2 = 405


y2 + 22y + 117= 0
(A) x > y (B) x<y
(C) x = y or no relation can be established (D) x≤y
(E) x ≥ y
2
20. 3x + 16x – 35 = 0
3y2 – 20y + 25 = 0
(A) x > y (B) x<y
(C) x = y or no relation can be established (D) x≤y
(E) x ≥ y

21. 11x – 7y = 12
9x + y = 30
(A) x > y (B) x<y
(C) x = y or no relation can be established (D) x≤y
(E) x ≥ y

22. 7x2 + 20x – 3 = 0


6y2 – 25y + 4= 0
(A) x > y (B) x<y
(C) x = y or no relation can be established (D) x≤y
(E) x ≥ y
3
23. x = -729
2
y + 14y + 45 = 0
(A) x > y (B) x<y
(C) x = y or no relation can be established (D) x≤y
(E) x ≥ y

24. 15x + 13y = 508


37y = 592
(A) x > y (B) x<y
(C) x = y or no relation can be established (D) x≤y
(E) x ≥ y

25. (x + 6)2 = 27x – 18


(y – 8)2 = 10 – y

PAGE h ps://t.me/studified h ps://instagram.com/aashisharorasocial?utm_medium=copy_link


Follow
21 Aashish Arora on: h ps://youtube.com/channel/UCYa4_JrOrf8R5Kz2uOtccXQ https://www.facebook.com/aashisharorasocial/
अ ास by Aashish Arora
(SBI/IBPS/RBI/LIC/All other banking and insurance Exams)

(A) x > y (B) x<y


(C) x = y or no relation can be established (D) x≤y
(E) x ≥ y

26. 4x2 +5x -3= 0


5y2 = 100
(A) x > y (B) x<y
(C) x = y or no relation can be established (D) x≤y
(E) x ≥ y
2
27. x – 8x + 15= 0
y2 – 4y -5= 0
(A) x > y (B) x<y
(C) x = y or no relation can be established (D) x≤y
(E) x ≥ y

28. 4x+3y -18= 0


7x +5y = 12
(A) x > y (B) x<y
(C) x = y or no relation can be established (D) x≤y
(E) x ≥ y

29. x2 + x – 56= 0
y2 – 26y + 144= 0
(A) x > y (B) x<y
(C) x = y or no relation can be established (D) x≤y
(E) x ≥ y
2
30. 2x = 1058
2
y – 28y + 115= 0
(A) x > y (B) x<y
(C) x = y or no relation can be established (D) x≤y
(E) x ≥ y

31. x2 – 33x + 252 = 0


5y2 – 74y + 209 = 0
(A) x > y (B) x<y
(C) x = y or no relation can be established (D) x≤y
(E) x ≥ y

32. 7x + 11y = 28
6x + 13y = 49
(A) x > y (B) x<y
(C) x = y or no relation can be established (D) x≤y
(E) x ≥ y
2
33. x = 169
2
y – 13y = 0
(A) x > y (B) x<y
(C) x = y or no relation can be established (D) x≤y
(E) x ≥ y

PAGE h ps://t.me/studified h ps://instagram.com/aashisharorasocial?utm_medium=copy_link


Follow
23 Aashish Arora on: h ps://youtube.com/channel/UCYa4_JrOrf8R5Kz2uOtccXQ https://www.facebook.com/aashisharorasocial/
अ ास by Aashish Arora
(SBI/IBPS/RBI/LIC/All other banking and insurance Exams)
2
34. x + 13x + 42 = 0
y2 + 5y = 0
(A) x > y (B) x<y
(C) x = y or no relation can be established (D) x≤y
(E) x ≥ y
2
35. x - 49 = 0
2
y + 58y + 165= 0
(A) x > y (B) x<y
(C) x = y or no relation can be established (D) x≤y
(E) x ≥ y

36. x2 – 34x + 225 = 0


y2 - 20y + 96 = 0
(A) x > y (B) x<y
(C) x = y or no relation can be established (D) x≤y
(E) x ≥ y
2
37. x + 5x + 6 = 0
2
y – 4y – 12 = 0
(A) x > y (B) x≥y (C) x < y
(D) x ≤ y (E) x = y or no relation can be established

38. x2 – 3 = 2x
y2 + 5y + 6 = 0
(A) x > y (B) x≥y (C) x < y
(D) x ≤ y (E) x = y or no relation can be established
2
39. x – 25x + 114 = 0
2
y – 10y + 24 = 0
(A) x > y (B) x≥y (C) x < y
(D) x ≤ y (E) x = y or no relation can be established

40. 48x2 – 24x + 3 = 0


55y2 + 53y + 12 = 0
(A) x > y (B) x≥y (C) x < y
(D) x ≤ y (E) x = y or no relation can be established
2
41. x – 7 √3 x + 36 = 0
2
y – 11 √3 y + 84 = 0
(A) x > y (B) x≥y (C) x < y
(D) x ≤ y (E) x = y or no relation can be established

PAGE h ps://t.me/studified h ps://instagram.com/aashisharorasocial?utm_medium=copy_link


Follow
24 Aashish Arora on: h ps://youtube.com/channel/UCYa4_JrOrf8R5Kz2uOtccXQ https://www.facebook.com/aashisharorasocial/
अ ास by Aashish Arora
(SBI/IBPS/RBI/LIC/All other banking and insurance Exams)

SOLUTION

1. Ans. (D) x2 - 256 = 0


6x2 + 17x - 3 = 0 x2 = 256
(6x - 1)(x + 3) = 0 x = ±16
x = 1/6, -3 y2 - 33y + 272 = 0
2
6y - 31y + 5 = 0 (y - 16)(y - 17) = 0
(6y - 1)(y - 5) = 0 y = 16, 17
x = 1/6, 5 x≤y
x≤y
7. Ans. (E)
2. Ans. (B) 3x2 - 22x + 40 = 0
9x + 7y = 112 – (Eq. 1) (x - 4)(3x - 10) = 0
7x - 9y = - 18 – (Eq. 2) x = 4, 10/3
By solving Eq.1 & Eq,2 3y2 + 11y - 70 = 0
x = 441/65 (3y - 10)(y + 7) = 0
y = 473/65 y = 10/3, -7
x<y x≥y

3. Ans. (E) 8. Ans. (C)


2x2 = 450 3x2 = 147
x2 = 225 x2 = 49
x = ±15 x = ±7
y2 + 32y + 255 = 0 5y3 = 1080
(y + 15)(y + 17) = 0 Y3 = 216
y = -15, - 17 y=6
x≥y Relationship cannot be established

4. Ans. (A) 9. Ans. (E)


x2 - 19x + 90 = 0 2
2x - 5x - 33 = 0
(x - 10)(x - 9) = 0 (x + 3)(2x - 11) = 0
x = 10, 9 x = -3, 11/2
y2 + y - 72 = 0 3y2 + 25y + 48 = 0
(y + 9)(y - 8) = 0 (y + 3)(3y + 16) = 0
x = - 9, 8 x = -3, -16/3
x>y x≥y

5. Ans. (B) 10. Ans. (D)


2 2
8x + 94x + 105 = 0 4x + 23x - 6 = 0
(4x + 5)(2x + 21) = 0 (x + 6)(4x - 1) = 0
x = -5/4, -21/2 x = -6, 1/4
15y2 - 41y - 34 = 0 4y2 - 29y + 7 = 0
(3y + 2)(5y – 17) = 0 (4y - 1)(y - 7) = 0
y = -2/3, 17/5 y = 1/4, 7
x<y x≤y

6. Ans. (D) 11. Ans. (C)

PAGE h ps://t.me/studified h ps://instagram.com/aashisharorasocial?utm_medium=copy_link


Follow
25 Aashish Arora on: h ps://youtube.com/channel/UCYa4_JrOrf8R5Kz2uOtccXQ https://www.facebook.com/aashisharorasocial/
अ ास by Aashish Arora
(SBI/IBPS/RBI/LIC/All other banking and insurance Exams)

2
2x + 5x - 18 = 0 17. Ans. (C)
2
(x - 2)(2x + 9) = 0 x – 21x + 108 = 0
x = 2, -9/2 (x – 9)(x – 12) = 0
y2 - 19y - 42 = 0 x = 9, 12
(y + 2)(y - 21) = 0 y2 = 100
y = -2, 21 y = ± 10
Relationship cannot be established Relationship cannot be established

12. Ans. (B) 18. Ans. (A)


21x + 8y - 95 = 0 – (Eq. 1) 7x – 5y = 25 ) × 3
14x + 5y - 62 = 0 – (Eq. 2) 4x + 3y = 26 ) × 5
By solving Eq.1 & 2 21x + 20x = 25 × 3 + 26 × 5
x=3 41x = 205
y=4 x = 5,
x<y y = 7 × 5 – 25 / 5 = 2
x>y
13. Ans. (C)
2x2 = 128 19. Ans. (E)
2 2
x = 64 5x = 405
2
x = ±8 x = 81
3
y + 816 = 87 x=±9
3 2
y = -729 y + 22y + 117 = 0
y=-9 (y + 9)(y + 13)
Relationship cannot be established y = -9, -13
x≥y
14. Ans. (C)
x2 – 19x + 78 = 0 20. Ans. (D)
(x – 6)(x – 13) = 0 3x2 + 16x – 35 = 0
x = 6, 13 (3x – 5)(x + 7) = 0
2
y – 16y + 48 = 0 x = 5/3, – 7
2
y = (y – 12)(y – 4) 3y – 20y + 25 = 0
y = 12, 4 (3y – 5)(y – 5) = 0
Relationship cannot be established y = 5/3, 5
x≤ y
15. Ans. (A)
11x – 5.5y = 0) × 4 21. Ans. (C)
4x + 2.5y = - 5) × 11 11x – 7y = 12
– 22y – 27.5y = 55 9x + y = 30
y = – 10/9 y = 30 – 9x
11x = 5.5 × (–10/9) 11x – 7(30 – 9x) = 12
x = – 5/9 11x – 210 + 63x = 12
x>y 74x = 222
x = 3,
16. Ans. (A) y = 30 – 9 × 3 = 3
5x2 – 20x + 100 = 4x2 + 5x – 36 x=y
x2 – 25x + 136 = 0
(x – 8)(x – 17) = 0 22. Ans. (B)
2
x = 8, 17 7x + 20x – 3 = 0
2
y + 18y = 0 (7x – 1)(x + 3) = 0
y(y + 18) = 0 x = – 3, 1/7
2
y = 0, - 18 6y – 25y + 4 = 0
x>y (6y – 1)(y – 4) = 0

PAGE h ps://t.me/studified h ps://instagram.com/aashisharorasocial?utm_medium=copy_link


Follow
26 Aashish Arora on: h ps://youtube.com/channel/UCYa4_JrOrf8R5Kz2uOtccXQ https://www.facebook.com/aashisharorasocial/
अ ास by Aashish Arora
(SBI/IBPS/RBI/LIC/All other banking and insurance Exams)

y = 4, 1/6 y = 5, - 1
x<y x=y or relationship cannot be
established
23. Ans. (D)
3
x = – 729 28. Ans. (B)
x= –9 4x + 3y = 18, 7x + 5y = 12
2
y + 14y + 45 = 0 5(4x + 3y) – 3(7x + 5y) = 5 × 18 – 3 × 12
(y + 9)(y + 5) = 0 x = - 54
y = – 9, -5 4 × (- 54) + 3y = 18
x≤y 3y = 18 + 216 = 234
y = 78
24. Ans. (A) x<y
15x + 13y = 508
37y = 592 29. Ans. (B)
y = 16 x2 + x – 56 = 0
15x + 13 × 16 = 508 (x + 8)(x – 7) = 0
15x = 508 – 208 = 300 X = -8. 7
x = 20 y2 – 26y + 144 = 0
x>y (y – 8)(y – 18) = 0
y = 8, 18
25. Ans. (C) x<y
(x + 6)2 = 27x – 18
2
x + 12x + 36 = 27x – 18 30. Ans. (C)
2
x – 15x + 54 = 0 2x2 = 1058
(x – 9)(x – 6) = 0 x2 = 529
x = 9, 6 x = ± 23
2
(y – 8) = 10 – y y2 – 128y + 115 = 0
y2 – 16y + 64 = 10 - y (y – 23)(y – 5) = 0
y2 – 16y + 54 = 0 y = 23, 5
(y – 9)(y – 6) = 0 x = y or No Relationship
y = 9, 6
x = y or relationship cannot be 31. Ans. (A)
2
established x – 33x + 252 = 0
(x – 12)(x – 21) = 0
26. Ans. (C) x = 12, 21
2
4x + 5x – 3 = 0 5y2 – 74y + 209 = 0
(y – 11)(5y – 19) = 0
- 5 ± 52 + 4 ´ 4 ´ 3 - 5 Ò 73
x= = y = 11, 3.8
2´ 4 8 x>y
x = 0.48, - 1.73
5y2 = 100 32. Ans. (B)
2
y = 20 6(7x + 11y) = 6 × 28
y = ± 4.47 – 7(6x + 13y) = 7 × 49
Relationship Cannot be established = 66y – 91y = 168 – 343
y=7
27. Ans. (C) 7x + 11 × 7 = 28
x2 – 8x + 15 = 0 x = 4 - 11 = - 7
(x – 3)(x – 5) = 0 x<y
x = 3, 5
y2 – 4y – 5 = 0 33. Ans. (C)
2
(y – 5)(y + 1) = 0 x = 169

PAGE h ps://t.me/studified h ps://instagram.com/aashisharorasocial?utm_medium=copy_link


Follow
27 Aashish Arora on: h ps://youtube.com/channel/UCYa4_JrOrf8R5Kz2uOtccXQ https://www.facebook.com/aashisharorasocial/
अ ास by Aashish Arora
(SBI/IBPS/RBI/LIC/All other banking and insurance Exams)

2
x = ± 13 x – 25x + 114 = 0
y2 – 13y = 0 (x – 6)(x - 19) = 0
y(y – 13) = 0 x = 6, 19
2
y = 0, 13 y – 10y + 24 =0
x = y or relationship can't be established (y – 4)(y – 6)
y = 4, 6
34. Ans. (B) x≥y
2
x + 13x + 42 = 0
(x + 6)(x + 7) = 0 40. Ans. (A)
x = - 6, - 7 48x2 – 24x + 3 = 0
y2 + 5y = 0 2
3(4x – 1) = 0
y(y + 5) = 0 x = 1/4
2
y = 0, - 5 55y + 53y + 12 = 0
x<y (11y + 4)(5y + 3) = 0
y = – 4/11, 3/5
35. Ans. (C) x > y Relation can’t be established
2
x – 49 = 0
2
x = 49 41. Ans. (D)
x=±7 x2 – 7√3x + 36 = 0
y2 + 58y + 165 = 0 (x - 4√3)(x - 3√3) = 0
(y + 3)(y + 55) = 0 x = 4√3, 3√3
2
y = - 3, - 55 y – 11√3y + 84 = 0
Relationship can't be established (y - 7√3)(y - 4√3)
y = 7√3, 4√3
36. Ans. (C) x≤y
2
x – 34x + 225 = 0
(x – 25)(x – 9) = 0
x = 25, 9
y2 – 20y + 96 = 0
(y – 12)(y – 8) = 0
y = 12, 8
Relationship can't be eastablished

37. Ans. (D)


2
x + 5x + 6 = 0
(x + 2)(x + 3) = 0
x = - 2, - 3
2
y – 4y – 12 = 0
(y – 6)(y + 2) = 0
y = 6, - 2
x≤y

38. Ans. (A)


2
x – 3 = 2x
(x – 3)(x + 1) = 0
x = 3, - 1
2
y + 5y + 6 = 0
(y + 2)(y + 3) = 0
y = - 2, - 3
x>y

39. Ans. (B)

PAGE h ps://t.me/studified h ps://instagram.com/aashisharorasocial?utm_medium=copy_link


Follow
28 Aashish Arora on: h ps://youtube.com/channel/UCYa4_JrOrf8R5Kz2uOtccXQ https://www.facebook.com/aashisharorasocial/
अ ास by Aashish Arora
(SBI/IBPS/RBI/LIC/All other banking and insurance Exams)

3 Missing Number Series

In each of these questions a number is missing in the series. Find out the missing
number.
इनम से ेक ृंखला म एक सं ा लु है , लु सं ा ात कीिजए

1. 56, 112, 162, ? , 244, 276


(A) 206 (B) 208 (C) 210
(D) 212 (E) None of these

2. 0, ? , 133, 1464, 6377, 18544


(A) 4 (B) 6 (C) 8
(D) 10 (E) None of these

3. 5, 12, 8, 40, 18, ?


(A) 122 (B) 120 (C) 130
(D) 140 (E) None of these

4. ? , 77, 91, 105, 126, 168


(A) 63 (B) 41 (C) 57
(D) 49 (E) None of these

5. 1, 8, 22, 50, 99, ?


(A) 126 (B) 196 (C) 186
(D) 176 (E) None of these

6. 7, ? , 19, 31, 67, 211


(A) 13 (B) 14 (C) 15
(D) 16 (E) None of these

7. 3, 6, ? , 18, 27, 38
(A) 11 (B) 13 (C) 15
(D) 17 (E) None of these

8. 990, 200, 54, 24, 20, ?


(A) 10 (B) 16 (C) 30
(D) 8 (E) None of these

9. 11, ? , 38, 64, 105, 166


(A) 19 (B) 22 (C) 25

PAGE h ps://t.me/studified h ps://instagram.com/aashisharorasocial?utm_medium=copy_link


Follow
29 Aashish Arora on: h ps://youtube.com/channel/UCYa4_JrOrf8R5Kz2uOtccXQ https://www.facebook.com/aashisharorasocial/
अ ास by Aashish Arora
(SBI/IBPS/RBI/LIC/All other banking and insurance Exams)

(D) 28 (E) None of these

10. 1, 3, 15, 51, ? , 281


(A) 196 (B) 116 (C) 126
(D) 131 (E) None of these

11. 8, 16, 20, 24, 30, ?


(A) 42 (B) 48 (C) 52
(D) 50 (E) None of these

12. 11, 22, 44, 88, ? , 286


(A) 165 (B) 99 (C) 110
(D) 132 (E) None of these

13. 448, 450, ? , 80, 25, 11


(A) 222 (B) 220 (C) 228
(D) 226 (E) None of these

14. 29, 33, 21, ? , 13, 49


(A) 43 (B) 41 (C) 47
(D) 48 (E) None of these

15. 10, ? , 16, 36, 78, 150


(A) 12 (B) 16 (C) 18
(D) 10 (E) None of these

16. ? , 51, 76, 96, 110, 117


(A) 22 (B) 24 (C) 21
(D) 17 (E) None of these

17. 4, 13, 36, 97, 268, ?


(A) 720 (B) 765 (C) 745
(D) 760 (E) None of these

18. -4, -3, ? 23, 252, 3121


(A) 3 (B) 2 (C) 0
(D) 1 (E) None of these

19. 100, ? , 108, 120, 140, 170


(A) 108 (B) 106 (C) 104
(D) 102 (E) None of these

20. 178, 181, 176, ? , 174, 185

PAGE h ps://t.me/studified h ps://instagram.com/aashisharorasocial?utm_medium=copy_link


Follow
30 Aashish Arora on: h ps://youtube.com/channel/UCYa4_JrOrf8R5Kz2uOtccXQ https://www.facebook.com/aashisharorasocial/
अ ास by Aashish Arora
(SBI/IBPS/RBI/LIC/All other banking and insurance Exams)

(A) 184 (B) 183 (C) 172


(D) 173 (E) None of these

21. 10, 13, 29, 90, ? , 1818


(A) 363 (B) 447 (C) 535
(D) 391 (E) None of these

22. 57, 72, 102, ?, 207, 282


(A) 147 (B) 162 (C) 153
(D) 157 (E) None of these

23. 3, 7, 13, 21, 31, ?


(A) 41 (B) 45 (C) 42
(D) 43 (E) None of these

24. 44, 47, 53, 65, 89, ?


(A) 137 (B) 185 (C) 125
(D) 172 (E) None of these

25. 9, 19, 40, ?, 170, 345


(A) 80 (B) 90 (C) 83
(D) 77 (E) 95

26. 111, 128, 147, 168, ?, 216


(A) 189 (B) 193 (C) 187
(D) 183 (E) 191

27. 8, 32, 33, 132, 133, ?


(A) 532 (B) 530 (C) 499
(D) 397 (E) 442

28. 7, 14, 27, 52, 101, ?


(A) 158 (B) 186 (C) 198
(D) 206 (E) 174

29. 10, 5, 5, 7.5, 15, ?


(A) 30 (B) 15 (C) 22.5
(D) 37.5 (E) 25

30. 43, 47, 53, 59, 61 , ?


(A) 63 (B) 65 (C) 71
(D) 67 (E) 73

PAGE h ps://t.me/studified h ps://instagram.com/aashisharorasocial?utm_medium=copy_link


Follow
31 Aashish Arora on: h ps://youtube.com/channel/UCYa4_JrOrf8R5Kz2uOtccXQ https://www.facebook.com/aashisharorasocial/
अ ास by Aashish Arora
(SBI/IBPS/RBI/LIC/All other banking and insurance Exams)

31. 43, 48, 63, 88, ?, 168


(A) 118 (B) 123 (C) 128
(D) 133 (E) 113

32. 5, 18, 39, ?, 105, 150


(A) 58 (B) 62 (C) 65
(D) 68 (E) 78

33. 1, 2, 6, 30, 210, ?


(A) 2310 (B) 1890 (C) 2100
(D) 1680 (E) None of these

34. 85, 89, 80, 96, ?, 107


(A) 83 (B) 90 (C) 77
(D) 71 (E) 103

35. 90, 114, 102, 126, ?, 138


(A) 114 (B) 148 (C) 116
(D) 118 (E) 134

36. 27, 28, 32, ?, 112 , 368


(A) 54 (B) 48 (C) 72
(D) 66 (E) None of these

37. 7, 12, 20, 31, ? , 62


(A) 51 (B) 48 (C) 52
(D) 45 (E) None of these

38. 34, 38, 47, 72, 121 , ?


(A) 256 (B) 278 (C) 242
(D) 224 (E) None of these

39. 133, 241, 507, ?, 2003 , 3981


(A) 989 (B) 1039 (C) 1074
(D) 991 (E) None of these

40. 48, 12, 6, ?, 12 , 48


(A) 3 (B) 4 (C) 6
(D) 9 (E) None of these

41. 127, 133, 121, 141, ? , 153


(A) 131 (B) 111 (C) 127
(D) 135 (E) None of these

PAGE h ps://t.me/studified h ps://instagram.com/aashisharorasocial?utm_medium=copy_link


Follow
32 Aashish Arora on: h ps://youtube.com/channel/UCYa4_JrOrf8R5Kz2uOtccXQ https://www.facebook.com/aashisharorasocial/
अ ास by Aashish Arora
(SBI/IBPS/RBI/LIC/All other banking and insurance Exams)

42. 9, 23, 85, 521, ? , 41567


(A) 4167 (B) 4015 (C) 4567
(D) 4155 (E) None of these

43. 12, 44, 164, ?, 2452, 9684


(A) 558 (B) 476 (C) 628
(D) 724 (E) None of these

44. 37, 45, 41, ?, 77 , 165


(A) 57 (B) 62 (C) 59
(D) 53 (E) None of these

45. 23, 34, 38, 53, 61 , ?


(A) 83 (B) 77 (C) 80
(D) 79 (E) None of these

46. 15, 23, ? , 87, 143, 215


(A) 24 (B) 47 (C) 18
(D) 44 (E) None of these

47. 20, 92, 20, ?, 24, 84


(A) 90 (B) 34 (C) 17
(D) 141 (E) None of these

48. 14, 15, 17, 23, 47, ?


(A) 185 (B) 148 (C) 167
(D) 112 (E) None of these

49. 6, 108, 218, 344, ?, 724


(A) 154 (B) 223 (C) 113
(D) 502 (E) None of these

50. 580, 145, 435, ?, 326.25, 81.5625


(A) 115.45 (B) 107 (C) 108.75
(D) 213.25 (E) None of these

51. 415, 440, 475, 520, 575, ?


(A) 645 (B) 625 (C) 640
(D) 595 (E) None of these

52. 1800, 1804, 1813, ?, 1854, 1890


(A) 1831 (B) 1815 (C) 1834
(D) 1829 (E) None of these

PAGE h ps://t.me/studified h ps://instagram.com/aashisharorasocial?utm_medium=copy_link


Follow
33 Aashish Arora on: h ps://youtube.com/channel/UCYa4_JrOrf8R5Kz2uOtccXQ https://www.facebook.com/aashisharorasocial/
अ ास by Aashish Arora
(SBI/IBPS/RBI/LIC/All other banking and insurance Exams)

53. 300, 292, 276, ?, 220, 180


(A) 252 (B) 260 (C) 254
(D) 258 (E) None of these

54. 535, 535, ?, 556, 577, 605


(A) 543 (B) 542 (C) 549
(D) 555 (E) None of these

55. 48, 52, ?, 104, 168, 268


(A) 65 (B) 68 (C) 56
(D) 78 (E) None of these

56. 30, 45, 59, ?, 84, 95


(A) 72 (B) 75 (C) 70
(D) 68 (E) None of these

57. 8, 21, 52, 119, ?, 541


(A) 25 (B) 355 (C) 258
(D) 350 (E) None of these
58. 54, 68, ?, 138, 194, 264
(A) 78 (B) 96 (C) 91
(D) 99 (E) None of these

59. 26, 78, ?, 117, 58.5, 175.5


(A) 49 (B) 41 (C) 39
(D) 51 (E) None of these

60. 14, 24, 44, ?, 114, 164


(A) 72 (B) 73 (C) 74
(D) 75 (E) None of these

61. 980, 392, 156.8, ?, 25.08, 10.03


(A) 24.24 (B) 47.15 (C) 18.25
(D) 62.72 (E) None of these

62. 5, 36, 218, 1093, 4376, ?


(A) 90365 (B) 34354 (C) 13133
(D) 14114 (E) None of these

63. 5, 36, 218, 1093, 4376, ?


(A) 1145 (B) 1017 (C) 1075
(D) 2915 (E) None of these

PAGE h ps://t.me/studified h ps://instagram.com/aashisharorasocial?utm_medium=copy_link


Follow
34 Aashish Arora on: h ps://youtube.com/channel/UCYa4_JrOrf8R5Kz2uOtccXQ https://www.facebook.com/aashisharorasocial/
अ ास by Aashish Arora
(SBI/IBPS/RBI/LIC/All other banking and insurance Exams)

64. 826, 701, ?, 610, 602


(A) 185 (B) 637 (C) 767
(D) 812 (E) None of these

65. 27, 45, 61, ?, 79


(A) 73 (B) 22 (C) 83
(D) 50 (E) None of these

66. 997920, 90720, ____, 1008, 126, 18


(A) 9072 (B) 8765 (C) 11567
(D) 9502 (E) 11009

67. 23, 70, 281, 1406, ____, 59060


(A) 8407 (B) 8437 (C) 8505
(D) 8369 (E) 8003

68. 37, 18, 17, ___, 46, 112.5


(A) 21 (B) 32 (C) 24
(D) 29 (E) 31

69. 92, 101, ___, 110, 74, 123


(A) 76 (B) 106 (C) 64
(D) 103 (E) 85

70. 59, 67, 83, ____, 179, 307


(A) 115 (B) 109 (C) 129
(D) 96 (E) 123

71. 665599, 60509, ____, 960.46, 192.09, 64.03


(A) 6723.22 (B) 8765.09 (C) 1167.86
(D) 9502.22 (E) 7196.08

72. 67, 130, 256, 508, _____, 2020


(A) 998 (B) 1012 (C) 850
(D) 770 (E) 803

73. 6.6, 3.3, 3.3, 4.95, ____, 24.75


(A) 6.6 (B) 7.2 (C) 9.9
(D) 2.9 (E) 7.8

74. 6, 288, 6912, _____,497664, 1492992


(A) 76944 (B) 81944 (C) 64069
(D) 14300 (E) 82944

PAGE h ps://t.me/studified h ps://instagram.com/aashisharorasocial?utm_medium=copy_link


Follow
35 Aashish Arora on: h ps://youtube.com/channel/UCYa4_JrOrf8R5Kz2uOtccXQ https://www.facebook.com/aashisharorasocial/
अ ास by Aashish Arora
(SBI/IBPS/RBI/LIC/All other banking and insurance Exams)

75. 32, 96, 312, 824, ____, 3552


(A) 1824 (B) 1560 (C) 1209
(D) 1324 (E) 1009

Join
Joinme
meonon

Join me on
Join me on

Join me on
Join me on
Join me on
BY: AASHISH ARORA

PAGE h ps://t.me/studified h ps://instagram.com/aashisharorasocial?utm_medium=copy_link


Follow
36 Aashish Arora on: h ps://youtube.com/channel/UCYa4_JrOrf8R5Kz2uOtccXQ https://www.facebook.com/aashisharorasocial/
अ ास by Aashish Arora
(SBI/IBPS/RBI/LIC/All other banking and insurance Exams)

SOLUTION

1. Ans. (A) 11 + 7 = 18
56 + 56 = 112 18 + 9 = 27
112 + 50 = 162 27 + 11 = 38
162 + 44 = 206
206 + 38 = 244 8. Ans. ©
244 + 32 = 276 990 ÷ 5 + 2 = 200
200 ÷ 4 + 4 = 54
2. Ans. © 54 ÷ 3 + 6 = 24
8 - 0 = 8 = 23 24 ÷ 2 + 8 = 20
133 - 8 = 125 = 53 20 ÷ 1 + 10 = 30
1464 - 133 = 1331 = 113
6377 - 1464 = 4913 = 173 9. Ans. (B)
3
18544 - 6377 = 12167 = 23 22 - 11 = 11
38 - 22 = 16, (16 - 11 = 5)
3. Ans. (A) 64 - 38 = 26, (26 - 16 = 10)
5 × 2 + 2 = 12 105 - 64 = 41, (41 - 26 = 15)
12 ÷ 3 + 4 = 8 166 - 105 = 61, (61 - 41 = 20)
8 × 4 + 8 = 40
40 ÷ 5 + 10 = 18 10. Ans. (D)
18 × 6 + 14 = 122 1 + 12 + 13 = 3
3 + 22 + 23 = 15
4. Ans. (D) 15 + 32 + 33 = 51
77 - 49 = 28 (28 × 0.5 = 14) 51 + 42 + 43 = 131
2 3
91 - 77 = 14 (14 × 1 = 14) 131 + 5 + 5 = 281
105 - 91 = 14 (14 × 1.5 = 21)
126 - 105 = 21 (21 × 2 = 42) 11. Ans. (A)
168 - 126 = 42 16 - 8 = 8 (8 × 0.5 = 4)
20 - 16 = 4 (4 × 1 = 4)
5. Ans. (D) 24 - 20 = 4 (4 × 1.5 = 6)
8-1=7 (14-7 = 7×1) 30 - 24 = 6 (6 × 2 = 12)
22 - 8 = 14 (28-14 = 7×2) 42 - 30 = 12
50 - 22 = 28 (49-28 = 7×3)
99 - 50 = 49 (77-49 = 7×4) 12. Ans. (A)
176 - 99 = 77 22 - 11 = 11 (22 – 11 = 11 × 1)
44 - 22 = 22 (44 – 22 = 11 × 2)
6. Ans. (A) 88 - 44 = 44 (77 – 44 = 11 × 3)
7 + 6 × 0! = 13 165 - 88 = 77 (121 – 77 = 11 × 4)
13 + 6 × 1! = 19 286 - 165 = 121
19 + 6 × 2! = 31
31 + 6 × 3! = 67 13. Ans. ©
67 + 6 × 4! = 211 448 ÷ 1 + 2 = 450
450 ÷ 2 + 3 = 228
7. Ans. (A) 228 ÷ 3 + 4 = 80
3+3=6 80 ÷ 4 + 5 = 25
6 + 5 = 11 25 ÷ 5 + 6 = 11

PAGE h ps://t.me/studified h ps://instagram.com/aashisharorasocial?utm_medium=copy_link


Follow
37 Aashish Arora on: h ps://youtube.com/channel/UCYa4_JrOrf8R5Kz2uOtccXQ https://www.facebook.com/aashisharorasocial/
अ ास by Aashish Arora
(SBI/IBPS/RBI/LIC/All other banking and insurance Exams)

14. Ans. (B) 21. Ans. (A)


29 + 4 = 33 10 × 1 + 3 = 13
33 - 12 = 21 12 × 2 + 3 = 29
21 + 20 = 41 29 × 3 + 3 = 90
41 - 28 = 13 90 × 4 + 3 = 363
13 + 36 = 49 363 × 5 + 3 = 1818

15. Ans. (D) 22. Ans. (A)


10 + 0 × 1 = 10 57 + (15 × 1) = 72
10 + 2 × 3 = 16 72 + (15 × 2) = 102
16 + 4 × 5 = 36 102 + (15 × 3) = 147
36 + 6 × 7 = 78 147 + (15 × 4) = 207
78 + 8 × 9 = 150 207 + (15 × 5) = 282

16. Ans. (A) 23. Ans. (D)


51 – 22 = 29 (29 – 25 = 4) 3+4=7
76 – 51 = 25 (25 – 20 = 5) 7 + 6 = 13
96 – 76 = 20 (20 – 14 = 6) 13 + 8 = 21
110 – 96 = 14 (14 – 7 = 7) 21 + 10 = 31
117 -110 = 7 31 + 12 = 43

17. Ans. (B) 24. Ans. (A)


1 2 0
3 +1 =4 44 + 3 × 2 = 47
2 2 1
3 + 2 = 13 47 + 3 × 2 = 53
33 + 32 = 36 53 + 3 × 22 = 65
34 + 42 = 97 65 + 3 × 23 = 89
35 + 52 = 268 89 + 3 × 24 = 137
6 2
3 + 6 = 765
25. Ans. ©
18. Ans. © 9 × 2 + 1 = 19
0
0 –4=–4 19 × 2 + 2 = 40
1
1 –4=–3 40 × 2 + 3 = 83
2
2 –4=0 83 × 2 + 4 = 170
33 – 4 = 23 170 × 2 + 5 = 345
44 – 4 = 252
55 – 4 = 3121 26. Ans. (E)
111 + 17 = 128
19. Ans. (D) 128 + 19 = 147
100 + 1 × 2 = 102 147 + 21 = 168
102 + 2 × 3 = 108 168 + 23 = 191
108 + 3 × 4 = 120 191 + 25 = 216
120 + 4 × 5 = 140
140 + 5 × 6 = 170 27. Ans. (A)
8 × 4 = 32
20. Ans. (B) 32 + 1 = 33
178 + 3 = 181 33 × 4 = 132
181 – 5 = 176 132 + 1 = 133
176 + 7 = 183 133 × 4 = 532
183 – 9 = 174
174 + 11 = 185 28. Ans. ©
0
14 – 7 = 7 (13 – 7 = 6 × 2 )
1
27 – 14 = 13 (25 – 13 = 6 × 2 )

PAGE h ps://t.me/studified h ps://instagram.com/aashisharorasocial?utm_medium=copy_link


Follow
38 Aashish Arora on: h ps://youtube.com/channel/UCYa4_JrOrf8R5Kz2uOtccXQ https://www.facebook.com/aashisharorasocial/
अ ास by Aashish Arora
(SBI/IBPS/RBI/LIC/All other banking and insurance Exams)

0
52 – 27 = 25 (49 – 25 = 6 × 2 2 ) 27 + 4 = 28
1
101 – 52 = 49 (97 – 49 = 6 × 2 3 ) 28 + 4 = 32
2
198 – 101 = 97 32 + 4 = 48
3
48 + 4 = 112
29. Ans. (D) 112 + 44 = 368
10 × 0.5 = 5
5×1=5 37. Ans. (D)
5 × 1.5 = 7.5 7 + 5 = 12
7.5 × 2 = 15 12 + 8 = 20
15 × 2.5 = 37.5 20 + 11 = 31
31 + 14 = 45
30. Ans. (D) 45 + 17 = 62
43, 47, 53, 59, 61, 67
Successive Prime Numbers 38. Ans. ©
34 + 22 = 38
31. Ans. (B) 38 + 32 = 47
43 + 5 = 48 47 + 52 = 72
48 + 15 = 63 72 + 72 = 121
63 + 25 = 88 121 + 112 = 242
88 + 35 = 123
123 + 45 = 168 39. Ans. (A)
133 × 2 – 25 = 241
32. Ans. (D) 241 × 2 + 25 = 507
5 + 13 = 18 507 × 2 – 25 = 989
18 + 21 = 39 989 × 2 + 25 = 2003
39 + 29 = 68 2003 × 2 - 25 = 3981
68 + 37 = 105
105 + 45 = 150 40. Ans. ©
48 × 0.25 = 12
33. Ans. (A) 12 × 0.5 = 6
1×2=2 6×1=6
2×3=6 6 × 2 = 12
6 × 5 = 30 12 × 4 = 48
30 × 7 = 210
210 × 11 = 2310 41. Ans. (B)
127 + (22 + 2) = 133
34. Ans. (D) 133 – (32 + 3) = 121
85 + 22 = 89 121 + (42 + 4) = 141
89 – 32 = 80 141 – (52 + 5) = 111
2
80 + 42 = 96 111 + (6 + 6) = 153
96 – 52 = 71
71 + 62 = 107 42. Ans. (D)
9 × 2 + 5 = 23
35. Ans. (A) 23 × 4 – 7 =85
90 + 24 = 114 85 × 6 + 11 = 521
114 – 12 = 102 521 × 8 – 13 = 4155
102 + 24 = 126 4155 × 10 + 17 = 41567
126 – 12 = 114
124 + 24 = 138 43. Ans. ©
(12 – 1) × 4 = 44
36. Ans. (B) (44 – 3) × 4 = 164

PAGE h ps://t.me/studified h ps://instagram.com/aashisharorasocial?utm_medium=copy_link


Follow
39 Aashish Arora on: h ps://youtube.com/channel/UCYa4_JrOrf8R5Kz2uOtccXQ https://www.facebook.com/aashisharorasocial/
अ ास by Aashish Arora
(SBI/IBPS/RBI/LIC/All other banking and insurance Exams)

(164 – 7) × 4 = 628 51. Ans. ©


(628 – 15) × 4 = 2452 415 + 25 = 440
(2452 – 31) × 4 = 9684 440 + 35 = 475
475 + 45 = 520
44. Ans. (A) 520 + 55 = 575
0
37 + (3 + 7) = 45 575 + 65 = 640
1
45 + (3 – 7) = 41
2
41 + (3 + 7) = 57 52. Ans. (D)
57 + (33 – 7) = 77 1800 + 22 = 1804
77 + (34 + 7) = 165 1804 + 32 = 1813
2
1813 + 4 = 1829
2
45. Ans. © 1829 + 5 = 1854
2
23 + 11 = 34 1854 + 6 = 1890
34 + 4 = 38
38 + 15 = 53 53. Ans. (A)
53 + 8 = 61 300 – 8 = 292
61 + 19 = 80 292 – 16 = 276
276 – 24 = 252
46. Ans. (B) 252 – 32 = 220
15 + 8 × 1 = 23 220 – 40 = 180
23 + 8 × 3 = 47
47 + 8 × 5 = 87 54. Ans. (B)
87 + 8 × 7 = 143 535 + 0 = 535
143 + 8 × 9 = 215 535 + 7 = 542
542 + 14 = 556
47. Ans. (A) 556 + 21 = 577
20 + 8 × 9 = 92 577 + 28 = 605
92 - 9 × 8 = 20
20 + 10 × 7 = 90 55. Ans. (B)
2
90 - 11 × 6 = 24 48 + 2 = 52
2
24 + 12 × 5 = 84 52 + 4 = 68
2
68 + 6 = 104
2
48. Ans. © 104 + 8 = 168
2
14 + 1! = 15 168 + 10 = 268
15 + 2! = 17
17 + 3! = 23 56. Ans. (A)
23 + 4! = 47 30 + 15 = 45
47 + 5! = 167 45 + 14 = 59
59 + 13 = 72
49. Ans. (D) 72 + 12 = 84
108 – 6 = 102 (110 – 102 = 8 × 2 0 ) 84 + 11 = 95
218 – 108 = 110 (126 – 110 = 8 × 2 1 )
344 – 218 = 126 (158 – 126 = 8 × 2 2 ) 57. Ans. ©
3
502 – 344 = 158 (222 – 158 = 8 × 2 ) 08 × 2 + 5 = 21
724 – 502 = 222 21 × 2 + 10 = 52
52 × 2 + 15 = 119
50. Ans. © 119 × 2 + 20 = 258
580 ÷ 4 = 145 258 × 2 + 25 = 541
145 × 3 = 435
435 ÷ 4 = 108.75 58. Ans. (B)
108.75 × 3 = 326.25 54 + 14 × 1 = 68
326.25 ÷ 4 = 81.5625 68 + 14 × 2 = 96

PAGE h ps://t.me/studified h ps://instagram.com/aashisharorasocial?utm_medium=copy_link


Follow
40 Aashish Arora on: h ps://youtube.com/channel/UCYa4_JrOrf8R5Kz2uOtccXQ https://www.facebook.com/aashisharorasocial/
अ ास by Aashish Arora
(SBI/IBPS/RBI/LIC/All other banking and insurance Exams)

96 + 14 × 3 = 138 66. Ans. (A)


138 + 14 × 4 = 194 997920 ÷ 11 = 90720
194 + 14 × 5 = 264 90720 ÷ 10 = 9072
9072 ÷ 9 = 1008
59. Ans. © 1008 ÷ 8 = 126
26 × 3 = 78 126 ÷ 7 = 18
78 ÷ 2 = 39
39 × 3 = 117 67. Ans. (B)
117 ÷ 2 = 58 23 × 3 + 1 = 70
58.5 × 3 = 175.5 70 × 4 + 1 = 281
281 × 5 + 1 = 1406
60. Ans. © 1406 × 6 + 1 = 8437
14 + 10 = 24 8437 × 7 + 1 = 59060
24 + 20 = 44
44 + 30 = 74 68. Ans. ©
74 + 40 = 114 37 × 0.5 – 0.5 = 18
114 + 50 = 116 18 × 1 – 1 = 17
17 × 1.5 – 1.5 = 24
61. Ans. (D) 24 × 2 – 2 = 46
980 × 0.4 = 392 46 × 2.5 – 2.5 =112.5
392 × 0.4 = 156.8
156.8 × 0.4 = 62.72 69. Ans. (E)
2 2
62.72 × 0.4 = 25.08 9 + 3 = 101
25.08 × 0.4 = 10.03 101 – 42 = 85
85 + 52 = 110
62. Ans. © 110 – 62 = 74
5 × 7 + 1 = 36 74 + 72 = 123
36 × 6 + 2 = 218
218 × 5 + 3 = 1093 70. Ans. (A)
1093 × 4 + 4 = 4376 59 + 8 = 67
4376 × 3 + 5 = 13133 67 + 16 = 83
83 + 32 = 115
63. Ans. (D) 115 + 64 = 179
(8 + 7) × 1 = 15 179 + 128 = 307
(15 + 6) × 2 = 42
(42 + 5) × 3 = 141 71. Ans. (A)
(141 + 4) × 4 = 580 665599 ÷ 11 = 60509
(580 + 3) × 5 = 2915 60509 ÷ 9 = 6723.22
6723.22 ÷ 7 = 960.46
64. Ans. (B) 960.45 ÷ 5 = 192.09
826 – 53 = 701 192.09 ÷ 3 = 64.03
701 – 43 = 637
3
637 – 3 = 610 72. Ans. (B)
3
610 – 2 = 602 (67 – 2) × 2 = 130
(130 – 2) × 2 = 256
65. Ans. (A) (256 – 2) × 2 = 508
45 – 27 = 18 (18 – 16 = 2 × 1) (508 – 2) × 2 = 1012
61 – 45 = 16 (16 – 12 = 2 × 2) (1012 – 2) × 2 = 2020
73 – 61 = 12 (12 – 6 = 2 × 3)
79 – 73 = 6 73. Ans. ©
6.6 × 0.5 = 3.3

PAGE h ps://t.me/studified h ps://instagram.com/aashisharorasocial?utm_medium=copy_link


Follow
41 Aashish Arora on: h ps://youtube.com/channel/UCYa4_JrOrf8R5Kz2uOtccXQ https://www.facebook.com/aashisharorasocial/
अ ास by Aashish Arora
(SBI/IBPS/RBI/LIC/All other banking and insurance Exams)

3.3 × 1 = 3.3
3.3 × 1.5 = 4.95
4.95 × 2 = 9.9
9.9 × 2.5 = 24.75

74. Ans. (E)


6 × 48 = 288
288 × 24 = 6912
6912 × 12 = 82944
82944 × 6 = 497664
497664 × 3 = 1492992

75. Ans. (A)


32 + 43 = 96
96 + 63 = 312
3
312 + 8 = 824
3
824 + 10 = 1824
3
1824 + 12 = 3552

Join
Joinme
meonon

Join me on
Join me on

Join me on
Join me on
Join me on
BY: AASHISH ARORA

PAGE h ps://t.me/studified h ps://instagram.com/aashisharorasocial?utm_medium=copy_link


Follow
42 Aashish Arora on: h ps://youtube.com/channel/UCYa4_JrOrf8R5Kz2uOtccXQ https://www.facebook.com/aashisharorasocial/
अ ास by Aashish Arora
(SBI/IBPS/RBI/LIC/All other banking and insurance Exams)

4 Wrong Number Series

In each of the following number series. The wrong number is given, find out that number ?
िन िल खत म से ेक सं ा ृंखला म। गलत सं ा दी गई है , वह सं ा ात कीिजए?

1. 37, 53, 21, 69, 5, 88


(A) 88 (B) 5 (C) 69
(D) 21 (E) None of these

2. 40, 28, 32, 48, 104, 288


(A) 104 (B) 48 (C) 32
(D) 28 (E) None of these

3. 9, 7, 20, 30, 53, 73


(A) 73 (B) 7 (C) 30
(D) 20 (E) None of these

4. 1706, 427, 110, 29, 10, 6


(A) 427 (B) 110 (C) 10
(D) 6 (E) None of these

5. 1, 3, 6, 30, 210, 2310


(A) 2310 (B) 210 (C) 30
(D) 3 (E) None of these

6. 25, 33, 97, 313 , 875, 1825


(A) 313 (B) 1825 (C) 875
(D) 97 (E) None of these

7. 56, 28, 28, 46, 84, 210


(A) 46 (B) 210 (C) 28
(D) 84 (E) None of these

8. 2, 6, 15, 45, 89, 210


(A) 205 (B) 99 (C) 45
(D) 6 (E) None of these

9. 30, 47, 51, 77, 85, 115


(A) 77 (B) 85 (C) 115
(D) 30 (E) None of these

10. 24, 28, 34, 46, 66, 160


(A) 66 (B) 160 (C) 46
(D) 28 (E) None of these

11. 149, 163, 135, 177, 111, 191


(A) 149 (B) 111 (C) 135

PAGE h ps://t.me/studified h ps://instagram.com/aashisharorasocial?utm_medium=copy_link


Follow
43 Aashish Arora on: h ps://youtube.com/channel/UCYa4_JrOrf8R5Kz2uOtccXQ https://www.facebook.com/aashisharorasocial/
अ ास by Aashish Arora
(SBI/IBPS/RBI/LIC/All other banking and insurance Exams)

(D) 191 (E) None of these

12. 999, 899, 755, 559, 303, -15


(A) 899 (B) 999 (C) –15
(D) 559 (E) None of these

13. 16, 15, 19, 24, 38, 61


(A) 61 (B) 38 (C) 16
(D) 19 (E) None of these

14. 0.2, 0.4, 1, 3, 12, 42


(A) 12 (B) 42 (C) 0.4
(D) 0.2 (E) None of these

15. 25, 50, 70, 105, 155, 225


(A) 70 (B) 50 (C) 155
(D) 255 (E) None of these

16. 11, 41, 101, 181, 311, 461


(A) 181 (B) 311 (C) 11
(D) 101 (E) None of these

17. 15, 35, 55, 95, 165, 275


(A) 165 (B) 15 (C) 35
(D) 95 (E) None of these

18. 8, 9, 14, 26, 57, 145


(A) 9 (B) 26 (C) 145
(D) 57 (E) None of these

19. 16, 19, 12, 24, 6, 33


(A) 24 (B) 19 (C) 33
(D) 12 (E) None of these

20. 19, 31, 67, 127, 212, 319


(A) 127 (B) 67 (C) 19
(D) 212 (E) None of these

21. 200, 190, 210, 170, 240, 90


(A) 90 (B) 240 (C) 170
(D) 210 (E) None of these

22. 67, 78, 113, 166, 243, 342


(A) 78 (B) 166 (C) 113
(D) 342 (E) None of these

23. 204, 35, 8, 3, 2, 1


(A) 3 (B) 204 (C) 35
(D) 1 (E) None of these

24. 23, 21, 18, 16, 20, 23

PAGE h ps://t.me/studified h ps://instagram.com/aashisharorasocial?utm_medium=copy_link


Follow
44 Aashish Arora on: h ps://youtube.com/channel/UCYa4_JrOrf8R5Kz2uOtccXQ https://www.facebook.com/aashisharorasocial/
अ ास by Aashish Arora
(SBI/IBPS/RBI/LIC/All other banking and insurance Exams)

(A) 20 (B) 18 (C) 21


(D) 16 (E) None of these

25. 129, 131, 154, 209, 315, 489


(A) 154 (B) 131 (C) 315
(D) 209 (E) None of these

26. 24, 124, 293, 551, 910, 1394


(A) 551 (B) 910 (C) 293
(D) 124 (E) None of these

27. 3, 2, 3, 6, 14, 38
(A) 6 (B) 14 (C) 38
(D) 2 (E) None of these

28. 36, 42, 25, 54, 12, 66


(A) 36 (B) 25 (C) 12
(D) 66 (E) None of these

29. 7, 10, 14, 25, 48, 95


(A) 48 (B) 25 (C) 14
(D) 10 (E) None of these

30. 431, 519, 615, 709, 831


(A) 431 (B) 519 (C) 831
(D) 709 (E) None of these

31. 39, 30, 50, 17, 65, 2


(A) 2 (B) 65 (C) 50
(D) 39 (E) None of these

32. 998, 999, 1008, 883, 932, 203


(A) 1008 (B) 932 (C) 998
(D) 203 (E) None of these

33. 119, 126, 138, 162, 193, 234


(A) 162 (B) 138 (C) 193
(D) 234 (E) None of these

34. 2, 8, 20, 72, 304, 1540


(A) 304 (B) 72 (C) 20
(D) 8 (E) None of these

35. 5, 10, 26, 55, 102, 170


(A) 10 (B) 5 (C) 170
(D) 102 (E) None of these

36. 5, 9, 27, 34, 59, 95


(A) 27 (B) 34 (C) 9
(D) 59 (E) 95

PAGE h ps://t.me/studified h ps://instagram.com/aashisharorasocial?utm_medium=copy_link


Follow
45 Aashish Arora on: h ps://youtube.com/channel/UCYa4_JrOrf8R5Kz2uOtccXQ https://www.facebook.com/aashisharorasocial/
अ ास by Aashish Arora
(SBI/IBPS/RBI/LIC/All other banking and insurance Exams)

37. 4, 5, 14, 48, 200, 1010


(A) 5 (B) 4 (C) 1010
(D) 200 (E) 48

38. 100, 49, 52, 105, 421, 3369


(A) 52 (B) 49 (C) 100
(D) 421 (E) 105

39. 76, 306, 1345, 9202, 64419


(A) 76 (B) 306 (C) 1345
(D) 9202 (E) 64419

40. 7, 13, 27, 51, 104


(A) 7 (B) 13 (C) 27
(D) 51 (E) 104

41. 6, 13, 28, 62, 122, 249


(A) 28 (B) 13 (C) 62
(D) 249 (E) None of these

42. 17, 37, 71, 110, 157, 212


(A) 37 (B) 157 (C) 110
(D) 17 (E) None of these

43. 5, 13, 28, 52, 87, 135


(A) 28 (B) 13 (C) 87
(D) 135 (E) None of these

44. 6, 20, 43, 72, 110, 156


(A) 72 (B) 43 (C) 110
(D) 156 (E) None of these

45. 13, 32, 70, 130, 221, 348


(A) 70 (B) 130 (C) 32
(D) 13 (E) None of these

46. 82, 89, 68, 101, 54, 117


(A) 101 (B) 82 (C) 89
(D) 68 (E) None of these

47. 19, 7, 8, 14, 46, 220


(A) 7 (B) 8 (C) 46
(D) 14 (E) None of these

48. 1, 21, 42, 62, 120, 144


(A) 42 (B) 1 (C) 120
(D) 144 (E) None of these

49. 12, 14, 19, 30, 46, 72


(A) 72 (B) 30 (C) 19
(D) 12 (E) None of these

PAGE h ps://t.me/studified h ps://instagram.com/aashisharorasocial?utm_medium=copy_link


Follow
46 Aashish Arora on: h ps://youtube.com/channel/UCYa4_JrOrf8R5Kz2uOtccXQ https://www.facebook.com/aashisharorasocial/
अ ास by Aashish Arora
(SBI/IBPS/RBI/LIC/All other banking and insurance Exams)

50. 8, 14, 26, 50, 93, 194


(A) 14 (B) 50 (C) 93
(D) 160 (E) None of these

51. 8, 332, 440, 475, 488, 492


(A) 332 (B) 492 (C) 488
(D) 475 (E) None of these

52. 66, 71, 81, 96, 118, 141


(A) 96 (B) 17 (C) 118
(D) 141 (E) None of these

53. 15, 18, 23, 32, 49, 70


(A) 15 (B) 49 (C) 70
(D) 23 (E) None of these

54. 21, 25, 34, 59, 108, 232


(A) 232 (B) 108 (C) 225
(D) 134 (E) None of these

55. 14, 70, 35, 179, 87.5


(A) 170 (B) 141 (C) 135
(D) 179 (E) None of these

56. 75, 80, 95, 105, 125, 150


(A) 105 (B) 80 (C) 150
(D) 95 (E) 125

57. 8, 12, 23, 37, 62, 98


(A) 23 (B) 37 (C) 12
(D) 98 (E) 8

58. 1234, 3567, 617, 1851, 308.5, 925.5


(A) 617 (B) 3567 (C) 925.5
(D) 1234 (E) 1851

59. 30, 273, 364, 381, 390, 393


(A) 364 (B) 393 (C) 273
(D) 381 (E) 30

60. 6654, 6072, 6690, 6723, 6762, 6813


(A) 6690 (B) 6813 (C) 6072
(D) 6762 (E) 6654

61. 15, 19, 46, 148, 616, 3100


(A) 148 (B) 616 (C) 3100
(D) 46 (E) None of these

62. 193, 174, 157, 144, 132, 126


(A) 144 (B) 132 (C) 126
(D) 157 (E) None of these

PAGE h ps://t.me/studified h ps://instagram.com/aashisharorasocial?utm_medium=copy_link


Follow
47 Aashish Arora on: h ps://youtube.com/channel/UCYa4_JrOrf8R5Kz2uOtccXQ https://www.facebook.com/aashisharorasocial/
अ ास by Aashish Arora
(SBI/IBPS/RBI/LIC/All other banking and insurance Exams)

63. 47, 65, 41, 71, 35, 78


(A) 41 (B) 35 (C) 71
(D) 78 (E) None of these

64. 231, 250, 233, 252, 234, 254


(A) 233 (B) 252 (C) 234
(D) 254 (E) None of these

65. 12, 16, 24, 41, 66, 102


(A) 24 (B) 41 (C) 66
(D) 102 (E) None of these

66. 24, 12, 18, 44, 157.5


(A) 24 (B) 12 (C) 18
(D) 44 (E) None of these

67. 7, 8, 17, 34, 209, 1046


(A) 7 (B) 34 (C) 17
(D) 1046 (E) None of these

68. 12, 24, 48, 85, 132, 192


(A) 85 (B) 48 (C) 24
(D) 12 (E) None of these

69. 14, 14, 23, 48, 114, 238


(A) 14 (B) 23 (C) 113
(D) 48 (E) None of these

70. 108, 115, 129, 159, 213, 325


(A) 115 (B) 108 (C) 159
(D) 213 (E) None of these

Join
Joinme
meonon

Join me on
Join me on

Join me on
Join me on
Join me on
BY: AASHISH ARORA

PAGE h ps://t.me/studified h ps://instagram.com/aashisharorasocial?utm_medium=copy_link


Follow
48 Aashish Arora on: h ps://youtube.com/channel/UCYa4_JrOrf8R5Kz2uOtccXQ https://www.facebook.com/aashisharorasocial/
अ ास by Aashish Arora
(SBI/IBPS/RBI/LIC/All other banking and insurance Exams)

SOLUTION

1. Ans. (A) 42 × 2 = 84
37 + 16 × 1 = 53 84 × 2.5 = 210
53 - 16 × 2 = 21
21 + 16 × 3 = 69 8. Ans. (C)
2
69 - 16 × 4 = 5 2+2 =6
2
5 + 16 × 5 = 85 6 + 3 = 15
2
15 + 5 = 40
2
2. Ans. (A) 40 + 7 = 89
40 × 1 - 12 × 20 = 28 2
89 + 11 = 210
28 × 2 - 12 × 21 = 32
32 × 3 - 12 × 22 = 48 9. Ans. (D)
48 × 4 - 12 × 23 = 96 5 × 6 - 5 = 25
4
96 × 5 - 12 × 2 = 288 6 × 7 + 5 = 47
7 × 8 - 5 = 51
3. Ans. © 8 × 9 + 5 = 77
9 + 11 = 20 9 × 10 - 5 = 85
7 + 22 = 29 10 × 11 + 5 = 115
20 + 33 = 53
29 + 44 = 73 10. Ans. (A)
24 + 30 + 3 = 28
4. Ans. (B) 28 + 31 + 3 = 34
(1706 + 2) ÷ 4 = 427 34 + 32 + 3 = 46
(427 + 5) ÷ 4 = 108 46 + 33 + 3 = 76
4
(108 + 8) ÷ 4 = 29 76 + 3 + 3 = 160
(29 + 11) ÷ 4 = 10
(10 + 14) ÷ 4 = 6 11. Ans. (B)
149 + 14 × 1 = 163
5. Ans. (D) 163 – 14 × 2 = 135
1×2=2 135 + 14 × 3 = 177
2×3=6 177 – 14 × 4 = 121
6 × 5 = 30 121 + 14 × 5 = 191
30 × 7 = 210
210 × 11 = 2310 12. Ans. (C)
2
999 – 10 = 899
2
6. Ans. (C) 899 – 12 = 755
3 2
25 + 2 + 33 755 – 14 = 559
3 2
33 + 4 = 97 559 – 16 = 303
3 2
97 + 6 = 313 303 – 18 = - 21
313 + 83 = 825
825 + 103 = 1825 13. Ans. (D)
16 + (12 – 2) = 15
7. Ans. (A) 15 + (22 – 2) = 17
56 × 0.5 = 28 17 + (32 – 2) = 24
2
28 × 1 = 28 24 + (4 – 2) = 38
2
28 × 1.5 = 42 38 + (5 – 2) = 61

PAGE h ps://t.me/studified h ps://instagram.com/aashisharorasocial?utm_medium=copy_link


Follow
49 Aashish Arora on: h ps://youtube.com/channel/UCYa4_JrOrf8R5Kz2uOtccXQ https://www.facebook.com/aashisharorasocial/
अ ास by Aashish Arora
(SBI/IBPS/RBI/LIC/All other banking and insurance Exams)

2
14. Ans. (A) 210 – 10 × 2 = 170
3
0.2 × 2 = 0.4 170 + 10 × 2 = 250
4
0.4 × 2.5 = 1 250 – 10 × 2 = 90
1×3=3
3 × 3.5 = 10.5 22. Ans. (C)
10.5 × 4 = 42 67 + 11 × 1 = 78
78 + 11 × 3 = 111
15. Ans. (B) 111 + 11 × 5 = 166
25 + 5 × 4 = 45 166 + 11 × 7 = 243
45 + 5 × 5 = 70 243 + 11 × 9 = 342
70 + 5 × 7 = 105
105 + 5 × 10 = 155 23. Ans. (D)
155 + 5 × 14 = 225 204 ÷ 6 + 1 = 35
35 ÷ 5 + 1 = 8
16. Ans. (A) 8÷4+1=3
11 + 30 × 1 = 41 3÷3+1=2
41 + 30 × 2 = 101 2÷2+1=2
101 + 30 × 3 = 191
191 + 30 × 4 = 311 24. Ans. (A)
2
311 + 30 × 5 = 461 23 + 2 – 6 × 1 = 21
2
21 + 3 – 6 × 2 = 18
2
17. Ans. (B) 18 + 4 – 6 × 3 = 16
2
35 – 25 = 10 16 + 5 – 6 × 4 = 17
2
55 – 35 = 20 (20 – 10 = 10) 17 + 6 – 6 × 5 = 23
95 – 55 = 40 (40 – 10 = 20)
165 – 95 = 70 (70 – 10 = 30) 25. Ans. (B)
275 – 165 = 110 (110 – 70 = 40) 133 – 129 = 4 (21 – 4 = 17 × 1)
154 – 133 = 21 (55 – 21 = 17 × 2)
18. Ans. (C) 209 – 154 = 55 (106 – 55 = 17 × 3)
8 × 0.5 + 5 = 9 315 – 209 = 106 (174 – 106 = 17 × 4)
9 × 1 + 5 = 14 489 – 315 = 174
14 × 1.5 + 5 = 26
26 × 2 + 5 = 57 26. Ans. (A)
57 × 2.5 + 5 = 147.5 24 + 102 = 124
124 + 132 = 293
19. Ans. (D) 293 + 162 = 549
16 + 12 + 2 = 19 549 + 193 = 910
2
19 – 2 – 2 = 13 910 + 224 = 1394
2
13 + 3 + 2 = 24
2
24 – 4 – 2 = 6 27. Ans. (C)
5
6 + 5 + 2 = 33 3 × 0.5 + 0.5 = 2
2×1+1=3
20. Ans. (D) 3 × 1.5 + 1.5 = 6
19 + 12 × 1 = 31 6 × 2 + 2 = 14
31 + 12 × 3 = 67 14 × 2.5 + 2.5 = 37.5
67 + 12 × 5 = 127
127 + 12 × 7 = 211 28. Ans. (B)
211 + 12 × 9 = 319 36 + 6 × 1 = 42
42 – 6 × 3 = 24
21. Ans. (B) 24 + 6 × 5 = 54
0
200 – 10 × 2 = 190 54 – 6 × 7 = 12
1
190 + 10 × 2 = 210 12 + 6 × 9 = 66

PAGE h ps://t.me/studified h ps://instagram.com/aashisharorasocial?utm_medium=copy_link


Follow
50 Aashish Arora on: h ps://youtube.com/channel/UCYa4_JrOrf8R5Kz2uOtccXQ https://www.facebook.com/aashisharorasocial/
अ ास by Aashish Arora
(SBI/IBPS/RBI/LIC/All other banking and insurance Exams)

2
29. Ans. (D) 9 + 3 = 18
2
7×2–5=9 18 + 4 = 34
2
9 × 2 – 4 = 14 34 + 5 = 59
2
14 × 2 – 3 = 25 59 + 6 = 95
25 × 2 – 2 = 48
48 × 2 - 1 = 95 37. Ans. (B)
3×1+2=5
30. Ans. (D) 5 × 2 + 4 = 14
212 – 10 = 431 14 × 3 + 6 = 48
232 – 10 = 519 48 × 4 + 8 = 200
252 – 10 = 615 200 × 5 + 10 = 1010
272 – 10 = 719
2
29 – 10 = 831 38. Ans. (B)
100 × 0.5 + 1 = 51
31. Ans. (A) 51 × 1 + 1 = 52
39 – 9 × 1 = 30 52 × 2 + 1 = 105
30 + 10 × 2 = 50 105 × 4 + 1 = 421
50 – 11 × 3 = 17 421 × 8 + 1 = 3369
17 + 12 × 4 = 65
65 – 13 × 5 = 0 39. Ans. (C)
76 × 4 + 2 = 306
32. Ans. (C) 306 × 5 + 3 = 1533
3
1000 – 1 = 999 1533 × 6 + 4 = 9202
2
999 + 3 = 1008 9202 × 7 + 5 = 64419
3
1008 – 5 = 883
2
883 + 7 = 932 40. Ans. (E)
3
932 – 9 = 203 7 × 2 – 1 = 13
13 × 2 + 1 = 27
33. Ans. (B) 27 × 2 – 3 = 51
119 + 7 = 126 51 × 2 + 3 = 105
126 + 14 = 140
140 + 22 = 162 41. Ans. (C)
162 + 31 = 193 6 × 2 + 1 = 13
193 + 41 = 234 13 × 2 + 2 = 28
28 × 2 + 3 = 59
34. Ans. (D) 59 × 2 + 4 = 122
2×1+4×1=6 122 × 2 + 5 = 249
6 × 2 + 4 × 2 = 20
20 × 3 + 4 × 3 = 72 42. Ans. (A)
72 × 4 + 4 × 4 = 304 17 + 23 = 40
304 × 5 + 4 × 5 = 1540 40 + 31 = 71
71 + 39 = 110
35. Ans. (D) 110 + 47 = 157
10 – 5 = 5 157 + 55 = 212
26 – 10 = 16 (16 – 5 = 11)
55 – 26 = 29 (29 – 16 = 13) 43. Ans. (E)
101 – 55 = 46 (46 – 29 = 17) 5 + (32 – 1) = 13
170 – 101 = 69 (69 – 46 = 23) 13 + (42 – 1) = 28
2
28 + (5 – 1) = 52
2
36. Ans. (A) 52 + (6 – 1) = 87
2
5 + 22 = 9 87 + (7 – 1) = 135

PAGE h ps://t.me/studified h ps://instagram.com/aashisharorasocial?utm_medium=copy_link


Follow
51 Aashish Arora on: h ps://youtube.com/channel/UCYa4_JrOrf8R5Kz2uOtccXQ https://www.facebook.com/aashisharorasocial/
अ ास by Aashish Arora
(SBI/IBPS/RBI/LIC/All other banking and insurance Exams)

4
44. Ans. (B) 8 + 4 × 3 = 332
3
3×2=6 332 + 4 × 3 = 440
2
5 × 4 = 20 440 + 4 × 3 = 476
1
7 × 6 = 42 476 + 4 × 3 = 488
0
9 × 8 = 72 488 + 4 × 3 = 492
11 × 10 = 110
13 × 12 = 156 52. Ans. (C)
66 + 5 × 1 = 71
45. Ans. (A) 71 + 5 × 2 = 81
23 + 5 = 13 81 + 5 × 3 = 96
33 + 5 = 32 96 + 5 × 4 = 116
43 + 5 = 69 116 + 5 × 5 = 141
53 + 5 = 130
3
6 + 5 = 221 53. Ans. (B)
3
7 + 5 = 348 18 – 15 = 3
23 – 18 = 5 (5 – 3 = 2 × 1)
46. Ans. (A) 32 – 23 = 9 (9 – 5 = 2 × 2)
82 + 7 × 1 = 89 47 – 32 = 15 (15 – 9 = 2 × 3)
89 – 7 × 3 = 68 70 – 47 = 23 (23 – 15 = 2 × 4)
68 + 7 × 5 = 103
103 – 7 × 7 = 54 54. Ans. (A)
2
54 + 7 × 9 = 117 21 + 2 = 25
2
25 + 3 = 34
2
47. Ans. (A) 34 + 5 = 59
19 × 1 – 10 = 9 59 + 72 = 108
9 × 2 – 10 = 8 108 + 112 = 229
8 × 3 – 10 = 14
14 × 4 – 10 = 46 55. Ans. (D)
46 × 5 – 10 = 220 14 × 5 = 70
70 ÷ 2 = 35
48. Ans. (C) 35 × 5 = 175
1 + 20 = 21 175 ÷ 2 = 87.5
21 × 2 = 42
42 + 20 = 62 56. Ans. (D)
62 × 2 = 124 75 + 05 = 80
124 + 20 = 144 80 + 10 = 90
90 + 15 = 105
49. Ans. (B) 105 + 20 = 125
12 + (12 + 1) = 14 125 + 25 = 150
14 + (22 + 1) = 19
19 + (32 + 1) = 29 57. Ans. (A)
29 + (42 + 1) = 46 2
08 + 2 = 12
46 + (52 + 1) = 72 2
12 + 3 = 21
2
21 + 4 = 37
50. Ans. (C) 37 + 5 2 = 62
8 + 6 = 14 62 + 6 2 = 98
14 + 12 = 26
26 + 24 = 50 58. Ans. (B)
50 + 48 = 98 1234 × 3 = 3702
98 + 96 = 194 3702 ÷ 6 = 617
617 × 3 = 1851
51. Ans. (D) 1851 ÷ 6 = 308.5

PAGE h ps://t.me/studified h ps://instagram.com/aashisharorasocial?utm_medium=copy_link


Follow
52 Aashish Arora on: h ps://youtube.com/channel/UCYa4_JrOrf8R5Kz2uOtccXQ https://www.facebook.com/aashisharorasocial/
अ ास by Aashish Arora
(SBI/IBPS/RBI/LIC/All other banking and insurance Exams)

308.5 × 3 = 925.5 66. Ans. (D)


24 × 0.5 = 12
59. Ans. (A) 12 × 1.5 = 18
5
30 + 3 = 273 18 × 2.5 = 45
4
273 + 3 = 354 45 × 3.5 = 157.5
3
354 + 3 = 381
2
381 + 3 = 390 67. Ans. (B)
390 + 3 1 = 393 7×1+1=8
8 × 2 + 1 = 17
60. Ans. © 17 × 3 + 1 = 52
6654 + 5 × 3 = 6669 52 × 4 + 1 = 209
6669 + 7 × 3 = 6690 209 × 5 + 1 = 1046
6690 + 11 × 3 = 6723
6723 + 13 × 3 = 6762 68. Ans. (A)
6762 + 17 × 3 = 6813 12 + 12 × 1 = 24
24 + 12 × 2 = 48
61. Ans. (A) 48 + 12 × 3 = 84
(15 + 4) × 1 = 19 84 + 12 × 4 = 132
(19 + 4) × 2 = 46 132 + 12 × 5 = 192
(46 + 4) × 3 = 150
(150 + 4) × 4 = 616 69. Ans. (D)
3
(616 + 4) × 5 = 3100 14 + 1 – 1 = 14
3
14 + 2 + 1 = 23
3
62. Ans. (B) 23 + 3 – 1 = 49
193 – 19 = 174 49 + 4 3 + 1 = 114
174 – 17 = 157 114 + 5 3 – 2 = 238
157 – 13 = 144
144 – 11 = 133 70. Ans. (C)
133 - 7 = 126 108 + 7 × 2 0 = 115
115 + 7 × 2 1 = 129
63. Ans. (D) 129 + 7 × 2 2 = 157
47 + 6 × 3 = 65 157 + 7 × 2 3 = 213
65 – 6 × 4 = 41 213 + 7 × 2 4 = 325
41 + 6 × 5 = 71
71 – 6 × 6 = 35
35 + 6 × 7 = 77

64. Ans. (C)


231 + 19 = 250
250 – 17 = 233
233 + 19 = 252
252 – 17 = 235
235 + 19 = 254

65. Ans. (A)


2
12 + 2 = 16
2
16 + 3 = 25
25 + 42 = 41
41 + 52 = 66
66 + 62 = 102

PAGE h ps://t.me/studified h ps://instagram.com/aashisharorasocial?utm_medium=copy_link


Follow
53 Aashish Arora on: h ps://youtube.com/channel/UCYa4_JrOrf8R5Kz2uOtccXQ https://www.facebook.com/aashisharorasocial/
अ ास by Aashish Arora
(SBI/IBPS/RBI/LIC/All other banking and insurance Exams)

5 Arithmetic Word Problem

1. A mixture contains coconut oil and coconut water in the ratio of 17:18 respectively. If 175
ml of mixture is taken out, and then 8 ml of coconut oil and 62 ml of coconut water is added
into the remaining mixture such that the ratio of coconut oil and coconut water becomes
3:4 respectively. Find the original quantity of the mixture.
एक िम ण म ना रयल तेल और ना रयल पानी मशः 17:18 के अनुपात म है । यिद 175 िमलीलीटर िम ण
िनकाला जाता है , और िफर 8 िमलीलीटर ना रयल तेल और 62 िमलीलीटर ना रयल पानी शेष िम ण म
िमलाया जाता है , तो ना रयल तेल और ना रयल पानी का अनुपात मशः 3:4 हो जाता है । िम ण की मूल मा ा
का पता लगाएं ।
A. 480 litres B. 450 litres
C. 380 litres D. 560 litres
E. None of these

2. 45 children can complete a piece of work in 64 days. 57 children started the work and
worked for 'x' days. After that, one more child joined the work and the remaining work gets
completed in 30 days. Find the value of 'x'.
45 ब े 64 िदनो ं म एक काम पूरा कर सकते ह। 57 ब ो ं ने काम शु िकया और 'x' िदनो ं के िलए काम
िकया। उसके बाद, एक और ब ा काम म शािमल हो गया और शेष काम 30 िदनो ं म पूरा हो गया। 'x' का
मान ात कर।
A. 35 days B. 20 days
C. 26 days D. 32 days
E. None Of These

3. P and Q entered into business investing Rs. 2400 and Rs. 3600 respectively. After 4
months, Q left and R entered into the business investing Rs. 3200. If at the end of the year,
profit share of P was Rs. 1782, then find the total profit earned by all of them together.
P और Q ने एक वसाय म मशः 2400 और 3600 िनवेश िकए। 4 महीने के बाद, Q ने छोड़ िदया
और R ने वसाय म 3200 का िनवेश िकया। यिद वष के अंत म, P का लाभ िह ा 1782 था, तो िफर
उन सभी ारा एक साथ अिजत कुल लाभ को खोज।
A. Rs. 4499 B. Rs. 4339
C. Rs. 4257 D. Rs. 4099
E. None Of These

4. Sarvesh spent 58.33% of his monthly income on shopping, and 56.25% of the remaining
on rent. Out of the remaining amount, 20% is spent of food. If he is left with Rs. 1260, then
find the amount spent of rent.
सवश ने अपनी मािसक आय का 58.33% खरीदारी पर खच िकया, और शेष का 56.25% िकराए पर खच
िकया। शेष रािश म से, 20% भोजन के िलए खच िकया । यिद उसके पास 1260 बचते ह, तो िफर िकराए
पर खच की गई रािश का पता लगाएं ।

PAGE h ps://t.me/studified h ps://instagram.com/aashisharorasocial?utm_medium=copy_link


Follow
54 Aashish Arora on: h ps://youtube.com/channel/UCYa4_JrOrf8R5Kz2uOtccXQ https://www.facebook.com/aashisharorasocial/
अ ास by Aashish Arora
(SBI/IBPS/RBI/LIC/All other banking and insurance Exams)

A. Rs. 3140 B. Rs. 2025


C. Rs. 1735 D. Rs. 2945
E. None Of These

5. Rajeev saves 32% of his monthly income. If monthly savings of Rajeev is increased by
37.5% while his monthly expenditure is decreased by 25%, then find the increase or
decrease in his monthly income given that his monthly savings originally was Rs. 1440.
राजीव अपनी मािसक आय का 32% बचाता है । यिद राजीव की मािसक बचत म 37.5% की वृ ई है
जबिक उसके मािसक य म 25% की कमी आई है , तो उसकी मािसक आय म वृ या कमी को ात करे
यह दे खते ए िक उसकी मािसक बचत मूल प से 1440 है ।
A. Rs. 225 B. Rs. 205
C. Rs. 255 D. Rs. 245
E. None of these

6. The length and breadth of a rectangle is (x + 17) cm and (2x – 5) cm. Find the perimeter of
the rectangle if area of rectangle is 551 cm2.
एक आयत की लंबाई और चौड़ाई (x + 17) सेमी और (2x – 5) सेमी है । यिद आयत का े फल 551 वग
सेमी है तो आयत की प रिध ात कीिजए।
A. 96 cm B. 48 cm
C. 84 cm D. 108 cm
E. None of these

7. Kevin invested 55% of his monthly income in a SIP offering 10% per annum compound
interest compounded annually and rest amount in a bank offering 22.5% per annum
simple interest. If the total interest received by him after 2 years is Rs. 3816, then find the
monthly income of Kevin.
केिवन अपनी मािसक आय का 55% SIP म 10% ितवष च वृ ाज की दर पर िनवेश करता है और
शेष रािश को बक म 22.5% वािषक साधारण ाज की दर पर िनवेश करता है । यिद 2 वष के बाद उसके
ारा ा कुल ाज 3816 है , तो िफर केिवन की मािसक आय का पता लगाएं ।
A. Rs. 10000 B. Rs. 16000
C. Rs. 12000 D. Rs. 18000
E. None Of These

8. The ratio of speed of boat in still water to speed of stream is 21 : 2 respectively. The ratio of
time taken to go 'X' km upstream and (X + 164) km downstream is 9 : 11 respectively. Find
the value of 'X' in km.
थर पानी नाव की गित और धारा की गित का अनुपात मशः 21 : 2 है । 'X' िकमी धारा के िव और (X +
164) िकमी धारा के अनुकूल जाने के िलए नाव ारा िलया जाने वाला समय का अनुपात मशः 9 : 11 है ।
िकमी म 'X' का मान ात कर।
A. 352 B. 234
C. 244 D. 342
E. None of these

9. Average marks obtained by a class of 36 students is 75. Marks of three students were
misread as 89, 87 and 77 in place of 98, 67 and 13 respectively and also the number of
students was one less. Find the actual average marks of the class.

PAGE h ps://t.me/studified h ps://instagram.com/aashisharorasocial?utm_medium=copy_link


Follow
55 Aashish Arora on: h ps://youtube.com/channel/UCYa4_JrOrf8R5Kz2uOtccXQ https://www.facebook.com/aashisharorasocial/
अ ास by Aashish Arora
(SBI/IBPS/RBI/LIC/All other banking and insurance Exams)

36 छा ो ं की एक क ा ारा ा औसत अंक 75 ह। तीन छा ो ं के अंक मशः 98, 67 और 13 के थान पर


89, 87 और 77 के प म गलत पढ़े जाते ह और छा ो ं की सं ा भी एक कम थी। क ा के वा िवक औसत
अंक ात कीिजए।
A. 76 B. 77
C. 78
D. No correction Required/ कोई सुधार आव क नही ं है
E. None Of These
10. Ratio of age of Ramesh and Suresh after 12 years from now will be 10 : 13 respectively. 8
years ago from now, the ratio of their ages was 5 : 8 respectively. Find the present average
age of Ramesh and Suresh.
अब से 12 वष बाद रमेश और सुरेश की आयु का अनुपात मशः 10 : 13 होगा। अब से 8 साल पहले, उनकी
आयु का अनुपात मशः 5 : 8 था। रमेश और सुरेश की वतमान औसत आयु ात कीिजए।
A. 36 years B. 34 years
C. 35 years D. 38 years
E. None of these

11. In a mixture of milk and water, the quantity of milk is 70 litres more than that of water. If 80
litres of milk is added to the mixture then the quantity of water becomes 50% less than the
quantity of milk. Find the initial quantity of the mixture.
दू ध और पानी के िम ण म पानी की तुलना म दू ध की मा ा 70 लीटर अिधक है । यिद िम ण म 80 लीटर दू ध
डाला जाता है तो पानी की मा ा, दू ध की मा ा से 50% कम हो जाती है । िम ण की ारं िभक मा ा का पता
लगाएं ।
A. 390 litres B. 410 litres
C. 430 litres D. 370 litres
E. None of these

12. 18 men can complete a piece of work in 19 days. 9 men started the work and worked for 'x'
days. The remaining work is completed by 15 men in 12 days. Find the value of 'x'.
18 पु ष 19 िदनो ं म एक काम पूरा कर सकते ह। 9 पु षो ं ने काम शु िकया और केवल 'x' िदनो ं के िलए
काम िकया। शेष काय 15 पु षो ं ारा 12 िदनो ं म पूरा िकया जाता है । 'x' का मान ात कर।
A. 12 B. 18
C. 16 D. 15
E. None Of These

13. From a box containing 14 red pens and 16 blue pens, find the probability of picking 2 pens
of each type at random.
14 लाल पेन और 16 नीले पेन वाले एक बॉ से, अिनयिमत प से ेक कार के 2 पेन लेने की संभावना
का पता लगाए।
A. 107/261 B. 105/261
C. 104/261 D. 108/261
E. None Of These

14. Ankita, Beniwal and Chandra invested Rs. 6600, Rs, 7700 and Rs 'x' respectively in a
business for equal time. If the profit received by Chandra is Rs. 2320 out of total profit of
Rs. 6090, then find the value of 'x'.
अंिकता, बेनीवाल और चं ा ने मशः 6600, 7700 और 'x' एक वसाय म समान समय के िलए
िनवेश िकये। यिद कुल लाभ 6090 म से चं ा ारा ा लाभ 2320 है , तब 'x' का मान ात कीिजए।

PAGE h ps://t.me/studified h ps://instagram.com/aashisharorasocial?utm_medium=copy_link


Follow
56 Aashish Arora on: h ps://youtube.com/channel/UCYa4_JrOrf8R5Kz2uOtccXQ https://www.facebook.com/aashisharorasocial/
अ ास by Aashish Arora
(SBI/IBPS/RBI/LIC/All other banking and insurance Exams)

A. Rs. 9900 B. Rs. 8800


C. Rs. 5500 D. Rs. 6600
E. None Of These

15. Rahul and Mamta started moving towards each other at same time with a speed of 30
km/hr and 24 km/hr respectively. If the distance covered by Rahul is 108 km more than
that by Mamta when they meet then find the initial distance between them.
रा ल और ममता ने एक ही समय म मशः 30 िकमी/घंटा और 24 िकमी/घंटा की गित से एक-दू सरे की ओर
बढ़ना शु कर िदया। यिद रा ल ारा तय की गई दू री, ममता ारा तय की गई दू री से 108 िकमी अिधक है ,
जब वे िमलते ह, तो उनके बीच की ारं िभक दू री का पता लगाएं ।
A. 972 km B. 918 km
C. 864 km D. 1026 km
E. None of these

16. The speed of current is 15 km/hr. If the speed of boat in still water had been 12 km/hr more
than the original speed then it would take 6.8 hours to cover 374 km in downstream. Find
the original speed of boat in still water (in km/hr).
धारा की गित 15 िकमी/घंटा है । यिद थर पानी म नाव की गित, थर पानी म नाव की मूल गित से 12 िकमी /
घंटा अिधक होगी, तो धारा के अनुकूल 374 िकमी दू री तय करने म 6.8 घंटे का समय लगेगा। थर पानी म
(िकमी/घंटा म) नाव की मूल गित का पता लगाएं ।
A. 28 km/hr B. 30 km/hr
C. 32 km/hr D. 36 km/hr
E. None of these

17. The cost of painting a squared frame at the rate of Rs. 4.5/cm2 is Rs. 8712. Find the
perimeter of the frame.
4.5 ित वग सेमी की दर से एक चौकोर े म को पट करने की लागत 8712 है । े म की प रिध का पता
लगाएं ।
A. 172 cm B. 180 cm
C. 176 cm D. 188 cm
E. None Of These

18. When Rs. (Y + 800) is invested at 38% per annum simple interest for 3 years it gives an
interest of Rs. (2Y – 550). Find the value of 'Y'.
जब (Y + 800) को 3 साल के िलए 38% ित वष साधारण ाज की दर पर िनवेश िकया जाता है , तो (2Y
– 550) ाज िमलता है । 'Y' का मान ात कर।
A. 2500 B. 2000
C. 1800 D. 1700
E. None of these

19. Sapna sold an article to Satya at 5.55% profit who sold the article to Hatim at 33.33% loss.
If Hatim purchased the article from Satya at Rs. 646, then find the marked price of the
article if Sapna purchased it at Rs. 182 discount.
सपना ने स ा को 5.55% लाभ पर एक व ु बेची, स ा ने 33.33 % हािन पर हाितम को वही व ु बेच दी।
यिद हाितम ने स ा से व ु को 646 म खरीदा है , तो व ु का िचि त मू ात कीिजए यिद सपना ने इसे
182 की छूट पर खरीदा है ।
A. 1400 B. 1200

PAGE h ps://t.me/studified h ps://instagram.com/aashisharorasocial?utm_medium=copy_link


Follow
57 Aashish Arora on: h ps://youtube.com/channel/UCYa4_JrOrf8R5Kz2uOtccXQ https://www.facebook.com/aashisharorasocial/
अ ास by Aashish Arora
(SBI/IBPS/RBI/LIC/All other banking and insurance Exams)

C. 1250 D. 1100
E. None Of These

20. The ratio of present age of Deep and Darshan is 5:8 respectively. The present age of
Darshan is 8 years less than the age of Aditi 7 years ago from now. If the present age of
Aditi is 47 years, then find the sum of present ages of Deep and Darshan.
दीप और दशन की वतमान आयु का अनुपात मशः 5:8 है । दशन की वतमान आयु, अब से 7 वष पहले
अिदित की आयु से 8 वष कम है । यिद अिदित की वतमान आयु 47 वष है , तो दीप और दशन की वतमान
आयु का योग ात कीिजए।
A. 56 years B. 52 years
C. 65 years D. 58 years
E. None of these

21. Average of 30 number is 54. if average of first 15 numbers is 40 while average of last 12
numbers is 66 and 16th, 17th and 18th numbers are (2x – 42), (3x + 28) and 62
respectively. Find the 16th number.
30 सं ाओं का औसत 54 है । यिद पहली 15 सं ाओं का औसत 40 है , जबिक अंितम 12 सं ाओं का
औसत 66 और 16 व, 17 व और 18 व सं ाएँ मशः (2x – 42), (3x + 28) और 62 है । 16 व नंबर का
पता लगाएं ।
A. 30 B. 32
C. 40 D. 45
E. None of these

22. The ratio of time taken by a boat to go certain distance in downstream and return in
upstream one third of the same distance is 17 : 11 respectively. If speed of stream is 16
km/hr, then find the total time taken by boat to cover a distance of 280.5 km upstream and
544.5 km downstream.
एक नाव ारा धारा के अनुकूल िनि त दू री तय करने म और धारा के िव उसी दू री का एक ितहाई लौटने म
लगने वाले समय का अनुपात मशः 17 : 11 है । यिद धारा की गित 16 िकमी / घंटा है , तो 280.5 िकमी धारा
के िव और 544.5 िकमी धारा के अनुकूल दू री तय करने के िलए नाव ारा िलया गया कुल समय ात
कर।
A. 15 hours B. 16.5 hours
C. 16 hours D. 15.5 hours
E. None Of These

23. Dhruv invested Rs. (R + 760) in a bank offering a simple interest of 15% per annum and
Rs. (3R – 2500) in another bank offering a simple interest of 12.5% per annum. If total
interest earned after 4 years from both the banks is Rs. 2944. Find the interest earned at
the end of 2 years if (R + 220) is invested at a rate of 15% per annum compound interest
compounded annually.
ुव ने (R + 760) एक बक म 15% ित वष साधारण ाज की दर से और दू सरे बक म (3R – 2500)
12.5% ित वष साधारण ाज की दर से िनवेश िकए। यिद दोनो ं बको ं से 4 वष के बाद अिजत कुल ाज
2944 है । 2 वष के अंत म अिजत ाज ात कर यिद (R + 220), 15% ितवष च वृ ाज की दर से
िनवेश िकया जाता है ।
A. Rs. 605 B. Rs. 640

PAGE h ps://t.me/studified h ps://instagram.com/aashisharorasocial?utm_medium=copy_link


Follow
58 Aashish Arora on: h ps://youtube.com/channel/UCYa4_JrOrf8R5Kz2uOtccXQ https://www.facebook.com/aashisharorasocial/
अ ास by Aashish Arora
(SBI/IBPS/RBI/LIC/All other banking and insurance Exams)

C. Rs. 645 D. Rs. 635


E. None Of These

24. A rectangular field has dimensions of 56 metres and 36 metres. A walkway 16 metres wide
and parallel to the breadth of the field and another walkway 10 metres wide and parallel to
length of the field intersect each other at the centre of the field is made. Find the area of the
remaining field excluding the walkways.
एक आयताकार खेत का आयाम 56 मीटर और 36 मीटर ह। एक रा ा 16 मीटर चौड़ा और खेत की चौड़ाई
के समानां तर और दू सरा रा ा 10 मीटर चौड़ा और खेत की लंबाई के समानां तर एक दू सरे को खेत के क म
काटते ए बनाया जाता है । रा ा छोड़कर शेष खेत का े फल ात कर।
2 2
A. 1200 m B. 1040 m
2 2
C. 1100 m D. 1250 m
E. None Of These

25. 460 ml of mixture P containing spirit and soda in the ratio of 12:11 respectively is mixed
with 164 ml of mixture Q containing spirit and soda. If the ratio of spirit and soda in the
resultant mixture is 15:11 respectively, then find the ratio of spirit and soda in mixture Q.
460 िमलीलीटर िम ण P िजसमे ट और सोडा का अनुपात मशः 12:11 है , को 164 िमलीलीटर िम ण
Q( ट और सोडा) के साथ िमलाया जाता है । यिद प रणामी िम ण म ट और सोडा का अनुपात मशः
15:11 है , तो िम ण Q म ट और सोडा का अनुपात ात करे ।
A. 30 : 11 B. 29 : 12
C. 28 : 13 D. 27 : 13
E. None of these

26. Neha with the help of Rohan can complete a piece of work in 24 days while with the help of
Preet can complete the same work in 30 days. If all of them together can complete the
whole work in 15 days, then find the time taken by Neha to complete the whole work alone.
रोहन की मदद से नेहा 24 िदनो ं म एक काम पूरा कर सकती है जबिक ीत की मदद से 30 िदनो ं म वही काम
पूरा हो सकता है । अगर ये सभी िमलकर 15 िदनो ं म पूरा काम पूरा कर सकते ह, तो नेहा ारा पूरा काम
अकेले पूरा करने के िलए िलया जाने वाला समय िनकाल।
A. 120 days B. 125 days
C. 130 days D. 135 days
E. None of these

27. In how many ways, can the letters of the word MALEVOLENCEE be arranged such that
the consonants always occupy the even places ?
िकतने तरीको ं से, MALEVOLENCEE श के अ रो ं को ऐसे व थत िकया जा सकता है िक ंजन
हमेशा सम थान पर रहते ह?
A. 12,800 B. 12,400
C. 10,800 D. 10,400
E. None Of These

28. Kaka bought a shirt and a jeans for Rs. 5090. He sold the shirt at a profit of 43.75% and
jeans at a loss of 44%. If total selling price of a shirt and a jeans is Rs. 4465, then find the
difference between cost price of shirt and jeans.
काका ने एक शट और एक जीस ं 5090 म खरीदी। उसने शट को 43.75% और जीस ं को 44% की हािन

PAGE h ps://t.me/studified h ps://instagram.com/aashisharorasocial?utm_medium=copy_link


Follow
59 Aashish Arora on: h ps://youtube.com/channel/UCYa4_JrOrf8R5Kz2uOtccXQ https://www.facebook.com/aashisharorasocial/
अ ास by Aashish Arora
(SBI/IBPS/RBI/LIC/All other banking and insurance Exams)

पर बेचा। अगर एक शट और एक जीस


ं की कुल िब ी मू 4465 है , तो िफर शट और जीस
ं की लागत
मू के बीच का अंतर ात कर।
A. Rs. 1350 B. Rs. 1220
C. Rs. 1150 D. Rs. 1410
E. None of these

29. A train can cross 362 metres long bridge in 34 seconds while a 236 metres long tunnel in
27 seconds. Find the time taken by train to cross a man who is coming towards it at a rate
of 7.2 km/hr.
एक टे न 34 सेकंड म 362 मीटर लंबे पुल को पार कर सकती है , जबिक 236 मीटर लंबी सुरंग को 27 सेकंड
म पार कर सकती है । टे न ारा उस को पार करने म लगने वाले समय का पता लगाएं , जो 7.2
िकमी/घंटा की दर से टे न की ओर आ रहा है ।
A. 13.5 seconds B. 14.5 seconds
C. 15.5 seconds D. 12.5 seconds
E. None Of These

30. Ratio of ages of P and Q after 12 years from now will be 14:17 respectively. Ratio of
present ages of P to R is 6:7 respectively. Average ages of Q and R after 3 years will be 40.
Find the present age of S, if present average age of P, Q, R and S is 32 years.
अब से 12 वष बाद P और Q की आयु का अनुपात मशः 14:17 होगा। P और R की वतमान आयु का
अनुपात मशः 6:7 है । 3 वष के बाद Q और R की औसत आयु 40 वष होगी। यिद वतमान म P, Q, R और
S की औसत आयु 32 वष है , तो S की वतमान आयु ात कीिजए।
A. 26 years B. 24 years
C. 25 years D. 28 years
E. None of these

31. Mixture P containing oil and water is the ratio of 8:7 respectively is mixed with another
mixture Q containing oil and water in the ratio of 11:12 respectively. If quantity of oil in
mixture P is 9 litres more than that in mixture Q and total quantity of resultant mixture is 235
litres, then find the total quantity of mixture P.
िम ण P म तेल और पानी मशः 8:7 के अनुपात म है और इस िम ण को एक अ िम ण Q के साथ
िमलाया जाता है िजसम तेल और पानी मशः 11:12 के अनुपात म है । यिद िम ण P म तेल की मा ा िम ण
Q म तेल की मा ा से 9 लीटर अिधक है और प रणामी िम ण की कुल मा ा 235 लीटर है , तो िम ण P की
कुल मा ा ात कीिजए।
A. 115 litres B. 110 litres
C. 125 litres D. 120 litres
E. None of these

32. Kusum is 66.66% more efficient than Sanjana who is 20% less efficient than Priyansh and
all of them together can complete the whole work in 5 days. If Sanjana and Priyansh
together work on it for 5 days and left, then find the time taken by Kusum to complete rest
of the work alone.
कुसुम, संजना की तुलना म 66.66% अिधक कुशल है और संजना, ि यां श की तुलना म 20% कम कुशल है
और सभी एक साथ 5 िदनो ं म पूरा काम पूरा कर सकते ह। अगर संजना और ि यां श एक साथ 5 िदनो ं के िलए
इस पर काम करते ह और छोड़ दे ते है , तो कुसुम ारा अकेले शेष काम पूरा करने के िलए िलया गया समय
िनकाल।

PAGE h ps://t.me/studified h ps://instagram.com/aashisharorasocial?utm_medium=copy_link


Follow
60 Aashish Arora on: h ps://youtube.com/channel/UCYa4_JrOrf8R5Kz2uOtccXQ https://www.facebook.com/aashisharorasocial/
अ ास by Aashish Arora
(SBI/IBPS/RBI/LIC/All other banking and insurance Exams)

A. 8 days B. 5 days
C. 3 days D. 6 days
E. None Of These

33. A briefcase lock is a combination of 4 digits where each digit vary from 0 to 9. If a person
tries any combination randomly then what is the probability that he puts at least 2 digits
right but he is unable to unlock the briefcase.
एक ीफकेस लॉक 4 अंको ं का एक संयोजन है , जहां ेक अंक 0 से 9 तक िभ हो सकता है । यिद कोई
िकसी भी संयोजन को या क प से आज़माता है , तो ा संभावना है िक वह कम से कम 2 अंक
सही रखता है लेिकन वह अटै ची को खोलने म असमथ हो।
A. 0.0544 B. 0.0562
C. 0.0522 D. 0.0564
E. None Of These

34. Sagar invested Rs. (A + 1500) in fund A offering a simple interest of 18 % per annum for 4
years and Rs. (A + 600) in fund B offering compound interest of 7.5% per annum
compound interest compounded annually for 2 years. If the total amount received by him
from both the funds is Rs. 6149. Find the value of 'A'.
सागर ने (A + 1500) का िनवेश फंड A म 18% ित वष साधारण ाज की दर से 4 साल के िलए िकया
और (A + 600) का िनवेश फंड B म 7.5% ित वष सालाना च वृ ाज की दर से 2 साल के िलए
िकया। यिद दोनो ं िनवेशो ं से उसके ारा ा कुल रािश 6149 है , तो िफर 'A' का मान ात कीिजए।
A. 1200 B. 1000
C. 1100 D. 1250
E. None Of These

35. Shubham and Lamha entered into a business with an initial investments of Rs. 1054 and
Rs. 1178 respectively. After 8 months, Shubham and Lamha made additional investments
of in the ratio of 1:2 respectively. If after 15 months, profit share of Shubham out of total
profit of Rs. 12000 is Rs. 5550, then find the amount of additional investment made by
Shubham.
शुभम और लमहा ने मशः 1054 और 1178 के शु आती िनवेश के साथ एक वसाय म वेश िकया।
8 महीनो ं के बाद, शुभम और लमहा ने मशः 1:2 के अनुपात म अित र िनवेश िकया। यिद 15 महीने के
बाद, शुभम का लाभ िह ा कुल लाभ 12000 म से 5550 है , तो िफर शुभम ारा िकए गए अित र
िनवेश की रािश का पता लगाएं ।
A. Rs. 120 B. Rs. 130
C. Rs. 140 D. Rs. 150
E. None of these

36. Length and Breadth of a rectangular plot is (x + 4) metres and (x – 5) metres respectively. If
cost of purchasing the plot is Rs. 3,80,000 at the rate of Rs. 2,000 per metre square, then
find the cost of fencing the plot at the rate of Rs. 200 per metre.
एक आयताकार ॉट की लंबाई और चौड़ाई मशः (x + 4) मीटर और (x – 5) मीटर है । यिद ॉट खरीदने
की लागत 2,000 ित वग मीटर की दर से 3,80,000 है , तो िफर 200 ित मीटर की दर से भूखंड की
बाड़ लगाने की लागत का पता लगाएं ।
A. Rs. 11,600 B. Rs. 12,600
C. Rs. 13,500 D. Rs. 12,400
E. None of these

PAGE h ps://t.me/studified h ps://instagram.com/aashisharorasocial?utm_medium=copy_link


Follow
61 Aashish Arora on: h ps://youtube.com/channel/UCYa4_JrOrf8R5Kz2uOtccXQ https://www.facebook.com/aashisharorasocial/
अ ास by Aashish Arora
(SBI/IBPS/RBI/LIC/All other banking and insurance Exams)

37. Srishti and Rocky are 373 km apart and they started running towards each other with initial
speed of 32 km/hr and 42 km/hr respectively at the same time. If Srishti increased her
speed by 15 km/hr after every hour and Rocky decreased his speed by 10 km/hr after
every hour, then find the time after which they will meet.
सृि और रॉकी 373 िकमी दू र ह और उ ोन ं े एक ही समय म मशः 32 िकमी / घंटा और 42 िकमी / घंटा की
ारं िभक गित के साथ एक दू सरे की ओर दौड़ना शु कर िदया। अगर सृि ने हर घंटे के बाद 15 िकमी / घंटा
से अपनी गित बढ़ाई और रॉकी ने हर घंटे के बाद अपनी गित 10 िकमी / घंटा से कम कर दी, तो उस समय का
पता लगाएं िजसके बाद वे िमलगे।
A. 5.5 hours B. 5 hours
C. 4.5 hours D. 6 hours
E. None Of These

38. There are total 400 students in three sections A, B and C of a class. The average marks of
all three sections were 85. The average marks of students of section A and C was 88 and
the average marks of students of section B was 80. Ratio of number of students in section
A and C is 12:13 respectively. Find the number of students in section A.
एक क ा के तीन वग A, B और C म कुल 400 छा ह। तीनो ं वग के औसत अंक 85 थे। वग A और C के
छा ो ं के औसत अंक 88 थे और वग B के छा ो ं के औसत अंक 80 थे। वग A और C म छा ो ं की सं ा का
अनुपात मशः 12:13 है । वग A म छा ो ं की सं ा ात कीिजए।
A. 150 B. 140
C. 130 D. 120
E. None of these

39. Two students appeared in an exam and one secured 30 marks more than the other. If
marks of students who got more marks is 55% of the sum of the marks of both of them,
then find the marks obtained by them.
एक परी ा म दो छा उप थत ए और एक ने 30 अंक अिधक ा िकए। यिद अिधक अंक पाने वाले छा ो ं
के अंक उन दोनो ं के अंको ं के योग का 55% है , तो उनके ारा ा िकए गए अंको ं का पता लगाएं ।
A. 130 and 170 B. 152 and 182
C. 150 and 180 D. 135 and 165
E. None Of These

40. Present age of Pariniti is 30% more than the present age of Shalini. Pariniti is 11 years
elder than Bipasha. After 37 years, Vasundhra will be 23 years elder than Bipasha. If the
present average age of all four together is 37 years, then find the present age of
Vasundhra.
प रनीित की वतमान आयु शािलनी की वतमान आयु से 30% अिधक है । प रनीित, िबपाशा से 11 साल बड़ी
ह। 37 साल बाद, वसुंधरा िबपाशा से 23 साल बड़ी होग
ं ी। यिद एक साथ चारो ं की वतमान औसत आयु 37 वष
है , तो वसुंधरा की वतमान आयु ात कर।
A. 46 years B. 51 years
C. 55 years D. 48 years
E. None of these

41. A jar contains solution of sugar and water in the ratio of 9:11 respectively. If 1150 ml of
water is added to the solution, then the ratio of sugar and water becomes 2:5. What is the
quantity of water in the original solution?

PAGE h ps://t.me/studified h ps://instagram.com/aashisharorasocial?utm_medium=copy_link


Follow
62 Aashish Arora on: h ps://youtube.com/channel/UCYa4_JrOrf8R5Kz2uOtccXQ https://www.facebook.com/aashisharorasocial/
अ ास by Aashish Arora
(SBI/IBPS/RBI/LIC/All other banking and insurance Exams)

एक जार म मशः 9:11 के अनुपात म चीनी और पानी का घोल होता है । यिद घोल म 1150 िमलीलीटर पानी
डाला जाता है , तो चीनी और पानी का अनुपात 2 :5 हो जाता है । मूल घोल म पानी की मा ा िकतनी होती है ?
A. 1100 ml B. 1650 ml
C. 550 ml D. 900 ml
E. None of these

42. In an office there are 60 employees, there are some males and remaining are females.
Average weight of all the male and females in the office is 56 kg and 48 kg respectively. If
the average weight of all the employees in the office is 50 kg, then what is the difference
between the number of males and females in the office?
एक कायालय म 60 कमचारी ह, कुछ पु ष ह और शेष मिहलाएं ह। कायालय म सभी पु षो ं और मिहलाओं
का औसत वजन मशः 56 िकलो ाम और 48 िकलो ाम है । यिद कायालय म सभी कमचा रयो ं का औसत
वजन 50 िकलो ाम है , तो कायालय म पु षो ं और मिहलाओं की सं ाम ा अंतर है ?
A. 25 B. 37
C. 30 D. 45
E. None of these

43. A is 9 years older than C, who is 3 years younger than B. Average of ages of A,B and C after
5 years is 50 years. If the respective ratio of ages of A and D after 5 years is 11 : 13
respectively, then find the difference between the ages of B and D.
A, C से 9 वष बड़ा है , जो B की तुलना म 3 वष छोटा है । A , B और C की औसत आयु आज से 5 वष बाद 50
वष है । यिद 5 वष के बाद A और D की आयु का संबंिधत अनुपात मशः 11:13 है , तो B और D की आयु के
बीच का अंतर ात कीिजए।
A. 12 B. 16
C. 15 D. 18
E. None of these

44. A business was started together by Azeem, Sapan and Pratyush. The total investment
was 1.2 lakhs. Azeem invested Rs 5000 more than Sapan and Pratyush invested Rs 8000
more than Azeem. If at the end of a year, they earned Rs 36000 as profit, then what is the
total share profit of Sapan ?
अज़ीम, सपन और ूष ने िमलकर एक वसाय शु िकया था। कुल िनवेश 1.2 लाख था। अज़ीम ने सपन
से 5000 पये अिधक और ूष ने अज़ीम से 8000 पये अिधक लगाए। यिद एक वष के अंत म, उ ोन
ं े
लाभ के प म 36000 पये कमाए, तो सपन का कुल लाभ िकतना है ?
A. Rs 11500 B. Rs 12700
C. Rs 10200 D. Rs 9900
E. None of these

45. Joey's friend Ross gave him Rs. 8000 and Monica gave Joey 25% less than what Ross
gave to Joey. If Chandler gave 50% more than Monica to Joey, the amount received by
Joey from Chandler is what percentage of the amount he received from Ross ?
जोय के दो रॉस ने उसे 8000 पये िदए और मोिनका ने जोय को रॉस के मुक़ाबले 25% कम पये िदए।
यिद चां डलर ने जॉय को मोिनका से 50% अिधक पये िदए, तो जॉय को चां डलर से जो रािश िमली, वह रॉस
से िमली रािश का िकतना ितशत है ?
A. 106 % B. 112.5 %

PAGE h ps://t.me/studified h ps://instagram.com/aashisharorasocial?utm_medium=copy_link


Follow
63 Aashish Arora on: h ps://youtube.com/channel/UCYa4_JrOrf8R5Kz2uOtccXQ https://www.facebook.com/aashisharorasocial/
अ ास by Aashish Arora
(SBI/IBPS/RBI/LIC/All other banking and insurance Exams)

C. 107.25 % D. 115 %
E. None of these

46. Mr. Patel bought an item at 16% discount on the marked price and sold the item at 30%
profit on the purchased price. Find the profit percentage if Mr. Patel bought the item at the
marked price.
ी पटे ल ने िच त मू पर 16% छूट पर एक व ु खरीदी और खरीदी गई कीमत पर 30% लाभ पर व ु
बेची। यिद ी पटे ल ने िच त मू पर व ु खरीदी होती लाभ ितशत ात कीिजए।
A. 18.4 % B. 9.2 %
C. 15.7 % D. 11.9 %
E. None of these

47. The sum of simple interest earned on sum of Rs 12000 at a% per annum for 2 years and
the simple interest earned on sum of Rs. 15000 at 2a% per annum for 18 months is Rs.
10350. What is the value of a?
2 साल के िलए ित वष 12% की दर से 12000 पये की साधारण ाज की रािश और 18 महीने के िलए ित
वष 2a% की दर से 15000 पये की रािश पर अिजत ाज 10350 पये है । a का मान ात कर।
A. 16 % B. 12.5 %
C. 15 % D. 10 %
E. None of these

48. A boat HMS Queen Elizabeth can cover 616 km upstream in 15 hours 24 minutes and
speed of the stream is 4 km/h. If speed of another boat Charles De Gaulle in still water is
75% of the speed of HMS Queen Elizabeth in still water, then in how much time would
Charles De Gaulle take to cover 629 km downstream ?
एक नाव एचएमएस ीन एिलजाबेथ 15 घंटे 24 िमनट म 616 िकमी धारा के िवपरीत चल सकती है और धारा
की गित 4 िकमी / घंटा है । अगर थर पानी म एक और नाव चा डी गॉल की गित एचएमएस ीन
एिलजाबेथ की गित का 75% है , तो चा डी गॉल को 629 िकमी धारा के अनुकूल जाने म िकतना समय
लगेगा?
A. 17 hours B. 19 hours
C. 22.5 hours D. 14.6 hours
E. None of these

49. Robin takes approximately 15.07 days to complete a piece of work alone while Robin and
Lily together take approximately 7.97 days to complete the same work. In how many days
can Lily alone complete a work three and a half times as large?
रॉिबन को अकेले एक काम को पूरा करने म लगभग 15.07 िदन लगते ह जबिक रॉिबन और िलली को एक
साथ एक ही काम को पूरा करने म लगभग 7.97 िदन लगते ह। िलली अकेले िकतने िदनो ं म साढ़े तीन गुना
बड़ा काम कर सकती है ?
A. 52 days B. 90 days
C. 75 days D. 60 days
E. None of these

50. Biswa's age 15 years ago was 50% more than what Sapan's age was 10 years ago and
Sapan's age 16 years from now will be 10/7 times the current age of Biswa. The sum of
their present ages is divisible by ?

PAGE h ps://t.me/studified h ps://instagram.com/aashisharorasocial?utm_medium=copy_link


Follow
64 Aashish Arora on: h ps://youtube.com/channel/UCYa4_JrOrf8R5Kz2uOtccXQ https://www.facebook.com/aashisharorasocial/
अ ास by Aashish Arora
(SBI/IBPS/RBI/LIC/All other banking and insurance Exams)

15 साल पहले िब ा की उ 10 साल पहले सपन की उ की तुलना म 50% अिधक थी और अभी से 16


साल बाद सपन की उ िबसवा की मौजूदा उ के 10/7 गुना होगी। उनके वतमान युग का योग िकसके ारा
िवभा है ?
A. 2 B. 4
C. 7 D. 3
E. 8

51. A bag contains 4 yellow pins, 6 orange pins and 9 red pins. A person draws 2 pins at
random from the bag. What will be the probability that both pins are of different color?
एक बैग म 4 पीली कील, 6 नारं गी कील और 9 लाल कील ह। एक बैग से अिनयिमत प से 2 कील
खीच
ं ता है । ा संभावना होगी िक दोनो ं कील अलग-अलग रं ग की हो?

A. 2/3 B. 16/19
C. 123/171 D. 43/57
E. None of these

52. A glass contains a mixture of alcohol and soda in the ratio 6 : 11 respectively. 85 ml of
mixture is replaced with 170 ml of alcohol. If the ratio of alcohol to soda in the final mixture
becomes 8 : 9 respectively, then find the amount of soda in the mixture initially.
एक िगलास म मशः 6 : 11 के अनुपात म शराब और सोडा का िम ण है । 85 िमलीलीटर िम ण को 170
िमलीलीटर शराब के साथ बदल िदया जाता है । यिद अंितम िम ण म शराब और सोडा का अनुपात मशः 8 :
9 हो जाता है , तो शु म िम ण म सोडा की मा ा का पता लगाएं ।
A. 539 ml B. 550 ml
C. 555 ml D. 540 ml
E. None Of These

53. Parma, Bhagchand and Chagan alone can do a work in 21 days, 35 days and 30 days
respectively. All of them started working together but after 4 days of the start Parma left the
work and 2 days before the completion of the work Bhagchand left the work. For how many
days Bhagchand and Chagan worked together?
परमा, भागचंद और छगन अकेले मशः 21 िदन, 35 िदन और 30 िदन म िकसी काम को पूरा कर सकते ह।
उन सभी ने एक साथ काम करना शु कर िदया लेिकन शु आत के 4 िदन बाद परमा ने काम छोड़ िदया
और काम पूरा होने से 2 िदन पहले भागचंद ने काम छोड़ िदया। भागचंद और छगन ने िकतने िदनो ं तक साथ
काम िकया?
A. 6 days B. 2 days
C. 8 days D. 5 days
E. None Of These

54. You sold a product at 38% profit. If you had sold the same product at 21% loss then you
would have earned Rs. 324.5 less. Find the marked price of the product if you have
marked it 56% above its cost price.
आपने 38% लाभ पर एक उ ाद बेचा। यिद आपने उसी उ ाद को 21% हािन पर बेचा होता तो आप
324.5 कम कमाते। उ ाद का िचि त मू ात कर यिद आपने इसे लागत मू से 56% ऊपर िचि त
िकया है ।
A. Rs. 766 B. Rs. 692
C. Rs. 768 D. Rs. 858
E. None Of These

PAGE h ps://t.me/studified h ps://instagram.com/aashisharorasocial?utm_medium=copy_link


Follow
65 Aashish Arora on: h ps://youtube.com/channel/UCYa4_JrOrf8R5Kz2uOtccXQ https://www.facebook.com/aashisharorasocial/
अ ास by Aashish Arora
(SBI/IBPS/RBI/LIC/All other banking and insurance Exams)

55. Rashmi deposited Rs. 34000 in RBL bank at 25% per annum compound interest
compounded annually for 2 years. After 2 years, she deposited all amount got from RBL
bank in BRL bank at 20% per annum simple interest for another 4 years. Find the simple
interest earned by her.
र ने 34000 आरबीएल बक म सालाना 25% ित वष च वृ ाज के िहसाब से 2 वष के िलए जमा
िकए। 2 वष बाद, उसने आरबीएल बक ारा ा की गई सभी रािश को बीआरएल बक म 20% ित वष
साधारण ाज पर 4 वष के िलए जमा िकया। उसके ारा अिजत साधारण ाज का पता लगाएं ।
A. Rs. 42500 B. Rs. 41426
C. Rs. 42008 D. Rs. 41200
E. None of these

56. Rachit earns Rs. 55000 in a month. He spends 23% and 26% of his income in house rent
and vehicle maintenance respectively. If 40% of rest amount was spent by him in other
activities and the remaining amount is saved by him then find the amount saved by Rachit
in a month.
रिचत ने 55000 एक महीने म कमाता है । वह अपनी आय का मशः 23% और 26% घर के िकराए और
वाहन रखरखाव म खच करता है । यिद बाकी रािश का 40% वह अ गितिविधयो ं म खच करता है और शेष
रािश उसके ारा बचाई जाती है तो एक महीने म रिचत ारा बचाई गई रािश का पता लगाएं ।
A. Rs. 18540 B. Rs. 16830
C. Rs. 16240 D. Rs. 15130
E. None of these

57. Tinku is travelling from point P to point Q while Minku is travelling from point Q to point P.
Both start at the same time. After every hour, Tinku and Minku increases their speeds by 2
km/hr and 4 km/hr respectively. Both of them meet after 6 hours of start, and initially the
speed of Minku is 32 km/hr. Find the distance between point P and point Q if Tinku takes 6
hours to reach point Q after meeting Minku.
िटं कू िबंदु P से िबंदु Q तक या ा कर रहा है , जबिक Minku िबंदु Q से िबंदु P तक या ा कर रहा है । दोनो ं एक
ही समय पर शु होते ह। हर घंटे के बाद, िटं कू और िमंकू अपनी गित मशः 2 िकमी / घंटा और 4 िकमी /
घंटा बढ़ाते ह। शु आत के 6 घंटे बाद दोनो ं िमलते ह, और शु म िमंकू की गित 32 िकमी / घंटा है । िबंदु P
और िबंदु Q के बीच की दू री ात कर यिद िटं कू को िमंकू से िमलने के बाद िबंदु Q तक प ं चने म 6 घंटे लगते
ह।
A. 450 km B. 464 km
C. 432 km D. 350 km
E. None Of These

58. The ratio of number of apples to number of oranges sold by a shopkeeper is 7 : 9


respectively, and the ratio of number of bananas to number of apples sold by the same
shopkeeper is 4 : 3 respectively. If the price of each orange and each banana is Rs. 8 and
Rs. 9 respectively and total revenue earned by selling all bananas is Rs. 108 more than
total revenue earned by selling all oranges, then find the number of apples sold by the
shopkeeper.
एक दु कानदार ारा बेचे गए सेब की सं ा और संतरे की सं ा का अनुपात मशः 7 : 9 है , और उसी
दु कानदार ारा बेचे गए केले की सं ा और सेब की सं ा का अनुपात मशः 4 : 3 है । यिद ेक संतरे
और ेक केले की कीमत 8 और 9 है और सभी केलो ं को बेचकर अिजत कुल राज , सभी संतरे

PAGE h ps://t.me/studified h ps://instagram.com/aashisharorasocial?utm_medium=copy_link


Follow
66 Aashish Arora on: h ps://youtube.com/channel/UCYa4_JrOrf8R5Kz2uOtccXQ https://www.facebook.com/aashisharorasocial/
अ ास by Aashish Arora
(SBI/IBPS/RBI/LIC/All other banking and insurance Exams)

बेचकर कमाए गए कुल राज से 108 अिधक, तो िफर दु कानदार ारा बेचे गए सेब की सं ा का पता
लगाएं ।
A. 53 B. 61
C. 57 D. 63
E. None of these

59. Teekam and Teejada together started a business with a total investment of Rs. 9000. After
8 months, Teekam decreased his investment by 25% while Teejada increased his
investment by 30%. At the end of the year, Teekam and Teejada divided the profit in the
ratio 2 :3 respectively. Find the initial investment of Teekam.
टीकम और टीजादा ने िमलकर कुल 9000 के िनवेश के साथ एक वसाय शु िकया। 8 महीनो ं के बाद,
टीकम ने अपने िनवेश म 25% की कमी की जबिक टीजादा ने अपने िनवेश म 30% की वृ की। वष के अंत
म, टीकम और टीजादा ने मशः 2 : 3 के अनुपात म लाभ को िवभािजत िकया। टीकम का ारं िभक िनवेश
ात कीिजए।
A. Rs. 4000 B. Rs. 5000
C. Rs. 4500000
D. Cannot Be Determined/िनधा रत नही ं िकया जा सकता
E. None Of These

60. The curved surface area of a cylindrical vessel is 5544 cm2. The respective ratio of the
radius and the height of the vessel is 2 : 9 respectively. If the cylindrical vessel is full of
water, then find the time taken to empty the vessel at the rate of 539 cm3 of water per
minute.
एक बेलनाकार बतन की घुमावदार सतह े 5544 वग सेमी है । बतन की ि ा और ऊंचाई का अनुपात
मशः 2 : 9 है । यिद बेलनाकार बतन पानी से पूरा भरा है , तो ित िमनट 539 घन सेमी पानी की दर से बतन
को खाली करने म लगने वाले समय का पता लगाएं ।
A. 90 minutes B. 72 minutes
C. 36 minutes D. 18 minutes
E. None of these

61. The time taken by a boat to cover 'X' km downstream is 36 hours while 'Y' km upstream is
55 hours. If the ratio of speed of boat in still water to the speed of stream is 10 : 1
respectively, then find X : Y.
'X' िकमी धारा के अनुकूल जाने म एक नाव ारा िलया गया समय 36 घंटे है जबिक 'Y' िकमी धारा के िव
जाने म उसी नाव ारा िलया गया समय 55 घंटे है । यिद धारा की गित और थर पानी म नाव की गित का
अनुपात मशः 1 : 10 है , तो X : Y को खोजे।
A. 6 : 7 B. 7:6
C. 4 : 5 D. 5:4
E. None of these

62. The ratio of age of Pragati and Unnati is 12 : 13 respectively. After 12 years, the ratio of
their ages will become 16 : 17 respectively. Find the present age of Unnati.
गित और उ ित की आयु का अनुपात मशः 12 : 13 है । 12 वष के बाद, उनकी आयु का अनुपात मशः
16 : 17 हो जाएगा। उ ित की वतमान आयु ात कीिजए।
A. 36 years B. 38 years

PAGE h ps://t.me/studified h ps://instagram.com/aashisharorasocial?utm_medium=copy_link


Follow
67 Aashish Arora on: h ps://youtube.com/channel/UCYa4_JrOrf8R5Kz2uOtccXQ https://www.facebook.com/aashisharorasocial/
अ ास by Aashish Arora
(SBI/IBPS/RBI/LIC/All other banking and insurance Exams)

C. 37 years D. 39 years
E. None of these

63. 630 litres of mixture contains syrup and water in the ratio 4 : a respectively. After addition of
20 litres of syrup and 50 litres of water, the ratio of syrup to water in the resultant mixture
becomes 3 : 4 respectively. Find the quantity of water in the mixture initially.
630 लीटर िम ण म मशः 4 : a के अनुपात म िसरप और पानी है । 20 लीटर िसरप और 50 लीटर पानी
डालने के बाद, प रणामी िम ण म िसरप और पानी का अनुपात मशः 3 : 4 हो जाता है । शु म िम ण म
पानी की मा ा का पता लगाएं ।
A. 350 litres B. 450 litres
C. 300 litres D. 400 litres
E. None of these

64. Anamika, Bhavika and Chutkumal can complete the work in 30 days, 48 days and 60 days
respectively. All of them started working together but after 5 days Anamika left the work
and 4 days before the completion of the work Chutkumal also left the work. In how many
days the work is completed?
अनािमका, भािवका और छु टकुमल मशः 30 िदन, 48 िदन और 60 िदन म काम पूरा कर सकते ह। उन
सभी ने एक साथ काम करना शु कर िदया लेिकन 5 िदनो ं के बाद अनािमका ने काम छोड़ िदया और काम
पूरा होने से 4 िदन पहले छु टकुमल ने भी काम छोड़ िदया। काय िकतने िदनो ं म पूरा होगा?
A. 35 days B. 24 days
C. 26 days D. 32 days
E. None Of These

65. After 12 years, ratio of age of Kanta and Ramu will be 5 : 7 respectively. Age of Kanta 2
years ago was half of the age of Ramu 2 years hence. If the present average age of Bala,
Kanta and Ramu is 24 years and 4 months, then find the present age of Bala.
12 वष के बाद, कां ता और रामू की आयु का अनुपात मशः 5 : 7 होगा। 2 साल पहले कां ता की उ , 2 साल
बाद रामू की उ की आधी थी। यिद बाला, कां ता और रामू की वतमान औसत आयु 24 वष और 4 महीने है , तो
बाला की वतमान आयु ात कीिजए।
A. 28 years B. 26 years
C. 25 years D. 20 years
E. None Of These

66. Papli, Bittan and Carlos entered into a business with an initial investments of Rs. (x + 200),
Rs. (x – 300) and Rs. (x + 250) respectively. After 6 months, they made an additional
investment of Rs. 200, Rs. 350 and Rs. 600 respectively. If the total profit earned at the end
of the year is Rs. 6655, then find the profit share of Bittan.
पपली, िब न और काल स ने मशः (x + 200), (x - 300) और (x + 250) के शु आती िनवेश के
साथ एक वसाय शु िकया। 6 महीने के बाद, उ ोन
ं े मशः 200, 350 और 600 का अित र
िनवेश िकया। यिद वष के अंत म उनके ारा कुल अिजत लाभ 6655 है , तो िफर िब न के लाभ का िह ा
खोज।
A. Rs. 2440 B. Rs. 2655
C. Rs. 2755
D. Cannot be determined/ िनधा रत नही ं िकया जा सकता
E. None Of These

PAGE h ps://t.me/studified h ps://instagram.com/aashisharorasocial?utm_medium=copy_link


Follow
68 Aashish Arora on: h ps://youtube.com/channel/UCYa4_JrOrf8R5Kz2uOtccXQ https://www.facebook.com/aashisharorasocial/
अ ास by Aashish Arora
(SBI/IBPS/RBI/LIC/All other banking and insurance Exams)

67. Charu borrowed a certain amount from a bank on 20% per annum compound interest
compounded annually for 3 years, but she deposited Rs. 7020 at the end of 2nd year and
clear her debt by depositing Rs. 5400 at the end of 3rd year. Find the amount that Charu
borrowed.
चा ने एक बक से 3 साल के िलए सालाना 20% ित वष च वृ ाज पर एक िनि त रािश उधार ली,
लेिकन उसने 7020 दू सरे वष के अंत म जमा कराये और तीसरे वष के अंत म 5400 जमा करके कज
मु हो गयी। वह रािश ात कर जो चा ने उधार ली थी।
A. Rs. 8000 B. Rs. 8500
C. Rs. 9000 D. Rs. 7500
E. None of these

68. Car A starts at 10:15 AM from Manali and after 30 minutes Car B starts from Kullu. Car B
meets Car A after 3 hours 30 minutes of starting and the speed of Car A is 70 km/hr. If the
distance between Manali and Kullu is 455 km, then find the speed of Car B.
कार A सुबह 10:15 बजे मनाली से चलती है और 30 िमनट के बाद कार B कु ू से चलती है । कार B चलने
के 3 घंटे 30 िमनट के बाद कार A से िमलती है और कार A की गित 70 िकमी / घंटा है । यिद मनाली और कु ू
के बीच की दू री 455 िकमी है , तो कार B की गित का पता लगाएं ।
A. 60 km/hr B. 50 km/hr
C. 65 km/hr D. 55 km/hr
E. None of these

69. Speed of a boat in downstream is 36% more than speed of the boat in still water. If the boat
can travel a distance of 68 km upstream in 4 hours and 15 minutes, then find the time taken
by the boat to cover a distance of 221 km downstream.
धारा के अनुकूल एक नाव की गित, थर पानी म नाव की गित से 36% अिधक है । यिद धारा के िव 68
िकमी की दू री तय करने म नाव 4 घंटे और 15 िमनट लेती है , तो धारा के अनुकूल नाव ारा 221 िकमी की दू री
तय करने म लगने वाले समय का पता लगाएं ।
A. 5 hours 15 minutes B. 5 hours 30 minutes
C. 6 hours 30 minutes D. 6 hours 15 minutes
E. None Of These

70. Three members X, Y and Z contested the election. 25% of the votes cast are invalid. Out of
the valid votes, X secured 33% votes and Y secured 28.56% votes and lost by 110 votes
with respect to 7. Find the total number of votes cast.
तीन सद ो ं XY और Z ने चुनाव लड़ा डाले गए मतो ं म से 25% मत अमा ह। मा मतो ं म से, X ने 33 ( 1 /
3) % मत ा िकए और Y ने 28.56% मत ा िकए और 2 के संबंध म 110 मत से हार गया। कुल मत
की सं ा ात कीिजए।
A. 1155 B. 1355
C. 1440 D. 1540
E. None of these

71. The percentage loss incurred by selling the product at Rs. 3026 is equal to half of the
percentage profit earned by selling the same product at Rs. 4628. At what price should the
product be sold to make a profit of 25% ?
3026 म उ ाद को बेचने से होने वाली ितशत हािन, उसी उ ाद को 4628 म बेचकर अिजत लाभ
ितशत के आधे के बराबर है । 25% का लाभ कमाने के िलए उ ाद को िकस मू पर बेचा जाना चािहए?

PAGE h ps://t.me/studified h ps://instagram.com/aashisharorasocial?utm_medium=copy_link


Follow
69 Aashish Arora on: h ps://youtube.com/channel/UCYa4_JrOrf8R5Kz2uOtccXQ https://www.facebook.com/aashisharorasocial/
अ ास by Aashish Arora
(SBI/IBPS/RBI/LIC/All other banking and insurance Exams)

A. Rs. 4450 B. Rs. 4560


C. Rs. 4350 D. Rs. 4250
E. None Of These

72. A cylindrical well of diameter 14 cm is dig up to 27 cm deep in the center of a rectangular


field of dimensions 22cm × 28cm and the volume of the soil dig out is spread on the
remaining field evenly. Find the increase in the height of the rectangular field.
14 सेमी ास वाले एक बेलनाकार कुआं , 22 सेमी × 28 सेमी आयाम के आयताकार मैदान के क म 27
सेमी गहराई तक खोदा जाता है और खुदाई की गई िम ी की मा ा शेष े पर समान प से फैलाई जाित है ।
आयताकार मैदान की ऊंचाई म वृ का पता लगाएं ।
A. 27 cm B. 9 cm
C. 18 cm D. 6 cm
E. None of these

73. A 1125 ml mixture contains petrol and kerosene in the ratio of 11 : 14 respectively. 40% of the
mixture is taken out and some amount of petrol is added to the remaining mixture so that
the ratio of petrol to kerosene in the remaining mixture becomes 6 : 7. The amount of petrol
added is what percent of the initial mixture?
एक 1125 िमलीलीटर िम ण म मशः 11 : 14 के अनुपात म पेटोल और िम ी का तेल है । िम ण का 40%
िनकाल िलया जाता है और शेष िम ण म कुछ मा ा म पेटोल िमला िदया जाता है तािक शेष िम ण म पेटोल
और िम ी का तेल मशः के अनुपात म 6 : 7 हो जाए। िमलाये गए पेटोल की मा ा ारं िभक िम ण का
िकतना ितशत है ?
A. 5 % B. 4%
C. 3.6 % D. 2.4 %
E. None of these

74. To do a certain work, Q takes five times as long as P and R together to complete a piece of
work, while R takes seven times as long as time taken by P and Q together. If all the three
can complete the whole work in 5 days then find the time taken by R alone to complete the
whole work.
एक िनि त काय करने के िलए, Q ारा िलया गया समय P और R ारा साथ म िलए गए समय का पाँ च गुना है ,
जबिक R ारा िलया गया समय P और Q ारा साथ म िलए गए समय का सात गुना है । यिद तीनो ं पूरे काय को
5 िदनो ं म पूरा कर सकते ह तो पूरे काय को पूरा करने के िलए R ारा िलया गया समय ात कर।
A. 35 days B. 40 days
C. 46 days D. 42 days
E. None Of These

75. A box contains 'x' Hindi notebooks, 'x + 3' English notebooks and 'x – 6' Maths notebooks.
If the probability of drawing a Hindi notebook is 1/7 more than the probability of drawing a
Maths notebook, then find the value of 'x'.
एक बॉ म box 'x' िहं दी पु क, 'x + 3' अं ेजी पु क और 'x - 6' गिणत पु क ह। यिद िहं दी पु क
खीच
ं ने की संभावना, गिणत पु क खीच
ं ने की संभावना से 1/7 अिधक है , तो 'x' का मान ात कर।
A. 18 B. 16
C. 15 D. 10
E. None Of These

PAGE h ps://t.me/studified h ps://instagram.com/aashisharorasocial?utm_medium=copy_link


Follow
70 Aashish Arora on: h ps://youtube.com/channel/UCYa4_JrOrf8R5Kz2uOtccXQ https://www.facebook.com/aashisharorasocial/
अ ास by Aashish Arora
(SBI/IBPS/RBI/LIC/All other banking and insurance Exams)

76. Rajnish borrowed Rs. 5500 and Rs. 6500 at P% simple interest and (P + 4)% simple
interest respectively. After 4 years, he paid a total interest of Rs. 12560. Find the rate of
interest at which he borrowed Rs. 5500.
रजनीश ने मश: 5500 और 6500 P% और (P + 4)% साधारण ाज की दर से उधार िलए। 4 वष
के बाद, उसने कुल 12560 ाज का भुगतान िकया। उस ाज दर का पता लगाएं िजस पर उसने
5500 उधार िलए।
A. 28% B. 24%
C. 26%
D. Cannot be determined/ िनधा रत नही ं िकया जा सकता
E. None Of These

77. Average weight of four friends , P, Q, R and S is 36.5 kg. When S is replaced by T then the
average weight increases by 2.5 kg, while when R is replaced by T, average weight
increases by 2 kg. If average weight of R, S and T is 42 kg, then find the average weight of
all five friends.
चार दो , P, Q, R और S का औसत वजन 36.5 िकलो ाम है । जब S को T से बदला जाता है तो औसत
वजन 2.5 िकलो ाम बढ़ जाता है , जबिक जब R को T से बदल िदया जाता है , तो औसत वजन 2 िकलो ाम
बढ़ जाता है । यिद R, S और T का औसत वजन 42 िकलो ाम है , तो सभी पां च दो ो ं का औसत वजन ात
कर।
A. 38.8 B. 35.6
C. 39.6 D. 36.8
E. None of these

78. A shopkeeper mixes two types pulses of price Rs 70 per kg and Rs. 54 per kg in the ratio of
5 : 3 respectively. If he sold the mixture for Rs. 1024, then find the quantity of pulse of cost
Rs. 70 per kg sold by him.
एक दु कानदार पये 70 ित िकलो ाम और पये 54 ित िकलो ाम की दो कार की दालो ं को मशः 5 :
3 के अनुपात म िमलाता है । यिद उसने िम ण को 1024 म बेचा, तो िफर उसके ारा बेची गयी पये 70
ित िकलो ाम की दाल की मा ा ात करे ।
A. 10 kg B. 12 kg
C. 8 kg D. 6 kg
E. None of these

79. The cost of fencing a rectangular plot at Rs. 18 per meter is Rs. 936. Find the length of the
rectangular plot if its breadth is 11 meters.
एक आयताकार भूखंड पर 18 ित मीटर की दर से बाड़ लगाने की लागत 936 है । आयताकार भूखंड
की लंबाई ात कर यिद इसकी चौड़ाई 11 मीटर है ।
A. 10 meters B. 16 meters
C. 15 meters D. 18 meters
E. None Of These

80. 5 years ago, the ratio of age of Vikas and Hitesh was 9 : 10 respectively. After 7 years from
now, the respective ratio of their age becomes 13 : 14. If present average age of Jeetu,
Vikas and Hitesh is 37 years, then find the present age of Jeetu.
5 साल पहले, िवकास और िहतेश की उ का अनुपात मशः 9:10 था। अब से 7 वष बाद, उनकी आयु का

PAGE h ps://t.me/studified h ps://instagram.com/aashisharorasocial?utm_medium=copy_link


Follow
71 Aashish Arora on: h ps://youtube.com/channel/UCYa4_JrOrf8R5Kz2uOtccXQ https://www.facebook.com/aashisharorasocial/
अ ास by Aashish Arora
(SBI/IBPS/RBI/LIC/All other banking and insurance Exams)

संबंिधत अनुपात 13:14 हो जाता है । यिद वतमान म जीतू, िवकास और िहतेश की औसत आयु 37 वष है , तो
वतमान म जीतू की आयु ात कर।
A. 42 years B. 48 years
C. 46 years D. 44 years
E. None of these

81. Shivangi started a business with a capital of Rs. 1984. Harshangi joined the business
investing Rs. 1612 after certain months. At the end of one year, profit share of Shivangi out
of total profit of Rs. 9900 is Rs. 7040. Find the time after which Harshangi joined the
business.
िशवां गी ने 1984 की पूंजी के साथ एक वसाय शु िकया। कुछ महीनो ं के बाद, हषागी 1612 की पूंजी
के साथ वसाय म शािमल ई। एक वष के अंत म, 9900 के कुल लाभ म से िशवां गी का लाभ िह ा
7040 है , तो िशवां गी से िकतने समय बाद हषागी वसाय म शािमल हो गई ?
A. 4 months B. 7 months
C. 5 months D. 6 months
E. None Of These

82. Speed of a boat in still water is 120% more than the speed of stream. If the boat can cover
168 km downstream and 81 km upstream in 8 hours, then find the time taken by the boat to
cover 297 km in still water.
थर पानी म एक नाव की गित धारा की गित से 120% अिधक है । यिद नाव कुल 8 घंटे म 168 िकमी धारा के
अनुकूल और 81 िकमी धारा के िव चल सकती है , तो थर पानी म 297 िकमी चलने के िलए नाव ारा
िलया गया समय ढू ं ढ।
A. 10 hours B. 9 hours
C. 11 hours D. 7 hours
E. None of these

83. Nakul and Sakshi started a company together with an investment of Rs. 1200 and Rs.
1600 respectively. After 4 months, Chhavi joined them with 25% more investment than
initial investment of Nakul. After completion of a year, they received a total profit of Rs.
6080, then find the profit share of Nakul.
नकुल और सा ी ने मशः 1200 और 1600 के िनवेश के साथ एक कंपनी शु की। 4 महीने के बाद,
नकुल के शु आती िनवेश की तुलना म 25% अिधक िनवेश के साथ छवी शािमल ई। एक साल पूरा होने के
बाद, उ कुल लाभ 6080 िमला, तो िफर नकुल का लाभ िह ा खोज।
A. Rs. 1920 B. Rs. 2560
C. Rs. 1600 D. Rs. 1290
E. None of these

84. Venuka invested a sum of Rs. 'X' in a scheme and received Rs. 120 as interest after a year.
If she added Rs. 180 to it, and invested the total amount in the same scheme for one more
year, and received 1080 as total, then find the value of 'X'.
वेणुका ने 'X' एक योजना म िनवेश िकए और एक वष के बाद उसे ाज के प म 120 िमले। अगर वह
इसम 180 और जोड़ दे , और उसी योजना म िनवेश करे , तो एक वष बाद कुल िमलाकर उसे 1080
िमलगे। 'X' का मान ात कीिजए।
A. Rs. 600 B. Both (A) and (C)

PAGE h ps://t.me/studified h ps://instagram.com/aashisharorasocial?utm_medium=copy_link


Follow
72 Aashish Arora on: h ps://youtube.com/channel/UCYa4_JrOrf8R5Kz2uOtccXQ https://www.facebook.com/aashisharorasocial/
अ ास by Aashish Arora
(SBI/IBPS/RBI/LIC/All other banking and insurance Exams)

C. Rs. 60 D. Rs. 800


E. None Of These

85. 7 women and 6 children can complete a given work in 10 days. If additional 4 women
joined them, then they could finish the 3/4th of work in 6 days, find the time taken by 32
women to complete the work.
7 मिहलाएं और 6 ब े िदए गए काम को 10 िदनो ं म पूरा कर सकते ह। यिद अित र 4 मिहलाएं उनके साथ
जुड़ गईं, तो वे 3/4 व काम को 6 िदनो ं म पूरा कर सकते ह, काम पूरा करने के िलए 32 मिहलाओं ारा िलया
गया समय ढू ं ढ।
A. 10 days B. 8 days
C. 5 days D. 4 days
E. None Of These

86. The downstream speed and upstream speed of a boat is 38 km/hr and 22 km/hr
respectively. Time taken by the boat to travel (a + b) km downstream with stream speed
half that of normal stream speed is 10 hours, then find the average of value of a and b.
एक नाव की धारा के अनुकूल गित और धारा के िव गित मशः 38 िकमी / घंटा और 22 िकमी / घंटा है ।
सामा धारा की गित की आधी गित के साथ (a + b) िकमी धारा के अनुकूल नाव से या ा करने म लगने वाला
समय 10 घंटे है , तो िफर a और b के मान का औसत ात कर।
A. 340 B. 510
C. Cannot be determined/ िनधा रत नही ं िकया जा सकता
D. 170 E. None Of These

87. A vessel contains 525 litres mixture of petrol and diesel mixed in the ratio 7 ; 8 respectively.
'L' litres of mixture is taken out of the vessel and replaced with 30 litres of diesel such that
the ratio of the petrol to diesel becomes 13 : 17 respectively. Find the value of 'L'.
एक बतन म 7 : 8 के अनुपात म 525 लीटर पेटोल और डीजल का िम ण है । 'L' लीटर िम ण को बतन से
िनकाल िलया जाता है और इसे 30 लीटर डीजल के साथ बदल िदया जाता है , तािक पेटोल और डीजल का
अनुपात मशः 13 : 17 हो जाता है । 'L' का मान ात कीिजए।
A. 135 B. 105
C. 90 D. 750
E. None of these

88. A train can cross a pole and platform of 138 meters long in 19 seconds and 25 seconds
respectively. Find the time taken by the train to cross another platform having length 230
meters.
एक टे न मशः 19 सेकंड और 25 सेकंड म एक पोल और 138 मीटर लंबे एक ेटफॉम को पार कर सकती
है । 230 मीटर की लंबाई वाले दू सरे ेटफाम को पार करने के िलए टे न ारा िलया गया समय ात कर।
A. 33 seconds B. 29 seconds
C. 39 seconds D. 23 seconds
E. None of these

89. A shopkeeper marked an article 75% above the cost price and sold it after two consecutive
discounts of p% and 10%. Find the value of 'p', if in this transaction he had a profit of 19.7%
एक दु कानदार ने एक व ु को उसकी लागत मू से 75% ऊपर िचि त िकया और p% और 10% की
िमक छूट के बाद इसे बेच िदया। 'p' का मान ात कर, यिद इस लेनदे न म उसे 19.7% का लाभ आ।

PAGE h ps://t.me/studified h ps://instagram.com/aashisharorasocial?utm_medium=copy_link


Follow
73 Aashish Arora on: h ps://youtube.com/channel/UCYa4_JrOrf8R5Kz2uOtccXQ https://www.facebook.com/aashisharorasocial/
अ ास by Aashish Arora
(SBI/IBPS/RBI/LIC/All other banking and insurance Exams)

A. 26 B. 25
C. 24 D. 20
E. None Of These

90. Present ages of Nisha and Afreen are in the ratio 2 : 3 respectively. Difference between the
ages of Afreen and Leela is 6 years. Find the present age of Leela if the present age of
Nisha is 32 years.
िनशा और आफरीन की वतमान आयु मशः 2 : 3 के अनुपात म है । आफरीन और लीला की उ म 6 साल
का अंतर है । लीला की वतमान आयु ात कर यिद िनशा की वतमान आयु 32 वष है ।
A. 54 B. 42
C. 48
D. Cannot be determined/ िनधा रत नही ं िकया जा सकता
E. None of these

91. A cuboidal box has dimensions 12 cm, (2a + 3) cm and (4a – 3) cm. If the cost of polishing
the total surface area of the box at Rs. 5/cm2 is Rs. 7470, then find the value of 'a’
एक आयातफल की िड े का आयाम 12 सेमी, (2a + 3) सेमी और (4a - 3) सेमी ह। यिद 5/सेमी2 की दर
से िड े की कुल सतह े को चमकाने की लागत 7470 है , िफर 'a' का मान ात कीिजए |
A. 6 B. 5
C. 4 D. 2
E. None Of These

92. How many arrangements can be made from the word 'CLERKPRE', such that all vowels
do not come together?
'CLERKPRE' श से ऐसे िकतने श बनाये जा सकते है , जैसे िक सभी र एक साथ नही ं आये?
A. 7240 B. 7560
C. 8180 D. 6920
E. None of these
3
93. Volume of a cylindrical drum is 308 cm . Find the curved surface area of the cylindrical
drum if its base radius and height are in the ratio of 7 : 16 respectively.
एक बेलनाकार डम का आयतन 308 घन सेमी है । बेलनाकार डम के घुमावदार सतह े का पता लगाएं यिद
इसकी आधार ि ा और ऊंचाई मशः 7 : 16 के अनुपात म है ।
2 2
A. 136 cm B. 172 cm
2 2
C. 142 cm D. 176 cm
E. None of these

94. Sanjana bought a dress for Rs. 7400. She spent 25% of the amount for which she bought
the dress for its fitting but then she sold that dress to Sanju for Rs. 10,730. Find the profit
percentage for Sanjana in this transaction.
संजना ने 7400 म एक पोशाक खरीदी। उसने खरीद रािश का 25% पोशाक को सही करवाने म लगा िदया
लेिकन िफर उसने उस पोशाक को संजू को 10,730 म बेच िदया। इस लेनदे न म संजना के िलए लाभ
ितशत का पता लगाएं ।
A. 12 % B. 16 %
C. 20 % D. 24 %
E. None Of These

PAGE h ps://t.me/studified h ps://instagram.com/aashisharorasocial?utm_medium=copy_link


Follow
74 Aashish Arora on: h ps://youtube.com/channel/UCYa4_JrOrf8R5Kz2uOtccXQ https://www.facebook.com/aashisharorasocial/
अ ास by Aashish Arora
(SBI/IBPS/RBI/LIC/All other banking and insurance Exams)

95. Daksh and Suchitra can complete a work in 15 days and 25 days respectively. Daksh and
Suchitra started the work together and completed 64% of the work. If the rest work is
completed by Rudraksh alone in 4.5 days, then Rudraksh is how much percent more
efficient than Suchitra?
द और सुिच ा मशः 15 िदनो ं और 25 िदनो ं म एक काम पूरा कर सकते ह। द और सुिच ा ने िमलकर
काम शु िकया और 64% काम पूरा िकया। यिद शेष काय ा अकेले 4.5 िदनो ं म पूरा करता है , तो
ा सुिच ा की तुलना म िकतने ितशत अिधक कुशल है ?
A. 30% B. 60%
C. 100% D. 40%
E. None Of These

96. Shrinath sold an article for Rs. 800 at a profit of 33.33%. Jainath sold another article and
earned 25% more profit than the profit earned by Shrinath. Find the profit percent for
Jainath if the cost price of both the articles is same.
ीनाथ ने 33.33% के लाभ पर एक व ु को पये 800 म बेचा। जैनाथ ने एक व ु को बेचा और ीनाथ ारा
अिजत लाभ का 25% अिधक लाभ अिजत िकया। जैनथ के िलए लाभ ितशत ात कर यिद दोनो ं व ुओं की
लागत मू समान है ।
A. 38% B. 41.67%
C. 40%
D. Cannot be determined/ िनधा रत नही ं िकया जा सकता
E. None Of These

97. Avinash and Anurag together started a business with initial investment of Rs. 1450 and
Rs. 1050 respectively. After one year, Avinash increased his investment by 20%, while
Anurag decreased his investment by Rs. 150. Find the profit share of Avinash, out of total
profit of Rs. 6682 after 2 years.
अिवनाश और अनुराग ने िमलकर मशः 1450 और 1050 के शु आती िनवेश के साथ एक वसाय
शु िकया। एक वष के बाद, अिवनाश ने अपने िनवेश म 20% की वृ की, जबिक अनुराग ने अपने िनवेश
म से 150 कम िकए। 2 साल बाद 6682 के कुल लाभ म से, अिवनाश का लाभ िह ा ात करे ।
A. Rs. 4147 B. Rs. 2250
C. Rs. 3850 D. Rs. 3250
E. None of these

98. A container P contains mixture of milk and water in which 70% of mixture is water. If 36% of
the mixture is taken out, then the quantity of water in the container P becomes 168 litres. If
60 litres of another mixture of milk and water containing milk and water in the ratio of 4:1 is
added in the remaining mixture of container P, then find the new ratio of milk and water in
container P.
एक बतन P म दू ध और पानी का िम ण है िजसम 70% िम ण पानी है । यिद िम ण का 36% बाहर िनकाला
जाता है , तो बतन P म पानी की मा ा 168 लीटर हो जाती है । यिद बतन P के शेष िम ण म 60 लीटर दू ध और
पानी का िम ण िमलाया जाए िजसमे दू ध और पानी का अनुपात मश: 4:1 है , तो बतन P म दू ध और पानी के
नए अनुपात का पता लगाएं ।
A. 2 : 3 B. 4:5
C. 6 : 7 D. 8:9
E. None of these

PAGE h ps://t.me/studified h ps://instagram.com/aashisharorasocial?utm_medium=copy_link


Follow
75 Aashish Arora on: h ps://youtube.com/channel/UCYa4_JrOrf8R5Kz2uOtccXQ https://www.facebook.com/aashisharorasocial/
अ ास by Aashish Arora
(SBI/IBPS/RBI/LIC/All other banking and insurance Exams)

99. School A has 30% more students than in School B. The ratio of boys to girls in School A is 6
: 7 respectively. The ratio of girls to boys in School B is 3 : 2 respectively. If number of boys
in School A is 4080, then find the difference between number of girls in School A and than
in School B.
ू ल A म, ू ल B की तुलना म 30% अिधक छा ह। ू ल A म लड़को ं और लड़िकयो ं का अनुपात मशः
6 : 7 है । ू ल B म लड़िकयो ं और लड़को ं का अनुपात मशः 3 : 2 है । यिद ू ल A म लड़को ं की सं ा
4080 है , तो ू ल A और ू ल B म लड़िकयो ं की सं ा के बीच का अंतर ात कर।
A. 520 B. 480
C. 680 D. 580
E. None Of These

100. Champak deposited 40% of the money he had in FDHC Bank which offered 20% per
annum rate of compound interest compounded annually. If interest earned by him after 3
years was Rs. 2912, then find the money that Champak had initially.
चंपक के पास कुछ धनरािश थी। उसने FDHC बक म 40% धनरािश जमा की, जो 20% ितवष की दर से
च वृ ाज दान करता है । यिद 3 वष के बाद उसके ारा अिजत ाज 2912 था, तो शु म चंपक
के पास िकतनी धनरािश थी ?
A. Rs. 20,000 B. Rs. 16,000
C. Rs. 12,000 D. Rs. 10,000
E. None of these

101. Girish, Harish and Jagdish together started a business. The initial investments made by
Girish and Harish are 20% and 30% respectively more than that of initial investment made
by Jagdish. Find the profit share of Girish, if total profit earned by them in a year is Rs.
4,445.
िगरीश, हरीश और जगदीश ने िमलकर एक वसाय शु िकया। िगरीश और हरीश ारा िकया गया
ारं िभक िनवेश जगदीश ारा िकया गया ारं िभक िनवेश की तुलना म मशः 20% और 30% अिधक ह।
िगरीश के लाभ का िह ा ात कर, यिद एक वष म उनके ारा अिजत कुल लाभ 4,445 ह।
A. Rs. 1270 B. Rs. 1651
C. Rs. 1645 D. Rs. 1524
E. None Of These

102. A boat covers 108 km downstream and 56 km upstream together in 16 hours. If the ratio of
downstream speed and upstream speed is 3 : 2 respectively, then find the speed of
stream.
एक नाव 16 घंटे म एक साथ धारा के अनुकूल 108 िकमी और धारा के िव 56 िकमी चलती है । यिद धारा
के अनुकूल गित और धारा के िव गित का अनुपात मशः 3 : 2 है , तो धारा की गित का पता लगाएं ।
A. 3 km/hr B. 2 km/hr
C. 5 km/hr D. 6 km/hr
E. None of these

103. The average runs scored by Yugam in 18 matches was 29. In 19th match, he scored 86
runs. Find the average runs scored by him in all the matches.
18 मैचो ं म युगम ारा बनाए गए औसत रन 29 थे। 19 व मैच म उ ोन
ं े 86 रन बनाए। सभी मैचो ं म उनके ारा
बनाए गए औसत रन का पता लगाएं ।

PAGE h ps://t.me/studified h ps://instagram.com/aashisharorasocial?utm_medium=copy_link


Follow
76 Aashish Arora on: h ps://youtube.com/channel/UCYa4_JrOrf8R5Kz2uOtccXQ https://www.facebook.com/aashisharorasocial/
अ ास by Aashish Arora
(SBI/IBPS/RBI/LIC/All other banking and insurance Exams)

A. 40 runs B. 36 runs
C. 32 runs D. 38 runs
E. None of these

104. Yatin invested Rs. 6000 partially in Stock A offering 25% per annum compound interest
compounded annually and rest in Stock B offering 43% per annum simple interest. Find
the amount invested by Yatin in Stock A, if the total interest earned by him after two years
from both the stocks together is Rs. 4089.
यितन ने आं िशक प से 6000 ॉक A म सालाना 25% ित वष च वृ ाज पर िनवेश िकए और
ॉक B म सालाना 43% ित वष साधारण ाज पर िनवेश िकए। ॉक A म यितन ारा िनवेश की गई रािश
का पता लगाएं , अगर दोनो ं ॉक से एक साथ दो साल बाद उसके ारा अिजत कुल ाज 4089 है ।
A. Rs. 2400 B. Rs. 3600
C. Rs. 2800 D. Rs. 3200
E. None of these

105. Monthly savings of Vineet and Ryan are in the ratio of 2 : 3 respectively. Monthly salary of
Vineet is 5 times the monthly savings of Ryan , and monthly salary of Ryan is Rs. 2000
more than the monthly salary of Vineet. If monthly expenditure of Vineet is Rs. 15600 then
find the monthly salary of Ryan.
िवनीत और रयान की मािसक बचत मशः 2 : 3 के अनुपात म है । िवनीत का मािसक वेतन, रयान की
मािसक बचत का 5 गुना है , और रयान का मािसक वेतन, िवनीत के मािसक वेतन से 2000 अिधक ह। यिद
िवनीत का मािसक खच 15600 ह, तो िफर रयान का मािसक वेतन पता कर।
A. Rs. 20,000 B. Rs. 18,000
C. Rs. 16,000 D. Rs. 12,000
E. None of these

106. Present age of Yasin is 15% more than the present age of Upkaar. Difference in the
present ages of Upkaar and Sajal is 10 years. If Yasin is 6 years elder than Upkaar, then
find the present age of Sajal.
यासीन की वतमान आयु उपकार की वतमान आयु से 15% अिधक है । उपकार और सजल की वतमान आयु
म 10 वष का अंतर है । अगर यासीन, उपकार से 6 साल बड़े ह, तो सजल की वतमान उ का पता लगाएं ।
A. 50 years
B. Cannot Be Determined/ िनधा रत नही ं िकया जा सकता
C. 30 years D. 53 years
E. None of these

107. The incomes of Ronit and Ojas are in the ratio of a:6 respectively, and Ronit spends 48% of
his income. If the sum of income of Ronit and Ojas together is Rs. 13420 and savings of
Ronit is Rs. 3172, then find the value of 'a'.
रोिनत और ओजस की आय मशः a : 6 के अनुपात म है , और रोिनत अपनी आय का 48% खच करता है ।
यिद एक साथ रोिनत और ओजस की आय का योग 13420 है और रोिनत की बचत 3172 है , तो िफर 'a'
का मान ात कीिजए।
A. 1
B. Cannot Be Determined/ िनधा रत नही ं िकया जा सकता
C. 5 D. 7
E. None of these

PAGE h ps://t.me/studified h ps://instagram.com/aashisharorasocial?utm_medium=copy_link


Follow
77 Aashish Arora on: h ps://youtube.com/channel/UCYa4_JrOrf8R5Kz2uOtccXQ https://www.facebook.com/aashisharorasocial/
अ ास by Aashish Arora
(SBI/IBPS/RBI/LIC/All other banking and insurance Exams)

108. In an examination Ben got 25% marks and failed by 60 marks while James got 55% marks
and passed by 90 marks. Find the percentage of marks required to pass.
एक परी ा म, बेन ने 25% अंक ा िकए और 60 अंको ं से असफल आ जबिक जे ने 55% अंक ा
िकए और 90 अंको ं से पास आ । पास होने के िलए आव क अंको ं का ितशत ात कीिजए।
A. 42% B. 45%
C. 39% D. 33%
E. 37%

109. Perimeter of a Square field is 48m and the area of Rectangular field is 25% more than the
area of Square field. Find the perimeter of rectangular field if the difference between the
length and breadth of rectangular field is 3m.
एक वग े की प रिध 48 मी है और आयताकार े का े फल वग े के े फल से 25% अिधक है ।
यिद आयताकार े की लंबाई और चौड़ाई के बीच का अंतर 3 मी है । आयताकार े की प रिध ात कर।
A. 54 m B. 72 m
C. 47m D. 64 m
E. None Of These

110. Average of 48 randomly arranged numbers is 65. If the average of first 45 numbers is 62
and the ratio of last three numbers is 2:3:6 respectively, then find the product of the
smallest and largest of the three numbers.
48 अिनयिमत प से व थ सं ाओं का औसत 65 है । यिद पहली 45 सं ाओं का औसत 62 है और
अंितम तीन सं ाओं का अनुपात मशः 2 : 3 : 6 है , तो तीन सं ाओं म से सबसे छोटी और सबसे बड़ी
सं ा का गुणनफल ात कीिजए।
A. 10500 B. 16000
C. 10800 D. 10900
E. None Of These

111. Ratio of the present ages of Tom and Ben is 5:4 respectively. Ratio of age of Tom after 6
years and the age of Ben after 3 years is 7:5 respectively. If the present average age of
Tom, Ben and Ron is 15 years, then find out the present age of Ron.
टॉम और बेन की वतमान आयु का अनुपात मशः 5 : 4 है । 6 वष के बाद टॉम की आयु और 3 वष के बाद बेन
की आयु का अनुपात मशः 7: 5 है । यिद टॉम, बेन और रॉन की वतमान औसत आयु 15 वष है , तो रॉन की
वतमान आयु का पता लगाएं ।
A. 12 years B. 15 years
C. 21 years D. 18 years
E. None Of These

112. Jon and Robb together can complete 70% of a work in 42 days while Robb alone takes
50% more time then Jon and Robb together to complete the whole work. Find the time
taken by Jon alone to complete 80% of the whole work.
जॉन और रॉब एक साथ िमलकर 42 िदनो ं म 70% काम पूरा कर सकते ह, जबिक अकेले रॉब, जॉन और रॉब
एक साथ पूरा काम करने म िजतना समय लेते ह उस से 50% अिधक समय लेता ह। पूरे काम का 80% पूरा
करने के िलए जॉन ारा अकेले िलया गया समय खोज।
A. 82 days B. 172 days
C. 144 days D. 112 days
E. None of these

PAGE h ps://t.me/studified h ps://instagram.com/aashisharorasocial?utm_medium=copy_link


Follow
78 Aashish Arora on: h ps://youtube.com/channel/UCYa4_JrOrf8R5Kz2uOtccXQ https://www.facebook.com/aashisharorasocial/
अ ास by Aashish Arora
(SBI/IBPS/RBI/LIC/All other banking and insurance Exams)

113. Simple interest on a sum of Rs 'X' at a certain rate after 2 years is Rs 3420 and the
compound interest on the same sum, at the same rate (compounded annually ) for the
same time period is Rs 3727.80. Find the desired rate.
2 वष के बाद एक िनि त दर पर 'X' का साधारण ाज 3420 पये है और उसी समयाविध पर च वृ
ाज (सालाना आधार पर) 3727.80 पये है । वां िछत दर ात कीिजए।
A. 16% B. 20%
C. 12% D. 18%
E. None of these

114. Varsha and Nandini entered into a business with initial investment of Rs 6960 and Rs 3750
respectively. The ratio of the time for which they made their investments is 5:8
respectively. If profit share of Nandini is Rs 3250, then find out profit earned by Varsha and
Nandini together.
वषा और नंिदनी ने मशः 6960 पये और 3750 पये के शु आती िनवेश के साथ एक वसाय म वेश
िकया। िजस समय के िलए उ ोन
ं े अपना िनवेश िकया है , उसका अनुपात मशः 5 : 8 है । यिद नंिदनी का
लाभ िह ा 3250 पये है , तो वषा और नंिदनी ारा अिजत लाभ का पता लगाएं ।
A. Rs 7020 B. Rs 6720
C. Rs 3770 D. Rs 3580
E. None Of These

115. Ratio of speed of a boat in still water and the speed of stream is 11:3 respectively. If time
taken by a boat to go 112 km upstream and 168 km downstream together is 6 hours 30
minutes. Find the time taken by the boat to go 132 km in still water and 224 km
downstream together.
थर पानी म एक नाव की गित और धारा की गित का अनुपात मशः 11: 3 है । यिद एक नाव ारा एक साथ
112 िकमी धारा के िव जाने तथा 168 िकमी धारा के अनुकूल जाने के िलए 6 घंटे 30 िमनट का समय
लगता है । नाव ारा थर पानी म 132 िकमी और 224 िकमी धारा के अनुकूल जाने के िलए समय िनकाल।
A. 8 hours B. 9 hours 30 minutes
C. 7 hours D. 11 hours 30 minutes
E. None of these

116. Shatabdi Express running with a speed of 108km/h crosses a pole in 8.5 seconds. If the
time taken by Shatabdi Express to cross Rajdhani Express which is running with a speed
of 72km/h and coming from opposite direction is 10.5 seconds, then find the length of
Rajdhani Express.
108 िकमी / घंटा की गित से दौड़ने वाली शता ी ए ेस 8.5 सेकंड म एक पोल को पार करती है । यिद
राजधानी ए ेस जो की 72 िकमी / घंटा की गित से चल रही है , और िवपरीत िदशा से आ रही है , शता ी
ए ेस को पार करने म 10.5 सेकंड लगाती है , तो राजधानी ए ेस की लंबाई ात कीिजए।
A. 255 m B. 235 m
C. 270 m D. 295 m
E. None Of These

117. A mixture of oil and alcohol is mixed in the ratio 7:9 respectively. If 384 ml of mixture is
taken out and replaced with 72 ml of oil, then in final mixture the quantity of oil becomes 24
ml more than that of alcohol. Find the original quantity of the mixture.
तेल और इथेनॉल का िम ण मशः 7 : 9 के अनुपात म िमलाया जाता है । यिद 384 िमलीलीटर िम ण को

PAGE h ps://t.me/studified h ps://instagram.com/aashisharorasocial?utm_medium=copy_link


Follow
79 Aashish Arora on: h ps://youtube.com/channel/UCYa4_JrOrf8R5Kz2uOtccXQ https://www.facebook.com/aashisharorasocial/
अ ास by Aashish Arora
(SBI/IBPS/RBI/LIC/All other banking and insurance Exams)

बाहर िनकाला जाता है और 72 िमलीलीटर तेल के साथ ित थािपत िकया जाता है , तो अंितम िम ण म
इथेनॉल की तुलना म तेल की मा ा 24 िमलीलीटर अिधक हो जाती है । िम ण की मूल मा ा का पता लगाएं ।
A. 748 ml B. 768 ml
C. 792 ml D. 775 ml
E. None of these

118. The outer surface area of cube 'S' which is open from one side is 720 cm². Find the curved
surface area of cube 'T' whose edge is 3cm more than that of cube 'S'.
ूब 'S' का बाहरी सतह े जो एक तरफ से खुला है 720 वग सेमी है । ूब 'T 'के घुमावदार सतह े का
पता लगाएं िजसकी धार ूब 'S' से 3 सेमी अिधक है ।
A. 1350 cm² B. 1125 cm²
C. 900 cm² D. 825 cm²
E. 1050 cm²

119. In a box there are 30 red balls and rest are yellow balls. The probability of picking a yellow
ball at random is ¼. Find the probability of picking a red and a yellow ball.
एक बॉ म, 30 लाल गद ह और बाकी पीले रं ग की गद ह। अिनयिमत तौर पर एक पीले रं ग की गद लेने की
संभावना 1/4 है । लाल और पीले रं ग की गद को चुनने की संभावना का पता लगाएं ।
A. 25/39 B. 5/13
C. 9/26 D. 77/156
E. None of these

120. Jim is running with a speed of (x + 6) km/h and covers a certain distance in 7 hours. If his
speed had been (2x - 8) km/h, then he would have taken 1.5 hours more to cover the same
distance. Find the distance he ran.
िजम (x + 6) िकमी / घंटा की गित से चल रहा है और 7 घंटे म एक िनि त दू री तय करता है । यिद उसकी गित
(2x - 8) िकमी / घंटा होती, तो उसे समान दू री तय करने म 1.5 घंटे अिधक लगते। वह दू री खोज जो वह चला
रहा था।
A. 107 B. 131
C. 142 D. 119
E. None of these

121. Jeetu earned an average of Rs 5200 per month from January through May. Then he
earned Rs 6600 , Rs 6700, Rs 6800, Rs 6900 during the months June through September.
During October he earned 50% of what he earned in November and during December he
earned 200% of what he earned in November. If he averaged Rs 7100 per month for the
entire year, them find his earnings during the month of November.
जीतू ने जनवरी से मई तक औसतन 5200 पये ित माह कमाए। िफर उसने जून से िसतंबर के महीनो ं के
दौरान 6600 पये, 6700 पये, 6800 पये, 6900 पये कमाए। अ ू बर के दौरान उसने नवंबर म जो
कमाया उसका 50% िह ा कमाया और िदसंबर के दौरान उसने नवंबर म जो कमाया उसका 200% िह ा
कमाया। यिद वह पूरे वष के िलए ित माह औसतन 7100 पये कमाता है , तो नवंबर के महीने के दौरान
उसकी कमाई िकतनी ह।
A. Rs 9200 B. Rs 7600
C. Rs 10200 D. Rs 6400
E. None of these

PAGE h ps://t.me/studified h ps://instagram.com/aashisharorasocial?utm_medium=copy_link


Follow
80 Aashish Arora on: h ps://youtube.com/channel/UCYa4_JrOrf8R5Kz2uOtccXQ https://www.facebook.com/aashisharorasocial/
अ ास by Aashish Arora
(SBI/IBPS/RBI/LIC/All other banking and insurance Exams)

122. The average age of employees of a company was 46 years, when 6 new employees with
an average age of 43 years were hired, the average age went down to 44 years. Find the
initial number of employees in the company.
िकसी कंपनी के कमचा रयो ं की औसत आयु 46 वष थी, जब 43 वष की औसत आयु वाले 6 नए कमचा रयो ं
को काम पर रखा गया, औसत आयु घटकर 44 वष हो गई। कंपनी म कमचा रयो ं की ारं िभक सं ा ात
कीिजए।
A. 4 B. 3
C. 5 D. 7
E. None Of These

123. Two vessels contain milk solutions, with milk and water in the ratio of 3:11 in the first vessel
and 7:12 in the second vessel. In what ratio should the contents of these two vessels be
mixed such that the resultant mixture has milk and water in the ratio 3:7 ?
दो बतन म दू ध का िम ण है , पहले बतन म दू ध और पानी मशः 3 : 11 और दू सरे बतन म दू ध और पानी
मशः 7 : 12 के अनुपात म है । इन दो बतन की साम ी को िकस अनुपात म िमि त िकया जाना चािहए तािक
प रणामी िम ण म दू ध और पानी का अनुपात 3 : 7 हो।
A. 13:16 B. 83:111
C. 91:114 D. 7:11
E. None Of These

124. A vessel contains 12 liters of a mixture of milk and water containing 70% milk. 3 liters of
pure milk is added to it. Find the percentage of milk in the new mixture.
एक बतन म 12 लीटर दू ध और पानी का िम ण है िजसम 70% दू ध है । इसम 3 लीटर शु दू ध िमलाया जाता
है । नए िम ण म दू ध का ितशत ात कर।
A. 82% B. 76%
C. 72% D. 80%
E. None Of These

125. The present age of 'P' is 28 years which is 10 years less than the age of 'Q' 3 years hence
from now. The ratio of the present ages of 'Q' and 'R' is 7:4 respectively. If the age of 'S' 3
years hence from now is 25% more than the present age of 'R', then find the present age of
'S'.
'P' की वतमान आयु 28 वष है , जो 3 वष बाद 'Q' की आयु से 10 वष कम है । Q और R की वतमान आयु का
अनुपात मशः 7 : 4 है । यिद 3 वष बाद 'S' की आयु 'R' की वतमान आयु से 25% अिधक है , तो 'S' की
वतमान आयु को ात कर।
A. 22 years B. 26 years
C. 23 years D. 20 years
E. 18 years

126. Perimeter of a square field is 72 m and the area of a rectangular field is 22.22% more than
the area of square field. Find the perimeter of the rectangular field if the difference
between the length and the breadth of rectangular field is 4m.
एक वग े की प रिध 72 मीटर है और एक आयताकार े का े फल वग े के े फल से 22.22%
अिधक है । आयताकार े की प रिध ात कर यिद आयताकार े की लंबाई और चौड़ाई के बीच का अंतर
4 मी है ।

PAGE h ps://t.me/studified h ps://instagram.com/aashisharorasocial?utm_medium=copy_link


Follow
82 Aashish Arora on: h ps://youtube.com/channel/UCYa4_JrOrf8R5Kz2uOtccXQ https://www.facebook.com/aashisharorasocial/
अ ास by Aashish Arora
(SBI/IBPS/RBI/LIC/All other banking and insurance Exams)

A. 82 m B. 72 m
C. 88 m D. 78 m
E. 80 m

127. 30 men can complete a work in 16 days. 9 men started the same work and worked for 20
days, and then they were joined by 'Y' more men. Together they completed the remaining
work in 20 more days. Find the value of 'Y'.
30 आदमी 16 िदनो ं म एक काम पूरा कर सकते ह। 9 आदिमयो ं ने एक काम शु िकया और 20 िदनो ं तक
काम िकया, िफर 'Y' और अिधक आदमी उनके साथ शािमल हो गए। सभी ने िमलकर शेष काम को 20 और
िदनो ं म पूरा िकया। Y का मान ात कीिजए।
A. 5 B. 6
C. 4 D. 8
E. 7

128. Jamie and Ned invested Rs 6500 and Rs 5000 respectively in a business together. After 4
months, Jamie withdrew 20% of his initial investment. If the total profit received by them
after 1 year is Rs 12760, then find the profit received by Jamie.
जेमी और नेड ने एक वसाय म मशः पये 6500 और पये 5000 का िनवेश िकया। 4 महीने के बाद,
जेमी ने अपने शु आती िनवेश का 20% वापस ले िलया। यिद 1 वष के बाद उनके ारा ा कुल लाभ
12760 पये है , तो जेमी ारा ा लाभ पता लगाएं ।
A. Rs 5370 B. Rs 6000
C. Rs 6760 D. Rs 6420
E. Rs 5840

129. Item 'X' is sold at 25% profit. Rs 360 is added in the sum received from selling item 'X' and
then item 'Y' is purchased from the new amount. If item 'Y' is sold at 20% loss, then find the
overall profit or loss of the seller.
व ु 'X' को 25% लाभ पर बेचा जाता है । व ु 'X' को बेचने से ा रािश म 360 पये जोड़े जाते ह और िफर
व ु 'Y' को नई रािश से खरीदा जाता है । यिद व ु 'Y' को 20% हािन पर बेचा जाता है , तो िव े ता के सम
लाभ या हािन का पता लगाएं ।
A. Rs 66 B. Rs 84
C. Rs 96 D. Rs 72
E. Rs 90

130. The income of Prakash is 25% more than that of Sumeet. Prakash spends 72% of his
income. Prakash saves Rs 2200 less than Sumeet. If Summet saves 45% of his income,
them find out his expenditure.
काश की आय सुमीत की तुलना म 25% अिधक है । काश अपनी आय का 72% खच करते ह। काश
सुमीत से 2200 पये कम बचाता है । यिद सुमीत अपनी आय का 45% बचाता है , तो उसका खच पता कर।
A. Rs 12770 B. Rs 12100
C. Rs 11500 D. Rs 14250
E. Rs 13420

131. A box contains 3 blue balls, 3 red balls and 4 yellow balls. 2 balls are drawn at random. Find
the probability that none of them is yellow ball.
एक बॉ म 3 नीली गद, 3 लाल गद और 4 पीली गद ह। 2 गदो ं को अिनयिमत प से िनकाला गया है । इस

PAGE h ps://t.me/studified h ps://instagram.com/aashisharorasocial?utm_medium=copy_link


Follow
83 Aashish Arora on: h ps://youtube.com/channel/UCYa4_JrOrf8R5Kz2uOtccXQ https://www.facebook.com/aashisharorasocial/
अ ास by Aashish Arora
(SBI/IBPS/RBI/LIC/All other banking and insurance Exams)

संभावना को खोज िक उनम से कोई भी पीली गद नही ं है।


A. 1/3 B. 2/7
C. 5/8 D. 1/4
E. Cannot be Determined

132. The sum of length and height of a cuboid is 21 cm. If the breadth of the cuboid is 5 cm and
its volume is 540 cm3 then the length of the cuboid is how much percent more than its
height. (Length of the cuboid is more than its height.)
एक घनाभ की लंबाई और ऊंचाई का योग 21 सेमी है । यिद घनाभ की चौड़ाई 5 सेमी है और इसकी आयतन
540 घन सेमी है तो घनाभ की लंबाई उसकी ऊँचाई से िकतने ितशत अिधक है । (घनाभ की लंबाई इसकी
ऊंचाई से अिधक है ।)
A. 27.5% B. 25%
C. 22.22% D. 33.33%
E. None of these

133. The interest earned for an amount invested at 22.5% per annum simple interest for 4 years
is Rs 1800. The same initial amount when invested at (R + 9)% per annum compound
interest (compounded annually) amount to Rs 3380 at the end of 2 years. Find the value of
'R'.
4 साल के िलए 22.5% ित वष साधारण ाज पर िनवेश की गई रािश का ाज 1800 पये है । जब वही
ारं िभक रािश (R + 9)% ितवष च वृ ाज (च वृ वािषक) पर िनवेश की जाती है , तो यह 2 साल के
अंत म 3380 पये हो जाती है । 'R' का मान ात कर।
A. 23 B. 18
C. 21 D. 19
E. 17

134. A mixture contains paint and water in the ratio of 13 : 15 respectively. If 196 ml of mixture is
taken out, and 18 ml of paint and 23 ml of water is added in the remaining mixture such that
the ratio of water to paint in the resultant mixture becomes 7 : 6 respectively. Find the initial
quantity of the mixture.
एक िम ण म मशः 13 : 15 के अनुपात म पट और पानी है । यिद िम ण का 196 िमलीलीटर बाहर िनकाला
जाता है , और शेष िम ण म 18 िमलीलीटर पट और 23 िमलीलीटर पानी डाला जाता है , जैसे िक प रणामी
िम ण म पानी और पट का अनुपात मशः 7 : 6 हो जाता है । िम ण की ारं िभक मा ा का पता लगाएं ।
A. 526 ml B. 548 ml
C. 564 ml D. 532 ml
E. None of these

135. If Shilpa, Sunil and Lakshit can complete a piece of work in 'x' days, (x + 9) days and (x +
16) days respectively then find the value of x if the efficiency of Shilpa is as much as the
efficiency of Sunil and Lakshit together.
यिद िश ा, सुनील और लि त मशः 'x' िदनो,ं (x + 9) िदनो ं और (x + 16) िदनो ं म एक काम पूरा कर सकते
ह, तो 'x' का मान ात कर यिद िश ा की द ता उतनी ही है िजतनी एक साथ सुनील और लि त की द ता
ह।
A. 13 B. 12
C. 11 D. 14
E. None Of These

PAGE h ps://t.me/studified h ps://instagram.com/aashisharorasocial?utm_medium=copy_link


Follow
84 Aashish Arora on: h ps://youtube.com/channel/UCYa4_JrOrf8R5Kz2uOtccXQ https://www.facebook.com/aashisharorasocial/
अ ास by Aashish Arora
(SBI/IBPS/RBI/LIC/All other banking and insurance Exams)

136. A shopkeeper sold 4 tables and 7 chairs for Rs. 5400. He sold the table at a profit of 25%
and chair at a profit of 33.33% and thus makes a profit of Rs. 1220. Find the cost price of 2
chairs and 3 tables.
एक दु कानदार ने 4 टे बल और 7 कुिसयाँ 5400 म बेची।ं उसने टे बल को 25% के लाभ पर और कुस को
33.33% के लाभ पर बेचा और इस कार उसने 1220 का लाभ कमाया। 2 कुिसयो ं और 3 टे बल की
लागत मू ात कीिजए।
A. Rs. 3280 B. Rs. 1880
C. Rs. 2160 D. Rs. 4420
E. None Of These

137. Ratio of age of Gambhir and Ganguly 10 years ago was 5 : 4 respectively. After 8 years
from now, the ratio of age of Gambhir and Ganguly will become 7 : 6 respectively. Find the
sum of present ages of Gambhir and Ganguly.
10 साल पहले गंभीर और गां गुली की आयु का अनुपात मशः 5 : 4 था। अब से 8 साल बाद, गंभीर और
गां गुली की आयु का अनुपात मशः 7 : 6 हो जाएगा। गंभीर और गां गुली की वतमान आयु का योग ात
कीिजए।
A. 99 B. 101
C. 100 D. 98
E. None Of These

138. A sector making an angle of 200° at the center is cut out from a circle of diameter 378 mm,
and the remaining circle is converted into a cone. Find the radius of the cone thus formed.
क म 200° का कोण बनाने वाले े को 378 िममी ास के एक वृ से काट िदया जाता है , और शेष वृ को
शंकु म बदल िदया जाता है । इस कार बने शंकु की ि ा ात कीिजए।
A. 84 mm B. 80 mm
C. 72 mm D. 64 mm
E. None of these

139. A certain amount of sum at a rate of R % per annum compounded annually becomes Rs.
1665 in 4 years and Rs. 2775 in 8 years. Find the certain amount of sum.
एक िनि त रािश R% सालाना च वृ ाज की दर से 4 साल म 1665 हो जाती है और 8 साल म
2775 हो जाती है । वह िनि त रािश ात कीिजए।
A. Rs. 999 B. Rs. 990
C. Rs. 909 D. Rs. 900
E. None of these

140. The marked price of an article is Rs. 8125, and a retailer sold it after two successive
discounts of x % and (x + 8) % respectively for Rs. 5187. Find the value of x.
एक व ु की िचि त कीमत 8125 है , और एक फुटकर ापारी ने मशः x % और (x + 8) % के दो
िमक छूट के बाद इसे 5187 मे बेचा। x का मान ात कीिजए।
A. 14 B. 12
C. 16 D. 18
E. None Of These

141. Ikka and Diljit entered into a business with initial investments of Rs. 1419 and Rs. 1677
respectively, and the ratio of time period for which they made their investment is 3 : 2

PAGE h ps://t.me/studified h ps://instagram.com/aashisharorasocial?utm_medium=copy_link


Follow
85 Aashish Arora on: h ps://youtube.com/channel/UCYa4_JrOrf8R5Kz2uOtccXQ https://www.facebook.com/aashisharorasocial/
अ ास by Aashish Arora
(SBI/IBPS/RBI/LIC/All other banking and insurance Exams)

respectively. Find the profit share of Diljit out of total profit of Rs. 4543.
इ ा और िदलजीत ने मशः 1419 और 1677 के शु आती िनवेश के साथ एक वसाय म वेश
िकया और िजस अविध के िलए उ ोन
ं े अपना िनवेश िकया था, उस समय का अनुपात मशः 3 : 2 है ।
4543 के कुल लाभ म से िदलजीत का लाभ िह ा ात करे ।
A. Rs. 2541 B. Rs. 2718
C. Rs. 2371 D. Rs. 2002
E. None of these

142. A boat covers a distance of 704 km while going upstream in 22 hours, and the same
distance in 960 minutes while going downstream. Find the speed of boat in still water.
एक नाव 704 िकमी दू री धारा के िव 22 घंटे म तय करती है , और वही दू री धारा के अनुकूल 960 िमनट म
तय करती है । थर पानी म नाव की गित का पता लगाएं ।
A. 44 km/hr B. 32 km/hr
C. 8 km/hr D. 38 km/hr
E. None Of These

143. A piece of work is completed in 12 days, by 4 men working 8 hours per day. How much time
will be taken by 6 men working 7 hours a day to finish the work. 2 men of the former type
work as 1 men of latter type.
एक काम 12 िदनो ं म पूरा होता है यदी 4 पु ष ित िदन 8 घंटे काम करते ह। काम पूरा करने के िलए 7 घंटे
काम करने वाले 6 पु षो ं ारा िकतना समय िलया जाएगा। पूव कार के 2 पु ष, बाद के कार के 1 पु ष के
समान काम करते ह।
A. 4.571 days B. 5.412 days
C. 5.982 days D. 4.345 days
E. None of these

144. At present, ratio of ages of Kukku and Nikku are in the ratio 4:3 respectively. 7 years later
the ratio of their age will become 5:4. What is the ratio of their ages 21 years later from
present.
वतमान म, कु ू और िन ू की आयु का अनुपात मशः 4: 3 के अनुपात म है । 7 साल बाद उनकी उ का
अनुपात 5: 4 हो जाएगा। वतमान से 21 वष बाद उनकी आयु का अनुपात ा है ।
A. 5:3 B. 9:7
C. 7:6 D. 8:7
E. None of these

145. Rahul started a bakery business with investing Rs. 200000. Seeing his business
flourishing his cousin Mukul after 4 months invested Rs. 250000 in the same business with
him. Nakul also joined 2 months later than Mukul investing Rs. 400000. As the end of the
year, total profit from the business was Rs. 340000. Calculate the share of profit of Mukul.
रा ल ने 200000 पये के िनवेश के साथ एक बेकरी वसाय शु िकया। वसाय को फलता-फूलता
दे ख, रा ल के चचेरे भाई मुकुल ने उसी वसाय म 250000 पये का िनवेश िकया। 2 महीने बाद नकुल ने
भी 400000 पये का िनवेश िदया। वष के अंत तक, वसाय से कुल लाभ 340000 पये था। मुकुल के
लाभ के िह े की गणना कर।
A. Rs. 95000 B. Rs. 100000
C. Rs. 105000 D. Rs. 110000
E. None of these

PAGE h ps://t.me/studified h ps://instagram.com/aashisharorasocial?utm_medium=copy_link


Follow
86 Aashish Arora on: h ps://youtube.com/channel/UCYa4_JrOrf8R5Kz2uOtccXQ https://www.facebook.com/aashisharorasocial/
अ ास by Aashish Arora
(SBI/IBPS/RBI/LIC/All other banking and insurance Exams)

146. A person was offered 4 successive discounts of 30%, 30%, 25% & 20% or 50%, 20%,
10% & 5%. What is the differences in the discounts offered if the list price of the object is
Rs. 45000.
एक को 30%, 30%, 25% और 20% या 50%, 20%, 10% और 5% की 4 िमक छूट की
पेशकश की गई थी। यिद व ु की सूची मू 45000 पये है तो छूट म ा अंतर है ।
A. Rs. 3130 B. Rs. 2160
C. Rs. 1980 D. Rs. 2070
E. None of these

147. An amount increases by 72% in 6 years invested at a simple interest. How much will be an
amount of Rs. 72000 become in 2 years at the same compound interest rate .
एक रािश म साधारण ाज पर 6 साल म 72% की वृ होती है । इस ही च वृ ाज दर पर 2 साल म
72000 पये की रािश िकतनी हो जाएगी।
A. Rs. 90320 B. Rs. 90316
C. Rs. 90388 D. Rs. 92584
E. None of these

148. Shashank is travelling to Jaipur. If he moves 10 Kmph faster, he saves 60 minutes of his
time. Had he been moving 5 Kmph slower, he would have taken 90 minutes more. What is
the total distance he is travelling.
शशां क जयपुर की या ा कर रहे ह। यिद वह 10 िकमी ित घंटे से अपनी गित बढ़ाता, तो वह अपने समय के
60 िमनट बचाता है । अगर वह 5 िकलोमीटर ित घंटे से अपनी गित धीमी करता ह, तो उसे 90 िमनट और
लगते। वह िकतनी दू री की या ा कर रहा है ।
A. 27.325 km B. 31.925 km
C. 28.125 km D. 29.55 km
E. None of these

149. A boat takes 540 minutes to cover a certain distance upstream & takes 420 minutes to
cover the same distance downstream. Calculate the ratio of the speed of stream and the
speed of the boat.
एक नाव धारा के िव एक िनि त दू री 540 िमनट म तय करती है और धारा के अनुकूल समान दू री 420
िमनट म तय करती है । धारा की गित और नाव की गित के अनुपात की गणना कर।
A. 8:1 B. 1:8
C. 1:4 D. 4:1
E. None of these

150. Raman works as an sales officer. For every sale he does, he gets 3% commission on the
sales amount. If his monthly sales exceeds Rs 300000, he gets additional bonus
commission of 2% on sales exceeding Rs. 300000. He receives total commission for
month of February as Rs. 21000. What is the additional bonus amount earned by him.
रमन एक से ऑिफसर के प म काम करता है । ेक िब ी के िलए, वह िब ी रािश पर 3% कमीशन
ा करता है । यिद उसकी मािसक िब ी 300000 पये से अिधक है , तो उसे 300000 पये से अिधक की
िब ी पर 2% का अित र बोनस कमीशन िमलता है । वह 21000 पये फरवरी महीने के िलए कुल
कमीशन के प म ा करता है । उसके ारा अिजत अित र बोनस रािश ा है ।
A. Rs. 8000 B. Rs. 6000
C. Rs. 4800 D. Rs. 7500
E. None of these

PAGE h ps://t.me/studified h ps://instagram.com/aashisharorasocial?utm_medium=copy_link


Follow
87 Aashish Arora on: h ps://youtube.com/channel/UCYa4_JrOrf8R5Kz2uOtccXQ https://www.facebook.com/aashisharorasocial/
अ ास by Aashish Arora
(SBI/IBPS/RBI/LIC/All other banking and insurance Exams)

151. A Solid metallic hemisphere of radius 4 cm is melted & recasted into right circular cone of
base radius 3 cm. Find the height of cone.
ि ा 4 सेमी का एक ठोस धा क गोलाध िपघल जाता है और आधार ि ा 3 सेमी के सही प रप शंकु म
बदल जाता है । शंकु की ऊंचाई ात कीिजए।
A. 12.22 B. 13.45
C. 14.22 D. 15.56
E. None of these

152. To reach a certain destination, Anuj has to travel via road and ship. 40% of the distance he
needs to travel is a speedway built of tar, 20% of the road he travels is a broken road and
the remaining distance is to be covered by boat. His speed of traveling on broken road,
boat & tar road is in the ratio 2:3:4 respectively. Calculate the ratio of time Anuj travels on
the broken road, boat, and tar road.
एक िनि त गंत तक प ं चने के िलए, अनुज को सड़क और जहाज के मा म से या ा करनी पड़ता है । उसे
िजतनी दू री तय करनी है उसका 40% टार से बना एक ीडवे है , 20% टू टी सड़क है और शेष दू री को नाव
से कवर करना है । टू टी सड़क, नाव और टार रोड पर या ा करने की उनकी गित मश: 2: 3: 4 के अनुपात
म है । अनुज ने समय के िकस अनुपात म टू टी सड़क, नाव और टार रोड पर या ा की ?
A. 28 : 5 : 49 B. 30 : 40 : 30
C. 4 : 8 : 3 D. 45: 12 : 45
E. None of these

153. Devansh and Dhruv together can complete a piece of work in 15 days while Dhruv and
Dharmendra can do the same work in 18 days. If Dhruv alone takes 24 days to complete
the work, then find the time take by all the three together to complete 72.5 % of the work.
दे वां श और ुव िमलकर 15 िदनो ं म एक काम पूरा कर सकते ह जबिक ुव और धम वही काम 18 िदनो ं म
कर सकते ह। अगर ुव को अकेले काम पूरा करने म 24 िदन लगते ह, तो 72.5% काम पूरा करने के िलए
तीनो ं को एक साथ लगने वाला समय ात करे ।
A. 13 days B. 11 days
C. 8 days D. 9 days
E. None of these

154. P, Q and R invested Rs. 2500, Rs. 2800 and Rs. 3000 respectively in a business. After 17
months they increased their investments by 29 %. Then after 19 months they decreased
their investments by 23 %. If after 37 months total profit received by them is Rs. 13,695,
then find the difference between the profits received by P and R.
P, Q और R ने मशः 2500, 2800 और 3000 एक वसाय म िनवेश िकए। 17 महीनो ं के बाद
उ ोन ं े अपने िनवेश म 29% की वृ की। िफर 19 महीने बाद उ ोन ं े अपने िनवेश म 23% की कमी की।
यिद 37 महीनो ं के बाद उनके ारा ा कुल लाभ 13,695 है , तो िफर P और R ारा ा लाभ के बीच
का अंतर ात कर।
A. Rs. 725 B. Rs. 825
C. Rs. 775 D. Rs. 875
E. None Of These

155. A bike runs with a speed of 45 km/hr for (x + 7.5) hours, and then with 20% more speed for
(2x – 1) hours. If the distance covered in (x + 7.5) hours is 45 km more than the distance
covered in (2x – 1) hours, then find the value of 'x'.

PAGE h ps://t.me/studified h ps://instagram.com/aashisharorasocial?utm_medium=copy_link


Follow
88 Aashish Arora on: h ps://youtube.com/channel/UCYa4_JrOrf8R5Kz2uOtccXQ https://www.facebook.com/aashisharorasocial/
अ ास by Aashish Arora
(SBI/IBPS/RBI/LIC/All other banking and insurance Exams)

एक बाइक 45 िकमी / घंटा की गित के साथ (x + 7.5) घंटे चलती है , और िफर 20% अिधक गित के साथ (2x
– 1) घंटे चलती है । यिद (x + 7.5) घंटे म तय की गई दू री, (2x – 1) घंटे म तय की गई दू री से 45 िकमी अिधक
है , तो 'x' का मान ात कर।
A. 6.5 B. 5
C. 5.5 D. 6
E. None Of These

156. Rs. 2850 is invested at 36% simple interest for 3 years. The amount received after 3 years
is then invested at 20% per annum simple interest for 5 years. Find the interest received
from second investment.
2850 को 36% ित वष साधारण ाज पर 3 साल के िलए िनवेश िकया जाता है । 3 साल के बाद ा कुल
रािश को 5 साल के िलए 20% ित वष साधारण ाज पर िनवेश िकया जाता है । दू सरे िनवेश से ा ाज
का पता लगाएं ।
A. Rs. 4728 B. Rs. 5928
C. Rs. 3938 D. Rs. 4938
E. None Of These

157. The average marks obtained by 12 students in Social Science is 48. Later on it was found
that marks of two of the students were read as 34 and 44 in place of 43 and 71 respectively.
Find the correct average marks of 12 students.
सामािजक िव ान म 12 छा ो ं ारा ा िकए गए औसत अंक 48 ह। बाद म यह पाया गया िक दो छा ो ं के
अंको ं को मशः 43 और 71 के बजाय 34 और 44 के प म पढ़ा गया था। 12 छा ो ं के सही औसत अंक
ा कर।
A. 51 B. 49
C. 50 D. 52
E. None of these

158. 44 kg of Tata salt is purchased for Rs. 9 per kg while 28 kg of RMD salt is purchased for Rs.
5 per kg. If the mixture of both types of salt is sold at Rs. 10 per kg, then find the profit or
loss incurred to the seller.
44 िक ा टाटा नमक 9 ित िकलो म खरीदा जाता है । जबिक 28 िकलो ाम RMD नमक 5 ित िकलो म
खरीदा जाता है । यिद दोनो ं कार के नमक का िम ण 10 ित िकलो म बेचा जाता है , तो िफर िव े ता को
होने वाला मुनाफा या नुकसान का पता लगाएं ।
A. Rs. 180 B. Rs. 184
C. Rs. 212 D. Rs. 248
E. None of these

159. The ratio of the present ages of Ankur and Shanti is 4 : 7 respectively. The ratio of the age
of Ankur 4 years ago from now to the present age of Ujala is 3 : 4 respectively. If the sum of
present ages of all the three is 109 years, then find the present age of Shanti.
अंकुर और शां ित की वतमान आयु मशः 4 : 7 है । अब से 4 वष पहले अंकुर की आयु और उजाला की
वतमान आयु का अनुपात मशः 3 : 4 है । यिद तीनो ं की वतमान आयु का योग 109 वष है , तो शां ित की
वतमान आयु ात कीिजए।
A. 32 years B. 28 years
C. 49 years D. 30 years
E. None Of These

PAGE h ps://t.me/studified h ps://instagram.com/aashisharorasocial?utm_medium=copy_link


Follow
89 Aashish Arora on: h ps://youtube.com/channel/UCYa4_JrOrf8R5Kz2uOtccXQ https://www.facebook.com/aashisharorasocial/
अ ास by Aashish Arora
(SBI/IBPS/RBI/LIC/All other banking and insurance Exams)

160. The ratio of tigers and lions in a wildlife sanctuary is 7 : 11 respectively. If 17.5% tigers and
25% lions died, then number of tigers that are alive is what percent of number of lions that
are alive ?
एक व जीव अभयार म बाघो ं और शेरो ं का अनुपात मशः 7 : 11 है । अगर 17.5% बाघ और 25% शेर
मर गए, तो बाघो ं की सं ा जो जीिवत ह, जीिवत शेरो ं की सं ा का िकतना ितशत ह?
A. 94% B. 85%
C. 74% D. 70%
E. None of these

161. A mixture contains 55% sugarcane juice and rest water. 7 ml of water is added to the
mixture such that the quantity of sugarcane juice and water becomes equal. Find the initial
quantity of the mixture.
एक िम ण म 55% ग े का रस और बाकी पानी है । िम ण म 7 िमलीलीटर पानी डाला जाता है तािक ग े के
रस और पानी की मा ा बराबर हो जाए। िम ण की ारं िभक मा ा का पता लगाएं ।
A. 77 ml B. 84 ml
C. 91 ml D. 70 ml
E. None Of These

162. There are 30 nails (2 cm long and 3 cm long) in a box. The probability of picking a 3 cm long
nail is 3/5. Find the probability of picking two nails, that are 2 cm long, from the box.
एक ड े म 30 कील (2 सेमी लंबी और 3 सेमी लंबी) ह। 3 सेमी लंबी कील को चुनने की संभावना 3/5 है ।
ड े म से दो कील (2 सेमी लंबी) चुनने की संभावना ात करे ।
A. 15/29 B. 44/290
C. 13/29 D. 37/290
E. None Of These

163. An article is sold for Rs. 2691 at 38 % profit. If the article was marked up by 50 % above its
cost price, then find the discount offered.
एक व ु 38% लाभ पर 2691 म बेची जाती है । यिद व ु को इसकी लागत मू से 50% ऊपर िचि त
िकया गया था, तो ािवत छूट को ढू ं ढ।
A. Rs. 354 B. Rs. 234
C. Rs. 324 D. Rs. 274
E. None Of These

164. The ratio of income of Kaalia in the year 2018 and 2019 is 14 : 15 respectively. In 2019,
Kaalia spends 55% of his income and saves Rs. 4320. Find the savings of Kaalia in 2018 if
he spends 70% of his income in that year.
वष 2018 और 2019 म कािलया की आय का अनुपात मशः 14 : 15 है । 2019 म, कािलया ने अपनी आय
का 55% खच िकया और 4320 बचाये। 2018 म कािलया की बचत का पता लगाएं यिद वह उस वष अपनी
आय का 70% खच करता है ।
A. Rs. 2388 B. Rs. 2488
C. Rs. 2688 D. Rs. 2588
E. None Of These

165. A container has only two varieties (A and B) of rice in ratio 7 : 5 respectively. When 8 kg of
rice 'B' is added to the container , the ratio of quantities of rice 'A' to rice 'B' in the container
becomes 11 : 9. Find the capacity of the container if the container becomes full after
adding 8 kg of rice 'B'.

PAGE h ps://t.me/studified h ps://instagram.com/aashisharorasocial?utm_medium=copy_link


Follow
90 Aashish Arora on: h ps://youtube.com/channel/UCYa4_JrOrf8R5Kz2uOtccXQ https://www.facebook.com/aashisharorasocial/
अ ास by Aashish Arora
(SBI/IBPS/RBI/LIC/All other banking and insurance Exams)

एक कंटे नर म चावल की केवल दो िक (ए और बी) मशः 7: 5 अनुपात म ह। जब कंटे नर 'बी' म 8 िकलो


चावल डाला जाता है , तो कंटे नर 'ए' और कंटे नर 'बी' म चावल की मा ा का अनुपात मशः 11: 9 हो जाता
है । यिद कंटे नर 8 िकलो चावल 'बी' को डालने के बाद पूण हो जाता है । कंटे नर की मता का पता लगाएं ।
A. 142 kg B. 140 kg
C. 137 kg D. 133 kg
E. 127 kg

166. 6 men and 10 boys can complete a work in 24 days. If each boy is at least 1/5th as efficient
as each man, but not more efficient than man, then find the maximum and minimum
number of days required by 4 men and 20 boys to complete the same work.
6 पु ष और 10 लड़के 24 िदनो ं म एक काम पूरा कर सकते ह। यिद ेक लड़का ेक आदमी के
मुक़ाबले कम से कम 1/5th कुशल है , लेिकन आदमी की तुलना म अिधक कुशल नही ं है , तो उसी काय को
पूरा करने के िलए 4 पु षो ं और 20 लड़को ं ारा आव क अिधकतम और ूनतम िदनो ं की आव कता है ।
A. 24 and 16 days B. 20 and 15 days
C. 25 and 20 days D. 30 and 24 days
E. None Of These

167. A 5 digit number is formed using the digits 1,2,3,4 and 5. If each digit can be used only
once, then find the probability that the number formed will be divisible by 5 ?
5 अंको ं की सं ा 1,2,3,4 और 5 का उपयोग करके बनाई गई है । यिद ेक अंक को केवल एक बार
उपयोग िकया जा सकता है , तो इस संभावना को ढू ं ढ िक गिठत सं ा 5 से िवभा होगी?
A. 1/6 B. 2/7
C. 1/5 D. 1/4
E. 3/7

168. A girl deposited Rs 'Y' in a bank in the denomination of Rs 500, Rs 200 and Rs 100 only.
The ratio of the number of notes of the given denomination is 1 : 2 : 3 respectively. If the
amount deposited in Rs 100 denomination is Rs 1000 less than the amount deposited in
Rs 500 denomination, then find out the value of 'Y'.
एक लड़की ने पये 'Y' बक म 500 पये, 200 पये और 100 पये के मू वग म जमा िकया। िदए गए
मू वग के नोटो ं की सं ा का अनुपात मशः 1: 2: 3 है । यिद पये 100 के मू वग म जमा की गई रािश
500 के मू वग म जमा की गई रािश से 1000 पये कम है , तो 'Y' का मान ात कीिजए।
A. Rs 4500 B. Rs 5000
C. Rs 5500 D. Rs 7500
E. Rs 6000

169. 'Raju', 'Shyam' and 'Baburao' invested Rs 4000, Rs 3400 and Rs 5000 respectively
together in a business. After 6 months 'Raju' withdrew 20% of his initial investment while
'Shyam' added Rs 600 and 'Baburao' added Rs 1000 to their initial investments. Find out
the ratio of the profit received by 'Raju', 'Shyam' and 'Baburao' respectively at the end of
the year.
'राजू', ' ाम' और 'बाबूराव 'ने मश: 4000 पये, 3400 पये और 5000 पये का िनवेश िकया। 6 महीने
के बाद 'राजू' ने अपने शु आती िनवेश का 20% वापस ले िलया, जबिक ' ाम' ने और 'बाबूराव' ने अपने
शु आती िनवेशो ं म मश: 600 पये और 1000 पये जोड़े । वष के अंत म मशः 'राजू', ' ाम' और
'बाबूराव' ारा ा लाभ का अनुपात ात कीिजए।

PAGE h ps://t.me/studified h ps://instagram.com/aashisharorasocial?utm_medium=copy_link


Follow
91 Aashish Arora on: h ps://youtube.com/channel/UCYa4_JrOrf8R5Kz2uOtccXQ https://www.facebook.com/aashisharorasocial/
अ ास by Aashish Arora
(SBI/IBPS/RBI/LIC/All other banking and insurance Exams)

A. 12 : 15 : 19 B. 36 : 37 : 55
C. 23 : 24 : 47 D. 10 : 11 : 17
E. None of these

170. The ratio of speed of boat in still water and the speed of current is 11 : 7 respectively. The
boat can travel 162 km downstream and 44 km upstream in 4 hours . Find the speed of
boat in still water.
थर पानी म नाव की गित और धारा की गित का अनुपात मशः 11: 7 है । नाव 4 घंटे म 162 िकमी धारा के
अनुकुल और 44 िकमी धारा के िवपरीत या ा कर सकती है । थर पानी म नाव की गित का पता लगाएं ।
A. 42 kmph B. 37 kmph
C. 49 kmph D. 55 kmph
E. 62 kmph

171. Time taken to travel 'X' km at a speed of 20km/h is 2 hours less than the time taken to travel
(3X + 15) km with a speed of 30km/h. Find the time taken to cover (X + 48) km with a
speed of 26 km/h.
20 िकमी / घंटा की गित पर 'X' िकमी की या ा करने म लगने वाला समय 30 िकमी / घंटा की गित के साथ
(3X + 15) िकमी की या ा करने म लगने वाले समय से 2 घंटे कम है । 26 िकमी / घंटा की गित के साथ (X +
48) िकमी की या ा करने के िलए िलया गया समय ात कीिजए।
A. 2.5 hours B. 3.5 hours
C. 3 hours D. 1 hour
E. 1.5 hours

172. The area of a square 'X' is 196 m2 . Find the area of square 'Y' whose each side is 50%
more than that of square 'X'.
एक वग 'X' का े फल 196 वग मी है । वग 'Y' का े फल ात कीिजए िजसका ेक भुजा वग 'X' की
भुजा से 50% अिधक है ।
A. 441 m2 B. 294 m2
C. 306.25 m2 D. 150.06 m2
E. None of these

173. An article 'A' is marked up by 45% above its cost price. If after giving a discount of Rs 120,
the seller earned a profit of 25%, then find the cost price of the article.
एक लेख 'ए' को उसकी लागत मू से 45% ऊपर िचि त िकया गया है । यिद 120 पये की छूट दे ने के बाद,
िव े ता ने 25% का लाभ कमाया, तो लेख की लागत मू ात कर।
A. Rs 600 B. Rs 720
C. Rs 1020 D. Rs 640
E. Rs 800

174. The ratio of ages of 'Dwight' and 'Michael', 8 years ago from now was 4:7 respectively. If
the present age of 'Ryan' is 25% more than the age of 'Dwight' 8 years ago from now and
the present average age of 'Dwight', 'Michael' and 'Ryan' is 48 years, then find present age
of 'Michael'.
अब से 8 साल पहले 'ड् वाइट' और 'माइकल' की आयु का अनुपात, मशः 4: 7 था। यिद 'रे यान' की वतमान
आयु 'ड् वाइट' की 8 वष पहले की आयु से 25% अिधक है और 'ड् वाइट', 'माइकल' और 'रयान' की वतमान
औसत आयु 48 वष है । िफर 'माइकल' की वतमान आयु ात कर।

PAGE h ps://t.me/studified h ps://instagram.com/aashisharorasocial?utm_medium=copy_link


Follow
92 Aashish Arora on: h ps://youtube.com/channel/UCYa4_JrOrf8R5Kz2uOtccXQ https://www.facebook.com/aashisharorasocial/
अ ास by Aashish Arora
(SBI/IBPS/RBI/LIC/All other banking and insurance Exams)

A. 48 years B. 68 years
C. 72 years D. 60 years
E. None of these

175. Find the cost of painting a hemispherical structure along with its base at the rate of Rs
10.5/m2 . The base radius of the dome is 6 metre.
10.5/ वग मीटर की दर से एक अधगोल संरचना को इसके आधार के साथ िचि त करने की लागत का पता
लगाएं । गुंबद का आधार ि ा 6 मीटर है ।
A. Rs 2886 B. Rs 3564
C. Rs 3270 D. Rs 4425
E. Rs 2550

176. A bag contains 30 green balls and 45 yellow balls. Some balls were taken out of the bag
such that the ratio of yellow to green balls in the bag becomes 3 : 2. If the no of yellow balls
drawn is 5 more than that of the green balls, then find the total number of balls drawn from
the bag.
एक बैग म 30 हरी गद और 45 पीले रं ग की गद होती ह। कुछ गदो ं को बैग से बाहर िनकाल िदया गया, जैसे
बैग म पीले से हरे रं ग की गदो ं का अनुपात मशः 3: 2 हो जाता है । यिद खीच
ं ी गई पीली गदो ं की सं ा हरे रं ग
की गदो ं की तुलना म 5 अिधक है , तो बैग से िनकाले गए गदो ं की कुल सं ा का पता लगाएं ।
A. 30 B. 45
C. 25 D. 15
E. 35

177. The ratio of the perimeter of a rectangular and square wall is 5 : 6 respectively. The cost of
painting the square wall at the rate of Rs 5.5/m2 is Rs 19800. If the length of the rectangle
is 55 meter, then find the breadth of the rectangle.
एक आयताकार और वग दीवार की प रिध का अनुपात मशः 5: 6 है । 5.5 / वग मीटर की दर से वग दीवार
को पट करने की लागत 19800 पये है । यिद आयत दीवार की लंबाई 55 मीटर है , तो आयत दीवार की
चौड़ाई का पता लगाएं ।
A. 60 m B. 55 m
C. 50 m D. 45 m
E. 65 m

178. Raghu scored 55 marks in Quants. He scored 60% marks in reasoning and 'X' marks in
English. Maximum marks in each subject were 125. The overall percentage marks
obtained by Raghu in all the three subjects together were 56%. How many marks did he
score in English?
रघु ने ट म 55 अंक हािसल िकए। उ ोन ं े तक म 60% अंक और अं ेजी म 'X' अंक हािसल िकए। ेक
िवषय म अिधकतम अंक 125 थे। तीनो ं िवषयो ं म रघु ारा ा कुल ितशत अंक 56% थे। उ ोन
ं े अं ेजी म
िकतने अंक ा िकए?
A. 85 B. 75
C. 80 D. 65
E. 90

179. A shopkeeper mixed two solutions 'A' and 'B' of milk and water in the ratio of 5 : 7
respectively, than a new solution is obtained in which the concentration of milk was 67%.

PAGE h ps://t.me/studified h ps://instagram.com/aashisharorasocial?utm_medium=copy_link


Follow
93 Aashish Arora on: h ps://youtube.com/channel/UCYa4_JrOrf8R5Kz2uOtccXQ https://www.facebook.com/aashisharorasocial/
अ ास by Aashish Arora
(SBI/IBPS/RBI/LIC/All other banking and insurance Exams)

In 6 liters solution 'A', the quantity of water is 2.4 liters, then in 15 liters solution 'B', what
was the quantity (in ml) of milk ?
एक दु कानदार ने दू ध और पानी के दो समाधान 'ए' और 'बी' को मशः 5: 7 के अनुपात म िमलाया, और एक
नया समाधान बनाया िजसम दू ध की एका ता 67% थी। 6 लीटर समाधान 'ए 'म, पानी की मा ा 2.4 लीटर है ,
िफर 15 लीटर समाधान 'बी' म, दू ध की मा ा (एमएल म) ा होगी ?
A. 11400 ml B. 13600 ml
C. 12080 ml D. 10800 ml
E. 9950 ml

180. A lady bought 65 bottles at Rs 230 each and some glasses at Rs 258 each. If the average
cost of all the glasses and bottels is Rs 245, then how many glasses did she buy ?
एक मिहला ने 230 पये ेक म 65 बोतल और कुछ ास 258 पये ेक म खरीदे । यिद सभी ास
और बोतलो ं की औसत लागत 245 है , तो उसने िकतने िगलास खरीदे ?
A. 75 B. 70
C. 80 D. 73
E. 82

181. A 2 m wide path is carved outside a circular field of circumference 44m. What is the ratio of
the area of the path to the area of the circular field respectively.
एक 2 मीटर चौड़ा रा ा 44 मीटर प रिध के एक गोलाकार े के बाहर उकेरा गया है । मशः प रप पथ
के े और गोलाकार े के े का अनुपात ा है ।
A. 1 : 7 B. 32 : 49
C. 5 : 14 D. 15 : 49
E. 7 : 17

182. Keshav invested two same amounts in two different schemes, one at 50% annual
compound interest for 2 years and other one at 25% simple interest for 'n' years. If the
interest amount is same from both the schemes , then what is the value of 'n' ?
केशव ने दो अलग-अलग योजनाओं म दो समान रािश का िनवेश िकया, एक 50% वािषक च वृ ाज पर
2 साल के िलए और दू सरा 25% साधारण ाज पर 'n' वष के िलए। यिद ाज रािश दोनो ं योजनाओं से
समान है , तो 'n' का मू ा है ?`
A. 7 B. 6
C. 4 D. 5
E. 3

183. Surya and Rohit can complete a certain task in 24 days. Rohit alone can complete the task
in 40 days. They both start working together but Rohit leaves after 6 days. In how many
days did Surya complete the rest of the work?
सूय और रोिहत एक िनि त काय को 24 िदनो ं म पूरा कर सकते ह। अकेले रोिहत 40 िदनो ं म काय पूरा कर
सकते ह। वे दोनो ं एक साथ काम करना शु करते ह लेिकन रोिहत 6 िदनो ं के बाद िनकल जाता है । सूया ने
बाकी काम िकतने िदनो ं म पूरा िकया?
A. 32.5 B. 45
C. 37.33 D. 42.5
E. 47

184. Sansa and Maisie started a business by investing in the ratio 7 : 9 respectively. Maisie left

PAGE h ps://t.me/studified h ps://instagram.com/aashisharorasocial?utm_medium=copy_link


Follow
94 Aashish Arora on: h ps://youtube.com/channel/UCYa4_JrOrf8R5Kz2uOtccXQ https://www.facebook.com/aashisharorasocial/
अ ास by Aashish Arora
(SBI/IBPS/RBI/LIC/All other banking and insurance Exams)

the business at the end of 3 months but joined again 4 months later. At the end of the year,
the total profit received by them was Rs 5850. What is the difference between the profit
received by Sansa and Maisie?
संसा और मैसी ने मशः 7: 9 के अनुपात म िनवेश करके एक वसाय शु िकया। मैसी ने 3 महीने के अंत
म वसाय छोड़ िदया लेिकन 4 महीने बाद िफर से शािमल हो गयी। वष के अंत म, उनके ारा ा कुल
लाभ 5850 पये था। सां सा और मैसी को ा लाभ के बीच अंतर ा है ?
A. Rs 450 B. Rs 475
C. Rs 425 D. Rs 525
E. Rs 390

185. A shopkeeper sold an article at a loss of 30%. If he sold same article for Rs 4800 more than
the previous selling price, then he would have earned 30% profit. What must be the selling
price of the article in order to gain a 25% profit?
एक दु कानदार ने 30% की हािन पर एक लेख बेचा। यिद वह िपछले िब ी मू से 4800 पये अिधक म वही
लेख बेचता है , तो उसने 30% लाभ अिजत िकया । 25% लाभ ा करने के िलए लेख का िव य मू ा
होना चािहए?
A. Rs 8600 B. Rs 9100
C. Rs 10000 D. Rs 11700
E. Rs 10400

186. A boat can cover 91 km in still water in 7 hours and the difference between the upstream
speed and downstream speed of the boat is 4 km/h. In how much time the boat can cover
77 km upstream and 105 km downstream?
एक नाव 7 घंटे म थर पानी म 91 िकमी चल सकती है और नाव की धारा के अनुकूल गित और धारा के
िवपरीत की गित के बीच का अंतर 4 िकमी / घंटा है । नाव िकतने समय म 77 िकमी धारा के िवपरीत और 105
िकमी धारा के अनुकूल को कवर कर सकती है ?
A. 11 hours B. 9.5 hours
C. 13 hours D. 14 hours
E. 16 hours

187. Biswa's age 15 years ago was 50% more than what Sapan's age was 10 years ago and
Sapan's age 16 years from now will be 10/7 times the current age of Biswa. The sum of
their present ages is divisible by ?
15 साल पहले िब ा की उ 10 साल पहले सपन की उ की तुलना म 50% अिधक थी और अभी से 16
साल बाद सपन की उ िबसवा की मौजूदा उ के 10/7 गुना होगी। उनके वतमान उ का योग िकसके ारा
िवभा है ?
A. 2 B. 4
C. 7 D. 3
E. 8

188. In an athletics meet, number of students participating from College X and College Y are in
the ratio 5 : 8 respectively and 168 male students participated from College Y. If the
respective ratio of male to female students participating from College Y is 3 : 4, then what
is the total number of participants from College X ?
एक एथलेिट मीट म, कॉलेज X और कॉलेज Y से भाग लेने वाले छा ो ं की सं ा मशः 5: 8 है और

PAGE h ps://t.me/studified h ps://instagram.com/aashisharorasocial?utm_medium=copy_link


Follow
95 Aashish Arora on: h ps://youtube.com/channel/UCYa4_JrOrf8R5Kz2uOtccXQ https://www.facebook.com/aashisharorasocial/
अ ास by Aashish Arora
(SBI/IBPS/RBI/LIC/All other banking and insurance Exams)

कॉलेज Y से 168 पु ष छा ो ं ने भाग िलया है । यिद कॉलेज Y से भाग लेने वाले पु ष और मिहला छा ो ं का
संबंिधत अनुपात 3: 4 है , तो कॉलेज X के ितभािगयो ं की कुल सं ा िकतनी है ?
A. 300 B. 280
C. 245 D. 392
E. None of these

189. Yennefer sold three perfumes at Rs 3600 each. The first perfume was sold at 28% loss,
second perfume was sold with a profit of 25 % and the third perfume was sold at a loss of
10%. What is the approximate profit/loss percentage incurred by Yennefer?
यफर ने तीन इ 3600 पये ेक म बेचे। पहला इ 28% हािन पर बेचा गया, दू सरा इ 25% के लाभ के
साथ बेचा गया और तीसरा इ 10% की हािन पर बेचा गया। यफर ारा अनुमािनत लाभ / हािन ितशत ा
है ?
A. 9% B. 15 %
C. 7% D. 13 %
E. None of these

190. The ratio of length and breadth of a rectangle are 9 : 14 respectively and the diameter of a
circle is 50% more than the breadth of the rectangle, them what is ratio of area of the circle
to area of the rectangle respectively
एक आयत की लंबाई और चौड़ाई का अनुपात मशः 9: 14 है और एक वृ का ास आयत की चौड़ाई से
50% अिधक है , िफर वृ के े फल का आयत के े के अनुपात म मशः ा है ।
A. 5 : 13 B. 11 : 4
C. 4 : 11 D. 13 : 5
E. None of these

191. The ratio of time taken by two pipes X and Y to fill a swimming pool is 2 : 7 respectively.
When both pipes are opened together in a swimming pool which is already 2/11 filled, the
swimming pool is filled in 3.5 hours. In how much time can Pipe X alone fill the tank.?
एक िमंग पूल को भरने के िलए दो पाइप X और Y ारा िलए गए समय का अनुपात मशः 2: 7 है । जब
दोनो ं पाइप एक िमंग पूल म एक साथ खोले जाते ह जो पहले से ही 2/11 भरा आ है , तो िमंग पूल 3.5
घंटे म भर जाता है । पाइप X अकेले िकतने समय म टक को भर सकता है ?
A. 11 hours B. 7.5 hours
C. 5.5 hours D. 14 hours
E. None of these

192. The average price of a bat is Rs 1500. The average price of a English Willow bat is Rs 2500
which constitute 30 % of the total bats and the average price of a Kashmir Willow bat is Rs
1500 which constitute 20 % of the bats, rest of the bats are local bats. Find the average
price of local bats?
एक बैट की औसत कीमत 1500 पये है । एक इं श िवलो बैट की औसत कीमत 2500 पये है जो कुल
बैट का 30% है और एक क ीर िवलो बैट की औसत कीमत 1500 पये है जो िक बैट के 20% का गठन
करता है , बाकी बैट थानीय बैट ह। थानीय बैट की औसत कीमत ात कीिजये?
A. Rs 600 B. Rs 1200
C. Rs 1800 D. Rs 900
E. None of these

PAGE h ps://t.me/studified h ps://instagram.com/aashisharorasocial?utm_medium=copy_link


Follow
96 Aashish Arora on: h ps://youtube.com/channel/UCYa4_JrOrf8R5Kz2uOtccXQ https://www.facebook.com/aashisharorasocial/
अ ास by Aashish Arora
(SBI/IBPS/RBI/LIC/All other banking and insurance Exams)

193. The sum of three numbers is 480. Largest number is 40 % more than the middle number
and smallest number is 20% less than the middle no. Find the difference between the
largest and the smallest number.
तीन सं ाओं का योग 480 है । सबसे बड़ी सं ा म सं ा से 40% अिधक है और सबसे छोटी सं ा म
सं ा से 20% कम है । सबसे बड़ी और सबसे छोटी सं ा के बीच का अंतर ात कीिजए।
A. 120 B. 90
C. 150 D. 60
E. None of these

194. Abhijeet and Daya invested in a business together for 34 months and 51 months
respectively. Abhijeet invested Rs __ which is Rs 3700 more than what Daya invested. If
the ratio of profit earned by Abhijeet and Daya is 8 : 7 respectively, then what is the amount
invested by Abhijeet ?
अिभजीत और दया ने मशः 34 महीने और 51 महीनो ं के िलए एक साथ एक वसाय म िनवेश िकया।
अिभजीत ने __ का िनवेश िकया जो िक दया ने िजतना िनवेश िकया उससे 3700 पये अिधक है । यिद
अिभजीत और दया ारा अिजत लाभ का अनुपात मशः 8: 7 है , तो अिभजीत ारा िनवेश की गई रािश ा
है ?
A. Rs 5180 B. Rs 6250
C. Rs 7840 D. Rs 8880
E. None of these

195. Two trains Garibrath and Aligarh Express having lengths of 600 m and 450 m respectively
are coming towards each other and cross each other in 12 seconds. If Garibrath crosses a
bridge of length 300 m in 18 seconds, then in how much time will Aligarh Express cross a
tunnel of 150 m ?
एक दो टे न गरीबरथ और अलीगढ़ ए ेस, िजनकी लंबाई मशः 600 मीटर और 450 मीटर है , एक दू सरे
की ओर आ रही ह और 12 सेकंड म एक दू सरे को पार करती ह। यिद ग थ 18 सेकंड म 300 मीटर की
लंबाई के पुल को पार करता है , तो अलीगढ़ ए ेस 150 मीटर की सुरंग को िकतने समय म पार कर लेगी?
A. 11 seconds B. 16 seconds
C. 22 seconds D. 14.5 seconds
E. None of these

196. The present age of a mother is 8 times her daughter's age. In twelve years time her age will
be 4 times her daughter's age. Find the age of the mother six years from now ?
एक माँ की वतमान आयु उसकी बेटी की आयु से 8 गुना है । बारह साल के बाद म उसकी उ उसकी बेटी की
उ से 4 गुना होगी। अब से छह साल बाद माँ की उ का पता लगाएं ?
A. 82 years B. 66 years
C. 74 years D. 78 years
E. None of these

197. Ronny divided Rs 20000 into two parts. He invested one part at simple interest of 15% for
5 years and the other part at a compound interest of 10 % compounded annually for 3
years. If the total interest he received comes to Rs 11648, then find the ratio in which
divided the money ?
रॉनी ने 20000 पये को दो भागो ं म बां टा। उ ोन
ं े एक िह ा 5 साल के िलए 15% के साधारण ाज पर और

PAGE h ps://t.me/studified h ps://instagram.com/aashisharorasocial?utm_medium=copy_link


Follow
97 Aashish Arora on: h ps://youtube.com/channel/UCYa4_JrOrf8R5Kz2uOtccXQ https://www.facebook.com/aashisharorasocial/
अ ास by Aashish Arora
(SBI/IBPS/RBI/LIC/All other banking and insurance Exams)

दू सरा िह ा 3 साल के िलए 10% के च वृ ाज पर िनवेश िकया। अगर उसे िमलने वाला कुल ाज
11648 पये आता है , तो उस अनुपात का पता लगाएं , िजसम पैसे का बंटवारा आ हो?
A. 3 : 2 B. 5 : 7
C. 1 : 4 D. 3 : 7
E. None of these

198. 323 ml of mixture P containing cola and pepsi in the ratio of 6 : 11 respectively is mixed with
323 ml of mixture Q containing cola and pepsi. If ratio of cola and pepsi in the resultant
mixture is 13 : 21 respectively, then find the ratio of cola and pepsi in the mixture Q.
िम ण P का 323 िमलीलीटर िजसमे कोला और पे ी मशः 6 : 11 के अनुपात म है , 323 िमलीलीटर िम ण
Q कोला और पे ी यु के साथ िमलाया जाता है । यिद प रणामी िम ण म कोला और पे ी का अनुपात
मशः 13 : 21 है , तो िम ण Q म कोला और पे ी का अनुपात ात कर।
A. 7 : 10 B. 8 : 11
C. 9 : 11 D. 10 : 13
E. None of these

199. 7 men and 4 women can complete a piece of work in 15 days while 16 men and 18 women
can complete the same work in 5 days. In how many days 2 men, 3 women and 4 children
can complete the same work if efficiency of each child is 80% less than the efficiency of
each woman ?
7 पु ष और 4 मिहलाएं 15 िदनो ं म एक काम पूरा कर सकते ह जबिक 16 पु ष और 18 मिहलाएं 5 िदनो ं म
उसी काम को पूरा कर सकते ह। िकतने िदनो ं म 2 पु ष, 3 मिहलाएं और 4 ब े उसी काम को पूरा कर
सकते ह यिद ेक ब े की द ता ेक मिहला की द ता से 80% कम है ?
A. 25 days B. 28 days
C. 30 days D. 40 days
E. None Of These

200. The ratio of the speeds of boat P and boat Q in still water is 3 : 5 respectively. The speed of
the current is 14.28% of the speed of boat Q in still water. If boat P can travel 182 km
downstream and 144 km upstream in 8 hours, then find the time taken by boat Q to cover
435 km in upstream.
थर पानी म नाव P और नाव Q की गित का अनुपात मशः 3 : 5 है । धारा की गित, थर पानी म नाव Q की
गित का 14.28% है । यिद नाव P 8 घंटे म 182 िकमी धारा के अनुकूल और 144 िकमी धारा के िव दू री
तय कर सकती है , तो नाव Q ारा धारा के िव 435 िकमी की दू री तय करने म लगने वाला समय ात कर।
A. 7.25 hours B. 6.75 hours
C. 4.75 hours D. 7.5 hours
E. None Of These

201. Rs. 'Y' when invested at p% per annum simple interest for 15 years amounts to 4 times of
itself. If Rs. (Y + 150) when invested at (p + 10)% per annum compound interest
compounded annually for 2 years amounts to Rs. 2704, then find the value of 'Y'.
'Y' जब p% ित वष साधारण ाज पर की दर से 15 साल के िलए िनवेश िकया जाता है तो वह खुद का 4
गुना हो जाता है । यिद (Y + 150), (p + 10)% ित वष सालाना च वृ ाज की दर से 2 वष के िलए
िनवेश की जाती है तो वह 2704 हो जाते है , तो िफर 'Y' का मान ात कर।
A. 1200 B. 1450

PAGE h ps://t.me/studified h ps://instagram.com/aashisharorasocial?utm_medium=copy_link


Follow
98 Aashish Arora on: h ps://youtube.com/channel/UCYa4_JrOrf8R5Kz2uOtccXQ https://www.facebook.com/aashisharorasocial/
अ ास by Aashish Arora
(SBI/IBPS/RBI/LIC/All other banking and insurance Exams)

C. 1100 D. 1250
E. None Of These

202. The sum of incomes of Arnab, Bhupesh and Simar is Rs. 23,780. Arnab, Bhupesh and
Simar spend 95%, 80% and 85% of their respective incomes such that ratio of their
savings is 4 : 5 : 6 respectively. Find the sum of expenditures of Arnab, Bhupesh and
Simar.
अनब, भूपेश और िसमर की आय का योग 23,780 है । अनब, भूपेश और िसमर अपनी आय का 95%,
80% और 85% खच करते ह तथा उनकी बचत का अनुपात मशः 4 : 5 : 6 है । अनब, भूपेश और िसमर के
य का योग ात कीिजए।
A. Rs. 21,320 B. Rs. 21,420
C. Rs. 21,520 D. Rs. 21,620
E. None of these

203. The letters of the word 'SHADIKAB' is arranged at random without any repetitions. Find
the probability that there are exactly three letters between both 'A'.
श 'SHADIKAB' के अ रो ं को िबना िकसी दोहराव के अिनयिमत प से व थत िकया गया है । इस
संभावना को खोज िक 'A' के बीच ठीक तीन अ र ह।
A. 1/7 B. 2/7
C. 3/8 D. 1/8
E. None of these

204. Madan sold a book at 60% profit. If the profit percentage had been numerically equal to
cost price, then the profit earned had been 225% more. Find the amount for which Madan
sold the book initially.
मदन ने 60% लाभ पर एक पु क बेची। यिद लाभ ितशत सं ा क प से लागत मू के बराबर होता
है , तो अिजत लाभ 225% अिधक होता है । वह रािश ात कीिजए िजस पर मदन ने शु म पु क बेची थी।
A. Rs. 332 B. Rs. 342
C. Rs. 312 D. Rs. 322
E. None Of These

205. Pallavi and Tejas invested some amount in a business. Pallavi being a working partener,
received Rs. 33000 out of total profit as commission and 42% of the remaining amount as
her profit share. If Tejas received Rs. 25000 less profit than Pallavi, then find the total profit
received by them.
प वी और तेजस ने कुछ रािश एक वसाय म लगाई। प वी काम करने वाली भागीदार होने के नाते कुल
लाभ म से 33000 कमीशन के प म और शेष रािश का 42% उसके लाभ िह े के प म लेती है । अगर
तेजस को प वी की तुलना म 25000 कम लाभ िमला, तो िफर उनके ारा ा कुल लाभ का पता
लगाएं ।
A. Rs. 83500 B. Rs. 91000
C. Rs. 81000 D. Rs. 83000
E. None of these

206. 276 metre long train P is running with a speed of 79.2 km/hr. Train Q which is 384 metre
long is running with a speed of 100.8 km/hr in opposite direction of train P. For how much
time, the smaller train would get completely hidden by larger train?

PAGE h ps://t.me/studified h ps://instagram.com/aashisharorasocial?utm_medium=copy_link


Follow
99 Aashish Arora on: h ps://youtube.com/channel/UCYa4_JrOrf8R5Kz2uOtccXQ https://www.facebook.com/aashisharorasocial/
अ ास by Aashish Arora
(SBI/IBPS/RBI/LIC/All other banking and insurance Exams)

276 मीटर लंबी टे न P, 79.2 िकमी / घंटा की गित से चल रही है । टे न Q जो 384 मीटर लंबी है , टे न P के
िवपरीत िदशा म 100.8 िकमी / घंटा की गित से चल रही है । िकतने समय के िलए, छोटी टे न पूरी तरह से बड़ी
टे न के पीछे िछप जाएगी ?
A. 2.26 seconds B. 1.86 seconds
C. 1.96 seconds D. 2.16 seconds
E. None Of These

207. The average age of a family of 9 members in 2014 was 43 years. In 2019, a family member
of 48 years died and four children were born. Find the average age of the family in 2021.
2014 म 9 सद ो ं के एक प रवार की औसत आयु 43 वष थी। 2019 म, 48 साल के प रवार के एक सद
की मृ ु हो गई और चार ब ो ने ज िलया। 2021 म प रवार की औसत आयु ात कर।
A. 36 years B. 34 years
C. 35 years D. 38 years
E. None of these

208. In a school, there are total 858 students and the ratio of boys to girls is 6 : 5. If the school
admits 26 new boys, then how many girls should be admitted by the school so that the ratio
of boys to girls becomes 19 : 17 respectively?
एक ू ल म, कुल 858 छा ह और लड़को ं और लड़िकयो ं का अनुपात मशः 6 : 5 है । यिद ू ल 26 नए
लड़को ं को ीकार करता है , तो ू ल ारा िकतनी लड़िकयो ं को भत िकया जाना चािहए तािक लड़को ं और
लड़िकयो ं का अनुपात मशः 19 : 17 हो जाए?
A. 26 B. 42
C. 52 D. 13
E. None of these

209. There are 2800 students in a coaching who are preparing for either CAT or GMAT exam.
Number of male students are 20% more than the number of students preparing for GMAT
exam and 25% less than the number of female students. How many students are
preparing for the CAT exam ?
एक कोिचंग म 2800 छा ह जो CAT या GMAT परी ा की तैयारी कर रहे ह। पु ष छा ो ं की सं ा GMAT
परी ा की तैयारी करने वाले छा ो ं की सं ा से 20% अिधक है और मिहला छा ो ं की सं ा से 25% कम है ।
CAT परी ा के िलए िकतने छा तैयारी कर रहे ह?
A. 1200 B. 1800
C. 1400 D. 1500
E. None of these

210. A track of width of 10.5m is running around a circular park. Cost of gardening the park
(without road) is Rs 8624 at the rate of Rs 3.5 per m2, then what will be the difference
between the inner and outer perimeter of the track?
एक प रप पाक के चारो ं ओर 10.5m की चौड़ाई का एक टै क चल रहा है । पाक (सड़क के िबना) की
बागवानी की लागत 3.5 पये ित वग मीटर की दर से 8624 पये है , िफर टै क की आं त रक और बाहरी
प रिध म ा अंतर होगा?
A. 78 m B. 66 m
C. 52 m D. 58 m
E. None of these

PAGE h ps://t.me/studified h ps://instagram.com/aashisharorasocial?utm_medium=copy_link


Follow
100 Aashish Arora on: h ps://youtube.com/channel/UCYa4_JrOrf8R5Kz2uOtccXQ https://www.facebook.com/aashisharorasocial/
अ ास by Aashish Arora
(SBI/IBPS/RBI/LIC/All other banking and insurance Exams)

211. The speed of a boat in still water is 25 % more than its upstream speed. If the stream is
flowing with a speed of 3 km/hr, then what will be the downstream distance covered by the
boat in 11 hours 25 minutes ?
थर पानी म एक नाव की गित उसकी धारा के िव जाने की गित से 25% अिधक है । यिद धारा 3 िकमी /
घंटा की गित से बह रही है , तो 11 घंटे 25 िमनट म नाव धारा के अनुकूल िकतनी दू री तय करे गी?
A. 212.5 km B. 197 km
C. 175.75 km D. 205.5 km
E. None of these

212. Ted's age is 3 times Victoria's age seven years ago and Victoria is 12 years younger than
Marshall. The sum of the present ages of Ted and Marshall is 51 years, then what will be
the sum of present ages of Ted and Victoria?
टे ड की उ सात साल पहले िव ो रया की उ से 3 गुना और िव ो रया माशल से 12 साल छोटी है । टे ड
और माशल की वतमान आयु का योग 51 वष है , िफर टे ड और िव ो रया की वतमान आयु का योग ा
होगा?
A. 45 years B. 33 years
C. 42 years D. 39 years
E. None of these

213. Raghu alone can complete the whole work in 30 days and Amit alone can complete the
whole work in 21 days. If the ratio of efficiency of Raghu and Anamika is 7: 8 and the ratio of
efficiency of Amit and Simi is 5 : 6, then in how many days can Simi and Anamika together
complete the whole work.
अकेले रघु 30 िदनो ं म पूरा काम पूरा कर सकते ह और अकेले अिमत 21 िदनो ं म पूरा काम पूरा कर सकते ह।
यिद रघु और अनािमका की द ता का अनुपात मशः 7: 8 है और अिमत और िसमी की द ता का अनुपात
मशः 5: 6 है , तो िकतने िदनो ं म िसमी और अनािमका एक साथ पूरे काम को पूरा कर सकते ह।
A. 9 (3/8) days B. 10.5 days
C. 7 days D. 8 (9/11) days
E. None of these

214. Zidane starts a business with $775. He left the business after 8 months and at the same
time Beckham entered the business with $ 1250. If at the end of the year Zidane received
$1395 as his share out of the total profit. What was the share of profit received by
Beckham ?
िजदान $ 775 के साथ एक वसाय शु करता है । उ ोन ं े 8 महीने के बाद वसाय छोड़ िदया और उसी
समय, बेकहम ने $ 1250 के साथ वसाय म वेश िकया। यिद वष के अंत म िजदान को कुल लाभ म से
1395 डॉलर उनके िह े के प म िमले। बेकहम ारा ा लाभ का िह ा ा था?
A. $ 1175 B. $ 1025
C. $ 1125 D. $ 1075
E. None of these

215. A vendor sold 9 cups and 5 plates at Rs 150 and Rs 240 each respectively. If he gains a
overall profit of Rs 150 and the profit earned by him on each plate was 25%, then what was
the cost price of each cup ?
एक िव े ता ने मशः 9 कप और 5 ेट 150 पये ेक और 240 पये ेक म बेची।ं यिद वह 150

PAGE h ps://t.me/studified h ps://instagram.com/aashisharorasocial?utm_medium=copy_link


Follow
101 Aashish Arora on: h ps://youtube.com/channel/UCYa4_JrOrf8R5Kz2uOtccXQ https://www.facebook.com/aashisharorasocial/
अ ास by Aashish Arora
(SBI/IBPS/RBI/LIC/All other banking and insurance Exams)

पये का सम लाभ ा करता है और ेक ेट पर उसके ारा अिजत लाभ 25% था, तो ेक कप का


लागत मू ा था?
A. Rs 160 B. Rs 145
C. Rs 175 D. Rs 167.5
E. None of these

216. Mixture P of oil and water contains oil and water in the ratio 8 : 7 and 99 liters of mixture Q
contains oil and water in the ratio 6 : 5 and it contains 10 liters less oil than mixture P. If 30
liters of mixture P is removed and replaced by 30 liters of oil than what is the new
percentage of oil in mixture P.
िम ण पी म तेल और पानी मशः 8 : 7 के अनुपात म है और 99 लीटर िम ण Q म तेल और पानी 6: 5 के
अनुपात म है और इसम िम ण पी की तुलना म 10 लीटर कम तेल है । यिद िम ण पी का 30 लीटर िनकाला
जाता है और 30 लीटर तेल िमलाया जाता है । िम ण पी म तेल के ितशत का पता लगाएं ।
A. 65% B. 57.5%
C. 53.33% D. 47.75%
E. None of these

217. Russel invested Rs y in a scheme for 3 years at 10% SI. If he invested the same amount for
the same time and same annual rate of annual CI, then he would have gained Rs 6200
more, then what is the value of 'y'?
रसेल ने 3 साल के िलए 10% साधारण ाज पर एक योजना म 'y' का िनवेश िकया। यिद उसने समान रािश
समान वािषक दर पर सामान समय के िलए के िलए िनवेश िकया है , तो उसे 6200 पये अिधक िमलगे, तो 'y
'का ा मू है ?
A. Rs 180000 B. Rs 240000
C. Rs 200000 D. Rs 185000
E. None of these

218. The unit digit and ten's digit of two digit number is 'x' and 'y' respectively. If (x+y):(x-y)=4:1,
then find the value of x:y and also how many such numbers exists?
दो अंको ं की सं ा का इकाई अंक और दस का अंक मशः 'x' और 'y' है । यिद (x y):(x-y) = 4:1, तो x:y
का मान ात कीिजए और यह भी िक ऐसी िकतनी सं ाएँ मौजूद ह?
A. 5:3, 1 B. 2:5, 3
C. 10:7, 7 D. 20:17, 5
E. None of these

219. The time taken by A to build a wall alone is 20% more than the time taken by him to build it
along with B. B will take how much percent more time to build the same wall than the time
taken by him to build it along with A?
A ारा अकेले एक दीवार बनाने म लगने वाला समय B के साथ िमलकर उसे बनाने म लगने वाले समय से
20% अिधक है । B को उसी दीवार को बनाने म A के साथ लगने वाले समय की तुलना म िकतना ितशत
अिधक समय लगेगा?
A. 100% B. 720%
C. 500% D. 405%
E. None of these

220. The speed of a boat in still water is 24 km/hr while the speed of the current is 16 km/hr. If the

PAGE h ps://t.me/studified h ps://instagram.com/aashisharorasocial?utm_medium=copy_link


Follow
102 Aashish Arora on: h ps://youtube.com/channel/UCYa4_JrOrf8R5Kz2uOtccXQ https://www.facebook.com/aashisharorasocial/
अ ास by Aashish Arora
(SBI/IBPS/RBI/LIC/All other banking and insurance Exams)

boat travelled from point P to point Q and returned back to the same point P, then find the
average speed (in km/hr) of the boat during the whole journey.
शां त जल म एक नाव की गित 24 िकमी/घंटा है , जबिक धारा की गित 16 िकमी/घंटा है । यिद नाव िबंदु P से
िबंदु Q तक जाती है और वापस उसी िबंदु P पर लौटती है , तो पूरी या ा के दौरान नाव की औसत गित
(िकमी/घंटा म) ात कीिजए।
A. 40/3 B. 1/45
C. 5/17 D. 7/15
E. None of these

221. X and Y invested Rs 36000 and Rs 24000 in a business, together. After 6 months, X left
and Z joined the business with a capital of Rs 10000. Find the ratio of the profit received by
them at the end of the 2 years.
X और Y ने एक ापार म एक साथ 36000 पये और 24000 पये का िनवेश िकया। 6 महीने के बाद, X
छोड़ दे ता है और Z 10000 पये की पूंजी के साथ वसाय म शािमल हो जाता है । 2 वष के अंत म उनके
ारा ा लाभ का अनुपात ात कीिजए।
A. 1:5:5 B. 19:5:4
C. 5:4:5 D. 9:16:5
E. None of these

222. Mixture X ( petrol and diesel) contains petrol and diesel in the ratio 4:1 while mixture Y
contains petrol and diesel in the ratio 1:4. In what ratio of the two mixtures should be mixed
having petrol to diesel in ratio 1:3?
िम ण X (पेटोल और डीजल) म पेटोल और डीजल का अनुपात 4:1 है जबिक िम ण Y म पेटोल और डीजल
का अनुपात 1:4 है । पेटोल और डीजल के 1:3 के अनुपात म दोनो ं िम णो ं को िकस अनुपात म िमलाया जाना
चािहए?
A. 2:25 B. 21:98
C. 1:11 D. 2:23
E. None of these

223. X is 8 years older than Y. If the ratio of their ages 8 years hence from now will be 9:8, then
find the present age of X.
X, Y से 8 वष बड़ा है । यिद अब से 8 वष बाद उनकी आयु का अनुपात 9:8 होगा, तो X की वतमान आयु ात
कीिजए।
A. 30 B. 64
C. 30 D. 54
E. None of these

224. The income of X is Rs 25000 which is 25% more than that of Y. If Y spends Rs 10000 out of
his total income and the savings of X is Rs 2000 less than the Y, then how much
percentage of his income does X spend?
X की आय 25000 पये है जो िक Y से 25% अिधक है । यिद Y अपनी कुल आय म से 10000 पये खच
करता है और X की बचत Y से 2000 पये कम है , तो X अपनी आय का िकतना ितशत खच करता है ?
A. 20% B. 68%
C. 32% D. 21%
E. None of these

PAGE h ps://t.me/studified h ps://instagram.com/aashisharorasocial?utm_medium=copy_link


Follow
103 Aashish Arora on: h ps://youtube.com/channel/UCYa4_JrOrf8R5Kz2uOtccXQ https://www.facebook.com/aashisharorasocial/
अ ास by Aashish Arora
(SBI/IBPS/RBI/LIC/All other banking and insurance Exams)

225. The average weight of 12 persons is increased by 4.5 kg when one of them who weights
26 kg is replaced by a new man. Find the weight of new man.
12 यो ं के औसत भार म 4.5 िक ा की वृ होती है , जब उनम से 26 िक ा भार वाले को एक
नए ारा ित थािपत िकया जाता है । नए आदमी का वजन ात कीिजए।
A. 12 B. 140
C. 80 D. 105
E. None of these

226. The area of the square is 676 cm2. If a rectangle drawn whose length is twice of the side of
square and breadth is 10 cm less than the side of square, then find the ratio of the length
and breadth of the rectangle.
वग का े फल 676 सेमी2 है । यिद एक आयत बनाया गया है , िजसकी लंबाई वग की भुजा से दोगुनी है और
चौड़ाई वग की भुजा से 10 सेमी कम है । िफर आयत की लंबाई और चौड़ाई का अनुपात ात कीिजए।
A. 8:25 B. 7:15
C. 6:11 D. 13:4
E. None of these

227. A Contractor undertook a work to complete in 80 days and started the work with 20 men
but after 60 days he found that only 3/5th of the work was finished. Find how much more
men should work together to finish the work on time.
एक ठे केदार ने 80 िदनो ं म पूरा करने के िलए काम िकया और 20 पु षो ं के साथ काम शु िकया लेिकन 60
िदनो ं के बाद उसने पाया िक काम का केवल 3/5 िह ा ही समा आ था। ात कीिजए िक काम को समय
पर पूरा करने के िलए िकतने और पु षो ं को एक साथ काम करने चािहए।
A. 20 B. 15
C. 10 D. 14
E. None of these

228. The speed of X and Y is 160 km/hr and 64km/hr respectively. If distance covered by X in z
hours is 384 km more than that by Y in the same time, then find the value of z.
X और Y की गित मशः 160 िकमी/घंटा और 64 िकमी/घंटा है । यिद x ारा z घंटे म तय की गई दू री समान
समय म Y ारा तय की गई दू री से 384 िकमी अिधक है , तो z का मान ात कीिजए।
A. 6 B. 4
C. 7 D. 5
E. None of these

229. The cost price of two articles is same. One article is sold at 20% profit and another at 10%
loss. If the selling price of one article is Rs 568 more than that of another, then find the
selling price of article sold at loss.
दो व ुओं का य मू समान है । एक व ु को 20% लाभ पर और दू सरी को 10% हािन पर बेचा जाता है ।
यिद एक व ु का िव य मू दू सरी व ु के िव य मू से 568 पये अिधक है , तो हािन पर बेची गई व ु
का िव य मू ात कीिजए।
A. 1704 B. 7544
C. 1964 D. 1458
E. None of these

230. There are five numbers 28, 4, 6, 12 and 10. Which one of the given five numbers can be

PAGE h ps://t.me/studified h ps://instagram.com/aashisharorasocial?utm_medium=copy_link


Follow
104 Aashish Arora on: h ps://youtube.com/channel/UCYa4_JrOrf8R5Kz2uOtccXQ https://www.facebook.com/aashisharorasocial/
अ ास by Aashish Arora
(SBI/IBPS/RBI/LIC/All other banking and insurance Exams)

omitted such that the average of the remains the same?


पाँ च सं ाएँ 28, 4, 6, 12 और 10 ह। दी गई पाँ च सं ाओं म से िकस एक को इस कार छोड़ा जा सकता है
िक शेष का औसत समान रहे ?
A. 10 B. 4
C. 6 D. 12
E. None of these

231. 'P, Q and R entered in a partnership with the sums in the ratio of 4:1:7 respectively. The
time period of investment is 4 months, 3 months and 5 months respectively. The profit
received by P and Q together is how much percent more/less than the profit received by
R?
P, Q और R ने मशः 4:1:7 के अनुपात म रािश के साथ एक साझेदारी म वेश िकया। िनवेश की समयाविध
मशः 4 महीने, 3 महीने और 5 महीने है । P और Q को िमलाकर ा लाभ, R ारा ा लाभ से िकतने
ितशत अिधक/कम है ?
A. 22.58% B. 21.57%
C. 45.71% D. 42.25%
E. None of these

232. Mixture X contains 180 litres milk and water. 75% of the mixture is drawn and 180 litres of
mixture Y, which contains 50% milk is added to the remaining mixture such that ratio of
milk to the water becomes 5:4. Find the initial quantity of water in mixture X.
िम ण X म 180 लीटर दू ध और पानी है । िम ण का 75% िनकाला जाता है और 180 लीटर िम ण Y, िजसम
50% दू ध होता है , को शेष िम ण म इस कार िमलाया जाता है िक दू ध का पानी से अनुपात 5:4 हो जाता है ।
िम ण X म पानी की ारं िभक मा ा ात कीिजए।
A. 59 B. 40
C. 50 D. 55
E. None of these

233. The ratio of the speed of a boat in still water to the speed of the current is 7:5 respectively. If
the time taken by boat to travel (D + 20) km downstream is equal to the time taken by the
boat to travel (D - 30) km upstream, then find the value of D.
शां त जल म एक नाव की गित का धारा की गित से अनुपात मशः 7:5 है । यिद नाव ारा धारा के अनुकूल (D
+ 20) िकमी की या ा करने म िलया गया समय नाव ारा धारा के ितकूल (D - 30) िकमी की या ा करने म
लगने वाले समय के बराबर है , तो D का मान ात कीिजए।
A. 40 B. 44
C. 55 D. 45
E. None of these

234. The simple interest received on (x + 1200) after 3 years at the rate of 50% per annum is Rs
(3x - 3600). Find the interest earned on Rs 2x when invested at 20% per annum compound
interest
3 वष के बाद (x + 1200) पर 50% ित वष की दर से ा साधारण ाज (3x - 3600) है । 20% ित
वष च वृ ाज पर िनवेश करने पर 2x पर ा ाज ात कीिजए।
A. 3065 B. 6454
C. 3054 D. 3168
E. None of these

PAGE h ps://t.me/studified h ps://instagram.com/aashisharorasocial?utm_medium=copy_link


Follow
105 Aashish Arora on: h ps://youtube.com/channel/UCYa4_JrOrf8R5Kz2uOtccXQ https://www.facebook.com/aashisharorasocial/
अ ास by Aashish Arora
(SBI/IBPS/RBI/LIC/All other banking and insurance Exams)

235. The total surface area of cube X is 200 sq.cm more than its curved surface area. If each
edge of cube Y is 80% more than that of cube X, then find the sum of area of curved
surface area and total surface area of cube Y.
घन X का कुल पृ ीय े फल इसके व पृ ीय े फल से 200 वग सेमी अिधक है । यिद घन Y का ेक
िकनारा घन X से 80% अिधक है , तो व पृ ीय े फल और घन Y के कुल पृ ीय े फल का योग ात
कीिजए।
A. 1254 B. 1405
C. 3240 D. 1055
E. None of these

236. The ratio of the ages of X and Y, 4 years hence from now will be 5:7 respectively while the
ratio of their ages 2 years ago from now was 1:2 respectively. Find the difference between
their present ages.
X और Y की आयु का अनुपात अब से 4 वष बाद मशः 5:7 होगा जबिक अब से 2 वष पहले उनकी आयु का
अनुपात मशः 1:2 था। उनकी वतमान आयु के बीच का अंतर ात कीिजए।
A. 8 B. 4
C. 6 D. 13
E. None of these

237. If P can do a piece of work in 24 days and Q takes 32 days to complete the same work. If
they started the work together but after 9 days working together they stopped working.
Find the remaining part of the work.
यिद P िकसी काय को 24 िदनो ं म कर सकता है और Q उसी काय को पूरा करने म 32 िदन लेता है । यिद वे
एक साथ काम शु करते ह लेिकन 9 िदनो ं तक एक साथ काम करने के बाद उ ोन
ं े काम करना बंद कर
िदया। काय का शेष भाग ात कीिजए।
A. 11/32 B. 15/32
C. 10/31 D. 14/31
E. None of these

238. Find the speed of the stream, if a boat takes 12 hours to travel a distance of 252km
upstream and the speed of boat in still water is 28km/hr?
धारा की गित ात कीिजए, यिद एक नाव धारा के ितकूल 252 िकमी की दू री तय करने म 12 घंटे लेती है
और शां त पानी म नाव की गित 28 िकमी/घंटा है ?
A. 7 km/hr B. 9 km/hr
C. 10 km/hr D. 8 km/hr
E. None of these

239. A, B and C together can complete a work in 28 days. If the ratio of the efficiency of A, B and
C is 9:8:5, respectively, then find the time taken by B alone to complete the whole work?
A, B और C िमलकर एक काय को 28 िदनो ं म पूरा कर सकते ह। यिद A, B और C की द ता का अनुपात
मशः 9:8:5 है , तो अकेले B ारा पूरा काय पूरा करने म िलया गया समय ात कीिजए?
A. 88 days B. 77 days
C. 66 days D. 75 days
E. None of these

240. Present ages of Lavi and Anil are in the ratio 8:7, respectively. The ratio of their ages after

PAGE h ps://t.me/studified h ps://instagram.com/aashisharorasocial?utm_medium=copy_link


Follow
106 Aashish Arora on: h ps://youtube.com/channel/UCYa4_JrOrf8R5Kz2uOtccXQ https://www.facebook.com/aashisharorasocial/
अ ास by Aashish Arora
(SBI/IBPS/RBI/LIC/All other banking and insurance Exams)

16 years will be 12:11. The sum of the ages of Avi and Lavi is 40 years. Find the present
age of Avi?
लवी और अिनल की वतमान आयु मशः 8:7 के अनुपात म है । 16 वष बाद उनकी आयु का अनुपात 12:11
होगा। अिव और लवी की आयु का योग 40 वष है । अिव की वतमान आयु ात कीिजए?
A. 8 yrs B. 4 yrs
C. 5 yrs D. 7 yrs
E. None of these

241. A, B and C started a business with initial investment of Rs. 2500, Rs. 3500 and Rs. 4000
respectively. After one year A, B and C made additional invetsment of Rs. X+1300, Rs.
X+1000, Rs. X + 1700, respectively. Find the profit share of B out of the total profit of Rs.
6000, after two years?
A, Bऔर C ने मशः 2500 ., 3500 . और 4000 . के शु आती िनवेश के साथ एक वसाय शु
िकआ। एक वष बाद A, B और C ने मश: X + 1300 ., X+ 1000 . , X+ 1700 . का अित र
िनवेश िकया। दो वष बाद 6000 . के कुल लाभ म से B का िह ा ात कर ?
A. Rs. 3000 B. Rs. 2000
C. Rs. 3500 D. CND
E. None of these

242. Priya invested Rs. X in a scheme offering 15% p.a simple interest for 4 years while riya
invested same amount in another scheme offering 20% p.a. compound interest for 3
years, compounded annually. Find the value of X if the difference between the interests
earned by them after 3 years is Rs.1024?
ि या X . को एक योजना म तीन वष के िलए 15% ितवष साधारण ाज पर िनवेश करता है जबिक रया
सामान रािश को एक अ ीम म तीन वष के िलए 20% ितवष वािषक दे य च वृ ाज पर िनवेश
करती है । X का मान ात करे यिद तीन वष के बाद उनके ारा अिजत ाज के बीच का अंतर 1024 . है ।
A. Rs. 5000 B. Rs. 6000
C. Rs. 8000 D. Rs. 7000
E. None of these

243. Aditya , vicky and karan together can complete a work in 18 days. Vicky is 10% less
efficient than aditya . Aditya and karan together can complete the work in 27 days. In how
many days karan alone can complete the work ?
आिद , िव ी और करण िमलकर एक काम को 18 िदनो ं म पूरा कर सकते ह। िव ी, आिद से 10%
कम कायकुशल है । आिद और करण िमलकर काम को 27 िदनो ं म पूरा कर सकते ह। करण अकेला उस
काम को िकतने िदनो ं म पूरा कर सकता है ?
A. 50.75 days B. 40.65 days
C. 45 days D. 60.75 days
E. None of these

244. A bag contain 'x+1' red, 'x+4' blue and '2x' green balls. Two balls are randomly drawn from
the box and probability that a blue and a red ball are drawn is 1/6. Find the total number of
balls in the bag.
एक बैग म 'x + 1' लाल, 'x + 4' नीली और '2x' हरी गद ह। बॉ से से दो गद िनकाली जाती ह और एक नीली
और एक लाल गद के िनकाले जाने की ाियकता 1/6 है । बैग म गदो ं की कुल सं ा िकतनी ह ?

PAGE h ps://t.me/studified h ps://instagram.com/aashisharorasocial?utm_medium=copy_link


Follow
107 Aashish Arora on: h ps://youtube.com/channel/UCYa4_JrOrf8R5Kz2uOtccXQ https://www.facebook.com/aashisharorasocial/
अ ास by Aashish Arora
(SBI/IBPS/RBI/LIC/All other banking and insurance Exams)

A. 8 B. 7
C. 9 D. 6
E. None of these

245. Train X can cross a pole and a platform 150 meter long in 15 seconds and 45 seconds
respectively. Train X can cross train Y moving in opposite direction in 5 seconds. Find the
speed of train Y if its length is 20 meter less than that of train X ?
टे न X, 150 मीटर लंबे एक पोल और ेटफॉम को मश: 15 सेकंड और 45 सेकंड म पार कर सकती है ।
टे न X िवपरीत िदशा म चलती ई टे न Y को 5 सेकंड म पार कर सकती है । टे न Y की गित ात कीिजए यिद
इसकी लंबाई टे न X की लंबाई से 20 मीटर कम है ?
A. 20 m/s B. 21 m/s
C. 22 m/s D. 18 m/s
E. None of these

246. Radius of base and height of circular cylinder are in the ratio 3:14 respectively. Find its
curved surface area if its volume is 10692 cm3 ?
वृ ाकार बेलन के आधार की ि ा और ऊँचाई का अनुपात मशः 3:14 है । यिद इसका आयतन 10692
cm3 है तो इसका व पृ का े फल ात कीिजए।
A. 2376 cm2 B. 2354 cm2
2
C. 2556 cm D. 2761 cm2
E. None of these

247. Monthly income of Q is Rs.4000 more than the monthly income of P. Monthly savings of P
and Q are Rs. 4000 and Rs. 15000 respectively. Find the monthly income of P if the
monthly expenditures of P and Q are in the ratio 3:2, respectively?
Q की मािसक आय, P की मािसक आय से 4000 . अिधक है । P और Q की मािसक बचत मशः 4000 .
और 15000 . है । यिद P और Q के मािसक य मशः 3:2 के अनुपात म ह, तो P की मािसक आय ात
कीिजए?
A. Rs. 24000 B. Rs. 27000
C. Rs. 26000 D. Rs. 25000
E. None of these

248. A mixture (milk and water) contains milk and water in the ratio 5:7, respectively. When 240
liters of mixture is withdrawn and 21 liters of water is added to the remaining mixture, the
ratio of milk to water in the resultant mixture becomes 4:7. Find the initial quantity of
mixture.
एक िम ण (दू ध और पानी) म दू ध और पानी का अनुपात मशः 5:7 है । जब 240 लीटर िम ण िनकाल िलया
जाता है और शेष िम ण म 21 लीटर पानी िमला िदया जाता है , तो प रणामी िम ण म दू ध का पानी से अनुपात
4:7 हो जाता है । िम ण की ारं िभक मा ा ात कीिजए।
A. 296 B. 264
C. 474 D. 384
E. None of these

249. 'X' can complete a work in 24 days. 'Y' takes 16 days more than 'X' to complete the work. 'X'
and 'Y' worked together for 10 days and then left. if 'Z' completed the remaining work
alone, in 10 days, then find the efficiency of 'Z'.

PAGE h ps://t.me/studified h ps://instagram.com/aashisharorasocial?utm_medium=copy_link


Follow
108 Aashish Arora on: h ps://youtube.com/channel/UCYa4_JrOrf8R5Kz2uOtccXQ https://www.facebook.com/aashisharorasocial/
अ ास by Aashish Arora
(SBI/IBPS/RBI/LIC/All other banking and insurance Exams)

'X' एक काय को 24 िदनो ं म पूरा कर सकता है । 'Y' काय को पूरा करने म 'X' से 16 िदन अिधक लेता है । 'X'
और 'Y' ने 10 िदनो ं तक एक साथ काम िकया और िफर चले गए। यिद 'Z' शेष काय को अकेले 10 िदनो ं म
पूरा करता है , तो 'Z' की द ता ात कीिजए।
A. 4 unit/day B. 5 unit/day
C. 3 unit/day D. 6 unit/day
E. None of these

250. The ratio of men to women, who visited a fair is 9:7. The ratio of the cost of each ticket for
them is in the ratio 7:5, respectively. if the total amount collected from man is Rs. 100800,
then find the amount collected from woman.
मेले म जाने वाले पु षो ं का मिहलाओं से अनुपात 9:7 है । उनके िलए ेक िटकट की लागत का अनुपात
मशः 7:5 के अनुपात म है । अगर आदमी से एक की गई कुल रािश । 100800, तो मिहला से एक की
गई रािश ात कीिजए।
A. 56000 B. 25000
C. 30000 D. 34600
E. None of these

251. 'X', 'Y' and 'Z' invested Rs. 20000, Rs. 24000 and Rs. 32000, in a business for 8 months, 6
months and 10 months, respectively. If the profit received by 'X' is Rs. 7000, then find the
difference between the profits received by 'Y' and 'Z'.
A. 7000 B. 7700
C. 8700 D. 6700
E. None of these

252. The pocket money received by Rohit in three consecutive months is in the ratio 9:4:7,
respectively. He spends 20% of his pocket money in 1st month which is Rs. 180 less than
the money spent by him in 3rd month. If he saves 70% of his pocket money in 3rd month,
then find the pocket money received by him in 2nd month.
रोिहत ारा लगातार तीन महीनो ं म ा पॉकेट मनी मशः 9:4:7 के अनुपात म है । वह अपनी पॉकेट मनी
का 20% पहले महीने म खच करता है जो िक उसके ारा तीसरे महीने म खच की गई रािश से 180 कम है ।
यिद वह अपने पॉकेट मनी का 70% तीसरे महीने म बचाता है , तो दू सरे महीने म उसे ा पॉकेट मनी ात
कीिजए।
A. 2000 B. 2400
C. 1600 D. 1200
E. None of these

253. The speed of a boat in still water is 8 km/hr more than that of the current. If the speed of the
boat in still water had been twice, then it would take 2.5 hours to travel 125 km upstream.
Find the original downstream speed of the boat.
शां त जल म एक नाव की गित धारा की गित से 8 िकमी/घंटा अिधक है । यिद शां त जल म नाव की गित दु गनी
होती, तो वह धारा के ितकूल 125 िकमी की या ा करने म 2.5 घंटे का समय लेती। नाव की मूल अनु वाह
गित ात कीिजए।
A. 49 km/hr B. 56 km/hr
C. 76 km/hr D. 48 km/hr
E. None of these

PAGE h ps://t.me/studified h ps://instagram.com/aashisharorasocial?utm_medium=copy_link


Follow
109 Aashish Arora on: h ps://youtube.com/channel/UCYa4_JrOrf8R5Kz2uOtccXQ https://www.facebook.com/aashisharorasocial/
अ ास by Aashish Arora
(SBI/IBPS/RBI/LIC/All other banking and insurance Exams)

254. The length of a rectangular field is 6 m more than the radius of a circular field. If the
perimeter of the circular field and rectangular field is 264 m and 160 m, respectively, then
find the area of the rectangular field.
एक आयताकार मैदान की लंबाई एक वृ ाकार मैदान की ि ा से 6 मी अिधक है । यिद वृ ाकार खेत और
आयताकार खेत का प रमाप मशः 264 मी और 160 मी है , तो आयताकार े का े फल ात कीिजए।
A. 1126 B. 1584
C. 1536 D. 1234
E. None of these

255. A certain sum at certain rate becomes 4 times of itself in 4 years at compound interest,
compounded annually. In how many years will the sum becomes 256 times of itself at
same rate of compound interest, compounded annually?
एक िनि त रािश वािषक च वृ ाज पर 4 वष म यं का 4 गुना हो जाती है । वािषक च वृ ाज की
समान दर से िकतने वष म रािश यं का 256 गुना हो जाएगी?
A. 13 B. 16
C. 19 D. 12
E. None of these

256. The price of an article is increased by 20% such that its consumption decreased by 30%. If
the earlier expenditure on the article is Rs. 1000, then find the new expenditure on the
article.
एक व ु के मू म 20% की वृ इस कार की जाती है िक उसकी खपत म 30% की कमी हो जाती है ।
यिद व ु पर पहले का य . 1000 है , तो व ु पर नया य ात कीिजए।
A. 880 B. 900
C. 840 D. 560
E. None of these

257. If an article is sold at (25/19)th of the original selling price, then there would have been 25%
profit. Find the profit/loss percent on the original selling price.
यिद एक व ु को मूल िव य मू के (25/19)व पर बेचा जाता है , तो 25% लाभ होता। मूल िव य मू पर
लाभ/हािन ितशत ात कीिजए।
A. 3% loss B. 5% profit
C. 4% profit D. 5% loss
E. None of these

258. The amount compound annually on Rs. 1600 for 2 years at r% p.a. is Rs. 3600. Find the
simple interest received when Rs. 2400 is invested at (r-20)% p.a. for 5years.
. 1600 पर 2 वष के िलए r% ित वष च वृ ाज पर ा रािश . 3600. है । जब . 2400, (r-
20)% ित वष की दर से 5 साल के िलए िनवेश िकया जाता है , तब ा होने वाला साधारण ाज ात
कीिजए ।
A. Rs. 4200 B. Rs. 2700
C. Rs. 3600 D. Rs.1800
E. None of these

259. The sum invested by 'P' and 'Q', together in a business is 3600 such that the investment by
'P' is 25% of the total investment. If 'P' invested his sum for 8 months and 'Q' for 7 months,
then the ratio of the profit received by them is :

PAGE h ps://t.me/studified h ps://instagram.com/aashisharorasocial?utm_medium=copy_link


Follow
110 Aashish Arora on: h ps://youtube.com/channel/UCYa4_JrOrf8R5Kz2uOtccXQ https://www.facebook.com/aashisharorasocial/
अ ास by Aashish Arora
(SBI/IBPS/RBI/LIC/All other banking and insurance Exams)

एक वसाय म 'P' और 'Q' ारा एक साथ िनवेश की गई रािश 3600 इस कार है िक 'P' ारा िकया गया
िनवेश कुल िनवेश का 25% है । यिद 'P' ने उसकी रािश को 8 महीने और 'Q' ने 7 महीने के िलए िनवेश िकया,
तो उनके ारा ा लाभ का अनुपात है :
A. 21:9 B. 8:21
C. 9:8 D. 21:6
E. None of these

260. A taxi running with a speed of 'a' m/sec makes 7 rounds of a circular field having radius 45
meters, in 36seconds. Find the time taken by taxi to cover 640 meters with the speed of
(a-15) m/sec.
एक टै ी 'a' m/sec की गित से दौड़ती ई 45 मीटर ि ा वाले एक वृ ाकार मैदान के 36 सेकंड म 7
च र लगाती है । टै ी ारा (a-15) m/sec की गित से 640 मीटर की दू री तय करने म िलया गया समय
ात कीिजए।
A. 7 seconds B. 21 seconds
C. 16 seconds D. 29 seconds
E. None of these

261. Rima's present age is thrice of her neighbour neetu's age four year from now. If 20 years
later, the ratio is 20:9, what is the difference between their present age?
रीमा की वतमान आयु उसकी पड़ोसी नीतू की अब से चार वष बाद की आयु से तीन गुना है । यिद 20 वष बाद,
अनुपात 20:9 है , तो उनकी वतमान आयु म िकतना अंतर है ?
A. 40 years B. 34 years
C. 44 years D. 48 years
E. None of these
2
262. The difference between the total surface areas of two cubes is 504 cm . If the difference
between the lengths of the edges of the cubes is 4cm, then find the sum of the length of
edges of both cubes .
दो घनो ं के कुल पृ ीय े फलो ं के बीच का अंतर 504 cm2 है । यिद घनो ं के िकनारो ं की लंबाई के बीच का
अंतर 4 cm है , तो दोनो ं घनो ं के िकनारो ं की लंबाई का योग ात कीिजए।
A. 23cm B. 29cm
C. 17cm D. 21cm
E. None of these

263. If five men and four women can do the same piece of work as three men and six women,
then what is the ratio of efficiency of men and women?
यिद पां च पु ष और चार मिहलाएं , तीन पु षो ं और छह मिहलाओं के समान काय कर सकते ह, तो पु षो ं और
मिहलाओं की द ता का अनुपात ा है ?
A. 1:2 B. 2:1
C. 2:2 D. 1:1
E. None of these

264. The ratio of the number of shoes and sandals sold by shopkeeper is 16:9, respectively. If
the numbers of shoes and sandals sold had been 350 more and 250 more, respectively
then the respectively ratio would have been 3:2. Find the number of shoes sold.
दु कानदार ारा बेचे गए जूतो ं और सडल की सं ा का अनुपात मशः 16:9 है । यिद बेचे गए जूतो ं और

PAGE h ps://t.me/studified h ps://instagram.com/aashisharorasocial?utm_medium=copy_link


Follow
111 Aashish Arora on: h ps://youtube.com/channel/UCYa4_JrOrf8R5Kz2uOtccXQ https://www.facebook.com/aashisharorasocial/
अ ास by Aashish Arora
(SBI/IBPS/RBI/LIC/All other banking and insurance Exams)

सडल की सं ा मशः 350 अिधक और 250 अिधक होती, तो मशः अनुपात 3:2 होता। बेचे गए जूतो ं की
सं ा ात कीिजए।
A. 510 B. 180
C. 340 D. 160
E. None of these

265. A shopkeeper deals in milk and 49 litres mixture distributed in Milk & Water in the ratio of
5:2. If 5 litres milk & 4 litres water will be added in the mixture then what will be the new ratio
of water and milk?
एक दु कानदार दू ध का सौदा करता है और 49 लीटर िम ण को दू ध और पानी म 5:2 के अनुपात म बां टना है ।
यिद िम ण म 5 लीटर दू ध और 4 लीटर पानी िमला िदया जाए तो पानी और दू ध का नया अनुपात ा होगा?
A. 20:9 B. 3:7
C. 9:20 D. 9:7
E. None of these

266. Samarth spends 30% of his income on education, 20% of remaining on food and travel,
together. His expenditure on education, food and travel, together is Rs. 15840. Find his
savings if he saves 25% of his income.
समथ अपनी आय का 30% िश ा पर, शेष का 20% भोजन और या ा पर एक साथ खच करता है । उनका
िश ा, भोजन और या ा पर कुल िमलाकर खच . 15840. यिद वह अपनी आय का 25% बचाता है तो
उसकी बचत ात कीिजए।
A. 12000 B. 15000
C. 10450 D. 9000
E. None of these

267. A and B together started a business with initial investments in the ratio 14:x respectively.
After 3 months, C joined them in such a way that the ratio of initial investment of B and C is
3x:39. If after one year, A got Rs.7560 out of total profit of Rs.17685, then find the value of
x?
A और B ने एक साथ मश: 14:x के अनुपात म ारं िभक िनवेश के साथ एक वसाय शु िकया । 3 महीने
के बाद, C उनके साथ इस कार शािमल आ िक B और C के शु आती िनवेश का अनुपात 3x:39 है । यिद
एक वष के बाद, A को 17685 पये के कुल लाभ म से 7560 पये िमले , तो x का मान ात कीिजए ?
A. 13 B. 11
C. 15 D. 14
E. None of these

268. There are two natural numbers 'x' and 'y' such that xy=1575. If (x-y) is 1/8th of (x+y), then
find the ratio of 'x' to 'y'.
दो ाकृत सं ाएँ 'x' और 'y' इस कार ह िक xy=1575. यिद (x-y), (x+y) का 1/8वां है , तो 'x' से 'y' का
अनुपात ात कीिजए।
A. 1:5 B. 9:7
C. 5:7 D. 9:5
E. None of these

269. A bullet is fired towards a car which is 34 meters apart , with a speed of 1m/sec. The air is
flowing with a speed of 0.25m/sec in the direction same as that of car. If the car is moving

PAGE h ps://t.me/studified h ps://instagram.com/aashisharorasocial?utm_medium=copy_link


Follow
112 Aashish Arora on: h ps://youtube.com/channel/UCYa4_JrOrf8R5Kz2uOtccXQ https://www.facebook.com/aashisharorasocial/
अ ास by Aashish Arora
(SBI/IBPS/RBI/LIC/All other banking and insurance Exams)

with a speed of 27 km/hr towards the bullet then find the time after which bullet will hit the
car.
एक कार, जो 34 मीटर की दू री पर है , की ओर 1 मीटर/सेकंड की गित से एक गोली चलाई जाती है । हवा कार
के समान िदशा म 0.25m/sec की गित से बह रही है । यिद कार गोली की ओर 27 िकमी/घ ा की चाल से
चल रही है , तो वह समय ात कीिजए िजसके बाद गोली कार को लगेगी।
A. 2 sec B. 6 sec
C. 3 sec D. 4 sec
E. None of these

270. The cost price of cultivating a square field at the rate of Rs.9.5/m2 is Rs. 8550. If each side
of the field is increased by 20%, if the amount of cultivation remains same then find the
decreased cost of cultivation per m².
2
.9.5/m की दर से एक वगाकार खेत की खेती का लागत मू . 8550. यिद खेत के ेक प म 20%
2
की वृ हो जाती है , यिद खेती की मा ा समाने रहती है , तो ित m खेती की घटी ई लागत ात कीिजए।
A. Rs.2.9 B. Rs.1.5
C. Rs.3.9 D. Rs.2.4
E. None of these

271. A man invested Rs.3600 in each of different schemes for 2 years, at simple interest. The
rate of interest are 16%p.a, 13%p.a and 11%p.a respectively. Find the total amount
received.
एक ने साधारण ाज पर 2 वष के िलए ेक िविभ योजनाओं म 3600 पये का िनवेश िकया।
ाज दर मशः 16% ित वष, 13% ित वष और 11% ित वष है । ा कुल रािश ात कीिजए।
A. Rs.12540 B. Rs.15450
C. Rs.13680 D. Rs.14660
E. None of these

272. 5 years ago, the ratio of the age of Manish and Ravi was 5:3. three years hence, the ratio of
their ages will be 9:7. What is the present age of Ravi?
5 वष पहले, मनीष और रिव की आयु का अनुपात 5:3 था। तीन वष बाद, उनकी आयु का अनुपात 9:7 होगा।
रिव की वतमान आयु ा है ?
A. 12 years B. 17 years
C. 10 years D. 11 years
E. None of these

273. A box contains 8 yellow, 5 black, 3 and 4 red ball. If four ball are picked up at random, what
is probability that 2 are black and 2 are white?
एक बॉ म 8 पीली, 5 काली, 3 सफेद और 4 लाल गद ह। यिद या क प से चार गद उठाई जाती ह, तो
2 के काले और 2 के सफेद होने की ाियकता ा है ?
A. 12/323 B. 1/232
C. 2/323 D. 5/233
E. None of these

274. 320 litres of mixture of petrol and diesel, contains 65% petrol. When 'x' litres of diesel is
added in the mixture, the quantity of diesel becomes 25% less than that of petrol. Find the
value of x.
पेटोल और डीजल के 320 लीटर िम ण म 65% पेटोल होता है । जब िम ण म 'x' लीटर डीजल िमलाया जाता

PAGE h ps://t.me/studified h ps://instagram.com/aashisharorasocial?utm_medium=copy_link


Follow
113 Aashish Arora on: h ps://youtube.com/channel/UCYa4_JrOrf8R5Kz2uOtccXQ https://www.facebook.com/aashisharorasocial/
अ ास by Aashish Arora
(SBI/IBPS/RBI/LIC/All other banking and insurance Exams)

है , तो डीजल की मा ा पेटोल की मा ा से 25% कम हो जाती है । ए का मान ात कर।


A. 44 B. 45
C. 47 D. 55
E. None of these

275. The profit percent of an article is calculated as 38% on selling price. Find the actual profit
percent earned on the article.
एक व ु के लाभ ितशत की गणना िब ी मू पर 38% के प म की जाती है । लेख पर अिजत वा िवक
लाभ ितशत ात कीिजए।
A. 61.29% B. 25.26%
C. 74.25% D. 65.25%
E. None of these

276. X men working 16 hours a day can complete a work in 18 days while (5x-3) men working 9
hours a day can complete the same work in 8 days. In how many days can (x+3) men
working 16 hours a day complete the same work?
X पु ष एक िदन म 16 घंटे काय करते ए एक काय को 18 िदनो ं म पूरा कर सकते ह जबिक (5x-3) पु ष 9
घंटे ितिदन काय करके उसी काय को 8 िदनो ं म पूरा कर सकते ह। िकतने िदनो ं म (x +3) पु ष िदन म 16
घंटे काम करके उसी काम को पूरा कर सकते ह?
A. 10 B. 7
C. 9 D. 8
E. None of these

277. 'A' and 'B' started a business together such that the 'A' contributed 80% of total sum. After 6
months, C joined the business and the sum invested by 'A' becomes equal to 40% of total
sum invested by all three together. If the share of 'C' out of total profit received at the end of
the year is Rs 8525, then find the profit share of 'A' and 'B' together.
'A' और 'B' ने एक साथ एक वसाय शु िकया, िजसम 'A' ने कुल योग का 80% योगदान िदया। 6 महीने के
बाद, C वसाय म शािमल हो जाता है और 'A' ारा िनवेश की गई रािश तीनो ं ारा एक साथ िनवेश की गई
कुल रािश के 40% के बराबर हो जाती है । यिद वष के अंत म ा कुल लाभ म से 'C' का िह ा 8525 पये
है , तो 'A' और 'B' का एक साथ लाभ का िह ा ात कीिजए।
A. 22550 B. 21510
C. 12350 D. 17050
E. None of these

278. The present age of Ruhi is 75% more than the present age of Samrat. The sum of ages
Ruhi, Ricky, and Samrat after 6 years will be 129 years. Find the present age of Samrat, if
the present age of Ricky is 6 years less than the present age of Ruhi.
ही की वतमान आयु स ाट की वतमान आयु से 75% अिधक है । 6 वष बाद ही, रकी और स ाट की
आयु का योग 129 वष होगा। स ाट की वतमान आयु ात कीिजए, यिद रकी की वतमान आयु ही की
वतमान आयु से 6 वष कम है ।
A. 46 B. 34
C. 26 D. 58
E. None of these

279. When P starts a work at 08:00am, he finishes it at 02:00pm but when Q starts the same

PAGE h ps://t.me/studified h ps://instagram.com/aashisharorasocial?utm_medium=copy_link


Follow
114 Aashish Arora on: h ps://youtube.com/channel/UCYa4_JrOrf8R5Kz2uOtccXQ https://www.facebook.com/aashisharorasocial/
अ ास by Aashish Arora
(SBI/IBPS/RBI/LIC/All other banking and insurance Exams)

work at 11:00am, he finishes 80% of the work at 07:00pm. If P and Q together start the
work at 04:30pm, then, when they will finish it?
जब P िकसी काम को सुबह 08:00 बजे शु करता है , तो वह उसे दोपहर 02:00 बजे ख करता है , लेिकन
जब Q उसी काम को सुबह 11:00 बजे शु करता है , तो वह रात के 07:00 बजे काम का 80% पूरा करता
है । यिद P और Q िमलकर काम को दोपहर 04:30 बजे शु करते ह, तो वे इसे कब पूरा करगे?
A. 7:10 pm B. 8:15 pm
C. 8:45 pm D. 7:20 pm
E. None of these

280. Naveen and Nishant entered into a partnership and Naveen invested Rs. 2800 more than
Nishant. After 6 months, Naveen withdraws Rs. 800 and Nishant invested Rs. 900 more. If
the profit at the end of the year is 126,000 and share of Naveen is 72,000, then what is the
initial investment by Nishant?
नवीन और िनशां त ने एक साझेदारी म वेश िकया और नवीन ने. िनशां त से 2800 अिधक िनवेश िकया। 6
महीने के बाद, नवीन . 800 िनकालता है और िनशां त ने . 900 और िनवेश करता है । यिद वष के अंत म
लाभ 126,000 है और नवीन का िह ा 72,000 है , तो िनशां त ारा ारं िभक िनवेश ा है ?
A. Rs. 3600 B. Rs. 4200
C. Rs. 5400 D. Rs. 2900
E. None of these

281. Ratio of speed of a person to that of a train of length 560 meters is 2:9 and time taken by
train to cross the person running in the opposite direction is 36 seconds, then find the time
taken by train to cross the person when running in same direction.
एक की गित का 560 मीटर लंबी टे न की गित से अनुपात 2:9 है और िवपरीत िदशा म चल रहे
को पार करने म टे न ारा िलया गया समय 36 सेकंड है , तो समान िदशा म दौड़ते समय टे न ारा को
पार करने म लगने वाला समय ात कीिजए।
A. 37.9 seconds B. 25.6 seconds
C. 33.6 seconds D. 56.57 seconds
E. None of these

282. The circumference of a circular field is (1/2) times of the perimeter of a rectangular field.
The area of the circle is 7546 m2. What is the length of the rectangular field if the breadth of
the rectangular field is 240 meters?
एक वृ ाकार े की प रिध एक आयताकार े की प रिध की (1/2) गुना है । वृ का े फल 7546 वग
मीटर है । यिद आयताकार खेत की चौड़ाई 240 मीटर है तो आयताकार खेत की लंबाई ा होगी?
A. 68 m B. 52 m
C. 76 m D. 44 m
E. None of these

283. Pranav deposited some amount in PNB for 4 years and got a total interest (simple interest)
of Rs. 1200. Manav also deposited the same amount in Badhan bank for 2 years and got a
total interest of Rs. 640(compound interest). If both the banks are offering same rate of
interest, then what is the rate of interest offered by the bank?
णव ने पीएनबी म 4 साल के िलए कुछ रािश जमा की और कुल 1200 . का ाज (साधारण ाज) िमला।
मानव ने भी समान रािश 2 साल के िलए बाधन बक म जमा िकया और कुल 640 का ाज (च वृ ाज)
िमला। यिद दोनो ं बक समान ाज दर की पेशकश कर रहे ह, तो बक ारा दी जाने वाली ाज दर ा है ?

PAGE h ps://t.me/studified h ps://instagram.com/aashisharorasocial?utm_medium=copy_link


Follow
115 Aashish Arora on: h ps://youtube.com/channel/UCYa4_JrOrf8R5Kz2uOtccXQ https://www.facebook.com/aashisharorasocial/
अ ास by Aashish Arora
(SBI/IBPS/RBI/LIC/All other banking and insurance Exams)

A. 16% B. 24.8%
C. 13.33% D. 8.6%
E. None of these

284. Speed of a boat is 32km/hrs in still water and the speed of the stream is 8km/hrs. If the boat
takes 16hrs to go to a place and come back, the distance of the place is ?
शां त जल म एक नाव की गित 32 िकमी/घंटा है और धारा की गित 8 िकमी/घंटा है । यिद नाव को िकसी थान
पर जाने और वापस आने म 16 घंटे लगते ह, तो उस थान की दू री है ?
A. 386 km B. 240 km
C. 180 km D. 280 km
E. None of these

285. A teacher found that average marks of his class to be 43 misread 15 as 51 and hence the
average is increased by 0.9 units. Find the number of students in the class.
एक िश क ने पाया िक उसकी क ा के औसत अंक 43 ह और 15 को 51 के प म पढ़ा जाता है और
इसिलए औसत म 0.9 इकाई की वृ होती है । क ा म छा ो ं की सं ा ात कीिजए।
A. 40 B. 34
C. 56 D. 49
E. None of these

286. Atish donated 60% of his wealth to charity and 70% of the remaining for cancer research.
He gave 80% of his remaining wealth to his brother after which he had Rs. 49200. Left.
How much property did Aatish have in total?
आितश ने अपनी दौलत का 60% दान म िदया और शेष का 70% कसर अनुसंधान के िलए िदया। उसने
अपनी बची ई दौलत का 80% अपने भाई को दे िदया िजसके बाद उसके पास . 49200. बचे। आितश के
पास कुल िकतनी संपि थी।
A. Rs. 1,800,000 B. Rs. 2,050,000
C. Rs. 2,000,000 D. Rs. 3,080,000
E. None of these

287. Average of 5 consecutive odd numbers is 57. What will be average of next 5 consecutive
odd numbers ?
5 मागत िवषम सं ाओं का औसत 57 है । अगली 5 मागत िवषम सं ाओं का औसत ा होगा?
A. 335 B. 379
C. 280 D. 310
E. None of these

288. After spending 35% in equipment, 20% in building, 15% in raw material, 5% in computer
and 10% in furniture. Rahul had left a balance of Rs. 74250. The initial money with Rahul
was.
उपकरण पर 35 ितशत, भवन िनमाण म 20 ितशत, क े माल पर 15 ितशत, क ूटर पर 5 ितशत
और फन चर पर 10 ितशत खच करने के बाद रा ल के पास 74250 पये शेष थे। रा ल के पास शु आती
पैसा था।
A. Rs. 403500 B. Rs. 340500
C. Rs. 495000 D. Rs. 580000
E. None of these

PAGE h ps://t.me/studified h ps://instagram.com/aashisharorasocial?utm_medium=copy_link


Follow
116 Aashish Arora on: h ps://youtube.com/channel/UCYa4_JrOrf8R5Kz2uOtccXQ https://www.facebook.com/aashisharorasocial/
अ ास by Aashish Arora
(SBI/IBPS/RBI/LIC/All other banking and insurance Exams)

289. The length and breadth of a rectangular piece of land are in the ratio of 11:9 respectively.
The total cost of fencing the land is Rs. 10240 at the rate of Rs. 8 per meter. Then, what is
the area of the rectangular land?
भूिम के एक आयताकार टु कड़े की लंबाई और चौड़ाई मशः 11:9 के अनुपात म है । भूिम की बाड़ लगाने की
कुल लागत 8 ित मीटर की दर से 10240 पये है । तो आयताकार भूिम का े फल ा है ?
A. 110176 m2 B. 101376 m2
2
C. 120376 m D. 130450 m2
E. None of these

290. Rima and Sima entered into a partnership and Rima invested Rs. 3200 more than Sima.
After 4months, Rima withdraws Rs. 2700 and Sima invested Rs. 1200 more. If the share of
Rima is 72,000, and profit at the end of the year is 120,000, what is the initial investment by
sima?
रीमा और सीमा ने एक साझेदारी म वेश िकया और रीमा ने . सीमा से 3200 अिधक। 4 महीने के बाद, रीमा
पये िनकालती है । 600 और सीमा ने पये का िनवेश िकया। 1000 और। यिद रीमा का िह ा 72,000 है ,
और वष के अंत म लाभ 120,000 है , तो सीमा ारा ारं िभक िनवेश ा है ?
A. Rs. 400 B. Rs. 1200
C. Rs. 500 D. Rs. 800
E. None of these

291. The average number of printing errors per page in a book of 610 pages is 6.If the total
number of printing errors in the first 456 pages is 2736, the average number of printing
errors per page in the remaining page is:
610 पृ ो ं की एक पु क म ित पृ मु ण ुिटयो ं की औसत सं ा 6 है । यिद पहले 456 पृ ो ं म मु ण ुिटयो ं
की कुल सं ा 2736 है , तो शेष पृ म ित पृ मु ण ुिटयो ं की औसत सं ा है
A. 0 B. 8
C. 6 D. 154
E. None of these

292. P1 and P2 are two foods made of maida and besaan the proportion 5:4 and 5:13,
respectively. If equal quantities of the food are cooked to form a third food P3 the
proportion of maida and besaan in food P3 is?
P1 और P2 मैदा और बेसन से मशः 5:4 और 5:13 के अनुपात म बने दो खा पदाथ ह। यिद समान मा ा
म खाना पकाया जाता है तो तीसरा भोजन P3 बनता है , भोजन P3 म मैदा और बेसन का अनुपात िकतना है ?
A. 5:7 B. 13:7
C. 9:5 D. 5:9
E. None of these

293. A bike runs at the speed of 40 when not serviced and runs at 65km/h, when serviced. After
servicing, the bike covers a certain distance in 5hrs. How much approximate time will the
bike take to cover the same distance when not serviced?
एक बाइक सिवस न होने पर 40 की गित से चलती है और सिवस करने पर 65 िकमी/घंटा की गित से चलती
है । सिविसंग के बाद, बाइक 5 घंटे म एक िनि त दू री तय करती है । जब सिवस नही ं की जाएगी तो बाइक को
समान दू री तय करने म िकतना समय लगेगा?
A. 14 B. 24
C. 8 D. 6
E. None of these

PAGE h ps://t.me/studified h ps://instagram.com/aashisharorasocial?utm_medium=copy_link


Follow
117 Aashish Arora on: h ps://youtube.com/channel/UCYa4_JrOrf8R5Kz2uOtccXQ https://www.facebook.com/aashisharorasocial/
अ ास by Aashish Arora
(SBI/IBPS/RBI/LIC/All other banking and insurance Exams)

294. Speed of a boat is 72km/hrs in still water and the speed of the stream is 9km/hrs. If the boat
takes 32hrs to go to a place and come back, the distance of the place is ?
शां त जल म एक नाव की गित 32 िकमी/घंटा है और धारा की गित 8 िकमी/घंटा है । यिद नाव को िकसी थान
पर जाने और वापस आने म 32 घंटे लगते ह, तो उस थान की दू री है ?
A. 1386 km B. 1134 km
C. 1580 km D. 2180 km
E. None of these

295. If 8 women or 17 girls can do a piece of work in 26 days, how many days will it take for 4
women and 24 girls to a piece of work 50×0.9 times as great?
यिद 8 मिहलाएं या 17 लड़िकयां िकसी काम को 26 िदनो ं म कर सकती ह, तो 4 मिहलाओं और 24 लड़िकयो ं
को 50×0.9 गुना बड़े काम को पूरा करने म िकतने िदन लगगे?
A. 612 days B. 349 days
C. 568 days D. 62 days
E. None of these

296. Mr. Laheri completes 60% of the work in 15days. Then Mr. Sharma also joins and they
together finish the rest of the work in 3days. How long will it take for Mr. Sharma alone to
finish?
ी लाहे री 60% काय को 15 िदनो ं म पूरा करते ह। िफर ी शमा भी शािमल हो जाते ह और वे एक साथ शेष
काय को 3 िदनो ं म पूरा करते ह। अकेले ी शमा को समा करने म िकतना समय लगेगा?
A. 9*2/5 days B. 10*5/7 days
C. 10*3/5 days D. 10*1/5 days
E. None of these

297. A bed and a almirah were sold for Rs 15000 each. The bed was sold at a gain of 25% and
the almirah was sold at a loss of 25%. The gain or the loss percent on the whole
transaction is.
एक िब र और एक अलमारी 15000 ेक म बेची गई। िब र को 25% के लाभ पर बेचा गया और
अलमारी को 25% की हािन पर बेचा गया। पूरे लेन-दे न पर लाभ या हािन ितशत है ।
A. No loss or profit B. 4% gain
C. 4% loss D. 10% loss
E. None of these

298. Anuj can complete (1/3)rd of the work in 5 days and the rest of work is done be Bikash in 10
days. If Bikash is 20% less efficient than Dinesh, then how many days all together can
complete (13/15)th of the work?
अनुज काय का (1/3) भाग 5 िदनो ं म पूरा कर सकता है और शेष काय 10 िदनो ं म िबकेश करता है । यिद
िबकाश, िदनेश से 20% कम कुशल है , तो सभी िमलकर काय का 13/15वां भाग िकतने िदनो ं म पूरा करगे?
A. 5 days B. 4 days
C. 6 days D. 3 days
E. None of these

299. Rahul, Ramesh and Rohit entered in a partnership with sums in the ratio of 4:3:7,
respectively. Rahul, Ramesh and Rohit invested money for 9 months, 8 months and 10
months respectively. The profit received by Rahul and Ramesh together is how much
percent more/less than the profit received by Rohit?

PAGE h ps://t.me/studified h ps://instagram.com/aashisharorasocial?utm_medium=copy_link


Follow
118 Aashish Arora on: h ps://youtube.com/channel/UCYa4_JrOrf8R5Kz2uOtccXQ https://www.facebook.com/aashisharorasocial/
अ ास by Aashish Arora
(SBI/IBPS/RBI/LIC/All other banking and insurance Exams)

रा ल, रमेश और रोिहत ने मशः 4:3:7 के अनुपात म रािश के साथ एक साझेदारी म वेश िकया। रा ल,
रमेश और रोिहत ने मशः 9 महीने, 8 महीने और 10 महीने के िलए पैसा िनवेश िकया। रा ल और रमेश को
िमलाकर ा लाभ, रोिहत ारा ा लाभ से िकतने ितशत अिधक/कम है ?
A. 14.28 % B. 16.33 %
C. 13.66 % D. 19.28 %
E. None of these

300. Mixture 'X'(Fanta and Maaza) contain 150 liters Fanta and rest Maaza. 60% of the mixture
'X' is taken out and 120 liters of Mixture 'Y' of Fanta and Maaza only which contain 30% of
Fanta is added to remaining mixture such that ratio of Fanta and maaza becomes 4:5.
Find the initial quantity of Maaza in mixture 'X'.
िम ण ‘X’ (फटा और माज़ा) म 150 लीटर फटा और बाकी माज़ा है । िम ण ‘X’ का 60% िनकाल िलया जाता
है और केवल फटा और माज़ा के 120 लीटर िम ण ‘Y’ िजसम 30% फटा होता है , को शेष िम ण म इस
कार िमलाया जाता है िक फटा और माज़ा का अनुपात 4:5 हो जाता है । िम ण ‘X’ म माज़ा की ारं िभक मा ा
ात कीिजए।
A. 100 litres B. 80 litres
C. 70 litres D. 90 litres
E. None of these

301. The speed of two cars X and Y is 70km/hr and 28km/hr respectively. Find the time in which
distance covered by X is 189 km more than Y .
दो कारो ं X और Y की गित मशः 70 िकमी/घंटा और 28 िकमी/घंटा है । वह समय ात कीिजए िजसम X
ारा तय की गई दू री Y से 189 िकमी अिधक है ।
A. 3.5 hr B. 4.5 hr
C. 4 hr D. 6 hr
E. None of these

302. The ratio of the speed of boat in still water to the speed of the current is 8:5, respectively. If
time taken by Boat to travel (D+5) km downstream is equal to the time taken by boat to
travel (D-45)km upstream, then find the value of D.
शां त जल म नाव की गित की धारा की गित से अनुपात मशः 8:5 है । यिद नाव ारा धारा के अनुकूल
(D+5)km या ा करने म िलया गया समय, नाव ारा धारा के ितकूल (D-45)km या ा करने म िलए गए
समय के बराबर है , तो D का मान ात कीिजए।
A. 45 B. 60
C. 80 D. 75
E. None of These
2
303. The total surface area of A is cube is 288 cm more than its curved surface area. If each
edge o f cube B is 75% more than of cube A, then find the sum of curved surface area and
total surface area of cube B.
A का कुल पृ ीय े फल घन है , इसके व पृ ीय े फल से 288 cm2 अिधक है । यिद घन B का ेक
िकनारा घन A से 75% अिधक है , तो घन B के व पृ ीय े फल और कुल पृ ीय े फल का योग ात
कीिजए।
A. 3620 cm2 B. 4410 cm2
2
C. 5124 cm D. 6616 cm2
E. None of These

PAGE h ps://t.me/studified h ps://instagram.com/aashisharorasocial?utm_medium=copy_link


Follow
119 Aashish Arora on: h ps://youtube.com/channel/UCYa4_JrOrf8R5Kz2uOtccXQ https://www.facebook.com/aashisharorasocial/
अ ास by Aashish Arora
(SBI/IBPS/RBI/LIC/All other banking and insurance Exams)

304. The cost price of two articles is same. One article is sold at 20% profit and another at 15%
loss. If the selling price of one article is Rs.595/- more than that of another, then find the
selling price of article sold at loss?
दो व ुओं का य मू समान है । एक व ु को 20% लाभ पर और दू सरी को 15% हािन पर बेचा जाता है ।
यिद एक व ु का िव य मू दू सरी व ु के िव य मू से पये 595/- अिधक है , तो हािन पर बेची गई व ु
का िव य मू ात कीिजए ?
A. 1454 Rs. B. 1545 Rs.
C. 1554 Rs. D. 1445 Rs.
E. None of these

305. The ratio of the ages of A and B, 6 years hence from now will be 5:9, respectively while the
ratio of their ages 3 years ago from now was 13:27, respectively. Find the difference
between their present ages ?
अब से 6 वष बाद A और B की आयु का अनुपात मशः 5:9 होगा, जबिक 3 वष पहले उनकी आयु का
अनुपात अब से मशः 13:27 था। उनकी वतमान आयु के बीच का अंतर ात कीिजए?
A. 30 B. 26
C. 28 D. 22
E. None of these

306. There are six numbers 34, 18, 12, 14, 22 and 8. Which one of the given number can be
removed such that the average of the remaining numbers remains same?
छ: सं ाएँ 34, 18, 12, 14, 22 और 8 ह। दी गई सं ाओं म से िकस एक को इस कार हटाया जा सकता
है िक शेष सं ाओं का औसत समान रहे ?
A. 12 B. 18
C. 14 D. 22
E. None of these

307. A certain sum of money becomes Rs. 6000 in 5 years and Rs. 7200 in 7 years at any
certain rate of S.I. Find the principle amount.
एक िनि त रािश साधारण ाज की िकसी िनि त दर 5 साल म . 6000 और 7 वष म . 7200 पये हो
जाती है । मूलधन ात कीिजए।
A. Rs.2000 B. Rs.3600
C. Rs.2400 D. Rs.3000
E. None of these

308. A certain number of men were asked to complete a work in 12 days but 5 men didn't join the
work so, that work gets delayed by 12 days. Find the number of men asked to complete the
work in starting.
एक िनि त सं ा म पु षो ं को एक काम को 12 िदनो ं म पूरा करने के िलए कहा गया था लेिकन 5 आदमी
काम म शािमल नही ं ए, िजससे काम म 12 िदन की दे री हो गई। आरं भ म काय को पूरा करने के िलए कहे
जाने वाले पु षो ं की सं ा ात कीिजए।
A. 25 B. 22
C. 10 D. 20
E. None of these

309. Rs 410 has to be distributed in a group of 60 students of some boys and girls such that
each boy receives Rs 5 and each girl gets Rs 10. Find the difference between the number
of boys and girls.

PAGE h ps://t.me/studified h ps://instagram.com/aashisharorasocial?utm_medium=copy_link


Follow
120 Aashish Arora on: h ps://youtube.com/channel/UCYa4_JrOrf8R5Kz2uOtccXQ https://www.facebook.com/aashisharorasocial/
अ ास by Aashish Arora
(SBI/IBPS/RBI/LIC/All other banking and insurance Exams)

410 पये कुछ लड़को ं और लड़िकयो ं के 60 छा ो ं के समूह म इस कार िवत रत िकए जाने ह िक ेक
लड़के को 5 पये िमले और ेक लड़की को 10 पये िमले। लड़के और लड़िकयो ं की सं ा के बीच का
अंतर ात कीिजए।
A. 16 B. 72
C. 35 D. 40
E. None of these

310. There are 12 green balls, 36 red balls, 12 blue balls in a bag. Three balls are drawn at
random with replacement. Find the probability of getting a red ball on 1st draw, a green
ball on 2nd draw and a blue ball on 3rd draw.
एक थैले म 12 हरी गद, 36 लाल गद, 12 नीली गद ह। ित थापन के साथ या क प से तीन गद
िनकाली जाती ह। पहले डॉ पर लाल गद, दू सरे डॉ पर हरी गद और तीसरे डॉ पर नीली गद िमलने की
ाियकता ात कीिजए।
A. 2/125 B. 3/125
C. 5/137 D. 7/125
E. None of these

311. The ratio of the number of machines used by X and Y is 4:5 respectively while the ratio of
the amount of electricity consumed by each machine used by them is 1:3 respectively. If
the cost of each unit of electricity used is Rs 4 and total bill paid by Y is Rs 72180 then find
the bill paid by X.
X और Y ारा उपयोग की जाने वाली मशीनो ं की सं ा का अनुपात मशः 4:5 है जबिक उनके ारा
उपयोग की जाने वाली ेक मशीन ारा उपयोग की जाने वाली िबजली की मा ा का अनुपात मशः 1:3
है । यिद उपयोग की गई िबजली की ेक इकाई की लागत 4 पये है और Y ारा भुगतान िकया गया कुल
िबल 72180 पये है , तो X ारा भुगतान िकया गया िबल ात कीिजए।
A. v 19248 B. 19455
C. 18545 D. 18454
E. None of these

312. Sekhar and Mantoo invested Rs 20000 and Rs 16000, in a business together. After 4
months Sekhar withdrew Rs 8000 from his initial investment while Mantoo withdrew Rs
6000 from his initial investment. If the total profit received by them at the end of 1 year is Rs
50700, then find the profit share of Sekhar.
शेखर और मंटू ने एक साथ एक ापार म 20000 पये और 16000 पये का िनवेश िकया। 4 महीने के बाद
शेखर ने अपने शु आती िनवेश से 8000 पये वापस ले िलए जबिक मंटू ने अपने शु आती िनवेश से 6000
पये वापस ले िलए। यिद 1 वष के अंत म उनके ारा ा कुल लाभ 50700 पये है , तो शेखर का लाभ
िह ा ात कीिजए।
A. 25000 B. 21498
C. 24542 D. 27885
E. None of these

313. A boat can cover 105 km in downstream in 1.5 hours. If the speed of the boat in still water
has been 100% more, then it would travel 125 km upstream in 2.5 hours. Find the speed of
the current.
एक नाव 1.5 घंटे म धारा के अनुकूल 105 िकमी की दू री तय कर सकती है । यिद शां त जल म नाव की गित
100% अिधक होती, तो वह 2.5 घंटे म धारा के ितकूल 125 िकमी की या ा करती। धारा की गित ात
कीिजए।

PAGE h ps://t.me/studified h ps://instagram.com/aashisharorasocial?utm_medium=copy_link


Follow
121 Aashish Arora on: h ps://youtube.com/channel/UCYa4_JrOrf8R5Kz2uOtccXQ https://www.facebook.com/aashisharorasocial/
अ ास by Aashish Arora
(SBI/IBPS/RBI/LIC/All other banking and insurance Exams)

A. 32 B. 16
C. 30 D. 54
E. None of these

314. The ratio of the length and breadth of a rectangle is 9:4 respectively. If the perimeter of the
rectangle is 780 meters, then find the area of the rectangle.
एक आयत की लंबाई और चौड़ाई का अनुपात मशः 9:4 है । यिद आयत का प रमाप 780 मीटर है , तो
आयत का े फल ात कीिजए।
A. 20544 B. 18100
C. 32400 D. 21200
E. None of these

315. A certain money amounts to Rs 144 in 4 years and Rs 400 in 6 years, when invested at X%
p.a. compound interest compounded annually. Find the amount received when Rs 840 is
invested at X% p.a. compound interest, compounded annually for 1 year.
एक िनि त रािश 4 वष म 144 पये और 6 वष म 400 पये हो जाती है , जब सालाना च वृ ाज X%
पर िनवेश िकया जाता है । ा रािश का पता लगाएं , जब 840 पये को X% ित वष च वृ ाज पर
िनवेश िकया जाता है , जो 1 वष के िलए वािषक प से संयोिजत होता है ।
A. 1200 B. 1400
C. 1000 D. 1025
E. None of these

316. The ratio of the cost prices of two article X and Y is 4:9 respectively. If article X is sold at
25% profit while article Y is sold at 20% loss, then find the overall profit/loss percent on the
given two articles.
दो व ु X और Y के य मू का अनुपात मशः 4:9 है । यिद व ु X को 25% लाभ पर बेचा जाता है
जबिक व ु Y को 20% हािन पर बेचा जाता है , तो दी गई दो व ुओं पर कुल लाभ/हािन ितशत ात
कीिजए।
A. 8.5%profit B. 7.15%loss
C. 6.25%profit D. 6.15% loss
E. None of these

317. The ratio of the income and savings of Subham, in 2020 was 8:5. In 2021 his income
increased by 30% while his savings decreased by 20%, If his expenditure in 2021 was Rs
32000, then find his expenditure in 2020.
2020 म शुभम की आय और बचत का अनुपात 8:5 था। 2021 म उसकी आय म 30% की वृ ई जबिक
उसकी बचत म 20% की कमी आई, यिद 2021 म उसका य 32000 पये था, तो 2020 म उसका य
ात कीिजए।
A. 14000 B. 15000
C. 15500 D. 14500
E. None of these

PAGE h ps://t.me/studified h ps://instagram.com/aashisharorasocial?utm_medium=copy_link


Follow
122 Aashish Arora on: h ps://youtube.com/channel/UCYa4_JrOrf8R5Kz2uOtccXQ https://www.facebook.com/aashisharorasocial/
अ ास by Aashish Arora
(SBI/IBPS/RBI/LIC/All other banking and insurance Exams)

SOLUTION

1. Ans. (D) Area of Rectangle,


Initially, Coconut oil = 17x L, Coconut (x + 17)(2x - 5) = 551
Water = 18x L 2x2 + 29x - 85 = 551
2x2 + 29x - 636 = 0
17x - 85 + 8 3
= (x - 12)(2x + 53) = 0
18x - 90 + 62 4 x = 12
4(17x - 77) = 3(18x - 28) Length = 29 cm, Breadth = 19 cm
68x - 308 = 54x - 84 Perimeter = 2(29 + 19) = 96 cm
14x = 224
x = 16 7. Ans. (C)
Initial Quantity of Mixture = 35x = 35 × 16 Income of Kevin = x Rs.
= 560 Liters 0.55x(1.12 – 1) + 0.45x × 0.225 × 2 =
3816
2. Ans. (B) 0.55x × 0.21 + 0.2025x = 3816
Total work = 45 × 64 = 57x + 58 × 30 0.1155x + 0.2025x = 3816
2880 - 1740 = 57x = 1140, x = 20 days 0.318x = 3816
x = 12000 Rs
3. Ans. (C)
Total investment of P = 2400 × 12 Rs 8. Ans. (D)
Total investment of Q = 3600 × 4 Rs x (21 + 2) 9
Total investment of R = 3200 × 8 Rs ´ =
Profit Ratio = P:Q:R = (2400 × 12):(3600 (21 - 2) (x + 164) 11
× 4):(3200 × 8) = 18:9:16 x 23 9
´ =
1782 ´ 43 19 (x + 164) 11
Total Profit = = 4257 Rs.
18 11 × 23x = 9 × 19(x + 164)
4. Ans. (B) 253x = 171x + 28044
82x = 28044
1260 x = 342
Total Salary = = 8640 Rs.
æ 5 öæ 7 ö
ç ÷ç ÷ ´ 0.8 9. Ans. (D)
è 12 øè 16 ø
Money Spent on Rent = 8640 × (5/12) × 36 ´ 75 - (89 + 87 + 77) + (98 + 67 + 13)
(9/16) = 2025 Rs. 35
5. Ans. (A) 2700 - 253 + 178 2625
= = = 75
Initial Salary = 1440 ÷ 0.32 = 4500 Rs. 35 35
Initial Expense = 4500 - 1440 = 3060 Rs. No Correction Required
New Savings = 1440 × 1.375 = 1980 Rs.
New Expense = 0.75 × 3060 = 2295 Rs. 10. Ans. (B)
New Salary = 2295 + 1980 = 4275 Rs. 8 year ago, Age of Ramesh = 5x, Suresh
Salary Decrease by = 4500 – 4275 =
= 8x
225 Rs. 12 year later
6. Ans. (A)

PAGE h ps://t.me/studified h ps://instagram.com/aashisharorasocial?utm_medium=copy_link


Follow
123 Aashish Arora on: h ps://youtube.com/channel/UCYa4_JrOrf8R5Kz2uOtccXQ https://www.facebook.com/aashisharorasocial/
अ ास by Aashish Arora
(SBI/IBPS/RBI/LIC/All other banking and insurance Exams)

5x + 8 + 12 10 6090x = 2320(14300 + x)
= 6090x - 2320x = 2320 × 14300
8x + 8 + 12 13
3770x = 2320 × 14300
13(5x + 20) = 10(8x + 20) x = 8800 Rs.
65x + 260 = 80x + 200
15x = 60 15. Ans. (A)
x=4 Let they meet after t hour
Present Age of Ramesh = 5 × 4 + 8 = 28 30t - 24t = 108
year 6t = 108
Present Age of Suresh = 8 × 4 + 8 = 40 t = 18 hour
year Initial distance between them = 30 × 18
+ 24 × 18 = 972 km
28 + 40
Average = = 34 years
2 16. Ans. (A)
11. Ans. (D) Original speed of boat in still water = x
Initial amount of milk = x L, water = x – km/hr
70 L 6.8(x + 12 + 15) = 374
(x + 80) × 0.5 = x - 70 x + 27 = 55
0.5x + 40 = x - 70 x = 28 km/hr
0.5x = 110
x = 220 L 17. Ans. (C)
Initial Quantity of Mixture = 2x - 70 = Area of frame = 8712/4.5 = 1936 cm2
2×220 - 70 = 370 Litres Side of square = √1936 = 44 cm
Perimeter of frame = 4 × 44 = 176 cm
12. Ans. (B)
Total work 18. Ans. (D)
18 × 19 = 9x + 15 × 12 (Y + 800) × 0.38 × 3 = (2Y - 550)
9x = 342 - 180 = 162 1.14Y + 912 = 2Y - 550
x = 18 days 0.86Y = 1462
Y = 1700
13. Ans. ©
Two pen of each type, then total 4 pens. 19. Ans. (D)
Number or ways, picking 2 red pen = 646 ´ 3
14
CP for Satya = = 969
C2 = 91 2
Number of ways, picking 2 blue pen = 969 ´18
16
C2 = 120 CP for Sapna = = 918
30 19
Number of ways, picking 4 pen = C4 =
27405 MP = 918 + 182 = 1100 Rs.
91´120 104
Probability = = 20. Ans. (B)
27405 261
Present Age of Darshan = (47 - 7) - 8 =
14. Ans. (B) 32 year
Investment of Present Age of Deep = 32 × 5 / 8 = 20
Ankita = 6600 years
Beniwal = 7700 Sum = 32 + 20 = 52 year
Chandra = x
Profit Ratio, Ankita:Beniwal:Chandra = 21. Ans. (A)
6600:7700:x 30 × 54 = 15 × 40 + 12 × 66 + (2x - 42) +
6090 ´ x (3x + 28) + 62
= 2320
(6600 + 7700 + x) 1620 = 600 + 792 + 5x + 48

PAGE h ps://t.me/studified h ps://instagram.com/aashisharorasocial?utm_medium=copy_link


Follow
124 Aashish Arora on: h ps://youtube.com/channel/UCYa4_JrOrf8R5Kz2uOtccXQ https://www.facebook.com/aashisharorasocial/
अ ास by Aashish Arora
(SBI/IBPS/RBI/LIC/All other banking and insurance Exams)

5x = 180 Consonants have to be placed on even


x = 36 place so vowel will be placed on odd
16th Number = 2 × 36 - 42 = 30 place.
Total even place = Total odd place = 6
22. Ans. (B) Vowels & Consonants can be arranged
Speed of boat in still water = x km/h
in = 6!/4! × 4!/2! = 30 × 360 = 10,800 Rs.
3 (x - 16) 17
´ =
(x + 16) 1 11 28. Ans. (D)
CP of Shirt = x Rs., CP of Jeans = (5090 -
33(x - 16) = 17(x + 16)
33x - 17x = 272 + 528 x) Rs.
16x = 800 1.4375x + 0.56(5090 - x) = 4465
x = 50 km/h 1.4375 + 2850.4 – 0.56x = 4465
0.8775x = 1614.6
280.5 544.5 x = 1840 Rs. = CP of Shirt
+ = 8.25 + 8.25 = 16.5 hours
(50 - 16) (50 + 16) CP of Jeans = 3250 Rs.
Difference = 3250 - 1840 = 1410 Rs
23. Ans. (C)
(R + 760) × 0.15 × 4 + (3R - 2500) × 0.125 29. Ans. (D)
Length of Train = l m, Speed of train = s
× 4 = 2944
0.6R + 760) + 0.5(3R - 2500) = 2944 m/s
0.6R + 456 + 1.5R - 1250 = 2944 l = 34s - 362 = 27s - 236
2.1R = 2944 + 794 = 1780 7s = 126
2
CI = (1780 + 220)(1.15 - 1) = 2000 × s = 18 m/s
l = 34 × 18 - 362 = 250 m
0.3225 = 645 Rs.
Time required to cross a man on other
24. Ans. (B) train = t seconds
Remaining Area of field = 56 × 36 – (16 × 250 = (18 + 2)t
7.2 km/h = 7.2 × 5/18 = 2m/s
36 + 10 × 56 - 16 × 10)
20t = 250
= 2016 - (576 + 560 - 160)
t = 12.5 seconds
= 2016 – 976
2
= 1040 m
30. Ans. (B)
Present Age of P = 6x, R = 7x
25. Ans. (A)
Present average age of Q & R = 40 - 3 =
In Q, Spirit:Soda = (360 - 240):(264 -
37 year
220) = 30:11
Present age of Q = 2 × 37 - 7x = 74 - 7x
12 year later
26. Ans. (A)
Neha + Rohan => 24 5 6x + 12 14
=
Neha + Preet => 30 } 120 { 4 (74 - 7x) + 12 17
Neha + Rohan + Preet ==> 15 8
17(6x + 12) = 14(86 - 7x)
N + R = 5, N + P = 4, N + R + P= 8
102x + 204 = 1204 - 98x
Efficiency of Neha = 5 + 4 - 8 = 1
200x = 1000
Time taken by Neha = = 120 Days
x=5
Present Age of P = 6 × 5 = 30 year
27. Ans. (C)
Present Age of Q = 74 - 7 × 5 = 39 year
In MALEVOLENCEE,
Present Age of R = 7 × 5 = 35 year
Vowel = 6 (A – 1, E – 4, O – 1),
Present Age of S = 4 × 32 – (30 + 39 +
Consonant = 6 (M – 1, L – 2, V – 1, N – 1,
35) = 128 – 104 = 24 year
C – 1)

PAGE h ps://t.me/studified h ps://instagram.com/aashisharorasocial?utm_medium=copy_link


Follow
125 Aashish Arora on: h ps://youtube.com/channel/UCYa4_JrOrf8R5Kz2uOtccXQ https://www.facebook.com/aashisharorasocial/
अ ास by Aashish Arora
(SBI/IBPS/RBI/LIC/All other banking and insurance Exams)

31. Ans. (D) x = 120 Rs. = Extra investment of


In Mixture P, Oil = 8x L, Water = 7x L, In Shubham
Mixture Q, Oil = 11y L, Water = 12y L
8x - 11y = 9, 15x + 23y = 235 36. Ans. (A)
From solving equations, x = 8, 2000(x + 4)(x - 5) = 380000
Total Quantity of Mixture P = 15x = 15 × x2 - x - 20 = 190
8 = 120 Litres x2 - x - 210 = 0
(x + 14)(x - 15) = 0
32. Ans. (B) x = 15
Efficiency of Kusum = 20, Sanjana = 12, Length = 19 m, Breadth = 10 m
Priyansh = 15 Cost of fencing = 200 × 2(19 + 10) =
Let Kusum alone complete rest of work 11,600 Rs.
in x days.
Total work = 5(20 + 12 + 15) = 5(12 + 15) 37. Ans. (C)
In 1st hour they will cover = 32 + 42 =
+ 12x
x = 5 days 74km
In 2nd hour, (32 + 15) + (42 - 10) = 79 km
33. Ans. (C) Similary..
If 2 digits are correct, Number of In 3rd hour = 84 km, 4th hour = 89 km
4 Total distance traveled in 4 hour = 74 +
combination = C2 × 9 × 9 = 6 × 81 = 486
If 3 digits are correct, Number of 79 + 84 + 89 = 326 km
4 Remain = 373 - 326 = 47 km
combination = C3 × 9 = 4 × 9 = 36 47 km in = 47/94 = 0.5 hour
Wrong combination with atleast 2 Total time = 4 + 0.5 = 4.5 Hour
correct digit = 486 + 36 = 522
Total Number of combination = 104 = 38. Ans. (D)
10000 Students in A = 12x, C = 13x, B = 400 -
522 25x
Probability = = 0.0522
10000 400 × 85 = 88 × 25x + 80(400 - 25x)
400 × 5 = 8 × 25x
x = 10
34. Ans. (B) Student in Class A = 120
(A + 1500)(1 + 0.18 × 4) + (A + 600) ×
1.0752 = 6149 39. Ans. (D)
1.72(A + 1500) + (1849/1600)(A + 600) 1 get = x, 2nd get = x + 30
= 6149 (2x + 30) × 0.55 = x + 30
2.875625A + 3273.375 = 6149 1.1x + 16.5 = x + 30
A = 2875.625×2.875625 = 1000 Rs. 1.1x - x = 30 - 16.5
0.1x = 13.5
35. Ans. (A) x = 135
Investment of Shubham = 15 × 1054 + 7 Marks of students = 135 & 165
× x = 15810 + 7x
Investment of Lamha = 15 × 1178 + 7 × 40. Ans. (B)
Present Age of Shalini = x, Pariniti =
2x = 17670 + 14x
1.3x, Bipasha = 1.3x - 11, Vasundhra =
15810 + 7x 5550
= 1.3x + 12
33480 + 21x 12000 x + 1.3x + (1.3x - 11) + (1.3x + 12) = 4 ×
80(15810 + 7x) = 37(33480 + 21x) 37
1264800 + 560x = 1238760 + 777x 4.9x + 1 = 148
217x = 26040 4.9x = 147

PAGE h ps://t.me/studified h ps://instagram.com/aashisharorasocial?utm_medium=copy_link


Follow
126 Aashish Arora on: h ps://youtube.com/channel/UCYa4_JrOrf8R5Kz2uOtccXQ https://www.facebook.com/aashisharorasocial/
अ ास by Aashish Arora
(SBI/IBPS/RBI/LIC/All other banking and insurance Exams)

x = 30 × 8000 Rs.,
Present Age of Vasundhra = 1.3 × 30 + 12 Chandler give = 1.5 × 0.75 × 8000 =
= 51 year 1.125 × 8000 Rs
by Chandler ´100 1.125 ´ 8000 ´100
41. Ans. (A) = = 112.5%
In Mixture, Sugar = 9x ml, Water = 11x ml, by Ross 8000
9x 2
= 46. Ans. (B)
11x + 1150 5 MP = 100 Rs., CP = 84 Rs., SP = 1.3 × 84
5 × 9x = 2(11x + 1150) = 109.2 Rs.
45x - 22x = 2300 If item bought on MP, then P% = 109.2 -
23x = 2300 100 = 9.2%
x = 100
Original quantity of water = 11x = 11 × 47. Ans. (C)
100 = 1100 ml 12000 × 0.01a × 2 + 15000 × 0.02a × 1.5
= 10350
42. Ans. (C) 240a + 450a = 10350
Males = x, Females = 60 - x a = 15%
60 × 50 = 56x + 48(60 - x)
3000 = 56x + 2880 - 48x 48. Ans. (A)
8x = 120 Speed of HMS QE in still water = x km/h
x = 15 = Males, Females = 60 - 15 = 45 (x - 4) × 15.4 = 616
Difference = 45 - 15 = 30 x - 4 = 40
x = 44 km/h
43. Ans. (B) Speed of Charles De Gaulle in still water
Age of B = x year, C = x - 3 year, A = x + 6
= 0.75 × 44 = 33 km/h
Present average age of A, B & C = 55 - 5
Time taken by Ch. De Ga.= 629/(33+4) =
= 45
17 hours
(x + 6) + x + (x - 3) = 3 × 45
3x + 3 = 3 × 45
49. Ans. (D)
x = 44 = Age of B
15r = 8 (r + l)
(44 + 6 + 5) ´13 7r = 8l
Present age of D = - 5 = 60
11 3.5 × 15r = d × l
3.5 × 15r = d × (7r/8)
Difference of age of B & D = 60 - 44 = 16
d = 60 Days
44. Ans. (C)
50. Ans. (C)
Sapan invest = x Rs, Azeem invest = x +
Present age of Sapan = x year
5000 Rs, Pratyush invest = x + 13000 Rs Present age of Biswa = 1.5(x - 10) + 15 =
x + (x + 5000) + (x + 13000) = 120000
1.5x year
3x + 18000 = 120000
1.5x × 10/7 = x + 16
3x = 102000
15x = 7x + 112
x = 34000 Rs.
8x = 112
Profit Ratio = Sapan:Azeem:Pratyush =
x = 14 year = Present age of Sapan
34000:39000:47000 = 34:39:47 Present age of Biswa = 21 year
36000 ´ 34 Sum = 14 + 21 = 35 year = Divisible by 7
Profit of Sapan = = 10200 Rs.
120
51. Ans. (A)
45. Ans. (B) No of ways, in which 2 ball can be drawn
Ross give = 8000 Rs., Monica give = 0.75
= 19C2 = 171

PAGE h ps://t.me/studified h ps://instagram.com/aashisharorasocial?utm_medium=copy_link


Follow
127 Aashish Arora on: h ps://youtube.com/channel/UCYa4_JrOrf8R5Kz2uOtccXQ https://www.facebook.com/aashisharorasocial/
अ ास by Aashish Arora
(SBI/IBPS/RBI/LIC/All other banking and insurance Exams)

Probability of getting 2 balls of different 57. Ans. (C)


Initial speed of Tinku = x km/h,
(4 ´ 6) + (6 ´ 9) + (9 ´ 4) 114 2 Total distance =
colour = = =
171 171 3
6[2 ´ (32 + x) + 5 ´ 6] 12[2 ´ x + 11 Ì2]
=
2 2
52. Ans. (B)
Let initially, Total Quantity = 17x ml 2x + 94 = 2(2x + 22)
Alcohol = 6x ml, Soda = 11x ml 2x + 94 = 4x + 44
After replacing 85 ml mixture with 170 ml 2x = 50
alcohol, x = 25 km/h
Total quantity become = 17x – 85 + 170 = Distance = 6(2 × 25 + 22) = 432 km
17x + 85 ml
In new mixture, Quantity of Alcohol, 58. Ans. (D)
(17x + 85) × = 6x – 85 × + 170 Number of sold Banana = 28x, Apple =
8x + 40 = 6x – 30 + 140 21x, Orange = 27x
2x = 100 28x × 9 = 27x × 8 + 108
x = 50 ml 36x = 108
Initial Quantity of Soda = 11 × 50 = 550 ml x=3
Number of Apple sold = 21 × 3 = 63

53. Ans. (C) 59. Ans. (A)


Parma => 21 10 Investment of Teekam = 8 ×x +4 × 0.75x
Bhagchand => 35 } 210 { 6 = 11x Rs
Chagan => 30 7 Investment of Teejada = 8 × (9000 – x) +
Let work complete in x days then 4 × 1.3(9000 – x) = 13.2(9000 – x) Rs
210 = 4 × 10 + (x – 2) × 6 + x × 7 Profit Ratio
210 = 40 + 6x + 7x – 12 11x 2
13x = 182 =
x = 14 days 13.2(9000 - x) 3
Bhagchand & Chagan worked together 3x = 2 × 1.2(9000 – x)
for = 14 – 4 – 2 = 8 days 3x = 21600 – 2.4x
5.4x = 21600
54. Ans. (D) x = 4000 Rs
CP of Product = x Rs
1.38x – 0.79x = 324.5 60. Ans. (B)
0.59x = 324.5 Radius of cylinder = 2x cm, height = 9x
x = 550 Rs cm
MP = 1.56 × 550 = 858 Rs Curved surface area
22
55. Ans. (A) = 2 ´ ´ 2x ´ 9x = 5544
7
Amount got from RBL Bank = 34000 ×
2
2
1.25 = 53125 Rs x = 49
SI after 2 year = 53125 × 4 × 0.2 = 42500 x=7
Rs Radius = 2 × 7 = 14 cm, Height = 9 × 7 =
63 cm
2
56. Ans. (B) Volume of cylinder = 22/7 × 14 × 63 =
% of Amount saved = (100 – 23 – 26) × 38808 cm3
0.6 = 30.6% Time required to empty the tank =
Saving = 55000 × 0.306 = 16830 Rs 38808/539 = 72 minutes

PAGE h ps://t.me/studified h ps://instagram.com/aashisharorasocial?utm_medium=copy_link


Follow
128 Aashish Arora on: h ps://youtube.com/channel/UCYa4_JrOrf8R5Kz2uOtccXQ https://www.facebook.com/aashisharorasocial/
अ ास by Aashish Arora
(SBI/IBPS/RBI/LIC/All other banking and insurance Exams)

61. Ans. (C) Present age of Bala = 3 × (24 + 1/3) – 18


Let, Speed of boat in still water = 10 – 30 = 25 years
unit/hour, Speed of stream = 1 unit/hour
X = 36 × (10 + 1) = 36 × 11 66. Ans. (D)
Y = 55 × (10 – 1) = 55 × 9 Can't be determined
X:Y = (36 × 11):(55 × 9) = 4:5
67. Ans. (A)
62. Ans. (D) P × 1.23 = 7020 × 1.2 + 5400
At present, Age of Pragati = 12x year, P × 1.728 = 13824
Unnati = 13x year P = 8000 Rs
After 12 year,
12x + 12 16 68. Ans. (B)
= 4 × 70 + 3.5 × x = 455
13x + 12 17 280 + 3.5x = 455
17(12x + 12) = 16(13x + 12) 3.5x = 175
204x + 204 = 208x + 192 x = 50 km/hr
4x = 12
x=3 69. Ans. (C)
Present age of Unnati = 13 × 3 = 39 years Speed of Boat in still water = x km/h,
Speed of Boat in Downstream = 1.36x
63. Ans. (A) km/h
After adding 20 L of syrup & 50 L of water, Speed of stream = 1.36x – x = 0.36x
in 630 L mixture, km/h
New quantity of Mixture = 630 + 20 + 50 = 4.25(x – 0.36x) = 68
700 L 4.25 × 0.64x = 68
In new mixture, Syrup = 700 ×3 / 7 = 300L x = 25 km/h
, Water = 700 × 4 / 7 = 400 L Time required to cover 211 km in
In original mixture, Water = 400 – 50 = downstream = 221 / (1.36 × 25) = 6
350 L hour 30 minute
64. Ans. B) 70. Ans. (D)
A => 30 8 Total Votes = x, Valid votes = 0.75x
B => 48 } 240 { 5 X get = 0.75x × 1/3 = 0.25x, Y get =
C => 60 4 0.75x × 2/7 = 3x/14, Z get = 0.75x –
Let work complete in x days then
240 = 5 × 8 + x × 5 + (x – 4) × 4 0.25x – 3x/14 = 2x/7
240 = 40 + 5x + 4x – 16 3x/14 = 2x/7 – 110
9x = 216 x / 14 = 110
x = 24 days x = 1540

65. Ans. (C) 71. Ans. (A)


12 year later, Age of Kanta = 5x year, Age CP of Product = x Rs
x – 3026 = 0.5(4628 – x)
of Ramu = 7x year x – 3026 = 2314 – 0.5x
5x - 14 = 0.5 × (7x - 10) 1.5x = 5340
10x - 28 = 7x - 10 x = 3560 Rs
3x = 18 SP at 25% profit = 1.25×3560=4450 Rs
x=6
Present Age of Kanta = 5 × 6 – 12 = 18 72. Ans. (B)
year, Present Age of Ramu = 7 × 6 – 12 = Volume of soil dig out = 22/7 × 72 × 27 =
30 year 4158 cm3

PAGE h ps://t.me/studified h ps://instagram.com/aashisharorasocial?utm_medium=copy_link


Follow
129 Aashish Arora on: h ps://youtube.com/channel/UCYa4_JrOrf8R5Kz2uOtccXQ https://www.facebook.com/aashisharorasocial/
अ ास by Aashish Arora
(SBI/IBPS/RBI/LIC/All other banking and insurance Exams)

Remaining Area of field = 22 × 28 – 22/7 P = 24%


×72 = 462 cm2
Increase in height of remaining area of 77. Ans. (A)
field = 4158/462 = 9cm P + Q + R + S = 4 × 36.5 = 146,
P + Q + R + T = 4 × 39 = 156,
73. Ans. (D) P + Q + S + T = 4 × 38.5 = 154
Initial Quantity of Petrol = 495 L, Initial 3P + 3Q + 2R + 2S + 2T = 456
Quantity of Kerosene = 630 L 3(P + Q + R + S + T) = 456 + 3 × 42 = 582
In new mixture, Quantity of Kerosene = P + Q + R + S + T = 194
Average of all 5 = 194/5 = 38.8
0.6 × 630 = 378 L
Quantity of Petrol = 278 × 6 / 7 = 324 L, 78. Ans. (A)
Amount of Petrol added = 324 – 0.6 × 70 × 5x + 54 × 3x = 1024
495 = 27 L 512x = 1024
% of mixed petrol with initial quantity of x=2
mixture = 27× 100 / 1125 = 2.4% Quantity of 70 Rs/kg price pulse = 5 × 2 =
10 kg
74. Ans. (B)
5q = (p + r), 7r = (p + q) 79. Ans. (C)
Let total work = 5(p + q + r) = 100 18 × 2(11 + b) = 936
(p + q + r) = 20 11 + b = 26
(7r + r) = 20 b = 15 meters
8r = 20
r = 2.5 80. Ans. (D)
Time taken by R to complete the work = 5 year ago, Vikas = 9x year, Hitesh = 10x
100/2.5 = 40 days year
7 year later,
75. Ans. (C)
Total books = x + (x + 3) + (x – 6) = 3x – 3 9x + 12 13
=
Probability of drawing Hindi notebook = 10x + 12 14
Probability of drawing Math notebook + 7(9x + 12) = 13(5x + 6)
1/7 63x + 84 = 65x + 78
x x -6 1 2x = 6
= + x=3
3x - 3 3x - 3 7 Present age of Jitu = 3 × 37 – (9 × 3 + 5) –
x x -6 1 (10 × 3 + 5) = 44 years
- =
3x - 3 3x - 3 7
81. Ans. (D)
6 1
= Investment of Shivangi = 1984 × 12 =
3x - 3 7
23808 Rs, Investment of Harshangi =
42 = 3x – 3 1612x Rs
3x = 45
X = 15 23808 7040 32
= =
1612x 9900 - 7040 13
76. Ans. (B) 23808 × 13 = 32 × 1612x
P (P + 4) x=6
5500 ´ 4 ´ + 6500 ´ 4 ´ = 12560
100 100 Sharshangi joined after = 6 month
220P + 260(P + 4) = 12560 82. Ans. (B)
480P = 12560 – 1040 Speed of stream = x km/h, Speed of Boat
480P = 11520

PAGE h ps://t.me/studified h ps://instagram.com/aashisharorasocial?utm_medium=copy_link


Follow
130 Aashish Arora on: h ps://youtube.com/channel/UCYa4_JrOrf8R5Kz2uOtccXQ https://www.facebook.com/aashisharorasocial/
अ ास by Aashish Arora
(SBI/IBPS/RBI/LIC/All other banking and insurance Exams)

in still water = 2.2x km/h 86. Ans. (D)


Speed of Boat in still water = (38+22)/ 2=
168 81
+ =8 30 km/h, Speed of Steam = (38–22)/ 2 =
2.2x + x 2.2x - x
8 km/h
52.5 67.5 120 (a + b) = (30 + 4) × 10
+ =8= (a + b) = 340
x x x
(a+b)/2 = 170
x = 15 km/h
Speed in still water = 2.2 × 15 = 33 km/h 87. Ans. (A)
Time required to cover 297 km in still In 525 L mixture, Petrol = 245 l, Diesel =
water = 297/33 = 9 hours 280 l
New quantity of Mixture = 525 – L + 30 =
83. Ans. (A) 555 – L
Investment of Nakul = 1200 × 12 = Quantity of petrol in new mixture
14400 Rs 7 13
Investment of Sakshi = 1600 × 12 = (245 - L) ´ = (555 - L) Ì
15 30
19200 Rs
Investment of Chhavi = 1.25 × 1200 × 8 L/30 = 245 – 240.5
= 12000 Rs L = 135 I
Profit Ratio (Nakul:Sakshi:Chhavi) =
144:192:120 = 6:8:5 88. Ans. (B)
Profit of Nakul = 6080 × 6 / 19 = 1920 Rs Speed of train=s m/s, Length of train= lm
l = 19s
84. Ans. (B) l + 138 = 25s
19s + 138 = 25s
xr s = 23 m/s, l = 437 m
= 120
100 Time required to cross another platform
æ r ö = 437 + 230 / 23 = 29 seconds
(x + 120 + 180)ç1 + ÷ = 1080
è 100 ø
r 89. Ans. (C)
(x + 300) + (x + 300) ´ = 1080 1.75 × 0.9 × (1 – P) = 1.197
100
1.575(1 – P) = 1.197
xr
x+ + 3r = 780 1 – P = 0.76
100 P = 0.24
x + 3r + 120 = 780 p = 24%
x + 36000/x = 660
2
x – 660x + 36000 = 0 90. Ans. (D)
(x – 600)(x – 60) = 0 Present age of Nisha = 32 year, Afreen =
x = 600, 60 48 year
Leela = 48 – 6 = 42 years or 48 + 6 = 54
85. Ans. (C) year
10(7w + 6c) × 0.75 = 6(11w + 6c) Cannot be determined
52.5w + 45c = 66w + 36c
9c = 13.5w 91. Ans. (A)
c = 1.5w 5 × 2[12 × (2a + 3) + (2a + 3)(4a – 3) + 12
10(7w + 6c) = d × 32w × (4a – 3)] = 7470
10(7w + 6 × 1.5w) = d × 32w 24a + 36 + 8a2 + 6a – 9 + 48a – 36 = 747
10 × 16w = d × 32w 8a2 + 78a - 756 = 0
d = 5 days 2
4a + 39a – 378 = 0
(4a + 63)(a – 6) = 0

PAGE h ps://t.me/studified h ps://instagram.com/aashisharorasocial?utm_medium=copy_link


Follow
131 Aashish Arora on: h ps://youtube.com/channel/UCYa4_JrOrf8R5Kz2uOtccXQ https://www.facebook.com/aashisharorasocial/
अ ास by Aashish Arora
(SBI/IBPS/RBI/LIC/All other banking and insurance Exams)

a=6 41.67%

92. Ans. (B) 97. Ans. (A)


In 'CLERKPRE', Consonant = 6, Vowel Investment of Avinash = 1450 + 1.2 ×
=2 1450 = 3190 Rs
Total number of arrangement = Investment of Anurag = 1050 + (1050 –
8! 150) = 1950 Rs
= = 10080
2!´2! Profit ratio, Avinash:Anurag =
3190:1950 = 319:195
Number of arrangement with vowel
6682 ´ 319
7!´ 2! Profit of Avinash = = 4147 Rs
together = = 2520 514
2!´ 2!
Number of arrangement in which 98. Ans. (A)
vowels are not together = 10080 – 2520 Initial quantity of Water in mixture
= 7560 168 262.5 ´ 3
P= = 262.5 L, Milk = = 112.5L
0.64 7
93. Ans. (D)
Radius of cylinder = 7x cm, Height = New quantity of Milk = 112.5 × 0.64 + 60
16x cm × 4/5 = 120 L
2/7 × (7x)2 × 16x= 308 New quantity of Water = 168 + 60 × 1/5 =
x3 = 0.125 180 L
x = 0.5 New Ratio, Milk : Water = 120:180 = 2:3
Radius = 7 × 0.5 = 3.5 cm, Height = 16 ×
0.5 = 8 cm
Area of curved surface = 2 × 22/7 × 3.5 99. Ans. (C)
× 8 = 176 cm
2 Boys in school A = 4080, Girls = 4760,
Total = 4080 + 4760 = 8840
94. Ans. (B) Student in school B = 8840/1.3 = 6800,
Total money spent by Sanjana = 1.25 × Boys = 2720, Girls = 4080
7400 = 9250 Rs Girls in A – Girls in B = 4760 – 4080 = 680
(10730 - 9250) ´100
Profit % for Sanjana = = 16% 100. Ans. (D) 2912
9250 Champak had = = 10,000 Rs
0.4 ´ (1.23 - 1)
95. Ans. (C)
Daksh=> 15 5
} 75 { 101. Ans. (D)
Suchitra=>25 3 Profit ratio, Girish:Harish:Jagdish =
0.36 ´ 75 12:13:10
Efficiency of Rudraksh = =6 4445 ´12
4.5 Profit of Girish = = 1524 Rs
Efficiency of Rudraksh is more than 35

(6 - 3) ´100 102. Ans. (B)


Suchitra by = 100%
3 Downstream speed = 3x km/h,
96. Ans. (B) Upstream speed = 2x km/h
CP of both article = 800×3 / 4= 600 Rs,
108 56
Profit of Shrirant = 600 × 1/3 = 200 Rs. + = 16
Profit for Jainath = 1.25 × 200 = 250 Rs 3x 2x
Profit % for Jainath = 250×100/600 =

PAGE h ps://t.me/studified h ps://instagram.com/aashisharorasocial?utm_medium=copy_link


Follow
132 Aashish Arora on: h ps://youtube.com/channel/UCYa4_JrOrf8R5Kz2uOtccXQ https://www.facebook.com/aashisharorasocial/
अ ास by Aashish Arora
(SBI/IBPS/RBI/LIC/All other banking and insurance Exams)

36 28 Passing marks = 0.25x + 60 = 0.55x – 90


+ = 16
x x 0.3x = 150
64 x = 500
= 16 Passing marks = 0.25 × 500 + 60 = 185
x
x=4 185 ´100
Passing% = = 37%
3´ 4 - 2 ´ 4 500
Speed of stream = = 2km/hr
2
109. Ans. (A)
103. Ans. (C) Side of Square field = 48/4 = 12 cm
Average Run in all matches Area of Rectangular field = 1.25 × 122 =
18 ´ 29 + 86 180 cm2
= = 32 Runs Length of rectangle = x cm, Width = x –
19
3cm
x(x – 3) = 180
104. Ans. (B) 2
x – 3x – 180 = 0
Amount invested in stock A = x Rs, stock (x – 15)(x + 12) = 0
B = 6000 – x Rs x = 15 cm = Length, Width = 15 – 3 =
x × (1.252 – 1) + (6000 – x) × 2 × 0.43 = 12cm
4089 Perimeter of Rectangle = 2(15 + 12) =
0.5625x + (6000 – x) × 0.86 = 4089 54cm
0.86x – 0.5625x = 5160 – 4089
0.2975x = 1071 110. Ans. (C)
x = 3600 Rs 48 × 65 = 45 × 62 + 2x + 3x + 6x
11x = 3120 – 2790 = 330
105. Ans. (A) x = 30
Saving of Vineet = 2x Rs, Saving of From last 3 numbers,
Ryan = 3x Rs Smallest number = 2 × 30 = 60, Largest
Salary of Vineet = 5 × 3x = 15x Rs, number = 6 × 30 = 180
Salary of Ryan = 15x + 2000 Rs Product of these 2 numbers = 60 × 180 =
Expenditure of Vineet = 15600 = 15x – 10800
2x
x = 1200 Rs 111. Ans. (D)
Monthly Salary of Ryan = 15 × 1200 + Present age of Tom = 5x year, Ben = 4x
2000 = 20,000 Rs year
5x + 6 7
106. Ans. (B) =
Present age of Upkaar = x Rs, Yasin = 4x + 3 5
1.15x Rs, Sajal = x – 10 or 10 – x 5(5x + 6) = 7(4x + 3)
Cannot be determined 25x + 30 = 28x + 21
3x = 9
107. Ans. (C) x=3
Income of Ronit = 3172/0.52 = 6100 Rs, Present age of, Tom = 5 × 3 = 15 year,
Income of Ojas = 13420 – 6100 = 7320 Ben = 4 × 3 = 12 year
Rs Present age of Ron = 3 × 15 – 15 – 12 =
6100:7320 = a:6 18 years
a=5
112. Ans. (C)
108. Ans. (E) Jon & Rob complete total work in
Let Maximum marks in exam = x = 42/0.7 = 60 days

PAGE h ps://t.me/studified h ps://instagram.com/aashisharorasocial?utm_medium=copy_link


Follow
133 Aashish Arora on: h ps://youtube.com/channel/UCYa4_JrOrf8R5Kz2uOtccXQ https://www.facebook.com/aashisharorasocial/
अ ास by Aashish Arora
(SBI/IBPS/RBI/LIC/All other banking and insurance Exams)

Robb alone complete work in = 1.5 × 60 Length of Rajdhani express


= 90 days 5
90r = 60(j + r) = 10.5 ´ (108 + 72) ´ - 255 = 270 m
18
30r = 60j
90r = 180j
Jon complete whole work in = 180 days 117. Ans. (B)
80% work completed by Jon in = 0.8 × Initially, Oil = 7x ml, Alcohol = 9x ml
180 = 144 days 7x – 168 + 72 = 9x – 216 + 24
2x = 96
113. Ans. (D) x = 48
x × 2r = 3420 Original quantity of mixture = 16 × 48 =
xr = 1710 768 ml
2
x[(1 + r) – 1] = 3727.8
2
x[2r + r ] = 3727.8 118. Ans. (C)
2xr + xr × r = 3727.8 Side of cube S = a cm
3420 + 1710r = 3727.8 5a2 = 720
1710r = 307.8 a2 = 144
r = 0.18 a = 12 cm
Rate of interest = r × 100 = 18% Side of cube T = 12 + 3 = 15 cm
Curved surface Area of cube T = 4 × 152
2
114. Ans. (A) = 900 cm
Profit Ratio, Varsha:Nandini = (6960 ×
5):(3750 × 8) = 29:25 119. Ans. (B)
Yellow balls = x,
3250 ´ 54 Probability of picking 1 yellow ball
Total Profit = = 7020 Rs
25 x 1
= =
30 + x 4
115. Ans. (C)
Speed of Boat in still water = 11x km/h, 4x = 30 + x
x = 10
Speed of stream = 3x km/h Total balls = 10 + 30 = 40, Number of
112 168 40
way of picking 2 ball = C2 = 780
+ = 6.5
11x - 3x 11x + 3x Probability of getting 1 yellow & 1 red
112 168 10 ´ 30 5
+ = 6.5 =
8x 14x ball =
780 13
14 12
+ = 6.5
x x 120. Ans. (D)
26 Distance = 7(x + 6) = 8.5(2x – 8)
= 6.5
x 7x + 42 = 17x – 68
x=4 10x = 110
Speed of Boat = 11 × 4 = 44 km/h, x = 11 km
Speed of stream = 3 × 4 = 12 km/h Distance traveled = 7(11 + 6) = 119
132 224 121. Ans. (A)
+ = 3 + 4 = 7 hours
44 44 + 12 In November he earned = x Rs
116. Ans. (C) 12 × 7100 = 5 × 5200 + 6600 + 6700 +
Length of Shatabdi express 6800 + 6900 + 0.5x + x + 2x
3.5x = 85200 – 26000 – 27000 = 32200
5 x = 9200 Rs
= 8.5 ´108 ´ = 255m
18

PAGE h ps://t.me/studified h ps://instagram.com/aashisharorasocial?utm_medium=copy_link


Follow
134 Aashish Arora on: h ps://youtube.com/channel/UCYa4_JrOrf8R5Kz2uOtccXQ https://www.facebook.com/aashisharorasocial/
अ ास by Aashish Arora
(SBI/IBPS/RBI/LIC/All other banking and insurance Exams)

122. Ans. (B) 48 = 18 + 18 + 2y


Let initial number of employee = x 2y = 48 – 36
46x + 6 × 43 = 44(x + 6) 2y = 12
46x + 258 = 44x + 264 y=6
2x = 6
x=3 128. Ans. (C)
Investment of Jamie = 4 × 6500 + 8 × 0.8
123. Ans. (C) × 6500 = 67600 Rs
Let x L of 1st mixture & y L of 2nd Investment og Ned = 12 × 5000 = 60000
mixture is taken, Rs
3x 7y Profit Ratio, Jamie:Ned = 676:600 =
+ 169/150
14 19 = 3
11x 12y 7 169
+ Profit of Jamie = 12760 ´ = 6760 Rs
14 19 319
7(57x + 98y) = 3(209x + 168y)
399x + 686y = 627x + 504y 129. Ans. (D)
228x = 182y CP of X = x Rs, SP of X = 1.25x Rs
x:y = 91:114 CP of Y = 1.25x + 360 Rs, SP of Y =
0.8(1.25x + 360) = x + 288
124. Ans. (B) Loss of seller = (x + 360) – (x + 288) = 72
In 12 + 3 = 15 L of mixture, Milk = 0.7 ×
Rs
12 + 3 = 11.4 L
% of milk in new mixture 130. Ans. (B)
11.4 ´100 Income of Sumeet = x Rs, Income of
= 76%
15 Prakash = 1.25x Rs
Saving of Prkash = 0.28 × 1.25x = 0.35x,
125. Ans. (A) Saving of Sumit = 0.35x + 2200 Rs
Present age of P = 28 year 0.45x = 0.35x + 2200
Present age of Q = 28 + 10 – 3 = 35 year 0.1x = 2200
Present age of R = 20 year x = 22000 Rs
Present age of S = 20 × 1.25 – 3 = 22 Expense of Sumit = 0.55 × 22000 =
years 12100 Rs

126. Ans. (E) 131. Ans. (A)


Side of Square = 72/4 = 18 m Total balls = 3 + 3 + 4 = 10, Number of
10
Area of Rectangular field way of picking 2 ball = C2 = 45
11 Number of way of picking 2 ball which
= ´182 = 396 m 2 6
are not yellow = C2 = 15
9
Probability of not getting yellow ball
Length of rectangular field = x m, 15 1
breadth = x – 4 m = =
45 3
x(x – 4) = 396
2
x – 4x – 396 = 0 132. Ans. (D)
(x + 18)(x – 22) = 0 Length = x cm, Height = 21 – x cm,
Length = x = 22m, Breadth = 22-4=18 m breadth = 5 cm
Perimeter = 2 × (22 + 18) = 80 m Volume, x × 5 × (21 – x) = 540
2
21x – x = 108
127. Ans. (B) x2 – 21x + 108 = 0
30 × 16 = 9 × 20 + (9 + y) × 20

PAGE h ps://t.me/studified h ps://instagram.com/aashisharorasocial?utm_medium=copy_link


Follow
135 Aashish Arora on: h ps://youtube.com/channel/UCYa4_JrOrf8R5Kz2uOtccXQ https://www.facebook.com/aashisharorasocial/
अ ास by Aashish Arora
(SBI/IBPS/RBI/LIC/All other banking and insurance Exams)

(x – 12)(x – 9) = 0 15x + 28y = 16200


x = 12 cm = Length, Height = 21 – 12 = 9 1
cm 0.25 ´ 4x + ´ 7y = 5400
3
Length is more than height by=
3x + 7y = 3660
(12 - 9) ´100
= 33.33% 15x + 28y – 4(3x + 7y) = 16200 – 4 ×
9
3660
15x – 12x = 16200 – 14640
133. Ans. (C) 3x = 1560
1800 ´100 x = 520
Amount invested = = 2000 Rs 7y = 3660 – 3 × 520 = 2100
22.5 ´ 4
2 y = 300
é R + 9ù Cost price of 2 chairs & 3 tables = 3 ×
2000 ê1 + = 3380
ë 100 úû 520 + 2 × 300 = 2160 Rs
2
é R + 9ù
êë1 + 100 úû = 1.69 137. Ans. (B)
10 year ago, Gambhir = 5x year,
R +9 Ganguly = 4x years
1+ = 1.3
100
5x + 18 7
R +9 8 year later, =
= 0.3 4x + 18 6
100
6(5x + 18) = 7(4x + 18)
R + 9 = 0.3 × 100 30x + 6 × 18 = 28x + 7 × 18
R + 9 = 30 2x = 18
R = 21 x = 9,
Sum of present ages = (5 × 9 + 10) + (4 ×
134. Ans. (D)
Initial quantity of paint = 13x ml, water = 9 + 10) = 101 years
15x ml 138. Ans. (A)
13x - 91 + 18 6 Angle of remaining sector of circle =
=
15x - 105 + 23 7 360° – 200° = 160°
7(13x – 73) = 6(15x – 82) Radius of cone = r mm
91x – 511 = 90x – 492 Area of sector = Curved surface area of
x = 19 cone
Initial quantity of mixture = 28 × 19 = (160°/360°) × 1892 = r × 189
532 ml r = 84 mm

135. Ans. (B) 139. Ans. (A)


1 1 1 4
= + æ R ö 1665
x x + 9 x + 16 ç1 + ÷ =
è 100 ø P
(x + 9)(x + 16) = x(x + 16 + x + 9) 8 2
x2 + 25x + 144 = 2x2 + 25x æ R ö æ 1665 ö
ç1 + ÷ =ç ÷
x2 = 144 è 100 ø è P ø
x = 12 8 2
æ R ö æ 1665 ö
Pç1 + ÷ =ç ÷
136. Ans. (C) è 100 ø è P ø
CP of 1 table = x Rs, CP of 7 chair = y Rs (1665) 2
4 2775 =
1.25 ´ 4x + ´ 7y = 5400 P
3

PAGE h ps://t.me/studified h ps://instagram.com/aashisharorasocial?utm_medium=copy_link


Follow
136 Aashish Arora on: h ps://youtube.com/channel/UCYa4_JrOrf8R5Kz2uOtccXQ https://www.facebook.com/aashisharorasocial/
अ ास by Aashish Arora
(SBI/IBPS/RBI/LIC/All other banking and insurance Exams)

16652 145. Ans. (B)


P= = 999 Rs Investment of Rahul = 20 × 12 = 240
2775
Investment of Mukul = 25 × 8 = 200
140. Ans. (C) Investment of Nakul = 40 × 6 = 240
Profit Ratio = Rahul:Mukul:Nakul =
æ 8 öæ x + 8 ö 240:200:240 = 6:5:6
8125ç1 + ÷ç1 + ÷ = 5187
è 100 øè 100 ø Profit of Mukul = 340000 × 5/17 =
x x + 8 x(x + 8) 399 100000 Rs
1- - + =
100 100 100 2 625
2
146. Ans. (B)
100 – 100(x + x + 8) + x(x + 8) = 6384 Total Discount %
2
x + 8x – 200x – 800 + 3616 = 0 1st case = (1 – 0.7 × 0.7 × 0.75 × 0.8) ×
2
x – 192x + 2816 = 0 100 = 70.6%
(x – 16)(x – 176) = 0 2nd case = (1 – 0.5 × 0.8 × 0.9 × 0.95) ×
x = 16
100 = 65.8%
141. Ans. (D) Difference in Discount = 45000(0.706 –
Profit Ratio, Ikka:Diljt = (1419 × 0.658) = 2160 Rs
3):(1677 × 2) = 33:26
147. Ans. (B)
4543 ´ 26 6 × r/100 = 0.72
Profit of Diljit = = 2002 Rs
59 r = 12%
2
Amount on CI, 72000 × 1.12 = 90316.8
142. Ans. (D) Rs
Upstream speed = 704/22 = 32 km/h,
Downstream speed = 704/16 = 44 km/h 148. Ans. (C)
D = st = (s + 10)(t – 1) = (s – 5)(t + 1.5)
Speed of Boat in still water st = (s + 10)(t – 1)
st = st + 10t – s – 10
32 + 44 s = 10t – 10
= = 38 km/hr
2 st = (s – 5)(t + 1.5)
st = st – 5t + 1.5s – 7.5
143. Ans. (A) 1.5s – 5t = 7.5
2a = b 1.5(10t – 10) – 5t = 7.5
12 × 8 × 4a = d × 7 × 6b 15t – 15 – 5t = 7.5
64a = d × 7 × 2a 10t = 22.5
d = 4.571 days t = 2.25 hour
s = 10 × 2.25 – 10 = 12.5 km/h
144. Ans. (C) D = st = 12.5 × 2.25 = 28.125 km
Present Age of Kukku = 4x year, Nikku =
149. Ans. (A)
3x year Speed of boat = x km/h, Speed of stream
4x + 7 5 = y km/h
7 year later, =
3x + 7 4 540(x – y) = 420(x + y)
4(4x + 7) = 5(3x + 7) 9x – 9y = 7x + 7y
16x + 28 = 15x + 35 2x = 16y
x=7 x:y = 8:1
After 21 year, Ratio of ages = (4 × 7 +
150. Ans. (C)
21) + (3 × 7 + 21) = 7:6 Total sales of Raman = x Rs
0.03x + 0.02(x – 300000) = 21000

PAGE h ps://t.me/studified h ps://instagram.com/aashisharorasocial?utm_medium=copy_link


Follow
137 Aashish Arora on: h ps://youtube.com/channel/UCYa4_JrOrf8R5Kz2uOtccXQ https://www.facebook.com/aashisharorasocial/
अ ास by Aashish Arora
(SBI/IBPS/RBI/LIC/All other banking and insurance Exams)

0.05x – 6000 = 21000 157. Ans. (A)


0.05x = 27000 Correct average marks
x = 540000 Rs 12 ´ 48 - (34 + 44) + (43 + 71)
Extra bonus earned = 0.02(540000 – = = 51
12
300000) = 4800 Rs

151. Ans. (C) 158. Ans. (B)


Volume of Hemisphere = Volume of CP = 44 × 9 + 28 × 5 = 536 Rs, SP =
Cone 10(44 + 28) = 720 Rs
2 3 1 2 Profit =720 – 536 = 184 Rs
´4 = ´3 ´h
3 3 159. Ans. (C)
2 ´ 43 Present age of Ankur = 4x year, Shanti =
h = 2 = 14.22 cm
3 7x year, Ujala = (4x – 4) × 4/3
4
152. Ans. (B) 4x + 7x + (4x - 4) ´ = 109
Ratio of time = Time on broken Road : 3
Time on Boat : Time on Tar Road 3 × 11x + 4(4x – 4) = 3 × 109
33x + 16x – 16 = 327
20 40 40
= : : = 3 : 4 : 3 = 30 : 40 : 30 49x = 343
2 3 4 x=7
Present Age of Shanti = 7 × 7 = 49 years
153. Ans. (D)
Devansh+Dhruv=> 15 24
Dhruv + Dharmendra => 18 } 360 { 20 160. Ans. (D)
Dhruv => 24 15 Number of Tigers = 7x, Lions = 11x
Efficiency of Dhruv = 15, Devansh = 9, Alive Tigers = 0.825 × 7x = 5.775, Alive
Dhrmendra = 5 Lions = 0.75 × 11x = 8.25x
0.725 × 360 = d(15 + 9 + 5)
261 = d × 29 % of alive tigers with alive lions =
d = 9 days 5.775x ´100
= = 70%
8.25x
154. Ans. (B)
Profit Ratio, P:Q:R = 2500:2800:3000 =
25:28:30 161. Ans. (D)
Difference between profit of P & R Initial quantity of mixture = x ml
0.55x – 0.45x = 7
30 - 25 0.1x = 7
= 13695 ´ = 825 Rs
83 x = 70 ml
155. Ans. (C)
45(x + 7.5) = 1.2 × 45 × (2x – 1) + 45 162. Ans. (B)
45x + 337.5 = 108x – 54 + 45 Number of 3 cm nail = x
x 3
63x = 346.5 =
x = 5.5 30 5
x = 18,
156. Ans. (B) 2 cm nails = 30 – 18 = 12
Amount received after 3 year = 2850(1 Number of way of picking 2, 2 cm nails =
12
C2 = 66
+ 0.36 × 3) = 5928 Rs 30
Number of way of picking 2 nails = C2 =
Interest received on amount after 3 year
435
= 5928 × 0.2 × 5 = 5928 Rs Probability of picking 2 nails of 2 cm

PAGE h ps://t.me/studified h ps://instagram.com/aashisharorasocial?utm_medium=copy_link


Follow
138 Aashish Arora on: h ps://youtube.com/channel/UCYa4_JrOrf8R5Kz2uOtccXQ https://www.facebook.com/aashisharorasocial/
अ ास by Aashish Arora
(SBI/IBPS/RBI/LIC/All other banking and insurance Exams)

66 44 4! 1
= = divisible by 5 = =
435 290 5 ´ 4! 5
163. Ans. (B)
2691 168. Ans. (E)
CP of article = = 1950 Rs 500 × x – 100 × 3x = 1000
1.38 200x = 1000
Discount = 1.5 × 1950 – 2691 = 234 Rs x=5
Y = 500 × 5 + 200 × 2 × 5 + 100 × 3 × 5 =
164. Ans. (C) 6000 Rs
In 2019, Income of Kaalia =
4320 169. Ans. (B)
= 9600 Rs, Investment of Raju = 4000 × 6 + 4000 ×
0.45
0.8 × 6 = 43200 Rs
In 2018, Income of Kaalia = Investment of Shyam = 3400 × 6 + (3400
9600 ´14 + 600) × 6 = 44400 Rs
= 8960 Rs Investment of Baburao = 5000 × 6 +
15
Savings in 2018 = 0.3 × 8960 = 2688 Rs (5000 + 1000) × 6 = 66000 Rs
Profit Ratio = 43200:44400:66000 =
165. Ans. (B) 36:37:55
Let Initial mixture is 132x, Rice A = 77x,
Rice B = 55x 170. Ans. (D)
After adding 8 kg of Rice, Rice A = 77x, Speed of Boat in still water = 11x km/h,
Rice B = 63x Speed of stream = 7x km/h
55x + 8 = 63x 162 44
+ =4
x=1 11x + 7x 11x - 7x
Capacity of Container = 77 × 1 + 63 × 1= 162 44
+ =4
140 kg 18x 4x
9 11
166. Ans. (A) + =4
When Efficiency of Man is 5 & Efficiency x x
of Boy is 1 20
=4
24(6 × 5 + 10 × 1) = d(4 × 5 + 20 × 1) x
24 × 40 = d × 40 x =5
d = 24 Days Speed of Boat = 11 × 5 = 55 km/h

When Efficiency of Man & Boy is 1 171. Ans. (C)


24(6 × 1 + 10 × 1) = d(4 × 1 + 20 × 1)
3x + 15 x
24 × 16 = d × 24 - =2
d = 16 days9 30 20
24 days & 16 days 2(3x + 15) – 3x = 120
6x + 30 – 3x = 120
167. Ans. (C) 3x = 90
Total possible number can be formed = x = 30 km
5! = 5 × 4! Time required to cover (x + 48) = (30 +
Possible number formed, which are 48) = 78 km with 26 km/h speed = 78/26
divisible by 5 = 4! = 3 hours
Probability that formed number is
172. Ans. (A)

PAGE h ps://t.me/studified h ps://instagram.com/aashisharorasocial?utm_medium=copy_link


Follow
139 Aashish Arora on: h ps://youtube.com/channel/UCYa4_JrOrf8R5Kz2uOtccXQ https://www.facebook.com/aashisharorasocial/
अ ास by Aashish Arora
(SBI/IBPS/RBI/LIC/All other banking and insurance Exams)

2 2
Area of square Y = 1.5 × 196 = 441 m 178. Ans. ©
55 + 125 × 0.6 + X = 3 × 125 × 0.56
173. Ans. (A) 55 + 75 + X = 210
Cost price of Article = x Rs X = 210 – 130
1.45x – 120 = 1.25x X = Marks in English = 80
1.45x – 1.25x = 120
0.2x = 120 179. Ans. (D)
X = 600 Rs 3.6 ´100
In mixture A, % of Milk = = 60%
6
174. Ans. (E)
Present age of Dwight = 4x + 8 year, Let, in mixture B, Milk is x%
x
Michael = 7x + 8 year, 1.25 × 4x = 5x 5 ´ 0.6 + 7 ´ = 12 Ì0.67
year 100
(4x + 8) + (7x + 8) + 5x = 3 × 48 3 + 0.07x = 8.04
16x + 16 = 144 0.07x = 5.04
16x = 128 x = 72%
x = 8 year In 15 L of mixture B, Milk is = 15 × 0.72 =
Present age of Michael = 7 × 8 + 8 = 64 10.8 L = 10800 ml
year 180. Ans. (A)
Let she bought x glasses
175. Ans. (B) 65 × 230 + x × 258 = 245(65 + x)
Cost of painting of Hemisphere = 258x – 245x = 245 × 65 – 230 × 65
22 13x = 15 × 65
10.5 ´ 3 ´ ´ 6 2 = 3564 Rs
7 x = 15 × 5
x = 75
176. Ans. (C)
Let Drawn Green Balls = x, Yellow Balls 181. Ans. (B)
=x+5 Radius of field = r m
45 - (x + 5) 3 22
= 2 ´ ´ r = 44
30 - x 2 7
r=7m
2(40 – x) = 3(30 – x) 2 2
Area of Path:Area of Field = (9 – 7 ):7
2

80 – 2x = 90 – 3x = (81 – 49):49 = 32:49


x = 10
Number of total drawn ball = 10 + (10 + 182. Ans. (D)
5) = 25 Interest from CI = Interest from SI
1.52 – 1 = 0.25n
177. Ans. (D) 2.25 – 1 = 0.25n
19800 1.25 = 0.25n
Area of square wall = = 3600 m 2 n=5
5.5
Side of Square wall = 60 cm 183. Ans. (B)
Perimeter of Rectangular wall 5 Surya + Rohit => 24 5
= } 120 {
Perimeter of Square wall 6
Rohit => 40 3
2(55 + b) 5 Efficiency of Surya = 5 – 3 = 2
=
6 ´ 60 6 120 = 6 × 5 + 2 × x
6 × 2(55 + b) = 5 × 240 2x = 120 – 30
55 + b = 100 2x = 90
b = 45 m x = 45 days

PAGE h ps://t.me/studified h ps://instagram.com/aashisharorasocial?utm_medium=copy_link


Follow
140 Aashish Arora on: h ps://youtube.com/channel/UCYa4_JrOrf8R5Kz2uOtccXQ https://www.facebook.com/aashisharorasocial/
अ ास by Aashish Arora
(SBI/IBPS/RBI/LIC/All other banking and insurance Exams)

184. Ans. (A) 189. Ans. (A)


Investment of Sansa = 7 × 12 = 84 3600 3600 3600
Investment of Maisie = 9 × (3 + 5) = 72 Total CP = + + = 5000 +
0.72 1.25 0.9
Profit Ratio, Sansa : Maisie = 84:72 =
7:6 2880 + 4000 = 11880 Rs
Difference between Profit of Sansa & Total SP = 3 × 3600 = 10800 Rs
Loss = 11880 – 10800 = 1080 Rs
7-6 1080 ´100
Maisie = 5850 ´ = 450 Rs Loss% = = 9.09% = 9%
7+6 11880

185. Ans. (C) 190. Ans. (B)


CP of Article = x Rs Rectangle, Length = 9, Breadth = 14,
0.7x + 4800 = 1.3x Circle, Diameter = 1.5 × 14 = 21, Radius
0.6x = 4800 = 10.5
x = 8000 Rs Area of Circle : Area of Rectangle =
SP at 25% profit = 1.25 × 8000 = 10,000 æ 22 2ö
ç ´10.5 ÷ : (9 ´14) = 346.5 : 126 = 11 : 4
Rs è 7 ø

186. Ans. (D) 191. Ans. (C)


Speed of Boat in still water = 91 / 7 = 13 Time taken by X = 2a hours, Time taken
km/h by Y = 7a hours
Speed of Stream = x km/h X=> 2a 7
Downstream Speed – Upstream Speed } 14a {
=4 Y=> 7a 2
(13 + x) – (13 – x) = 4 11 × 3.5 × 9 = 14a × 9
2x = 4 2a = 5.5
x = 2 km/h Time taken by X alone = 2a = 5.5 hours
77 105
+ = 7 + 7 = 14 hour
13 - 2 13 + 2 192. Ans. (D)
Average price of local bats = x Rs
1500 × 100 = 2500 × 30 + 1500 × 20 + x ×
187. Ans. (C) 50
Present Age of Biswa = 7x year, Present 15000 = 7500 + 3000 + 5x
Age of Sapan = 10x – 16 year 5x = 15000 – 10500
7x – 15 = 1.5(10x – 16 – 10) 5x = 4500
7x – 15 = 1.5(10x – 26) x = 900
7x – 15 = 15x – 39
8x = 24 293. Ans. (B)
x=3 Let the middle number = x
Sum of Present Age of Biswa & Sapan = 1.4x + x + 0.8x = 480
7 × 3 + 10 × 3 – 16 = 35 = 7 × 5 3.2x = 480
Sum is Divisible by = 7 x = 150
Difference of largest & middle number =
188. Ans. (C) 150 × (1.4 – 0.8) = 90
Male student from College Y = 168,
Female Student from College Y 394. Ans. (D)
4 Investment of Abhijeet = x × 34 = 34x,
= 168 ´ = 224
3 Investment of Daya = (x – 3700) × 51 =
Total Participants from college Y = 168 + 51(x – 3700)
224 = 392 Profit Ratio, Abhijeet : Daya = 34x:51(x –

PAGE h ps://t.me/studified h ps://instagram.com/aashisharorasocial?utm_medium=copy_link


Follow
141 Aashish Arora on: h ps://youtube.com/channel/UCYa4_JrOrf8R5Kz2uOtccXQ https://www.facebook.com/aashisharorasocial/
अ ास by Aashish Arora
(SBI/IBPS/RBI/LIC/All other banking and insurance Exams)

3700) = 8:7 Mixture P + Mixture Q = 323 + 323 = 646


34x 8 ml
=
51(x - 3700) 7
In final Mixture, Cola =
7x = 4 × 3(x – 3700)
13 21
7x = 12x – 44400 646 ´ = 247 ml, Pepso = 323 ´ = 399 ml
5x = 44400 34 34
x = 8880 Rs In Mixture Q, Cola:Pepsi = (247 – 114) :
(399 – 209) = 7:10
195. Ans. (B)
600 + 300
Speed of Garibrath = = 50 m/s 199. Ans. (C)
18 c = 0.2w
Speed of Aligarh Express =
15(7m + 4w) = 5(16m + 18w) = d(2m +
600 + 450 3w + 4c)
- 50 = 87.5 - 50 = 37.5 m/s
12 3(7m + 4w) = (16m + 18w)
Time Required by Aligarh Express to 21m + 12w = 16m + 18w
5m = 6w
450 + 150 m = 1.2w
cross the tunnel = = 16 second
37.5
15(7m + 4w) = d(2m + 3w + 4c)
15(7 × 1.2w + 4w) = d(2 × 1.2w + 3w + 4
196. Ans. (D) × 0.2w)
Present age of daughter = x year, 15(8.4 + 4) = d(2.4 + 3 + 0.8)
Mother = 8x year 15 × 12.4 = d × 6.2
12 year later, 8x + 12 = 4(x + 12) d = 30 days
8x + 12 = 4x + 48
4x = 36 200. Ans. (A)
x=9 In still water, Speed of Boat P = 21x
Mother's age after 6 year from now = 8 × km/h, Speed of Boat Q = 35x km/h
9 + 6 = 78 years Speed of stream = 5x km/h

197. Ans. (A) 182 144


+ =8
Money invested in CI = x Rs, SI = 20000 21x + 5x 21x - 5x
– x Rs 7 9 16
3 + =8=
(20000 – x) × 5 × 0.15 + x × (1.1 – 1) = x x x
11648 x=2
0.75(20000 – x) + 0.331x = 11648
15000 – 0.75x + 0.331x = 11648 Speed of Boat Q = 35 × 2 = 70 km/h,
0.419x = 3352 Speed of Stream = 5 × 2 = 10 km/h
x = 8000 Time taken by Q to travel in upstream =
Money invested in SI : Money invested 435
= 7.25 hours
in CI = (20000 – 8000):8000 = 70 - 10
12000:8000 = 3:2
201. Ans. (B)
198. Ans. (A) p ´15
SI, Y + Y ´ = 4Y
In Mixture P, Cola = 100
p = 20%
6 11 p + 10 = 20 + 10 = 30%
323 ´ = 114 ml, Pepsi = 232 ´ = 209 ml 2
CI, (Y + 150) × 1.3 = 2704
17 17
(Y + 150) × 1.69 = 2704

PAGE h ps://t.me/studified h ps://instagram.com/aashisharorasocial?utm_medium=copy_link


Follow
142 Aashish Arora on: h ps://youtube.com/channel/UCYa4_JrOrf8R5Kz2uOtccXQ https://www.facebook.com/aashisharorasocial/
अ ास by Aashish Arora
(SBI/IBPS/RBI/LIC/All other banking and insurance Exams)

Y + 150 = 1600 5
Speed of P = 79.2 ´ = 22 m/s,
Y = 1450 Rs 18
5
202. Ans. (A) Speed of Q = 100.8 ´ = 28 m/s
Let Income of Arnab, Bhupesh & Simar 18
is x Rs, y Rs & z Rs respectively Smaller train hide behind larger train for
Ratio of Saving, Arnab:Bhupesh:Simar
384 - 276
= 5x:20y:15z = x:4y:3z = 4:5:6 time of = = 2.16 seconds
x:y:z = 16:5:8 2 + 28
16
Income of Arnab = 23780 ´ = 13120 Rs,
29 207. Ans. (B)
5 In 2019, Average age become = 43 + 5 =
Income of Bhupesh = 23780 ´ = 4100 Rs,
29 48 year
8 When 48 year member died & 4 children
Income of Simar = 23780 ´ = 6560 Rs born,
29
Sum of Expenditure = 0.95 × 13120 + 8 ´ 48
Average age become = = 32 year
0.8 × 4100 + 0.85 × 6560 = 21,320 Rs 8+ 4
In 2021, Average age become = 32 + 2 =
203. Ans. (A) 34 year
8!
Total possible arrangement = 208. Ans. (C)
2!
Initially, Boys = 858 × 6/11 = 468, Girls =
Number of Arrangement, with exactly 3
858 ×5/11 = 390
letters between A = 4 × 6! When 26 boys added, Number of boys
2 ´ 4 ´ 6! 8 ´ 6! 1
Probability = = = become = 468 + 26 = 494
8! 8 ´ 7 ´ 6! 7 To make Boys:Girls = 19:17, Number of
17
204. Ans. (C) Girls should be = 494 ´ = 442
19
CP = x Rs, Profit = 0.6x Rs
If P% = x% Number of New girls that should be
Then, x × x/100 = 3.25 × 0.6x added = 442 – 390 = 52
x = 100 × 1.95
x = 195 Rs 209. Ans. (B)
SP of Book = 1.6 × 195 = 312 Rs Student preparing for CAT = x, preparing
for GMAT = 2800 – x
205. Ans. (D) Boys = 1.2(2800 – x) = 0.75 × Girls, Girls
Let Total Profit is = x Rs = 1.6(2800 – x)
Profit of Pallavi = 33000 + 0.42(x – 1.2(2800 – x) + 1.6(2800 – x) = 2800
33000) = 0.42x + 19140 Rs 2.8 × 2800 – 2.8x = 2800
Profit of Tejas = x – (0.42x + 19140) = 2.8x = 1.8 × 2800
0.58x – 19140 Rs x = 1800
(0.42x + 19140) – (0.58x – 19140) =
25000 210. Ans. (B)
38280 – 0.16x = 25000 Radius of park = Inner Radius of track =
0.16x = 13280 r, Outer Radius = r + 10.5 m
x = 83000 Rs Outer Perimeter – Inner Perimeter =
22 22 22
206. Ans. (D) 2´ ´ (r + 10.5) - 2 ´ ´ r = 2 ´ ´10.5 = 66 m
7 7 7

PAGE h ps://t.me/studified h ps://instagram.com/aashisharorasocial?utm_medium=copy_link


Follow
143 Aashish Arora on: h ps://youtube.com/channel/UCYa4_JrOrf8R5Kz2uOtccXQ https://www.facebook.com/aashisharorasocial/
अ ास by Aashish Arora
(SBI/IBPS/RBI/LIC/All other banking and insurance Exams)

211. Ans. (D) 9x = 1440


Upstream speed = x, Speed of Boat in x = 160 Rs
still water = 1.25x
1.25x – 3 = x 216. Ans. (A)
0.25x = 3 In mixture Q, Oil =
x = 12 km/h 6 5
Distance covered in Downstream = 99 ´ = 54 L, Water = 99 ´ = 45 L
11 11
æ 25 ö 137
= (1.25 ´12 + 3) ´ ç11 + ÷ = 18 ´ = 205.5 km In Mixture P, Oil = 54 + 10 = 64 L, Water =
è 60 ø 12
7
64 ´ = 65 L, Total = 64 + 56 = 120 L
8
212. Ans. (D)
Present Age of Victoria = x year, Ted = When 30 L of mixture P is replaced by 30
3(x – 7) year, Marshal = x + 12 year L oil
3(x – 7) + (x + 12) = 51 Oil in Mixture P =
3x – 21 + x + 12 = 51 8
4x – 9 = 51 64 - 30 ´ + 30 = 64 - 16 + 30 = 78 L
15
4x = 60 78 ´100
x = 15 % on Oil = = 65%
Present age of Victoria + Ted = 15 + 120
3(15 – 7) = 39 years
217. Ans. (C)
213. Ans. (B) Y(1.13 – 1) – Y × 0.1 × 3 = 6200
Ragh => 30 7 0.331Y – 0.3Y = 6200
} 210 { 0.031Y = 6200
Amit => 21 10 Y = 200000 Rs
8 218. Ans. (A)
Efficiency of Anamika = 7 ´ = 8,
7 (x + y) 4
6 =
Efficiency of Simi = 10 ´ = 12 (x - y) 1
5
Anamika & Simi Together complete the (x + y) = 4(x – y)
4x – x = y + 4y
210 3x = 5y
work in = = 10.5 Days
8 + 12 x 5
=
y 3
214. Ans. (C) x:y = 5:3
Profit Ratio, Zidane:Beckham = (775 × Only 1 number is possible, 35
8):(1250 × 4) = 31:25
25 219. Ans. (C)
Profit of Beckham = 1395 ´ = $1125 Let time taken by A & B = 5 days
31
A alone take = 6 days
A+ B = 5 6
215. Ans. (A) } 30 {
CP of Plate = 240/1.25 = 192 Rs, CP of A= 6 5
Each Cup = x Rs Efficiency of B = 6 - 5 = 1
CP = 9 × x + 5 × 192 = 9x + 960 Rs Time taken by B = 30/1 = 30 days
SP = 9 × 150 + 5 × 240 = 2550 Rs B take = (30 - 5)×100/5 = 500%
9x + 960 + 150 = 2550
9x = 2550 – 1110 220. Ans. (A)

PAGE h ps://t.me/studified h ps://instagram.com/aashisharorasocial?utm_medium=copy_link


Follow
144 Aashish Arora on: h ps://youtube.com/channel/UCYa4_JrOrf8R5Kz2uOtccXQ https://www.facebook.com/aashisharorasocial/
अ ास by Aashish Arora
(SBI/IBPS/RBI/LIC/All other banking and insurance Exams)

Let water flowing from P to Q Side of square = 26 cm


Downstream speed = 24 + 16 = 40 km/h In Rectangle
Upstream speed = 24 - 16 = 8 km/h Length (2 ´ 26) 52
Average speed = (2×40×8)/(40 + 8) = = =
Breadth (26 - 10) 16
40/3 km/h
Length:Breadth = 13:4
221. Ans. (D)
Total investment of X = 36000×6 Rs 227. Ans. (A)
Total investment of Y = 24000×24 Rs Let x more man work for 20 days to
Total investment of Z = 10000×18 Rs complete work on time
Profit Ratio= X:Y:Z =(36000×6):(24000× 20 ´ 60 20(20 + x )
24):(10000×18) = 216:576:180 = 9:16:5 =
3 2
222. Ans. (C) 40 = 20 + x
Let 5x L of mixture X is mixed with 5y L of x = 20
mixture Y, then
228. Ans. (B)
(4x + y) 1 (160 – 64) × z = 384
=
(x + 4y) 3 96 × z = 384
3(4x + y)= (x + 4y) z=4
12x + 3y = x + 4y
11x = y 229. Ans. (A)
x:y = 1:11 CP of each article = x Rs
1.2x – 0.9x = 568
223. Ans. (B) 0.3x = 568
Present age of X = x, Y = y SP at loss = 0.9x = 1704 Rs
x=y+8
230. Ans. (D)
( x + 8) 9
= (28 + 4 + 6 + 12 + 10)
( y + 8) 8 Average = = 12
5
8(x + 8) = 9(y + 8)
8x + 64 = 9x Removed number = 12 × 5 – 12 × 4 = 12
x = 64
231. Ans. (C)
224. Ans. (B) Profit Ratio = P:Q:R = (4 × 4):(1 × 3):(7 ×
Income of X = 25000 Rs. 5) = 16:3:35
Income of Y = 25000/1.25 = 20000 Rs. Profit of P & Q = 16 + 3 = 19,
Savings of Y = 20000 - 10000 = 10000Rs Profit of R = 35
Savings of X = 10000 - 2000 = 8000 Rs Profit of P & Q together is less then Profit
Expense of X = 25000 - 8000 = 17000 Rs (35 - 19) ´100
% = 17000×100/25000 = 68% of R by = = 45.71%
35
225. Ans. (C)
Actual average weight = x 232. Ans. (B)
Weight of new man = y In 0.25 × 180 + 180 = 225 L, Milk = 125 L,
12x - 26 + y = 12(x + 4.5) Water = 100 L
– 26 + y = 54 In Final mixture, Water of mixture Y = 90
y = 80 kg L, Water of mixture X = 10 L
Initial quantity of water in mixture
226. Ans. (D) 10
Area of square = 676 cm
2
X= = 40 L
0.25

PAGE h ps://t.me/studified h ps://instagram.com/aashisharorasocial?utm_medium=copy_link


Follow
145 Aashish Arora on: h ps://youtube.com/channel/UCYa4_JrOrf8R5Kz2uOtccXQ https://www.facebook.com/aashisharorasocial/
अ ास by Aashish Arora
(SBI/IBPS/RBI/LIC/All other banking and insurance Exams)

233. Ans. (A) 239. Ans. (B)


D + 20 D - 30 Time taken by B alone =
= 28(9 + 8 + 5)
7+5 7-5 = 77 days
8
(D + 20) = 6(D – 30)
D + 20 = 6D – 180
5D = 200 240. Ans. (A)
D = 40 Present age of Lavi = 8x, Anil = 7x
After 16 year,
234. Anc. (D) 8x + 16 12
(x + 1200) × 3 × 0.5 = (3x – 3600) =
7x + 16 11
1.5x + 1800 = 3x – 3600
1.5x = 5400 11(8x + 16) = 12(7x + 16)
x = 3600 Rs 88x + 11 × 16 = 84x + 12 × 16
2
CI = (2 × 3600) × (1.2 – 1) = 3168 Rs 4x = 16
x=4
235. Ans. (C) Present age of Avi = 40 – 8 × 4 = 8 years
Side of cube X = a cm
Total Surface Area – Curved Surface 241. Ans. (B)
Area = 200 = 6a2 – 4a2 Investment of A = (2 × 2500) + (x + 1300)
2a2 = 200 = x + 6300 Rs
a2 = 100 Investment of B = (2 × 3500) + (x + 1000)
a = 12 cm = x + 8000 Rs
Side of Cube Y = 1.8 × 10 = 18 cm Investment of C = (2 × 4000) + (x + 1700)
Total Surface Area + Curved Surface = x + 9700 Rs
2 2
Area = 6 × 18 + 4 × 18 = 10 × 324 = Total profit = 3x + 24000 Rs
3240 cm2 6000 ´ (x + 8000)
Profit of B = = 2000 Rs
236. Ans. (B) (3x + 24000)
4 year later, age of X = 5x, Y = 7x
5x - 6 1 242. Ans. (C)
2 year ago, = x(1.23 – 1) – 4 × 0.15x = 1024
7x - 6 2 0.728x – 0.6x = 1024
2(5x – 6) = 7x – 6 0.128x = 1024
3x = 6 x = 8000 Rs.
x=2
Difference of their ages = 7x – 5x = 2x = 243. Ans. (D)
4 year A+ V + K = 18 3
} 54 {
237. Ans. (A) A+ K = 27 2
P = 24 4 Efficiency of Vicky = 3 – 2 = 1, Efficiency
} 96 { of Aditya = 10/9
Q = 32 3 10 8
In 9 day complete = 9(4 + 3) = 63 Efficiency Karan = 3 - 1 - =
Remaining part = (96 – 63)/96 = 11/32 9 9
Time Taken by Karan =
238. Ans. (A)
Speed of stream = s km/h 54 243
= = 60.75 days
12(28 – x) = 252 8 4
28 – x = 21 9
x = 7 km/hr

PAGE h ps://t.me/studified h ps://instagram.com/aashisharorasocial?utm_medium=copy_link


Follow
146 Aashish Arora on: h ps://youtube.com/channel/UCYa4_JrOrf8R5Kz2uOtccXQ https://www.facebook.com/aashisharorasocial/
अ ास by Aashish Arora
(SBI/IBPS/RBI/LIC/All other banking and insurance Exams)

244. Ans. (E) 5x – 100 = 4x – 68


Total Balls = (x + 1) + (x + 4) + x = 4x + 5 x = 32
Probability of getting 1 red ball & 1 blue Initial quantity of mixture = 12 × 32 = 384
ball Ans. (D)
2(x + 1)(x + 4) 1
= 249. X = 24 5
(4x + 5)(4x + 4) 6
} 120 {
12(x2 + 5x + 4) = 16x2 + 36x + 20 Y= 40 3
4x2 – 24x – 28 = 0 120 = 10(5 + 3) + 10z
x2 – 6x – 7 = 0 Efficiency of c = z = 4 unit/yay Ans. (A)
(x – 7)(x + 1) = 0
x=7 250. 100800/(9×7) = 1600
Total Balls = 4 × 7 + 5 = 33 7×5×1600 = 5600 Ans. (A)

245. Ans. (B) 251. Profit Ratio = X:Y:c = (20×8) : (24×6) :


Speed of X = s m/s, Length of train = l m (32×10) = 10:9:20
l = 15s Difference betweeen profit of Y &o c =
l + 150 = 45s 7000×(20 – 9)/10 = 7700 Ans. (B)
s = 5 m/s, l = 75 m
Speed of Y = y m/s, Length of Y = 75 – 252. 9x×0.2 + 180 = 0.3×7x
20 = 55 m 1.8x + 180 = 2.1x
75 + 55 = 5(y + 5) x = 600
y + 5 = 26 Pocket money in 2ny month = 4x = 2400
Speed of Y = 21 m/s Rs.Ans. (B)
246. Ans. (A) 253. Speey of Current = x km/h, Speey of
Radius of Cylinder = 3x cm, Height =
Boat in Still Water = x + 8 km/h,
14x cm 2(x + 8) – x = 125/2.5
22 2 x = 34 km/h
´ (3x ) ´14x = 10692 Dowenstream speey = (34 + 8) + 34 = 76
7
3
396x = 10692 km/h Ans. (C)
3
x = 27
x = 3 cm 254. Rayius of Circle = 264/(2×22/7) = 42 m
Area of curved surface = Length of Rectangle = 42 + 6 = 48 m
2(48 + b) = 160
22 b = 32 m
2 ´ ´ (3 ´ 3) ´ (14 ´ 3) = 2376 cm 2
7 Area = 48 × 32 = 1536 m2 Ans. ©
247. Ans. (D) 4
Income of P = x Rs, Income of Q = x + 255. (1 + r/100) = 4
n
(1 + r/100) = 256 = 44
4000 Rs n 4×4
(1 + r/100) = (1 + r/100) = (1 + r/100)
16

x - 4000 3 N = 16 Ans. (B)


=
(x + 4000) - 15000 2
256. Initially, Price = x Rs./unit, Consumption
2(x – 4000) = 3(x – 11000)
2x – 8000 = 3x – 33000 = y unit
3x – 2x = 33000 - 8000 xy = 1000
x = 25000 Rs Newe Expenyiture = 1.2x×0.7y =
0.84×1000 = 840 Rs.Ans. (C)
248. (5x – 100)/(7x – 140 + 21) = 4/7
7(5x – 100) = 4(7x – 119) 257. Actual, CP = x Rs, SP = y Rs

PAGE h ps://t.me/studified h ps://instagram.com/aashisharorasocial?utm_medium=copy_link


Follow
147 Aashish Arora on: h ps://youtube.com/channel/UCYa4_JrOrf8R5Kz2uOtccXQ https://www.facebook.com/aashisharorasocial/
अ ास by Aashish Arora
(SBI/IBPS/RBI/LIC/All other banking and insurance Exams)

x×1.25 = y×25/19 266. Total Income = x Rs


x×19/20 = y = 0.95x 0.3x + 0.2×0.7x = 15840
Loss% = (x – 0.95x)×100/x = 5% 0.44x = 15840
Ans. (D) x = 36000 Rs
Saving = 0.25×3600 = 9000 Rs Ans. (D)
2
258. (1 + r/100) = 3600/1600
1 + r/100 = 1.5 267. Profit Ratio, A:B:C = (14 × 12) : (x × 12) :
r = 50% (13 × 9) = 56:4x:39
2400(50 – 20)×5/100 = 3600 Rs. 7560/17685 = 56/(48 + 4x + 39)
Ans. (C) 87 + 4x = 131
x = 11 Ans. (B)
259. Investment of P = 0.25×3600 = 900 Rs.
Investment of Q = 2700 Rs. 268. Ans. (B)
Profit Ratto, P:Q = (900×8):(2700×7) = 8(x – y) = (x + y)
8:21 Ans. (B) 8x – x = 8y + y
7x = 9y
260. a×36 = 7×2×(22/7)×45 x:y = 9:7
a = 55 m/sec
640/(55 – 15) = 16 second Ans. (C) 269. Ans. (D)
Let after t seconds, bullet hit the car.
261. Present age of Neetu = x year, Present Speed of car = 27×5 / 18 = 7.5 m/s
age of Rtma = 3(x + 4) 7.5t + 1t = 34
(3x + 12 + 20)/(x + 20) = 20/9 8.5t = 34
27x + 288 = 20x + 400 t = 4 sec
7x = 112
x = 16 270. Ans. (A)
2
Neetu = 16 year, Rtma = 60 year Area of field = 8550/9.5 = 900 m
60 – 16 = 44 year Ans. © If each side is increased by 20%, Area
will be increased by = 1.22 = 1.44
262. 6a2 – 6b2 = 504 New Area = 1.44 × 900 = 1296 m2
a2 – b2 = 84 Cost of cultivating per m2 = 8550/1296 =
(a – b)(a + b) = 84 6.59 Rs.
4(a + b) = 84 Decreased = 9.5 – 6.6 = 2.9 Rs
a + b = 21 cm Ans. (D)
271. Ans. ©
263. 5m + 4w = 3m + 6w Total Received amount = 3 × 3600 +
2m = 2w 3600 × 2 × (0.16 + 0.13 + 0.11) = 13680
m/w = 1/1
m:w = 1:1 Ans. (D) Rs

264. (16x + 350)/(9x + 250) = 3/2 272. Ans. (D)


2(16x + 350) = 3(9x + 250) 5 year ago, Age of Manish = 5x, Ravi =
32x + 700 = 27x + 750 3x
5x = 50 3 year later,
x = 10 5x+8 / 3x+8 = 9/7
Number of Shoes sold = 16x = 160 35x + 56 = 27x + 72
Ans. (D) 8x = 16
x=2
265. Water/Mtlk = (14 + 4)/(35 + 5) = 9/20 = Present Age of Ravi = 3 × 2 + 5 = 11 year
9:20 Ans. (C)
273. Ans. (C)

PAGE h ps://t.me/studified h ps://instagram.com/aashisharorasocial?utm_medium=copy_link


Follow
148 Aashish Arora on: h ps://youtube.com/channel/UCYa4_JrOrf8R5Kz2uOtccXQ https://www.facebook.com/aashisharorasocial/
अ ास by Aashish Arora
(SBI/IBPS/RBI/LIC/All other banking and insurance Exams)

Number of way of getting 2 black ball = P & Q together complete the work in =
5
C2 = 10 30/8 = 3.75 hour
Number of way of getting 2 white ball = They will complete work after 3.75 hour
3
C2 = 3 at = 8:15 PM
Number of way of getting 4 ball out of all
= 20C4 = 4845 280. Ans. (C)
Probability of getting 2 black & 2 white Nishant’s investment = x × 6 + (x + 900)
ball = (10 × 3)/4845 = 2/323 × 6 = 12x + 5400 Rs.
Naveen’s investment = (x + 2800) × 6 +
274. Anc. (A) (x + 2000) × 6 = 12x + 28800 Rs.
Petrol = 0.65×320 = 208 L, Diesel = 112 L 12x + 28800 72000 4
208 × 0.75 = 112 + x = =
x = 156 - 112 24x + 34200 126000 7
x = 44 7(x + 2400) = 4(2x + 2850)
7x + 16800 = 8x + 11400
275. Ans. (A) x = 5400 Rs.
SP = 100 Rs. Profit = 38 Rs
CP = 100 – 38 = 62 Rs 281. Ans. (D)
Actual profit% = 38 ×100 / 62 = 61.29% Speed of Person = 2x m/s, Speed of
Train = 9x m/s
276. Ans. (C) 560 = (2x + 9x) × 36
18 × 16x = 8 × 9 (5x – 3) = d × 16(x + 3) x = 140/99
18 × 16x = 8 × 9(5x – 3) In same direction, speed = 9x – 2x = 7x =
4x = 5x – 3 7 × 140/99 = 980/99 m/s
x=3
560
18 × 16 × 3 = d × 16 × (3 + 3) Time = = 56.57 seconds
d=9 980
99
277. Ans. (D) 282. Ans. (A)
Initial Investment of A = 4x, B = x Radius of circle = r m
After 6 month, Investment of C = c 22/7 × r2 = 7546
(4x + x + c) × 0.4 = 4x 2
r = 2401
5x + c = 10x r = 49 m
c = 5x 2 × 22/7 × 49 = 240 + x
Profit Ratio = (4x × 12):(x × 12):(5x × 6) = x = 308 – 240
8:2:5 Breadth of Rectangle = 68 m
Profit of A & B = 8525 × 10 / 5 = 17050 Rs
283. Ans. ©
278. Ans. (C) Let rate of interest offered is = r%
Present age of Samrat = x year, Ruhi =
1.75x year, Rinki = 1.75x – 6 year æ r ö
4ç ÷
Sum of present age of all 3 è 10 ø 1200 15
1
= =
x + (1.75x – 6) + 1.75x = 129 – 3 × 6 æ r ö 640 8
4.5x = 111 + 6 ç1 + ÷ -1
è 100 ø
x = 26 year
r éæ r ö 2 æ r öù
8´ 4´ = 15êç ÷ + 30ç ÷ú
279. Ans. (B) 100 êëè 100 ø è 100 øúû
P = 6 hour 5
} 30 { æ r ö
32 = 15ç ÷ + 30
Q = 10 Hour 3 è 100 ø

PAGE h ps://t.me/studified h ps://instagram.com/aashisharorasocial?utm_medium=copy_link


Follow
149 Aashish Arora on: h ps://youtube.com/channel/UCYa4_JrOrf8R5Kz2uOtccXQ https://www.facebook.com/aashisharorasocial/
अ ास by Aashish Arora
(SBI/IBPS/RBI/LIC/All other banking and insurance Exams)

æ r ö Investment of Sima = 12x + 1200 × 8 =


15ç ÷=2 12x + 9600 Rs
è 100 ø Investment of Rima = (x + 3200) × 4 + (x
200 100 + 500) × 8 = 12x + 16800 Rs
r= = = 13.33%
15 3 12x + 16800 72000 3
= =
284. Anc. (B) 24x + 26400 120000 5
d d 5(12x + 16800) = 3(24x + 26400)
+ = 16 60x + 84000 = 72x + 79200
32 + 8 32 - 8
12x = 4800
d d
+ = 16 x = Initial Investment of Sima = 400 Rs.
40 24
d 291. Ans. (C)
= 16
15 Let numbers of error in remaining pages
d = 240 km =x
610 × 6 = 2736 + 154x
285. Ans. (A) 154x = 3660 – 2736
Number of student in class = x 154x = 924
43x – 15 + 51 = 43.9x x=6
0.9x = 36
x = 40 292. Ans. (A)
Let, 18 kg of P1 & P2 is taken, & Mixed to
286. Ans. (B)
Total property of Atish = form P3
In P3, Maida:Besaan = (10 + 5):(8 + 13)
49200
= 20,50,000 Rs = 15:21 = 5:7
0.4 ´ 0.3 ´ 0.2
293. Ans. (C)
287. Ans. (A) Time taken by not serviced bike to cover
5 consecutive odd numbers are = (x – same distance =
4), (x – 2), x , (x + 2) + (x + 4) 5 ´ 65
= 8.125 = 8 hour (approx)
Average = x = 57 40
Then Numbers are = 53, 55, 57, 59, 61
Next 5 odd numbers are = 63, 65, 67, 294. Anc. (B)
69, 71 d d
+ = 32
Average = 67 72 + 9 72 - 9
Sum of Numbers = 5 × 67 = 335 d d
+ = 32
81 63
288. Ans. (C)
Money that Rahul had = 74250/0.15 = 16d
= 32
495000 Rs 567
d = 1134 km
289. Ans. (B)
Length = 11x m, Width = 9x m 295. Ans. (A)
Cost of fencing = 2(11x + 9x) × 8 = 8w = 17g
10240 26 × 17g × (50 × 0.9) = d(4w + 24g)
320x = 10240 26 × 17g × 45 = d(8.5g + 24g)
x = 32 26 × 17g × 45 = d × 32.5g
2
Area = (11 × 32) × (9 × 32) = 101376 m d = 612 days

290. Ans. (A) 296. Ans. (B)

PAGE h ps://t.me/studified h ps://instagram.com/aashisharorasocial?utm_medium=copy_link


Follow
150 Aashish Arora on: h ps://youtube.com/channel/UCYa4_JrOrf8R5Kz2uOtccXQ https://www.facebook.com/aashisharorasocial/
अ ास by Aashish Arora
(SBI/IBPS/RBI/LIC/All other banking and insurance Exams)

Mr. Laheri complete total work in = 120 = 0.4x + 84


15/0.6 = 25 0.4x = 36
Let efficiency of Mr. Laheri = 1, x = 90 Litres
Efficiency of Mr. Sharma = x
25 × 0.4 = 3(1 + x) 301. Ans. (B)
x= 7/3 Time = 189/(70 – 28) = 4.5 Hour
Alone Mr. Sharma complete total work
302. Ans. (B)
25 75 5
in = 7 = = 10 days D + 5 D - 45
7 7 =
3 8+5 8-5
3(D + 5) = 13(D – 45)
297. Ans. (E) 3D + 15 = 13D – 585
2
10D = 600
æ 25 ö D = 60 km
Loss% = ç ÷ = 6.25%
è 10 ø
303. Ans. (B)
298. Ans. (B) Side of cube A = a cm
Time take to complete the work by Total Surface Area – Curved Surface
2 2
Anuj = 5 × 3 = 15 days, Bikash = 10 × 1.5 Area = 288 = 6a – 4a
2

= 15 days 2a = 288
2

Efficiency of Anuj = Efficiency of Bikash a = 144


a = 12 cm
=1 For cube B
Efficiency of Dinesh = 1.25 Side of B = 1.75 × 12 = 21 cm
th
æ 13 ö Total Surface Area + Curved Surface
All together complete ç ö of work in =
è 15 ø Area = 6 × 212 + 4 × 212 = 10 × 441 =
2
æ 13 ö 4410 cm
15 ´ ç ÷
è 15 ø = 4 days 304. Anc. (D)
1 + 1 + 1.25
595
CP of 1 article = = 1700 Rs
1.2 - 0.85
299. Ans. (A)
Profit Ratio = Rahul:Ramesh:Rohit = (4 SP of article sold at loss = 0.85 × 1700 =
× 9):(3 × 8):(7 × 10) = 18:12:35 1445 Rs.
Profit of Rahul & Ramesh = 18 + 12 =
30, Profit of Rohit = 35 305. Ans. (C)
Profit of Rahul & Ramesh is less then 6 year hence, Age of A = 5x, B = 9x
5x - 9 13
(35 - 30) ´100 3 year ago, =
Profit of Rohit by = = 14.28% 9x - 9 27
35
3(5x – 9) = 13(x – 1)
300. Ans. (D) 15x – 27 = 13x – 13
In Mixture X, Fanta = 150 L, Maaza = x L 2x = 14
After mixing 120 L of mixture Y, with x=7
40% of mixture X Difference of present age = (9x – 6) – (5x
Fanta in new mixture = 150 × 0.4 + 36 = – 6) = 4x = 4 × 7 = 28 year
96 L
96 ´ 5 306. Ans. (B)
Maaza in new mixture = = 120 L Current average =
4

PAGE h ps://t.me/studified h ps://instagram.com/aashisharorasocial?utm_medium=copy_link


Follow
151 Aashish Arora on: h ps://youtube.com/channel/UCYa4_JrOrf8R5Kz2uOtccXQ https://www.facebook.com/aashisharorasocial/
अ ास by Aashish Arora
(SBI/IBPS/RBI/LIC/All other banking and insurance Exams)

34 + 18 + 12 + 14 + 22 + 8 314. Length = 9x m, Width = 4x m


= 18 2×(9x + 4x) = 780
6 x = 30
Removed number = 18 × 6 – 18 × 5 = 18 Area = 270 × 120 = 32400 m2
(Ans.)
307. Ans. (D)
Interest in 2 year = 7200 – 6000 = 1200 315. (1 + x/100)2 = 400/144
Rs (1 + x/100) = 5/3
Interest in 1 year = 600 Rs 840(1 + x/100) = 840 × 5/3 = 1400 Rs
Interest in 5 year = 5 × 600 = 3000 Rs (Ans.)
Principal = 6000 – 3000 = 3000 Rs
316. CP = 4 + 9 = 13
308. Number of man asked to complete the SP = 4×1.25 + 9*0.8 = 12.2
work = x Loss% = (13 – 12.2) × 100/13 = 6.15%
12x = 24(x – 5) (Ans.)
12x = 24x – 24×5
x = 10 (Ans.) 317. 8x × 1.3 - 5x×0.8 = 32000
x = 5000
309. Number of Boys = x, Girls = 60 – x Expenditure in 2020 = (8 – 5) × 5000 =
5x + 10(60 – x) = 410 15000 Rs
x = 38 = Boys
Girls = 60 – 38 = 22
38 – 22 = 16 (Ans.)

310. Probability of getting 1st red ball, 2nd


green ball & 3rd blue ball
12 36 12 3
= ´ ´ =Ì (Ans.)
16 60 60 125
72180
311. = 1203
4´ 5´ 3
Bill paid by X = 4×4×1×1203 = 19248 Rs
(Ans.)

312. Shekhar = 20000×4 + 12000×8 =


176000 Rs.
Mantoo = 16000×4 + 10000×8 =
144000 Rs.
Pro fit R a ti o , Sh e kh a r/Ma n to o =
176000/144000 = 11/9
Profit Share of Shekhar = 50700×11/20
= 27885 Rs (Ans.)

313. Speed of Boat in Still Water = x km/h,


Speed of Current = y km/h
1.5(x + y) = 105, x + y = 70
2.5(2x – y) = 125, 2x – y = 50
y = 30 km/h (Ans.)

PAGE h ps://t.me/studified h ps://instagram.com/aashisharorasocial?utm_medium=copy_link


Follow
152 Aashish Arora on: h ps://youtube.com/channel/UCYa4_JrOrf8R5Kz2uOtccXQ https://www.facebook.com/aashisharorasocial/
अ ास by Aashish Arora
(SBI/IBPS/RBI/LIC/All other banking and insurance Exams)

6 Data Interpretation (Caselet)

Read the given data carefully and answer the following questions.
Three companies namely Sony, HP and Dell sold different number of laptops in three different
months. In April, Sony sold 30% more laptops than that in May. Ratio of laptops sold by HP in
May to Dell in June is 8 : 9 respectively. Total number of laptops sold by Sony in three months
together was 2770. HP sold 120 more laptops than Dell in May. In May, Sony sold 50% more
laptops than Dell in same month. There was 10% decrease in number of laptops sold by HP in
May to June. Total number of laptops sold in June by three companies together was 2158. Total
number of laptops sold by Dell in three months together was 2110 and HP sold 800 laptops in
April.

दी गयी जानकारी को ान से पढ़ और िन िल खत ो ं के उ र द।
सोनी, एचपी और डे ल जैसी तीन कंपिनयो ं ने तीन अलग-अलग महीनो ं म अलग-अलग सं ा म लैपटॉप बेचे। सोनी
ने अ ैल म, मई की तुलना म ३०% अिधक लैपटॉप बेचे। मई म एचपी ारा और जून म डे ल ारा बेचे गए लैपटॉप का
अनुपात मशः ८ : ९ है । सोनी ारा तीन महीनो ं म एक साथ बेचे गए लैपटॉप की कुल सं ा २७७० थी। एचपी ने मई
म, उसी महीने डे ल की तुलना म १२० अिधक लैपटॉप बेचे। मई म, सोनी ने उसी महीने डे ल की तुलना म ५०% अिधक
लैपटॉप बेचे। मई से जून म एचपी ारा बेचे जाने वाले लैपटॉप की सं ा म १०% की कमी थी। एक साथ तीन कंपिनयो ं
ारा जून म बेचे गए लैपटॉप की कुल सं ा २१५८ थी। डे ल ारा तीन महीनो ं म एक साथ बेचा गया लैपटॉप की कुल
सं ा २११० थी और एचपी ने अ ैल म ८०० लैपटॉप बेचे थे।

1. Find the difference between total number of laptops sold by Sony and HP in all the three
months together.
सभी तीन महीनो ं म सोनी और एचपी ारा बेचे गए लैपटॉप की कुल सं ा के बीच का अंतर ात कीिजए।
A. 698 B. 602 C. 616
D. 694 E. None of these/इनम से कोई नही ं

2. Find the ratio of number of laptops sold by Sony in May and June together to number of
laptops sold by Dell in April and May together.
सोनी ारा मई और जून म बेचे गए लैपटॉप की सं ा और डे ल ारा अ ैल और मई म बेचे गए लैपटॉप की
सं ा का अनुपात पता लगाएं ।
A. 16 : 15 B. 16 : 17 C. 16 : 13
D. 16 : 11 E. None of these/इनम से कोई नही ं

3. The number of laptops sold by Sony and Dell together in June is approximately what
percent of the total number of laptops sold in April?
जून म सोनी और डे ल ारा एक साथ बेचे गए लैपटॉप की सं ा अ ैल म बेचे गए लैपटॉप की कुल सं ा का

PAGE h ps://t.me/studified h ps://instagram.com/aashisharorasocial?utm_medium=copy_link


Follow
153 Aashish Arora on: h ps://youtube.com/channel/UCYa4_JrOrf8R5Kz2uOtccXQ https://www.facebook.com/aashisharorasocial/
अ ास by Aashish Arora
(SBI/IBPS/RBI/LIC/All other banking and insurance Exams)

लगभग िकतने ितशत है ?


A. 53% B. 56.5% C. 60.5%
D. 58% E. None of these/इनम से कोई नही ं

4. Average rate per laptop sold by Sony in April was Rs. 30,000 and same for Dell in June was
Rs. 35,000. What was the total revenue earned by Sony in April and Dell in June ?
सोनी ारा अ ैल म ित लैपटॉप की औसत दर 30,000 थी और डे ल ारा जून म ित लैपटॉप की औसत
दर 35,000 थी । अ ैल म सोनी और जून म डे ल ारा अिजत कुल राज ा था?
A. Rs. 6,34,50,000 B. Rs. 6,46,50,000 C. Rs. 6,57,50,000
D. Rs. 6,32,50,000 E. None of these/इनम से कोई नही ं

5. Find the total number of laptops sold by all three companies in three months together.
तीन महीनो ं म एक साथ सभी तीन कंपिनयो ं ारा बेचे गए लैपटॉप की कुल सं ा ात कीिजए।
A. 7218 B. 7048 C. 7118
D. 7028 E. None of these/इनम से कोई नही ं

Read the given data carefully and answer the following questions.
A company sold two types of printers that is laser and inkjet on different days of a week starting
from Monday. Total printers sold till Thursday is 88.88% more than the same sold on Monday.
Ratio of total printers sold till Tuesday to total printers sold till Friday is 2 : 3 respectively. Total
printers sold till Thursday is 41.66% more than the total printers sold till Tuesday which is 1080.
Total printers sold till Wednesday is 75% of total printers sold till Friday. 58(14/17)% of the
printers sold till Thursday are laser printers which is 350 more than the laser printers sold till
Tuesday. Laser printers sold till Friday is 120% more than the same sold on Monday which is 470
less than the laser printers sold till Thursday. Laser printers sold till Wednesday is 14/27 of total
printers sold till Wednesday.

दी गयी जानकारी को ान से पढ़ और िन िल खत ो ं के उ र द।
एक कंपनी ने सोमवार से शु होने वाले स ाह के िविभ िदनो ं म दो कार के ि ंटर बेचे ह जो लेजर और इं कजेट
ह। गु वार तक बेचे गए कुल ि ंटर, सोमवार को बेचे गए कुल ि ंटर से 88.88% अिधक ह। मंगलवार तक बेचे गए
कुल ि ंटरो ं और शु वार तक बेचे गए कुल ि ंटरो ं का अनुपात मशः 2 : 3 है । गु वार तक िबकने वाले कुल ि ंटर,
मंगलवार तक िबकने वाले कुल ि ंटर जो िक 1080 है , की तुलना म 41.66% अिधक है । बुधवार तक िबकने वाले
कुल ि ंटर, शु वार तक िबकने वाले कुल ि ंटर का 75% है । गु वार तक िबकने वाले ि ंटरो ं म से, 58(14/17) %
लेजर ि ंटर ह जो मंगलवार तक िबकने वाले लेजर ि ंटरो ं से 350 अिधक ह। शु वार तक िबकने वाले लेजर ि ंटर,
सोमवार को िबकने वाले लेजर ि ंटर से 120% अिधक है जो गु वार तक िबकने वाले लेजर ि ंटर से 470 कम है ।
बुधवार तक बेचे जाने वाले लेजर ि ंटर, बुधवार तक बेचे गए कुल ि ंटर का (14/27) गुना है ।

6. What are the total number of printers sold on Thursday ?


गु वार को िबकने वाले ि ंटरो ं की कुल सं ा ा है ?
A. 330 B. 315 C. 270

PAGE h ps://t.me/studified h ps://instagram.com/aashisharorasocial?utm_medium=copy_link


Follow
154 Aashish Arora on: h ps://youtube.com/channel/UCYa4_JrOrf8R5Kz2uOtccXQ https://www.facebook.com/aashisharorasocial/
अ ास by Aashish Arora
(SBI/IBPS/RBI/LIC/All other banking and insurance Exams)

D. 440 E. None of these/इनम से कोई नही ं


7. Find the ratio of number of Laser printers to number of Inkjet printers sold on Tuesday.
मंगलवार को बेचे जाने वाले लेजर ि ंटर और इं कजेट ि ंटर की सं ा का अनुपात ात कीिजए।
A. 6 : 5 B. 6:7 C. 4:5
D. 4 : 7 E. None of these/इनम से कोई नही ं

8. Number of Inkjet printers sold on Wednesday is what how much percent more or less than
the number of Inkjet printers sold on Thursday ?
बुधवार को बेचे जाने वाले इं कजेट ि ंटर की सं ा गु वार को बेचे जाने वाले इं कजेट ि ंटर की सं ा से
िकतने ितशत अिधक या कम है ?
A. 25% B. 22.22% C. 33.33%
D. 40% E. None of these/इनम से कोई नही ं

9. On which day number of inkjet printers sold are minimum ?


िकस िदन बेचे गए इं कजेट ि ंटर की सं ा ूनतम है ?
A. Friday B. Thursday C. Wednesday
D. Tuesday E. None of these/इनम से कोई नही ं

10. Average number of Inkjet printers sold on Wednesday and Thursday together is :
बुधवार और गु वार को एक साथ बेचे जाने वाले इं कजेट ि ंटर की औसत सं ा :
A. 55.55% of total number of printers sold on Friday
/शु वार को बेचे जाने वाले कुल ि ंटरो ं का 55.55% है
B. Only (A), (C) and (D)
C. 41.66% of laser printers sold on Tuesday/मंगलवार को जाने वाले लेजर ि ंटर का 41.66% है
D. 50
E. None of these/इनम से कोई नही ं

Read the given data carefully and answer the following questions.
In Jaipur an Delhi there are four types of cars sold that are Nano, Kwid, Alto and Datsun. Nano
sold in Jaipur is 58.33% more than that in Delhi. Ratio of Kwid sold in Jaipur to that in Delhi is 10 :
13 respectively. Datsun sold in Jaipur is 39 less than Kwid sold in Delhi and Datsun sold in Delhi is
85 less than Nano sold in Jaipur. Total Datsun sold is 1066. Difference between Alto sold in Jaipur
to that in Delhi is 78. Ratio of Alto sold in Delhi to that in Jaipur is 17 : 15 respectively. Nano sold in
Delhi is 300.

दी गयी जानकारी को ान से पढ़ और िन िल खत ो ं के उ र द।
जयपुर और िद ी म चार कार की कार बेची जाती ह जो नैनो, ड, ऑ ो और डै टसन ह। जयपुर म िबकने वाली
नैनो िद ी म िबकने वाली नैनो से 58.33% अिधक है । जयपुर और िद ी म िबकने वाले ड का अनुपात मशः
10:13 है । जयपुर म िबकने वाली डै टसन िद ी म िबकने वाली ड से 39 कम है और िद ी म िबकने वाली
डै टसन जयपुर म िबकने वाली नैनो से 85 कम है । बेची गयी कुल डै टसन 1066 है । िद ी और जयपुर म बेची गई
ऑ ो के बीच अंतर 78 है । िद ी और जयपुर म ऑ ो की िब ी का अनुपात मशः 17:15 है । िद ी म िबकने

PAGE h ps://t.me/studified h ps://instagram.com/aashisharorasocial?utm_medium=copy_link


Follow
155 Aashish Arora on: h ps://youtube.com/channel/UCYa4_JrOrf8R5Kz2uOtccXQ https://www.facebook.com/aashisharorasocial/
अ ास by Aashish Arora
(SBI/IBPS/RBI/LIC/All other banking and insurance Exams)

वाली नैनो 300 है ।

11. What is the ratio of total number of Nano and Kwid sold in Jaipur to that in Delhi ?
जयपुर और िद ी म बेची जाने वाली नैनो और ड की कुल सं ा का अनुपात ा है ?
A. 204 :209 B. 205 : 203 C. 203 : 201
D. 211 : 217 E. None of these/इनम से कोई नही ं

12. Find the total number of Kwid sold.


बेची गयी ड की कुल सं ा ात कीिजए।
A. 1625 B. 1526 C. 1265
D. 1256 E. None of these/इनम से कोई नही ं

13. Find the difference between total number of cars sold in Jaipur to that in Delhi.
जयपुर और िद ी म बेची गई कारो ं की कुल सं ा के बीच का अंतर ात कीिजए।
A. 288 B. 218 C. 282
D. 222 E. None of these/इनम से कोई नही ं

14. Find the total number of cars sold in Jaipur and Delhi together.
जयपुर और िद ी म कुल बेची गई कारो ं की सं ा ात कीिजए।
A. 4354 B. 4914 C. 4944
D. 4114 E. None of these/इनम से कोई नही ं

15. Find the ratio of Kwid and Alto sold in Jaipur respectively.
जयपुर म बेची गयी मशः ड और ऑ ो के अनुपात का पता लगाएं ।
A. 110:117 B. 55 : 51 C. 55 : 41
D. 53 : 50 E. None of these/इनम से कोई नही ं

Read the given data carefully and answer the following questions.
On a particular day, 792 people went to gym by cycle, bike and car in three different shifts. Ratio
of number of people who went by cycle, bike and car is 34:55:43 respectively.
Morning Shift : Total number of people who went by cycle is 30 less than number of people who
w e n t b y b i ke w h i c h i s 3 0 l e s s t h a n n u m b e r of p e o p l e w h o w e n t b y c a r.
Afternoon Shift : Ratio of number of people who went by cycle and bike is 8:11 respectively and
number of people who went by cycle is 33.33% more than number of people who went by car.
Evening Shift : Ratio of number of people who went by cycle and car is 6:7 respectively and total
number of people who went by bike is 12 more than twice the number of people who went by
cycle.

दी गयी जानकारी को ान से पढ़ और िन िल खत ो ं के उ र द।
िकसी िवशेष िदन, 792 लोग तीन अलग-अलग पारीयो ं म साइिकल, बाइक और कार से िजम गए। साइिकल, बाइक
और कार से जाने वाले लोगो ं की सं ा का अनुपात मशः 34:55:43 है ।

PAGE h ps://t.me/studified h ps://instagram.com/aashisharorasocial?utm_medium=copy_link


Follow
156 Aashish Arora on: h ps://youtube.com/channel/UCYa4_JrOrf8R5Kz2uOtccXQ https://www.facebook.com/aashisharorasocial/
अ ास by Aashish Arora
(SBI/IBPS/RBI/LIC/All other banking and insurance Exams)

सुबह की पारी : साइिकल से जाने वाले लोगो ं की कुल सं ा बाइक से जाने वाले लोगो ं की सं ा से 30 कम है जो िक
कार से जाने वाले लोगो ं की सं ा से 30 कम है ।
दोपहर की पारी : साइिकल और बाइक से जाने वाले लोगो ं की सं ा का अनुपात मशः 8:11 है और जो लोग
साइिकल से गए, उनकी सं ा कार से जाने वालो ं की सं ा से 33.33% अिधक है ।
सं ा की पारी : साइिकल और कार से जाने वाले लोगो ं की सं ा का अनुपात मशः 6:7 है और बाइक से जाने वाले
लोगो ं की कुल सं ा साइिकल से जाने वाले लोगो ं की सं ा के दोगुने से 12 अिधक है ।

16. What is the total number of people who went by car ?


कार से जाने वालो ं लोगो ं की कुल सं ा िकतनी है ?
A. 234 B. 258 C. 264
D. 284 E. None of these/इनम से कोई नही ं

17. Find the ratio of number of people who went by bike in evening and in morning
respectively.
शाम और सुबह बाइक से जाने वालो ं लोगो ं की सं ा का मश: अनुपात ात कीिजए।
A. 5 : 6 B. 6:7 C. 6:5
D. 7 : 6 E. None of these/इनम से कोई नही ं

18. Number of people who went by cycle in morning is how much percent more or less than
the number of people who went by bike in afternoon.
सुबह साइिकल से जाने वाले लोगो ं की सं ा दोपहर म बाइक से जाने वाले लोगो ं की सं ा से िकतने ितशत
अिधक या कम है ।
A. 11.11% B. 9.09% C. 7.14%
D. 8.33% E. None of these/इनम से कोई नही ं

19. Find the average number of people who went in morning and evening shift.
सुबह और शाम की पारी म जाने वाले लोगो ं की औसत सं ा ात कीिजए।
A. 296 B. 256 C. 276
D. 286 E. None of these/इनम से कोई नही ं

20. Find the total number of people who went to gym in evening shift.
सं ा की पारी म िजम जाने वाले लोगो ं की कुल सं ा ात कीिजए।
A. 274 B. 262 C. 254
D. 242 E. None of these/इनम से कोई नही ं

Read the given data carefully and answer the following questions.
Directions:- Four trains are travelling from Brussels to Paris, Vienna, Berlin and Copenhagen. The
number of passengers in the train going to Paris is 250 more than the number of total seats in
the train to Copenhagen. 30% of the seats in the train to Copenhagen are vacant. The train to
Paris is fully occupied by passengers. The train to Vienna has 256 passengers while 20 % of the
seats are vacant. The ratio of the number of total seats in the train to Vienna to the numbers of

PAGE h ps://t.me/studified h ps://instagram.com/aashisharorasocial?utm_medium=copy_link


Follow
157 Aashish Arora on: h ps://youtube.com/channel/UCYa4_JrOrf8R5Kz2uOtccXQ https://www.facebook.com/aashisharorasocial/
अ ास by Aashish Arora
(SBI/IBPS/RBI/LIC/All other banking and insurance Exams)

passengers in the train to Berlin is 8:9 respectively. 10% of the seats in the train to Berlin are
vacant. The train to Berlin has 50 less seats than the train to Paris.

दी गयी जानकारी को ान से पढ़ और िन िल खत ो ं के उ र द।
चार टे न से से पे रस, िवयना, बिलन और कोपेनहे गन की या ा कर रही ह। पे रस जाने वाली टे न म याि यो ं की
सं ा कोपेनहे गन म टे न की कुल सीटो ं की सं ा से 250 अिधक है । कोपेनहे गन जाने वाली टे न की 30% सीट खाली
ह। पे रस जाने वाली टे न की सर सीट भरी ह । िवयना जाने वाली टे न म 256 या ी ह जबिक 20% सीट खाली ह।
िवयना जाने वाली टे न म कुल सीटो ं की सं ा और बिलन म टे न म याि यो ं की सं ा का अनुपात मशः 8:9 है ।
बिलन की टे न म 10% सीट खाली ह। बिलन की टे न म पे रस की टे न की तुलना म 50 कम सीट ह।

21. By what percentage approximately is the number of vacant seats in the train to Berlin more
or less than that in the train to Copenhagen ?
बिलन जाने वाली टे न म खाली सीटो ं की सं ा कोपेनहे गन जाने वाली टे न म खाली सीटो ं की सं ा से लगभग
िकतना ितशत कम या ादा है ?
A. 26% B. 41% C. 33 %
D. 30% E. None of these

22. What is the difference between the total number of seats in the trains to Paris and Vienna ?
पे रस और िवयना जाने वाली टे नो ं की कुल सीटो ं म ा अंतर है ?
A. 130 B. 100 C. 70
D. 250 E. None of these

23. What is the ratio of the number of passengers traveling in the train to Paris to the total
number of seats in the train to Berlin respectively?
पे रस जाने वाली टे न म या ा करने वाले याि यो ं की सं ा और बिलन जाने वाली टे न की कुल सीटो ं का मशः
अनुपात ा है ?
A. 17 : 15 B. 9:8 C. 23 : 20
D. 11 : 8 E. None of these

24. What is the average number of vacant seats per train ?


ित टे न खाली सीटो ं की औसत सं ा ा है ?
A. 47 B. 32 C. 60
D. 41 E. None of these

25. The number of vacant seats in the trains to Vienna and Berlin together is What percentage
of the total number of seats in the train to Copenhagen?
िवयना और बिलन जाने वाली गािड़यो ं म खाली सीटो ं की सं ा कोपेनहे गन म टे न की कुल सीटो ं का िकतना
ितशत है ?
A. 52 % B. 50 % C. 33.33 %
D. 75 % E. None of these/इनम से कोई नही ं

PAGE h ps://t.me/studified h ps://instagram.com/aashisharorasocial?utm_medium=copy_link


Follow
158 Aashish Arora on: h ps://youtube.com/channel/UCYa4_JrOrf8R5Kz2uOtccXQ https://www.facebook.com/aashisharorasocial/
अ ास by Aashish Arora
(SBI/IBPS/RBI/LIC/All other banking and insurance Exams)

Read the given data carefully and answer the following questions.
There are four different schools (A, B, C and D) with different number of students and teachers in
it. Ratio of number of students and teacher in school A is 13 : 4 respectively. Total number of
students and teachers in school D is 1068. Number of teachers in school C is 60% less than the
number of students in school A, while number of students in school C is 2 times more than
number of teachers in school A. Ratio of total number of students and teachers in school C to
that in school B is 13 : 14 respectively, and number of students in school B is 400 more than that
of teachers in the same school. Number of students in school D is 16 more than the number of
students in school B, while number of teachers in school D is 10 less than the number of teachers
in school A.

दी गयी जानकारी को ान से पढ़ और िन िल खत ो ं के उ र द।
चार अलग-अलग ू ल (A, B, C और D) ह, िजसम िविभ छा और िश क ह। ू ल A म छा ो ं और िश को ं की
सं ा का अनुपात मशः 13 : 4 है । ू ल D म कुल छा ो ं और िश को ं की सं ा 1068 है । ू ल C म िश को ं की
सं ा, ू ल A म छा ो ं की सं ा की तुलना म 60% कम है , जबिक ू ल C म छा ो ं की सं ा, ू ल A के िश को ं
की सं ा से 2 गुना अिधक है । ू ल C और ू ल B म छा ो ं और िश को ं की कुल सं ा का अनुपात मशः 13 :
14 है , और ू ल B म छा ो ं की सं ा उसी ू ल के िश को ं की सं ा से 400 अिधक है । ू ल D म छा ो ं की
सं ा ू ल B के छा ो ं की सं ा से 16 अिधक है , जबिक ू ल D म िश को ं की सं ा ू ल A के िश को ं की
सं ा से 10 कम है ।

26. Find the average number of students in school C and school D .


ू ल C और ू ल D म छा ो ं की औसत सं ा ात कीिजए।
A. 791 B. 809 C. 801
D. 799 E. None of these/इनम से कोई नही ं

27. Find the difference between number of students in school A and number of teachers in
school B.
ू ल A म छा ो ं की सं ा और ू ल B म िश को ं की सं ा के बीच का अंतर ात कर।
A. 443 B. 429 C. 397
D. 385 E. None of these/इनम से कोई नही ं

28. If in school B 202 students are absent on a particular day, and 40% of the rest are boys then
find the number of girls present on that day.
यिद ू ल B म 202 छा िकसी िवशेष िदन अनुप थत ह, और बाकी छा ो ं म से 40% लड़के ह, तो उस िदन
उप थत लड़िकयो ं की सं ा का पता लगाएं ।
A. 306 B. 360 C. 375
D. 284 E. None of these/इनम से कोई नही ं

29. If school A requires only 1 teacher on every 5 student, then how many teachers has to be
weeded out?

PAGE h ps://t.me/studified h ps://instagram.com/aashisharorasocial?utm_medium=copy_link


Follow
159 Aashish Arora on: h ps://youtube.com/channel/UCYa4_JrOrf8R5Kz2uOtccXQ https://www.facebook.com/aashisharorasocial/
अ ास by Aashish Arora
(SBI/IBPS/RBI/LIC/All other banking and insurance Exams)

यिद ू ल A को ेक 5 छा पर केवल 1 िश क की आव कता है , तो िकतने िश को ं को हटा दे ना


चािहए?
A. 84 B. 75 C. 91
D. 88 E. None of these/इनम से कोई नही ं

30. Find the average number of teachers in schools A, B and D.


ू लो ं A, B और D म िश को ं की औसत सं ा का पता लगाएं ।
A. 218 B. 304 C. 216
D. 228 E. None of these/इनम से कोई नही ं

Read the given data carefully and answer the following questions.
Different number of students joined three different clubs in a college. The ratio of number of
boys to girls who joined club A is 9 : 10 respectively. The number of boys who joined club B is
11.11% less than that of club A while the number of girls who joined club C is 55% more than that
of club A. Ratio of number of girls who joined club B and club C is 4 : 5 respectively. Number of
boys who joined club C is 20% more than the number of girls who joined club C. Total number of
members in club C is 341.

दी गयी जानकारी को ान से पढ़ और िन िल खत ो ं के उ र द।
एक कॉलेज म तीन अलग-अलग बो ं म िविभ छा शािमल ए। ब A म शािमल होने वाले लड़को ं और
लड़िकयो ं की सं ा का अनुपात मशः 9 : 10 है । ब B म शािमल होने वाले लड़को ं की सं ा, ब A म शािमल
होने वाले लड़को ं की सं ा की तुलना म 11.11% कम है , जबिक ब C म शािमल होने वाली लड़िकयो ं की सं ा
ब A म शािमल होने वाली लड़िकयो ं की सं ा की तुलना म 55% अिधक है । ब B और ब C म शािमल होने
वाली लड़िकयो ं की सं ा का अनुपात मशः 4 : 5 है । ब C म शािमल होने वाले लड़को ं की सं ा, बCम
शािमल होने वाली लड़िकयो ं की सं ा से 20% अिधक है । ब C म सद ो ं की कुल सं ा 341 है ।

31. Find the total number of members in club A.


ब A म सद ो ं की कुल सं ा ात कीिजए।
A. 170 B. 205 C. 190
D. 195 E. None of these/इनम से कोई नही ं

32. Number of boys who joined club B is how much percent more or less than the number of
girls who joined club A?
ब B म शािमल होने वाले लड़को ं की सं ा, ब A म शािमल होने वाली लड़िकयो ं की सं ा से िकतने
ितशत अिधक या कम है ?
A. 20% B. 40% C. 30%
D. 10% E. None of these/इनम से कोई नही ं

33. What is the ratio of number of boys to girls who joined club B?
ब B म शािमल होने लड़को ं और लड़िकयो ं की सं ा का अनुपात ा है ?
A. 20 : 31 B. 21 : 29 C. 23 : 30

PAGE h ps://t.me/studified h ps://instagram.com/aashisharorasocial?utm_medium=copy_link


Follow
160 Aashish Arora on: h ps://youtube.com/channel/UCYa4_JrOrf8R5Kz2uOtccXQ https://www.facebook.com/aashisharorasocial/
अ ास by Aashish Arora
(SBI/IBPS/RBI/LIC/All other banking and insurance Exams)

D. 19 : 28 E. None of these/इनम से कोई नही ं

34. What is the average number of girls who joined club A and club B?
ब A और ब B म शािमल होने वाली लड़िकयो ं की औसत सं ा ा है ?
A. 134 B. 112 C. 92
D. 100 E. None of these/इनम से कोई नही ं

35. What is the difference between number of member in club C and club B?
ब C और ब B म सद ो ं की सं ा म ा अंतर है ?
A. 133 B. 129 C. 137
D. 141 E. None of these/इनम से कोई नही ं

Read the given data carefully and answer the following questions.
There are four different hospitals (P, Q, R and S) with different number of male and female nurses
in it. The number of male nurses in hospital R is 100 more than the number of female nurses in
hospital P. The number of male nurses in hospital P is 30% more than that in hospital R. Total
number of nurses in hospital R is 2200. The ratio of number of female nurses in hospital R to the
number of male nurses in hospital S is 6 : 7 respectively. Total number of male nurses in all the
four hospitals together is 4500. The number of female nurses in hospital Q and S is 1100 and
1000 respectively. The number of male nurses in hospital R is 400 less than that in hospital S.

दी गयी जानकारी को ान से पढ़ और िन िल खत ो ं के उ र द।
दी गई जानकारी म चार अलग-अलग अ ताल (P, Q, R और S) ह, िजनम पु ष और मिहला नस की सं ा के बारे
म है । अ ताल R म पु ष नस की सं ा, अ ताल P म मिहला नस की सं ा से 100 अिधक है । अ ताल P म
पु ष नस की सं ा, अ ताल R म पु ष नस की सं ा से 30% अिधक है । अ ताल R म नस की कुल सं ा
2200 है । अ ताल R म मिहला नस की सं ा और अ ताल S म पु ष नस की सं ा का अनुपात मशः 6 : 7
है । एक साथ सभी चारो ं अ तालो ं म कुल पु ष नस की सं ा 4500 है । अ ताल Q और अ ताल S म मिहला
नस की सं ा मशः 1100 और 1000 है । अ ताल R म पु ष नस की सं ाअ ताल S म पु ष नस की सं ा
से 400 कम है ।

36. Find the average number of female nurses in each of the given hospitals.
िदए गए ेक अ ताल म मिहला नस की औसत सं ा ात कीिजए।
A. 1010 B. 990 C. 1080
D. 1050 E. None of these/इनम से कोई नही ं

37. The number of male nurses in hospital S is how much percent more of less than the number
of female nurses in hospital P ?
अ ताल S म पु ष नस की सं ा, अ ताल P म मिहला नस की सं ा से िकतने ितशत अिधक या कम
है ?
A. 55.55% B. 57% C. 39%

PAGE h ps://t.me/studified h ps://instagram.com/aashisharorasocial?utm_medium=copy_link


Follow
161 Aashish Arora on: h ps://youtube.com/channel/UCYa4_JrOrf8R5Kz2uOtccXQ https://www.facebook.com/aashisharorasocial/
अ ास by Aashish Arora
(SBI/IBPS/RBI/LIC/All other banking and insurance Exams)

D. 66% E. None of these/इनम से कोई नही ं


38. Find the ratio of the number of male nurses in hospital R to the number of female nurses in
hospital S.
अ ताल R म पु ष नस की सं ा और अ ताल S म मिहला नस की सं ा का अनुपात पता लगाएं ।
A. 3 : 4 B. 1:1 C. 2:3
D. 2 : 5 E. None of these/इनम से कोई नही ं

39. Find the difference between the number of male nurses in hospital Q and total number of
nurses in hospital S.
अ ताल Q म पु ष नस की सं ा और अ ताल S म नस की कुल सं ा के बीच का अंतर ात कीिजए।
A. 1800 B. 1750 C. 1600
D. 1850 E. None of these/इनम से कोई नही ं

40. Which hospital has highest number of nurses?


िकस अ ताल म नस की सं ा सबसे अिधक है ?
A. Hospital P B. Hospital Q C. Hospital R
D. Hospital S E. None of these/इनम से कोई नही ं

Read the given data carefully and answer the following questions.
Different number of students attended a seminar on different days of the week. Number of girls
attending on Monday is 120 more than the number of boys attending on that day and the total
numbers of students attending is 920. Number of girls on Monday is 65% of the total number on
Tuesday. 42% of the students on Tuesday are boys. Ratio of girls on Tuesday to that on
Wednesday is 4:5 and number of male to female students on Wednesday is 15:10 respectively.
Total number of students on Thursday is 20% less than the same on Wednesday. Number of boys
on Thursday is 35% more than the same on Monday.

दी गयी जानकारी को ान से पढ़ और िन िल खत ो ं के उ र द।
स ाह के िविभ िदनो ं म एक सेिमनार म िविभ छा ो ं ने भाग िलया। सोमवार को भाग लेने वाली लड़िकयो ं की
सं ा उस िदन भाग लेने वाले लड़को ं की सं ा से 120 अिधक है और भाग लेने वाले छा ो ं की कुल सं ा 920 है ।
सोमवार को भाग लेने वाली लड़िकयो ं की सं ा मंगलवार को कुल सं ा का 65% है । मंगलवार को भाग लेने वाले
42% छा लड़के ह। मंगलवार और बुधवार को भाग लेने वाली लड़िकयो ं का अनुपात को 4 : 6 है । बुधवार को भाग
लेने वाले लड़को ं और लड़िकयो ं का अनुपात मशः 15 : 10 है ।। गु वार को भाग लेने वाले कुल छा ो ं की सं ा
बुधवार की तुलना म 20% कम है । सोमवार के मुकाबले गु वार को भाग लेने वाले लड़को ं की सं ा 35% अिधक
है ।

41. What is the ratio of boy to girl students on Thursday ?


गु वार को भाग लेने वाले लड़को ं और लड़िकयो ं का अनुपात ा है ?
A. 25:28 B. 9:11 C. 27:31
D. 11:13 E. None of these/इनम से कोई नही ं

PAGE h ps://t.me/studified h ps://instagram.com/aashisharorasocial?utm_medium=copy_link


Follow
162 Aashish Arora on: h ps://youtube.com/channel/UCYa4_JrOrf8R5Kz2uOtccXQ https://www.facebook.com/aashisharorasocial/
अ ास by Aashish Arora
(SBI/IBPS/RBI/LIC/All other banking and insurance Exams)

42. What is the average number of students on Tuesday and Wednesday together ?
मंगलवार और बुधवार को एक साथ भाग लेने वाले छा ो ं की औसत सं ा ा है ?
A. 1200 B. 1125 C. 1250
D. 1075 E. None of these/इनम से कोई नही ं

43. What is the difference between the number of boy students on Tuesday and that on
Wednesday ?
मंगलवार और बुधवार को भाग लेने वाले लड़को ं की सं ा म ा अंतर है ?
A. 534 B. 576 C. 542
D. 552 E. None of these/इनम से कोई नही ं

44. Number of girl students on Monday is how much percent more/less than the number of
boys on the same day.
सोमवार को भाग लेने वाली लड़िकयो ं की सं ा उसी िदन भाग लेने वाले लड़को ं की सं ा से िकतने ितशत
अिधक / कम है ।
A. 35% B. 25% C. 30%
D. 40% E. None of these/इनम से कोई नही ं

45. What is the total number of girl students on Monday and Thursday together ?
सोमवार और गु वार को एक साथ भाग लेने वाली लड़िकयो ं की कुल सं ा िकतनी है ?
A. 1140 B. 1000 C. 1120
D. 940 E. None of these/इनम से कोई नही ं

Read the given data carefully and answer the following questions.
The given data is about number of parks and museums built in three different years. The
numbers of museums built in 2018 is 30% less than the number of parks built in 2017. The ratio
of number of parks built in 2017 and 2018 is 5:7 respectively. Total number of parks built in these
three years is 700. Sum of parks and museums built in 2019 is 50 more than that of the same on
2018. The number of museums built in 2017 is 150. The ratio of number of museums built in
2017 and 2019 is 3:5 respectively.

दी गयी जानकारी को ान से पढ़ और िन िल खत ो ं के उ र द।
दी गयी जानकारी तीन अलग-अलग वष म िनिमत पाक और सं हालयो ं की सं ा है । 2017 म िनिमत पाक की
सं ा की तुलना म 2018 म िनिमत सं हालयो ं की सं ा 30% कम है । 2017 और 2018 म िनिमत पाक की सं ा
का अनुपात मशः 5 : 7 है । इन तीन वष म िनिमत पाक की कुल सं ा 700 है । 2019 म बनाए गए पाक और
सं हालयो ं का योग 2019 म बनाए गए पाक और सं हालयो ं के योग की तुलना म 50 अिधक है । 2017 म िनिमत
सं हालयो ं की सं ा 150 है । 2017 और 2019 म िनिमत सं हालयो ं की सं ा का अनुपात मशः 3 : 5 है ।

46. Find the average number of museums built in each of the given three years?
िदए गए तीन वष म िनिमत सं हालयो ं की औसत सं ा ात कीिजए?
A. 160 B. 200

PAGE h ps://t.me/studified h ps://instagram.com/aashisharorasocial?utm_medium=copy_link


Follow
163 Aashish Arora on: h ps://youtube.com/channel/UCYa4_JrOrf8R5Kz2uOtccXQ https://www.facebook.com/aashisharorasocial/
अ ास by Aashish Arora
(SBI/IBPS/RBI/LIC/All other banking and insurance Exams)

C. 180 D. 140
E. 175

47. The number of parks built in 2019 is how much percent more/less than the number of
museums built in 2019 ?
2019 म िनिमत पाक की सं ा 2019 म िनिमत सं हालयो ं की सं ा से िकतने ितशत अिधक / कम है ?
A. 12% B. 14%
C. 10% D. 11%
E. 16%

48. Find the ratio of number of museums built in 2018 to sum of number of museums and park
built in 2019.
2018 म िनिमत सं हालयो ं की सं ा और 2019 म िनिमत सं हालयो ं और पाक की सं ा का अनुपात ात
कीिजए।
A. 14:47 B. 16:57
C. 13:41 D. 11:35
E. None of these/इनम से कोई नही ं

49. Find the difference between sum of number of parks and museums built in 2017 and the
number of parks built in 2019 .
2017 म िनिमत पाक और सं हालयो ं की सं ा और 2019 म िनिमत पाक की सं ा के बीच का अंतर ात
कीिजए।
A. 170 B. 100
C. 120 D. 130
E. 80

50. Out of total number of parks built in 2017, 40% were for children. Out of the total number
of museums built in the same year 30% were heritage museums. Find the sum of number of
parks built for other than children and number of museums built for other purpose than
heritage in 2017.
2017 म िनिमत कुल पाक म से, 40% ब ो ं के िलए थे। उसी वष म िनिमत सं हालयो ं की कुल सं ा म से
30% िवरासत सं हालय थे। 2017 म ब ो ं के िलए ना बनाए गए पाक की सं ा और िवरासत के अलावा
अ उ े ो ं के िलए बनाए गए सं हालयो ं की सं ा का योग ात कीिजए।
A. 215 B. 205
C. 230 D. 225
E. None of these/इनम से कोई नही ं

Read the given data carefully and answer the following questions.
Government of Haryana constructed a total of 4400 toilets in four cities. The ratio of number of
male and female toilets constructed in city A is 5 : 6 respectively. Female toilets in city B are 110
more than the male toilets in the same city. Number of female toilets in city C are 12.5 % more

PAGE h ps://t.me/studified h ps://instagram.com/aashisharorasocial?utm_medium=copy_link


Follow
164 Aashish Arora on: h ps://youtube.com/channel/UCYa4_JrOrf8R5Kz2uOtccXQ https://www.facebook.com/aashisharorasocial/
अ ास by Aashish Arora
(SBI/IBPS/RBI/LIC/All other banking and insurance Exams)

than the number of male toilets in city C, which is 14.28 % more than the number of male toilets
in city B. Number of female toilets in city D is 0.8 times the number of male toilets in city B, and
the number of female toilets in city D is 122 more than the number of male toilets in city D.
Number of male toilets in city B is 595.

दी गयी जानकारी को ान से पढ़ और िन िल खत ो ं के उ र द।
ह रयाणा सरकार ने चार शहरो ं म कुल 4400 शौचालयो ं का िनमाण िकया। शहर A म िनिमत पु ष शौचालयो ं और
मिहला शौचालयो ं की सं ा का अनुपात मशः 5 : 6 है । शहर B म मिहला शौचालय, उसी शहर के पु ष शौचालयो ं
से 110 अिधक ह। शहर C म मिहला शौचालयो ं की सं ा, शहर C म पु ष शौचालयो ं की सं ा से 12.5% अिधक
है , शहर C म पु ष शौचालयो ं की सं ा शहर B म पु ष शौचालयो ं की सं ा से 14.28% अिधक है । शहर D म
मिहला शौचालयो ं की सं ा, शहर B म पु ष शौचालयो ं की सं ा का 0.8 गुना है , और शहर D म मिहला शौचालयो ं
की सं ा शहर D म पु ष शौचालयो ं की सं ा से 122 अिधक है । शहर B म पु ष शौचालयो ं की सं ा 595 है ।

51. What is the number of male toilets in city A ?


शहर A म पु ष शौचालयो ं की सं ा िकतनी है ?
A. 595 B. 375
C. 680 D. 450
E. None of these

52. What is the total number of toilets in city C ?


शहर C म शौचालयो ं की कुल सं ा िकतनी है ?
A. 1595 B. 1375
C. 1445 D. 1450
E. None of these

53. What is the difference between number of female toilets in city D and that of city B ?
शहर D और शहर B म मिहला शौचालयो ं की सं ा म ा अंतर है ?
A. 199 B. 229
C. 227 D. 197
E. None of these

54. Number of female toilets in city B is approximately what percent of the total number of
toilets in all four cities together ?
शहर B म मिहला शौचालयो ं की सं ा सभी चार शहरो ं म कुल शौचालयो ं की सं ा का लगभग िकतना
ितशत है ?
A. 16% B. 20%
C. 14% D. 18%
E. None of these

55. What is the number of male toilets in all the four cities together ?
सभी चार शहरो ं म एक साथ पु ष शौचालयो ं की सं ा िकतनी है ?

PAGE h ps://t.me/studified h ps://instagram.com/aashisharorasocial?utm_medium=copy_link


Follow
165 Aashish Arora on: h ps://youtube.com/channel/UCYa4_JrOrf8R5Kz2uOtccXQ https://www.facebook.com/aashisharorasocial/
अ ास by Aashish Arora
(SBI/IBPS/RBI/LIC/All other banking and insurance Exams)

A. 1994 B. 2004
C. 1904 D. 2094
E. None of these

The Paragraph given below gives the information about the number of male and female
passengers travelling in two different trains A and B and the fare of them.
In train A, the number of male passengers is 50 more than that of the female passengers and the
TTE gives 20% discount on original price of ticket to each female passenger but charges full
price to each male passenger. The average ticket price of each passenger was Rs.46 and the
sum of total money collected from all the passengers together was Rs. 11500.
In Train B, the ratio of male passengers to female passenger was in the ratio 2:1 and the TTE
gives 30% discount on original price of ticket to each female passenger but charges full price to
each male passenger. The difference between number of male passengers and female
passengers was 100 and the average of total money collected by all passengers was Rs. 63 .

नीचे िदए गए अनु े द म दो अलग-अलग टे नो ं ए और बी म या ा करने वाले पु ष और मिहला याि यो ं की सं ा


और उनके िकराए के बारे म जानकारी दी गई है ।
टे न ए म, पु ष याि यो ं की सं ा मिहला याि यो ं की तुलना म 50 अिधक है और टीटीई ेक मिहला या ी को
िटकट की मूल कीमत पर 20% की छूट दे ता है लेिकन ेक पु ष या ी से पूरी कीमत वसूलता है । ेक या ी का
औसत िटकट मू 46 पये था और सभी याि यो ं से िलया गया कुल धन का योग 11500 था |
टे न बी म, पु ष और मिहला याि यो ं का अनुपात मश: 2:1 है और टीटीई ेक मिहला या ी को िटकट की मूल
कीमत पर 30% की छूट दे ता है , लेिकन ेक पु ष या ी से पूरी कीमत वसूलता है । पु ष याि यो ं और मिहला
याि यो ं की सं ा का अंतर 100 था और सभी याि यो ं ारा एक िकए गए कुल धन का औसत पये 63 था |

56. In Trains A & B together, total number of Male passengers was how many more than that of
female passengers ?
टे न ए और बी म कुल िमलाकर पु ष याि यो ं की सं ा मिहला याि यो ं से िकतनी अिधक थी?
A. 100 B. 150
C. 200 D. 175
E. None of these

57. In Train A, total money collected by TTE from male passengers was how much more than
that collected from female passengers.
टे न ए म, पु ष याि यो ं से टीटीई ारा एक िकया गया पैसा मिहला याि यो ं से एकि त पये से िकतना
अिधक था।
A. Rs. 3600 B. Rs. 4000
C. Rs. 3000 D. Rs. 3500
E. None of these

58. What is the difference in the total money collected from both the trains.
दोनो ं टे नो ं से एकि त कुल धन म ा अंतर है ।
A. Rs. 7400 B. Rs. 8000
C. Rs. 6500 D. Rs. 8200
E. None of these

PAGE h ps://t.me/studified h ps://instagram.com/aashisharorasocial?utm_medium=copy_link


Follow
166 Aashish Arora on: h ps://youtube.com/channel/UCYa4_JrOrf8R5Kz2uOtccXQ https://www.facebook.com/aashisharorasocial/
अ ास by Aashish Arora
(SBI/IBPS/RBI/LIC/All other banking and insurance Exams)

59. The ticket price of each female passengers in train B was how much more than that of
Train A.
टे न B म ेक मिहला याि यो ं का िटकट मू टे न A की तुलना म िकतना अिधक था |
A. Rs. 11 B. Rs. 09
C. Rs. 15 D. Rs. 07
E. None of these

60. What is the approximate difference in the percentage of money collected from males and
females from both the trains taken together.
दोनो ं टे नो ं से पु षो ं और मिहलाओं से एक िकए गए धन के ितशत म अनुमािनत अंतर ा है ।
A. 41% B. 36%
C. 34% D. 47%
E. None of these

Read the given data carefully and answer the following questions.
A sports shop owner sold two types of items that is balls and bats on different days of a week.
Balls sold on Monday is [A] which is 50 less than number of bats sold on the same day. [B] balls
are sold on Tuesday which is 25% more than that on Monday and [C] bats are sold on Tuesday
such that total items sold on Tuesday is 492 which is 20% more than that on Monday. Bats sold
on Wednesday is [D] which is 50% more than balls sold on the same day and the ratio of total
items sold on Monday and Wednesday is 2 : 3 respectively. Balls sold on Wednesday is [E]
which is 33.33% less than the same sold on Thursday. Bats sold on Thursday is [F] such that
total items sold on Monday is 18% less than same on Thursday.

दी गयी जानकारी को ान से पढ़ और िन िल खत ो ं के उ र द।
एक ोट् स शॉप के मािलक ने स ाह के िविभ िदनो ं म दो तरह के उ ाद बेचे जो गद और ब े ह। सोमवार को
बेचे जाने वाली गद [A] ह जो िक उसी िदन म िबकने वाले ब ो ं की सं ा से 50 कम ह। [B] गद मंगलवार को बेची
जाती ह जो सोमवार को बेची जाने वाली गदो ं से 25% अिधक ह और [C] ब े मंगलवार को ऐसे बेचे जाते ह िक
मंगलवार को बेची जाने वाली कुल व ुएं 492 ह जो िक सोमवार की तुलना म 20% अिधक है । बुधवार को िबकने
वाले ब ो ं की सं ा [D] है जो उसी िदन िबकने वाली गदो ं की सं ा से 50% अिधक है और सोमवार और बुधवार
को बेची गई कुल व ुओं का अनुपात मश: 2 : 3 है । बुधवार को बेची गई गद [E] है जो गु वार को बेची गई गदो ं से
33.33% कम है । गु वार को बेचे जाने वाले ब े [F] है और सोमवार को बेची गई कुल व ुए गु वार की तुलना म
18% कम है ।
61. Find the ratio [E] : [A].
[E] : [A] का अनुपात ात कर।
A. 41 : 30 B. 37 : 39
C. 39 : 37 D. 30 : 41
E. None of these

62. Find the value of [B].


[B] का मान ात कीिजए।
A. 252 B. 225
C. 255 D. 552
E. None of these

63. Find the value of [C].

PAGE h ps://t.me/studified h ps://instagram.com/aashisharorasocial?utm_medium=copy_link


Follow
167 Aashish Arora on: h ps://youtube.com/channel/UCYa4_JrOrf8R5Kz2uOtccXQ https://www.facebook.com/aashisharorasocial/
अ ास by Aashish Arora
(SBI/IBPS/RBI/LIC/All other banking and insurance Exams)

[C] का मान ात कीिजए।


A. 139 B. 227
C. 267 D. 157
E. None of these

64. Find the value of [F].


[F] का मान ात कीिजए।
A. 161 B. 171
C. 141 D. 131
E. None of these

65. [D] is approximately how much percent more or less than the number of bats sold on
Monday?
[D] लगभग िकतने ितशत सोमवार को बेचे गए ब ो ं की सं ा से अिधक या कम है ?
A. 62% B. 64%
C. 60% D. 68%
E. None of these

Read the given data carefully and answer the following questions.
The given data is about number of questions of three subjects (History, Geography and
Zoology) created by a person in two different years namely 2020 and 2021. The sum of number
of questions of History and Geography created in 2020 is 16000. The number of questions of
Zoology created in 2021 is 20% less than that of Geography in 2020. Total number of questions
of Zoology created in two years is 12500. The ratio of number of questions of Zoology created in
2020 to that of Geography in 2021 is 3 : 7 respectively. The number of questions of History ,
created in 2021 is 5000 less than that of Geography in the same year. The total number of
questions of all three subjects created in 2021 is 24000.

दी गयी जानकारी को ान से पढ़ और िन िल खत ो ं के उ र द।
िदया गया डे टा तीन िवषयो ं (इितहास, भूगोल और जूलॉजी) के ो ं के बारे म है जो एक ारा दो अलग-अलग
वष 2020 और 2021 म बनाए गए ह। 2020 म बनाए गए इितहास और भूगोल के ो ं की सं ा का योग 16000
है । 2021 म बनाए गए जूलॉजी के 2020 म बनाए गए भूगोल के ो ं से 20% कम ह। दो वष म बनाए गए
जूलॉजी के कुल ो ं की सं ा 12500 है । 2020 म बनाए गए जूलॉजी के ो ं की सं ा 2021 म बनाए गए भूगोल
के ो ं की सं ा का अनुपात मशः 3 : 7 है । 2021 म बनाए गए इितहास के ो ं की सं ा उसी वष म बनाए गए
भूगोल के ो ं की तुलना म 5000 कम है । 2021 म बनाए गए तीनो ं िवषयो ं के कुल ो ं की सं ा 24000 है ।

66. Find the difference between total number of questions of all three subjects in 2021 and
number of questions of History in given 2 years?
2021 म तीनो ं िवषयो ं के कुल ो ं और िदए गए 2 वष म इितहास के ो ं की सं ा के बीच अंतर ात
कीिजए?
A. 12500 B. 11500
C. 13000 D. 12000
E. 10750

67. The number of History created in 2020 is how much percent more/less than the number of
questions of Zoology in 2021 ?

PAGE h ps://t.me/studified h ps://instagram.com/aashisharorasocial?utm_medium=copy_link


Follow
168 Aashish Arora on: h ps://youtube.com/channel/UCYa4_JrOrf8R5Kz2uOtccXQ https://www.facebook.com/aashisharorasocial/
अ ास by Aashish Arora
(SBI/IBPS/RBI/LIC/All other banking and insurance Exams)

2020 म बनाए गए इितहास के ो ं की सं ा 2021 म बनाए गए जूलॉजी के ो ं की सं ा से िकतना


ितशत अिधक / कम है ?
A. 25% B. 33.33%
C. 14.69% D. 22.22%
E. 27.25%

68. Find the average number of questions of Geography created in given 2 years.
िदए गए 2 वष म भूगोल के ो ं की औसत सं ा ात कीिजए।
A. 11200 B. 13000
C. 6250 D. 12000
E. 10250

69. Out of total number of questions of History and Geography, created in 2020, 75% were in
English language and rest were in Hindi language. If the number of questions of History
created in Hindi language in 2020 were 1850, then find the number of questions of
Geography created in Hindi language in 2020.
2020 म बनाए गए इितहास और भूगोल के कुल ो ं म से, 75% अं ेजी भाषा म और बाकी िहं दी भाषा म थे।
यिद 2020 म िहं दी भाषा म बनाए गए इितहास के ो ं की सं ा 1850 थी, तो 2020 म िहं दी भाषा म बनाए
गए भूगोल के ो ं की सं ा ात कीिजए।
A. 4000 B. 3250
C. 2250 D. 2150
E. 2725

70. Find the ratio of number of questions of Geography and History created in 2021.
2021 म बनाए गए भूगोल और इितहास के ो ं की सं ा का अनुपात ात कीिजए।
A. 11 : 17 B. 37 : 13
C. 11 : 21 D. 21 : 11
E. None of these/इनम से कोई नही ं

Read the given data carefully and answer the following questions.
The following information gives the data regarding the number of English, Mathematics and
Physics teachers in two different states.

State X :- Sum of the number of English, Mathematics and Physics teachers together is 940.
220 male mathematics teachers are there and respective ratio of the number of male and
female English teachers is 9 : 5 respectively. 100 female English teachers are there and male
English teachers are 90 more than female mathematics teachers. No of male Physics teachers
is 50 less than the number of female Physics teachers.

State Y :- Respective ratio of male English teachers, male Mathematics teachers and male
Physics teachers is 7 : 5 : 6 respectively. 180 female physics teachers are there and 380 English
teachers are there. Number of physics teachers is 20 less than the number of English teachers
and 10 more than the number of mathematics teachers.

दी गयी जानकारी को ान से पढ़ और िन िल खत ो ं के उ र द।
िन िल खत जानकारी दो अलग-अलग रा ो ं म अं ेजी, गिणत और भौितकी िश को ं की सं ा के बारे म डे टा दे ती

PAGE h ps://t.me/studified h ps://instagram.com/aashisharorasocial?utm_medium=copy_link


Follow
169 Aashish Arora on: h ps://youtube.com/channel/UCYa4_JrOrf8R5Kz2uOtccXQ https://www.facebook.com/aashisharorasocial/
अ ास by Aashish Arora
(SBI/IBPS/RBI/LIC/All other banking and insurance Exams)

है ।

रा X : - अं ेजी, गिणत और भौितकी के िश को ं की सं ा का योग एक साथ 940 है । 220 पु ष गिणत के


िश क ह और अं ेजी के पु ष और मिहला िश को ं की सं ा का अनुपात 9: 5 है । 100 मिहला अं ेजी िश क ह
और पु ष अं ेजी िश क मिहला गिणत िश को ं की तुलना म 90 अिधक ह। पु ष भौितकी िश को ं की सं ा
मिहला भौितकी िश को ं की सं ा से 50 कम है ।

रा Y: - पु ष अं ेजी िश को,ं पु ष गिणत िश को ं और पु ष भौितकी िश को ं का मशः अनुपात 7: 5: 6 है ।


180 मिहला भौितकी िश क ह और 380 अं ेजी िश क ह। भौितकी िश को ं की सं ा अं ेजी िश को ं की सं ा
से 20 कम और गिणत के िश को ं की सं ा से 10 अिधक है ।

71. What is the respective ratio of the number of English teachers in state 'X' to the number of
physics teachers in state 'Y'?
रा 'X' 'म अं ेजी िश को ं की सं ा और रा 'Y' 'म भौितकी िश को ं की सं ा का संबंिधत अनुपात ा
है ?
A. 5:7 B. 7:9
C. 13 : 17 D. 17 : 13
E. 29 : 36

72. Number of female mathematics teachers in State 'X' is what percent more/less than the
number of female physics teachers in state 'Y'?
रा 'X' म मिहला गिणत िश को ं की सं ा रा 'Y' म मिहला भौितकी िश को ं की सं ा से िकतने
ितशत अिधक / कम है ?
A. 50% B. 41.5%
C. 45% D. 33.33%
E. 25%

73. What is the difference between the number of male English teachers from both states
taken together and number of mathematics teachers in State 'Y'?
दोनो ं रा ो ं के पु ष अं ेजी िश को ं की सं ा और रा 'Y' म गिणत के िश को ं की सं ा म ा अंतर है ?
A. 30 B. 45
C. 40 D. 50
E. 60

74. What is the average of number of male Physics teachers in State 'Y' and number of male
mathematics teachers in State 'X' taken together?
रा 'Y' म पु ष भौितकी िश को ं की सं ा और रा 'X' म पु ष गिणत िश को ं की सं ा का औसत ा
है ?
A. 200 B. 380
C. 440 D. 400
E. 280

75. If the average number of female English teachers in State 'Y' and State 'Z' is 150, then
what is the difference between number of female English teachers in State 'X' and State
'Z'?

PAGE h ps://t.me/studified h ps://instagram.com/aashisharorasocial?utm_medium=copy_link


Follow
170 Aashish Arora on: h ps://youtube.com/channel/UCYa4_JrOrf8R5Kz2uOtccXQ https://www.facebook.com/aashisharorasocial/
अ ास by Aashish Arora
(SBI/IBPS/RBI/LIC/All other banking and insurance Exams)

यिद रा 'Y' और रा 'Z' म मिहला अं ेजी िश को ं की औसत सं ा 150 है , तो रा 'X' और रा 'Z' म


मिहला अं ेजी िश को ं की सं ा म ा अंतर है ?
A. 20 B. 30
C. 40 D. 35
E. None of these/इनम से कोई नहीं

Read the given data carefully and answer the following questions.
Directions:- The given data is about number of mangoes and oranges sold by a vendor on three
different days of the week. The number of mangoes sold on Monday is 540 which is 25% more
than the number of oranges sold on Tuesday. The ratio of number of oranges sold on Tuesday to
the number of oranges sold on Wednesday is 8 : 7 respectively. The total number of
fruits(oranges + mangoes ) sold on Wednesday is 828. The total number of mangoes sold in all
three days is 1500. The number of oranges sold on Monday is 110 less than the number of
mangoes sold on Tuesday.

दी गयी जानकारी को ान से पढ़ और िन िल खत ो ं के उ र द।
िदया गया डे टा स ाह के तीन अलग-अलग िदनो ं म एक िव े ता ारा बेचे गए आम और संतरे की सं ा के बारे म
है । सोमवार को िबकने वाले आमो ं की सं ा 540 है जो मंगलवार को िबकने वाले संतरे की तुलना म 25% अिधक
है । मंगलवार को बेचे गए संतरे की सं ा का अनुपात बुधवार को बेचे गए संतरे की सं ा के मुकाबले मश 8: 7
है । बुधवार को िबकने वाले फलो ं (संतरे + आम) की कुल सं ा 828 है । तीन िदनो ं म िबकने वाले आमो ं की कुल
सं ा 1500 है । सोमवार को िबकने वाले संतरे की सं ा मंगलवार को िबकने वाले आमो ं की सं ा से 110 कम है ।

76. The number of oranges sold on Wednesday is how much percent more/less than the
number of mangoes sold on Monday?
बुधवार को िबकने वाले संतरे की सं ा सोमवार को बेचे गए आमो ं की सं ा से िकतने ितशत अिधक / कम
है ?
A. 26% B. 41%
C. 33 % D. 30%
E. None of these

77. What is approximate average number of oranges sold oranges sold on all three days ?
सभी तीन िदनो ं म बेची गई संतरे की अनुमािनत औसत सं ा ा है ?
A. 400 B. 403
C. 451 D. 389
E. None of these

78. Find the ratio of number of mangoes sold on Monday to number of oranges sold on
Tuesday and Wednesday together respectively?
सोमवार को िबकने वाले आमो ं की सं ा और मंगलवार और बुधवार को एक साथ बेची गई संतरे सं ा का
मशः अनुपात पता लगाएं ?
A. 2 : 3 B. 4:5
C. 17 : 23 D. 7 : 12
E. None of these

79. The number of mangoes sold on Thursday is 30% more than the number sold mangoes

PAGE h ps://t.me/studified h ps://instagram.com/aashisharorasocial?utm_medium=copy_link


Follow
171 Aashish Arora on: h ps://youtube.com/channel/UCYa4_JrOrf8R5Kz2uOtccXQ https://www.facebook.com/aashisharorasocial/
अ ास by Aashish Arora
(SBI/IBPS/RBI/LIC/All other banking and insurance Exams)

sold on Wednesday and the total number of fruits sold on Thursday is 668 more than the
number of oranges sold on Tuesday. Find the number of oranges sold on Thursday ?
गु वार को िबकने वाले आमो ं की सं ा बुधवार को िबकने वाले आमो ं की सं ा से 30% अिधक है और
गु वार को िबकने वाले फलो ं की कुल सं ा मंगलवार को िबकने वाले संतरे की सं ा से 668 अिधक है ।
गु वार को बेचे गए संतरे की सं ा ात कीिजए?
A. 515 B. 735
C. 655 D. 615
E. None of these

80. In the next week on Monday the vendor sold 40 % less number of fruits than he sold on
previous Monday and 75% of the number of he sold that day were mangoes, then find the
number of oranges he sold on Monday in the next week.
यिद अगले स ाह म सोमवार को िव े ता ने िपछले सोमवार को बेचे गए फलो ं की तुलना म 40% कम फल
बेचे और उस िदन बेचे गए 75% फल आम थे, िफर अगले स ाह म सोमवार को उनके ारा बेचे गए संतरे की
सं ा ात कर।
A. 423 B. 141
C. 256 D. 319
E. None of these/इनम से कोई नही ं

Read the given data carefully and answer the following questions.
Given data is about AC and Coolers sold in month of February by four different shops. The ratio
of number of AC sold by shop P and number of Coolers sold by shop Q is 7 : 12 respectively. The
number of AC sold by shop S is 30 less than the number of AC sold by shop R. The number of
Coolers sold by shop S is 58.33% more than the number of Coolers sold by shop Q. The ratio of
number of AC sold by shop Q to the number of AC sold by shop R is 10 : 11 respectively. Total
number of AC and Coolers together sold by shop Q and shop S is 144 and 169 respectively.
Total number of AC and Coolers together sold by shop P is 31.25% less than that by shop Q. The
ratio of number of Coolers sold by shop P to the number of coolers sold by shop R is 2 : 3
respectively.

दी गयी जानकारी को ान से पढ़ और िन िल खत ो ं के उ र द।
िदए गए आं कड़ो ं के अनुसार फरवरी के महीने म चार अलग-अलग दु कानो ं ारा बेचे गए AC और कूलर के बारे म
ह। दु कान P ारा बेचे गए AC और दू कान Q कूलरो ं ारा बचे गए कूलर का अनुपात मशः 7 : 12 है । दु कान S
ारा बेचे जाने वाले AC की सं ा, दु कान R ारा बेचे जाने वाले AC की सं ा से 30 कम है । दु कान S ारा बेचे गए
कूलरो ं की सं ा, दु कान Q ारा बेचे गए कूलरो ं की सं ा से 58.33% अिधक है । दु कान Q ारा बेचे गए AC की
सं ा और दु कान R ारा बेचे गए AC की सं ा का अनुपात मशः 10 : 11 है । दु कान Q और दु कान S ारा बेचे
गए एक साथ AC और कूलर की कुल सं ा मशः 144 और 169 है । दु कान P ारा बेचे गए एक साथ AC और
कूलर की कुल सं ा, दु कान Q के मुक़ाबले 31.25% कम है । दु कान P और दु कान R ारा बेचे गए कूलर की
सं ा का अनुपात मशः 2 : 3 है ।

81. Find the difference between total number of AC and total number Coolers sold by all fours
shops together.
सभी चार दु कानो ं ारा एक साथ बेचे गए कुल AC और कुल कूलरो ं की कुल सं ा के बीच का अंतर ात कर।

PAGE h ps://t.me/studified h ps://instagram.com/aashisharorasocial?utm_medium=copy_link


Follow
172 Aashish Arora on: h ps://youtube.com/channel/UCYa4_JrOrf8R5Kz2uOtccXQ https://www.facebook.com/aashisharorasocial/
अ ास by Aashish Arora
(SBI/IBPS/RBI/LIC/All other banking and insurance Exams)

A. 121 B. 131
C. 127 D. 137
E. None of these/इनम से कोई नही ं

82. If the number of AC and number of Coolers sold by shop T is 15% more and 28.56% less
than that by shop Q respectively, then find the total number of AC and Coolers together
sold by shop T.
यिद दु कान T ारा बेचे गए AC और कूलर की सं ा दु कान Q से मशः 15% अिधक और 28.56% कम
है , तो दु कान T ारा एक साथ बेचे गए AC और कूलर की सं ा ात करे ।
A. 125 B. 126
C. 129 D. 121
E. None of these/इनम से कोई नही ं

83. If in the month of March, shop R sold 33.33% more AC and Coolers together, then find the
number of AC and Coolers together sold by shop R in March.
यिद माच के महीने म, दु कान R ने एक साथ एसी और कूलर 33.33% अिधक बेचे ह, तो माच म दु कान R
ारा बेचे गए AC और कूलर की सं ा का पता लगाएं ।
A. 178 B. 188
C. 202 D. 194
E. None of these/इनम से कोई नही ं

84. Number of Coolers sold by shop R is how much percent more or less than the number of
AC sold by shop Q.
दु कान R ारा बेचे गए कूलर की सं ा, दु कान Q ारा बेचे गए AC की सं ा से िकतने ितशत अिधक या
कम है ।
A. 25% B. 15%
C. 10% D. 20%
E. None of these/इनम से कोई नही ं

85. If price of one AC is Rs. 50,000 and price of one Cooler is Rs. 5000, then find the total
revenue earned by shop S.
यिद एक AC की कीमत 50,000 है और एक कूलर की कीमत 5000 है , तो दु कान S ारा अिजत कुल
राज का पता लगाएं ।
A. Rs. 23,65,000 B. Rs. 24,65,000
C. Rs. 25,65,000 D. Rs. 26,65,000
E. None of these/इनम से कोई नही ं

Read the given data carefully and answer the following questions.

Directions:- There are 4800 students in four different schools. The ratio of number of boys and
number of girls in school A 2:3 respectively. Girls in school Q are 150 more than the number of
boys in the same school. Number of girls in school R are 25% more than the number of boys in
the same school, which is 50% more than the number of boys in school Q. Number of girls in
school S is two and half times the number of boys in school Q, and number of girls in school S is
250 more than the number of boys in the same school. Number of boys in school Q is 400.

PAGE h ps://t.me/studified h ps://instagram.com/aashisharorasocial?utm_medium=copy_link


Follow
173 Aashish Arora on: h ps://youtube.com/channel/UCYa4_JrOrf8R5Kz2uOtccXQ https://www.facebook.com/aashisharorasocial/
अ ास by Aashish Arora
(SBI/IBPS/RBI/LIC/All other banking and insurance Exams)

दी गयी जानकारी को ान से पढ़ और िन िल खत ो ं के उ र द।
चार अलग-अलग ू लो ं म 4800 छा ह। ू ल ए म लड़को ं की सं ा और लड़िकयो ं की सं ा का अनुपात
मशः 2: 3 है । ूल ू म लड़िकयां उसी ू ल म लड़को ं की सं ा से 150 अिधक ह। ू ल R म लड़िकयो ं की
सं ा उसी ू ल के लड़को ं की सं ा से 25% अिधक है , जो िक ूल ू के लड़को ं की सं ा से 50% अिधक
है । ू ल एस म लड़िकयो ं की सं ा ूल ू म लड़को ं की सं ा से ढाई गुना है , और ू ल एस म लड़िकयो ं की
सं ा उसी ू ल म लड़को ं की सं ा से 250 अिधक है । ूल ू म लड़को ं की सं ा 400 है ।

86. What is the difference between the total number of girls and boys?
लड़िकयो ं और लड़को ं की कुल सं ा म ा अंतर है ?
A. 550 B. 700
C. 750 D. 650
E. None of these

87. What is ratio of number of boys in school S to the number of girls in school S ?
ू ल S म लड़को ं की सं ा और ू ल S म लड़िकयो ं की सं ा का अनुपात ा है ?
A. 5 : 7 B. 3 : 4
C. 2 : 5 D. 9 : 11
E. None of these

88. School R admitted 100 new boys. By what percentage is the number of boys in school R
more/less than the number of girls in the same school.
ू ल आर ने 100 नए लड़को ं को भत िकया। िकस ितशत से ू ल R म लड़को ं की सं ा उसी ू ल म
लड़िकयो ं की सं ा से अिधक / कम है ।
A. 6.66 % B. 7.75 %
C. 3.33 % D. 5.25 %
E. None of these

89. What is the difference between number of girls in School R and S together and number of
girls in School Q and number of boys in school R together.
ू ल आर और एस म लड़िकयो ं की सं ा एक साथ और ू ल ू म लड़िकयो ं की सं ा और ू ल आर म
लड़को ं की सं ा एक साथ म अंतर ा है ?
A. 550 B. 450
C. 750 D. 600
E. None of these

90. On a certain day 20% of the students in school R were absent and 4% of the students in
school Q were absent. What percentage of students were present in school Q compared
to school R.
एक िनि त िदन पर ू ल R म 20% छा अनुप थत थे और ू ल Q म 4% छा अनुप थत थे। िव ालय R
की तुलना म िव ालय Q म िकतने ितशत छा उप थत थे।
A. 75.5 % B. 84.4 %
C. 98.4 % D. 87.2 %
E. None of these/इनम से कोई नही ं

Direction: Study the passage given below and answer the following questions.
There are two institutes, A and B. Both of them provide test series for two different exams i.e.

PAGE h ps://t.me/studified h ps://instagram.com/aashisharorasocial?utm_medium=copy_link


Follow
174 Aashish Arora on: h ps://youtube.com/channel/UCYa4_JrOrf8R5Kz2uOtccXQ https://www.facebook.com/aashisharorasocial/
अ ास by Aashish Arora
(SBI/IBPS/RBI/LIC/All other banking and insurance Exams)

SSC and Banking. The ratio of the total number of test series provided by B for the SSC exam
and that of total number by A for banking exam is 6:7, respectively. The total number of test
series provided by B for Banking exam is 25% more than the total number of test series
provided by A for SSC exam. The total number of test series provided by A and B together for the
SSC and Banking exam is 56 and 67, respectively.

िनदश: नीचे िदए गये ग ांश का अ यन कीिजये और िन िल खत ो ं के उ र दीिजये।


दो सं थान ह, ए और बी। ये दोनो ं दो अलग-अलग परी ाओं यानी एसएससी और बिकंग के िलए टे सीरीज दान
करते ह। एसएससी परी ा के िलए बी ारा दान की गई टे सीरीज की कुल सं ा और बिकंग परी ा के िलए ए
ारा कुल सं ा का अनुपात मशः 6:7 है । बिकंग परी ा के िलए B ारा दान की गई टे सीरीज़ की कुल
सं ा, SSC परी ा के िलए A ारा दान की गई टे सीरीज़ की कुल सं ा से 25% अिधक है । एसएससी और
बिकंग परी ा के िलए ए और बी ारा दान की गई टे सीरीज की कुल सं ा मशः 56 और 67 है ।

91. What is the ratio of the number of test series provided by A for SSC exam to the number of
test series provided by B for Banking exam?
SSC परी ा के िलए A ारा दान की गई टे सीरीज़ की सं ा का बिकंग परी ा के िलए B ारा दान की
गई टे सीरीज़ की सं ा से अनुपात िकतना है ?
A. 5:4 B. 6:5
C. 4:5 D. 1:6
E. None of these
92. What is the total number of test series provided by institute A.
सं थान A ारा उपल कराई गई टे सीरीज की कुल सं ा िकतनी है ?
A. 80 B. 54
C. 72 D. 62
E. None of these

93. What is the difference between total number of test series provided by A for Banking exam
and total number of SSC exam by B?
A ारा बिकंग परी ा के िलए दान की गई टे सीरीज की कुल सं ा और B ारा एसएससी परी ा की कुल
सं ा के बीच का अंतर ा है ?
A. 8 B. 5
C. 7 D. 6
E. None of these

94. Total number of test series for Banking exam provided by A is how much percent more/less
that number of test series for SSC exam provided by A?
A ारा दान की गई बिकंग परी ा के िलए टे सीरीज़ की कुल सं ा, A ारा दान की गई SSC परी ा के
िलए टे सीरीज़ की सं ा से िकतने ितशत अिधक/कम है ?
A. 100% B. 110%
C. 120% D. 130%
E. None of these

95. Total number of test series provided by B for SSC exam is what fraction of total number of
test series provided by both institutes together for both exams?
SSC परी ा के िलए B ारा दान की गई टे सीरीज़ की कुल सं ा, दोनो ं सं थानो ं ारा दोनो ं परी ाओं के
िलए दान की गई टे सीरीज़ की कुल सं ा का िकतना अंश है ?

PAGE h ps://t.me/studified h ps://instagram.com/aashisharorasocial?utm_medium=copy_link


Follow
175 Aashish Arora on: h ps://youtube.com/channel/UCYa4_JrOrf8R5Kz2uOtccXQ https://www.facebook.com/aashisharorasocial/
अ ास by Aashish Arora
(SBI/IBPS/RBI/LIC/All other banking and insurance Exams)

A. 12/41 B. 13/27
C. 14/57 D. 17/21
E. None of these

Direction: Study the passage given below and answer the following questions.
The given data is about number of employee at five different company i.e. A, B, C, D and E. The
number of employee at company E is 25% less than that at company B. The ratio of the number
of employee at company B and company C is 9:10 respectively. The total number of employee
at company C and D is 12000. The number of employee at company A is 2000 more than that at
company D. The ratio of the number of employee at company A and E is 10:9 respectively.

िनदश: नीचे िदए गये ग ांश का अ यन कीिजये और िन िल खत ो ं के उ र दीिजये।


िदया गया डे टा पां च अलग-अलग कंपनी यानी A, B, C, D और E म कमचा रयो ं की सं ा के बारे म है । कंपनी ई म
कमचा रयो ं की सं ा कंपनी बी से 25% कम है । कंपनी बी म कमचा रयो ं की सं ा का अनुपात और कंपनी C
मशः 9:10 है । कंपनी C और D म कमचा रयो ं की कुल सं ा 12000 है । कंपनी A म कमचा रयो ं की सं ा
कंपनी D से 2000 अिधक है । कंपनी A और E म कमचा रयो ं की सं ा का अनुपात मशः 10:9 है ।

96. At which of the given five company following the number of employee is highest.
िन िल खत म से िकस पां च कंपनी म कमचा रयो ं की सं ा सबसे अिधक है ।
A. D B. B
C. E D. A
E. None of these

97. Find the difference between total number of employee at company C and D, together and
number of employee at Company A
कंपनी C और D म एक साथ कमचा रयो ं की कुल सं ा और कंपनी A म कमचा रयो ं की कुल सं ा के बीच
का अंतर ात कीिजए
A. 4000 B. 5400
C. 6000 D. 7400
E. None of these

98. 40% of employee at C and 25% of employee at B are female. Find the ratio of the number
of Female employee at C and B respectively.
C पर 40% कमचारी और B म 25% कमचारी मिहलाएँ ह। मशः C और B पर मिहला कमचा रयो ं की
सं ा का अनुपात ात कीिजए।
A. 16:9 B. 8:3
C. 11:8 D. 10:7
E. None of these

99. If the number of employee at F is 60% of the number of employee at C and E together. The
find the average of employee at A and F .
यिद F पर कमचा रयो ं की सं ा C और E म िमलाकर कमचा रयो ं की सं ा का 60% है । A और F पर
कमचारी का औसत ात कीिजए।
A. 6240 B. 4570
C. 7020 D. 5820
E. None of these

PAGE h ps://t.me/studified h ps://instagram.com/aashisharorasocial?utm_medium=copy_link


Follow
176 Aashish Arora on: h ps://youtube.com/channel/UCYa4_JrOrf8R5Kz2uOtccXQ https://www.facebook.com/aashisharorasocial/
अ ास by Aashish Arora
(SBI/IBPS/RBI/LIC/All other banking and insurance Exams)

100. The ratio of the number of male and female employee in company 'D' is 9:7, respectively.
The number of female employee at company 'D' is approximately how much percent
more/less than the total number of employee in company 'E'?
कंपनी 'D' म पु ष और मिहला कमचा रयो ं की सं ा का अनुपात मशः 9:7 है । कंपनी 'D' म मिहला
कमचा रयो ं की सं ा कंपनी 'E' म कमचा रयो ं की कुल सं ा से लगभग िकतने ितशत अिधक/कम है ?
A. 58% B. 68%
C. 78% D. 48%
E. None of these

Direction: Study the passage given below and answer the following questions.
The information given below is the investment of three Venture capitalists in a partnership for
the period of 1991 – 1995.
The investments made by an individual are for the same period. The investment of Bikram in
1991 is Rs. 40000 and is equal is to the investment of Chandan in 1993. The total investment in
1994 is Rs. 24000 and the ratio of investments of Arjun, Bikram and Chandan is 8 : 9 : 7
respectively. The investments of Arjun in 1991, 1992 and 1993 are Rs. 32000, Rs. 48000 and
Rs. 50000 respectively. The investment of Chandan in 1991 and 1992 are same i.e. Rs. 22000.
The investment of Bikram in 1993 is Rs. 6000 more than the investment by him in 1992 i.e. Rs.
30000.

िनदश: नीचे िदए गये ग ांश का अ यन कीिजये और िन िल खत ो ं के उ र दीिजये।


नीचे दी गई जानकारी 1991-1995 की अविध के िलए एक साझेदारी म तीन पूंजीपितयो ं का िनवेश है ।
िकसी भी एक के ारा िकया गया िनवेश समान अविध के िलए है । 1991 म िब म का िनवेश 40000 पये है
जो चंदन के 1993 म चंदन के िनवेश के बराबर है । 1994 म कुल िनवेश 24000 पये है और अजुन, िब म और
चंदन के िनवेश का अनुपात मशः 8:9:7 है । अजुन के िनवेश 1991, 1992 और 1993 म मशः 32000 पये,
48000 पये और 50000 पये है । चंदन का िनवेश 1991 और 1992 म समान अथात 22000 पये है । िब म
का िनवेश 1993 म, 1992 म उसके ारा िकए गए िनवेश से 6000 अिधक अथात 30000 पये है ।

101. Find the share of profit earned by Bikram in the year 1993, if the total profit in 1993 is Rs.
15000?
यिद 1993 म कुल लाभ 15000 पये है , तो 1993 म िब म ारा ा िकये गये लाभ का िह ा खोज ?
A. 4550 B. 5065
C. 3750 D. 4600
E. None of these

102. Suppose all the VCs invested for one more year i.e. 1995 and the total investment of Arjun
and Bikram is Rs. 56000 and invested their amounts for 24 and 16 months respectively,
find for how many months Chandan invested his amount of Rs. 64,000? [Given profits of
Arjun, Bikram and Chandan are Rs. 12600, Rs. 11200 and Rs. 16800 respectively]
मान लीिजए िक सभी पूंजीपितयो ं ने एक और वष यानी 1995 के िलए िनवेश िकया और अजुन और िब म का
कुल िनवेश 56000 पये है और उ ोने मशः 24 और 16 महीनो ं के िलए अपनी रािश का िनवेश िकया। तो
चंदन ने िकतने महीनो ं के िलए अपनी 64,000 पये िक रािश का िनवेश िकया ? [अजुन, िब म और चंदन के
िदये गये लाभ मशः 12600 पये, 11200 पये और 16800 पये है ।]
A. 12 months B. 15 months
C. 11 months D. 14 months

PAGE h ps://t.me/studified h ps://instagram.com/aashisharorasocial?utm_medium=copy_link


Follow
177 Aashish Arora on: h ps://youtube.com/channel/UCYa4_JrOrf8R5Kz2uOtccXQ https://www.facebook.com/aashisharorasocial/
अ ास by Aashish Arora
(SBI/IBPS/RBI/LIC/All other banking and insurance Exams)

E. None of these

103. If the share of profit of Chandan in 1991 and 1992 is Rs. 7700 and Rs. 8800 respectively,
find the ratio of profit of Arjun in 1991 to that in 1992?
यिद 1991 और 1992 म चंदन के लाभ का िह ा मशः 7700 पये और 8800 पये ह तो 1991 और
1992 म अजुन के लाभ का अनुपात ात कीिजए।
A. 5 : 13 B. 4 : 15
C. 7 : 12 D. 8:9
E. None of these

104. If the amount of profit shared by Arjun and Bikram in 1994 is Rs. 4000 and Rs. 4500
respectively and Chandan makes 3/4th of the profit in 1995 as compared to his profit in
1994. Find the amount of Profit shared by Chandan in 1995?
यिद 1994 म अजुन और िब म ारा साझा िकए गए लाभ की रािश मश: 4000 पये और 4500 पये है
और चंदन 1994 के अपने लाभ की तुलना म 1995 म लाभ का 3/4 िह ा ा करता है । तो 1995 म चंदन
ारा साझा की गई लाभ की रािश का पता लगाएं ।
A. 2526 B. 2625
C. 2895 D. 2425
E. None of these

105. The profit earned by Bikram in 1996 is 8% of the investment made by Bikram in 1993 and
the profit of Chandan in 1996 is 10% of the investment made by Chandan in 1992. Find the
ratio of profit of Chandan in 1996 to that of Bikram in 1996 ?
1996 म िब म ारा अिजत लाभ, 1993 म िब म ारा िकये गये िनवेश का 8% है और 1996 म चंदन का
लाभ 1992 म चंदन ारा िकये गये िनवेश का 10% है । 1996 म चंदन और 1996 म िब म के लाभ का
अनुपात ात कीिजए ?
A. 12 : 13 B. 11 : 12
C. 10 : 19 D. 8 : 15
E. None of these

Direction : Study the data carefully and answer the following questions
Two farmers Rohit and Rajat, grow five different grains i.e. wheat, rice, jowar, bajra, and
sorghum. The quantity of sorghum grown by Rohit is same as the quantity of jowar grown by
Rajat, and the ratio of the quantity of wheat and rice grown by Rohit is 3:4, respectively. The
quantity of wheat grown by Rajat is twice the quantity of bajra grown by Rohit. The total quantity
of rice grown by both Rohit and Rajat is 308 kg while the quantity of jowar grown by Rohit is 92
kg. The quantity of rice grown by Rajat is 57 kg more than the quantity of bajra grown by Rajat,
and the quantity of bajra grown by Rajat is 91 kg. The total quantity of grains grown by Rohit is
535 kg, and the quantity of jowar grown by Rajat is 92 kg less than the quantity of wheat grown
by Rajat. The quantity of sorghum grown by Rajat is 107 kg.

िनदश : आं कड़ो ं का ानपूवक अ यन कर और िन िल खत ो ं के उ र द


दो िकसान रोिहत और रजत, पां च अलग-अलग अनाज यानी गे ं , चावल, ार, बाजरा और ार उगाते ह। रोिहत
ारा उगाए गए ार की मा ा रजत ारा उगाए गए ार की मा ा के समान है , और रोिहत ारा उगाए गए गे ं और
चावल की मा ा का अनुपात मशः 3:4 है । रजत ारा उगाए गए गे ं की मा ा रोिहत ारा उगाए गए बाजरा की

PAGE h ps://t.me/studified h ps://instagram.com/aashisharorasocial?utm_medium=copy_link


Follow
178 Aashish Arora on: h ps://youtube.com/channel/UCYa4_JrOrf8R5Kz2uOtccXQ https://www.facebook.com/aashisharorasocial/
अ ास by Aashish Arora
(SBI/IBPS/RBI/LIC/All other banking and insurance Exams)

मा ा से दोगुनी है । रोिहत और रजत दोनो ं ारा उगाए गए चावल की कुल मा ा 308 िकलो ाम है जबिक रोिहत ारा
उगाए गए ार की मा ा 92 िकलो ाम है । रजत ारा उगाए गए चावल की मा ा रजत ारा उगाए गए बाजरा की
मा ा से 57 िकलो ाम अिधक है , और रजत ारा उगाए गए बाजरा की मा ा 91 िकलो ाम है । रोिहत ारा उगाए गए
अनाज की कुल मा ा 535 िकलो ाम है , और रजत ारा उगाए गए ार की मा ा रजत ारा उगाए गए गे ं की मा ा
से 92 िकलो ाम कम है । रजत ारा उगाए गए ार की मा ा 107 िकलो ाम है ।

106. What is total quantity of sorghum grown by both Rohit and Rajat ?
रोिहत और रजत दोनो ं ारा उगाए गए ार की कुल मा ा िकतनी है ?
A. 165 B. 185
C. 160 D. 175
E. None of these

107. What is total quantity of Wheat grown by Rajat?


रजत ारा उगाए गए गे ं की कुल मा ा िकतनी है ?
A. 185 B. 180
C. 175 D. 170
E. None of these

108. What is difference between the quantity of rice grown by Rohit and Rajat?
रोिहत और रजत ारा उगाए गए चावल की मा ा म िकतना अंतर है ?
A. 12 B. 15
C. 18 D. 22
E. None of these

109. What is total quantity of grains grown by Rajat?


रजत ारा उगाए गए अनाज की कुल मा ा िकतनी है ?
A. 448 B. 520
C. 594 D. 538
E. None of these

110. What is the total quantity of jowar grown by both Rohit and Rajat together?
रोिहत और रजत दोनो ं ारा िमलाकर उगाए गए ार की कुल मा ा िकतनी है ?
A. 165 B. 170
C. 175 D. 180
E. None of these

Direction: Answer the questions based on the information given below.


The given data is about number of Govt. sectors and Private sectors in three different cities. The
ratio of the number of private sectors in city 'P' to the number of govt. sectors in city 'Q' is 5:4,
respectively. The number of govt. sectors in city 'R' is 25% less than that in city 'Q'. The total
number of private sectors and govt. sectors , together in city P, Q and R is 1800, 900, 1200
respectively. The total number of private sectors in all three cities together is 1800.

िनदश: नीचे दी गई जानकारी के आधार पर ो ं के उ र द।


िदया गया डे टा तीन अलग-अलग शहरो ं म सरकारी े ो ं और िनजी े ो ं की सं ा है । 'P' म िनजी े ो ं की सं ा का
शहर 'Q' म सरकारी े ो ं की सं ा से अनुपात मशः 5:4 है । शहर 'R' म सरकारी े ो ं की सं ा शहर 'Q' की
तुलना म 25% कम है । शहर P, Q और R म एक साथ िनजी े ो ं और सरकारी े ो ं की कुल सं ा मशः 1800,

PAGE h ps://t.me/studified h ps://instagram.com/aashisharorasocial?utm_medium=copy_link


Follow
179 Aashish Arora on: h ps://youtube.com/channel/UCYa4_JrOrf8R5Kz2uOtccXQ https://www.facebook.com/aashisharorasocial/
अ ास by Aashish Arora
(SBI/IBPS/RBI/LIC/All other banking and insurance Exams)

900, 1200 है । सभी 3 शहरो ं म सरकारी े ो ं की कुल सं ा 1800 है ।

111. Find the average of number of private sectors in the given three cities.
िदए गए तीन शहरो ं म िनजी े ो ं की सं ा का औसत ात कीिजए।
A. 700 B. 900
C. 600 D. 1800
E. None of these

112. If the number of private sectors in city 'S' is 600 more than in city 'Q', then find the ratio of
the number of private sectors in city 'S' to the number of government sectors in city 'Q'.
यिद शहर 'S' म िनजी े ो ं की सं ा शहर 'Q' से 600 अिधक है , तो शहर 'S' म िनजी े ो ं की सं ा का
शहर 'Q' म सरकारी े ो ं की सं ा से अनुपात ात कीिजए।
A. 2:3 B. 3:2
C. 3:4 D. 5:2
E. None of these

113. Find the difference between total number of government sectors in city 'P' and city 'R'
together and total number of private sectors and government sectors together in city 'R'.
शहर 'P' और शहर 'R' म िमलाकर सरकारी से रो ं की कुल सं ा और शहर 'R' म िमलाकर िनजी से रो ं
और सरकारी से रो ं की कुल सं ा के बीच का अंतर ात कीिजए।
A. 300 B. 400
C. 350 D. 200
E. None of these

114. The number of government sectors in city 'Q' is how much percent more/less than the
number of private sectors in city 'P'?
शहर 'Q' म सरकारी े ो ं की सं ा, शहर 'P' म िनजी े ो ं की सं ा से िकतने ितशत अिधक/कम है ?
A. 6% B. 30%
C. 25% D. 20%
E. None of these

115. In city 'T' the number of government sectors is 20% less than that in city 'P'. If number of
government sectors in city 'T' 25% of total number of government sectors and private
sectors together of city 'T', then find the number of private sectors in city 'T' .
शहर 'T' म सरकारी े ो ं की सं ा शहर 'P' की तुलना म 20% कम है । यिद शहर 'T' म सरकारी े ो ं की
सं ा शहर 'T' के कुल सरकारी े ो ं और िनजी े ो ं की कुल सं ा का 25% है , तो शहर 'T' म िनजी े ो ं
की सं ा ात कीिजए।
A. 4302 B. 3200
C. 4280 D. 2520
E. None of these

Direction: Study the passage given below and answer the following questions.
There are three college i.e. 'A', 'B', and 'C'. Total Number of students is 1800. The ratio of the
number of girls is class B to the number of boys in class C is 5:9 respectively. Total number of
students in class C is 600.
Class A: Ratio of the number of boys and girls is 7:5 respectively.
Class B: Number of boys is 40% more than the Girls in the class.

PAGE h ps://t.me/studified h ps://instagram.com/aashisharorasocial?utm_medium=copy_link


Follow
180 Aashish Arora on: h ps://youtube.com/channel/UCYa4_JrOrf8R5Kz2uOtccXQ https://www.facebook.com/aashisharorasocial/
अ ास by Aashish Arora
(SBI/IBPS/RBI/LIC/All other banking and insurance Exams)

Class C: Total number of students is 25% more then that in Class B. Ratio of the number of girls
in class B and class C is 5:6 respectively.

िनदश: नीचे िदए गये ग ांश का अ यन कीिजये और िन िल खत ो ं के उ र दीिजये।


तीन कॉलेज यानी 'A', 'B', और 'C' ह। क ा B म लड़िकयो ं की सं ा का क ा C म लड़को ं की सं ा से अनुपात
मशः 5:9 है । क ा सी म छा ो ं की कुल सं ा 600 है ।
क ा A: लड़को ं और लड़िकयो ं की सं ा का अनुपात मशः 7:5 है ।
क ा B: क ा म लड़िकयो ं की तुलना म लड़को ं की सं ा 40% अिधक है ।
क ा C: क ा बी म छा ो ं की कुल सं ा 25% अिधक है । क ा B और क ा C म लड़िकयो ं की सं ा का अनुपात
मशः 5:6 है ।

116. Find the number of Boys in Class A.


क ा A म लड़को ं की सं ा ात कीिजए।
A. 140 B. 230
C. 450 D. 360
E. None of these

117. Ratio of the number of boys and girls in class C is.


क ा C म लड़को ं और लड़िकयो ं की सं ा का अनुपात है ।
A. 3:2 B. 5:4
C. 7:6 D. 9:8
E. None of these

118. Number of girls in Class B is how much percentage more/less than the number of girls in
Class A.
क ा B म लड़िकयो ं की सं ा, क ा A म लड़िकयो ं की सं ा से िकतने ितशत अिधक/कम है ।
A. 13 (1/3) % B. 23 (1/3) %
C. 43 (1/3) % D. 33 (1/3) %
E. None of these

119. Average number of students in A and C together is-


A और C म िमलाकर िव ािथयो ं की औसत सं ा है -
A. 560 B. 520
C. 660 D. 460
E. None of these

120. If the total number of Students in Class D is 40% more than that of Class B and ratio of
number of boys in Class C and D is 15:19, then find the difference between number of girls
in class B and D
यिद क ा D म छा ो ं की कुल सं ा क ा B से 40% अिधक है और क ा C और D म लड़को ं की सं ा का
अनुपात 15:19 है , तो क ा B और D म लड़िकयो ं की सं ा के बीच का अंतर ात कीिजए।
A. 20 B. 16
C. 18 D. 17
E. None of these

Direction: Study the following data are given below to answer the question.
There are total 24000 Students enroll in a school, each one is from one among the five districts

PAGE h ps://t.me/studified h ps://instagram.com/aashisharorasocial?utm_medium=copy_link


Follow
181 Aashish Arora on: h ps://youtube.com/channel/UCYa4_JrOrf8R5Kz2uOtccXQ https://www.facebook.com/aashisharorasocial/
अ ास by Aashish Arora
(SBI/IBPS/RBI/LIC/All other banking and insurance Exams)

of Karnataka viz. Mysore, Tumkur, Bellary, Bangalore and Kolar. Respective ratio of boys and
girls among them is 7:5. 25% of the total number of boys is from Bangalore. Respective ratio of
number of girls from Bellary and number of boys from Bangalore is 4:7. Number of boys from
Bellary is 10% more than the number of girls from that state. 18% of the girls are from Mysore.
25% of the total number of students is from Mysore. Number of boys from Kolar is 80% of the
number of boys from Mysore. Number of girls from Tumkur is 20% more than the number of
boys from that state. 30% of the total number of girls is from Bangalore.

िनदश: का उ र दे ने के िलए नीचे िदए गए डे टा का अ यन कर।


एक ू ल म कुल 24000 छा नामां िकत ह, ेक कनाटक के पां च िजलो ं अथात मैसूर, तुमकुर, बे ारी, बगलोर
और कोलार म से एक से है । उनम लड़के और लड़िकयो ं का मशः अनुपात 7:5 है । लड़को ं की कुल सं ा का
25% बगलोर से है । बे ारी से लड़िकयो ं की सं ा और बगलोर से लड़को ं की सं ा का अनुपाितक अनुपात 4:7
है । बे ारी से लड़को ं की सं ा उस रा की लड़िकयो ं की सं ा से 10% अिधक है । 18% लड़िकयां मैसूर से ह।
छा ो ं की कुल सं ा का 25% मैसूर से है । कोलार से लड़को ं की सं ा मैसूर के लड़को ं की सं ा का 80% है ।
तुमकुर की लड़िकयो ं की सं ा उस रा के लड़को ं की सं ा से 20% अिधक है । लड़िकयो ं की कुल सं ा का
30% बगलोर से है ।

126. The total number of boys from Bangalore is approximately what percent of the total
number of students from Bellary?
बगलोर से लड़को ं की कुल सं ा बे ारी के छा ो ं की कुल सं ा का लगभग िकतना ितशत है ?
A. 67% B. 83%
C. 53% D. 79%
E. None of these

127. Find the respective ratio of the total number of students from Mysore and the total number
of students from Bangalore.
मैसूर से छा ो ं की कुल सं ा और बगलोर से छा ो ं की कुल सं ा का संबंिधत अनुपात ात कीिजए।
A. 17:11 B. 9:11
C. 11:13 D. 12:13
E. None of these

128. Find the average of the number of boy students Mysore, Tumkur, Bellary and Kolar.
मैसूर, तुमकुर, बे ारी और कोलार म लड़को ं की औसत सं ा ात कीिजए।
A. 2625 B. 2984
C. 3387 D. 2280
E. None of these

129. The number of girl students are highest from which state?
छा ाओं की सं ा सबसे अिधक िकस रा से है ?
A. Kolar B. Tumkur
C. Bangalore D. Mysore
E. None of these

130. Find the difference between the number of girl students from Bellary and Bangalore
together and the number of girl students from Tumkur and Kolar together
बे ारी और बगलोर से िमलाकर छा ाओं की सं ा और तुमकुर और कोलार से िमलाकर छा ाओं की

PAGE h ps://t.me/studified h ps://instagram.com/aashisharorasocial?utm_medium=copy_link


Follow
182 Aashish Arora on: h ps://youtube.com/channel/UCYa4_JrOrf8R5Kz2uOtccXQ https://www.facebook.com/aashisharorasocial/
अ ास by Aashish Arora
(SBI/IBPS/RBI/LIC/All other banking and insurance Exams)

सं ा के बीच का अंतर ात कीिजए।


A. 1760 B. 2535
C. 1230 D. 1800
E. None of these

Direction: Study the passage given below and answer the following questions.
Three different cities Kolkata, Mumbai and Delhi use shopping app of three different sites
Flipkart, Amazon and Myntra. The ratio of number of people who shop from Flipkart in Kolkata to
that shop from Amazon in Delhi is 5:4. The number of people who shop from Myntra in Kolkata is
25% more than that shop from Amazon in Delhi. The total number of people who shop from any
of the three sites in Kolkata is 12000. The number of people who shop from amazon in Mumbai
is 1200 less than that shop from Myntra in Kolkata. Total number of people who shop from
Flipkart and Amazon in all three cities is 12000 and 10000 respectively. The number of people
who shop from Flipkart and Myntra in Mumbai is 3000 and 1000 respectively. Number of people
who shop from Myntra in Delhi is 2000.

िनदश: नीचे िदए गये ग ांश का अ यन कीिजये और िन िल खत ो ं के उ र दीिजये।


तीन अलग-अलग शहर कोलकाता, मुंबई और िद ी तीन अलग-अलग साइटो ं Flipkart, Amazon और Myntra
के शॉिपंग ऐप का उपयोग करते ह। कोलकाता म पकाट से खरीदारी करने वाले लोगो ं की सं ा का िद ीम
अमेज़न से उस दु कान से अनुपात 5:4 है । कोलकाता म िमं ा से खरीदारी करने वालो ं की सं ा िद ी म अमेज़न
की उस दु कान से 25% अिधक है । कोलकाता म तीन साइटो ं म से िकसी एक से खरीदारी करने वालो ं की कुल सं ा
12000 है । मुंबई म ऐमज़ॉन से खरीदारी करने वालो ं की सं ा कोलकाता के िमं ा की उस दु कान से 1200 कम है ।
तीनो ं शहरो ं म Flipkart और Amazon से खरीदारी करने वालो ं की कुल सं ा मशः 12000 और 10000 है ।
मुंबई म पकाट और िमं ा से खरीदारी करने वालो ं की सं ा मश: 3000 और 1000 है । िद ी म िमं ा से
खरीदारी करने वालो ं की सं ा 2000 है ।

131. Find the average number of people who shop from any of three sites in Delhi ?
िद ी म तीन साइटो ं म से िकसी एक से खरीदारी करने वाले लोगो ं की औसत सं ा ात कीिजए?
A. 3200 B. 3400
C. 3600 D. 3800
E. None of these

132. Find the ratio of the number of people who shop from Flipkart in Mumbai to the number of
people who shop from Myntra in Kolkata.
मुंबई म पकाट से खरीदारी करने वाले लोगो ं की सं ा का कोलकाता म िमं ा से खरीदारी करने वाले
लोगो ं की सं ा से अनुपात ात कीिजए।
A. 3:4 B. 3:5
C. 4:5 D. 7:6
E. None of these

133. The number of Male who shop from amazon in Delhi is 72% less than that of females. Find
the number of males who shop from amazon.
िद ी म अमेज़न से खरीदारी करने वाले पु षो ं की सं ा मिहलाओं की तुलना म 72% कम है । अमेज़न से
खरीदारी करने वाले पु षो ं की सं ा ात कीिजए।
A. 800 B. 700

PAGE h ps://t.me/studified h ps://instagram.com/aashisharorasocial?utm_medium=copy_link


Follow
183 Aashish Arora on: h ps://youtube.com/channel/UCYa4_JrOrf8R5Kz2uOtccXQ https://www.facebook.com/aashisharorasocial/
अ ास by Aashish Arora
(SBI/IBPS/RBI/LIC/All other banking and insurance Exams)

C. 600 D. 500
E. None of these

134. The number of people who shop from Myntra in Mumbai is how much percentage
more/less that the number of people who shop from amazon in Kolkata ?
मुंबई म िमं ा से खरीदारी करने वाले लोगो ं की सं ा कोलकाता म अमेज़न से खरीदारी करने वाले लोगो ं की
सं ा से िकतने ितशत अिधक/कम है ?
A. 45% B. 60%
C. 75% D. 80%
E. None of these

135. Find the difference between the number of people who shop from Flipkart and Amazon
together in Kolkata and the number of people who shop from Myntra and amazon together
in Delhi.
कोलकाता म Flipkart और Amazon से िमलाकर खरीदारी करने वाले लोगो ं की सं ा और िद ी म
Myntra और Amazon से िमलाकर खरीदारी करने वाले लोगो ं की सं ा के बीच का अंतर ात कीिजए।
A. 2200 B. 2400
C. 2600 D. 2800
E. None of these

Direction: Study the passage given below and answer the following questions.
In an examination, there are three subjects i.e. English, Math And Science, having maximum
Marks 30, 35 and 35 respectively. Ramesh got 60% marks in Math. The overall percentage of
marks obtained by Anish is 65%. Anish got 70% marks in English. Anish got 80% marks in
English and Science together. The marks obtained by Ramesh in Science are 4/3 times of the
marks obtained by Ramesh in Maths. The overall percentage of marks by Ramesh is 75%.

िनदश: नीचे िदए गये ग ांश का अ यन कीिजये और िन िल खत ो ं के उ र दीिजये।


एक परी ा म, तीन िवषय अथात अं ेजी, गिणत और िव ान ह, िजनके अिधकतम अंक मश: 30, 35 और 35 ह।
रमेश को गिणत म 60% अंक िमले। अनीश ारा ा अंको ं का कुल ितशत 65% है । अनीश को अं ेजी म 70%
अंक िमले। अनीश ने अं ेजी और िव ान म िमलाकर 80% अंक ा िकए। रमेश ारा िव ान म ा अंक रमेश
ारा गिणत म ा अंको ं के 4/3 गुना ह। रमेश ारा कुल अंको ं का ितशत 75% है ।

136. Find the marks obtained by Anish in Science.


अनीश ारा िव ान म ा अंक ात कीिजए।
A. 31 B. 30
C. 29 D. 28
E. None of these

137. Find the total marks obtained by Ramesh.


रमेश ारा ा कुल अंक ात कीिजए।
A. 80 B. 75
C. 78 D. 74
E. None of these

138. Find the marks obtained by Ramesh in English is how much less/more than the marks
obtained by Ramesh in Science.

PAGE h ps://t.me/studified h ps://instagram.com/aashisharorasocial?utm_medium=copy_link


Follow
184 Aashish Arora on: h ps://youtube.com/channel/UCYa4_JrOrf8R5Kz2uOtccXQ https://www.facebook.com/aashisharorasocial/
अ ास by Aashish Arora
(SBI/IBPS/RBI/LIC/All other banking and insurance Exams)

रमेश ारा अं ेजी म ा अंक, रमेश ारा िव ान म ा अंको ं से िकतने कम/अिधक ह।


A. 8 B. 6
C. 4 D. 2
E. None of these

139. If the percentage of passing marks in the examination is 40%, then Anish passed by how
many marks?
यिद परी ा म उ ीण अंको ं का ितशत 40% है , तो अनीश िकतने अंको ं से उ ीण आ?
A. 36 B. 37
C. 25 D. 39
E. None of these

140. Find the difference between total marks obtained by Ramesh and Anish together in maths
and sum of mark obtained by both in English?
रमेश और अनीश ारा गिणत म ा कुल अंको ं और अं ेजी म दोनो ं ारा ा अंको ं के योग के बीच का
अंतर ात कीिजए?
A. 10 B. 9
C. 12 D. 11
E. None of these

Join
Joinme
meonon

Join me on
Join me on

Join me on
Join me on
Join me on
BY: AASHISH ARORA

PAGE h ps://t.me/studified h ps://instagram.com/aashisharorasocial?utm_medium=copy_link


Follow
185 Aashish Arora on: h ps://youtube.com/channel/UCYa4_JrOrf8R5Kz2uOtccXQ https://www.facebook.com/aashisharorasocial/
अ ास by Aashish Arora
(SBI/IBPS/RBI/LIC/All other banking and insurance Exams)

SOLUTION

(1 – 5)

In June, Laptop Sold by Sony


2770 - 1.3 × 1.5x - 1.5x = 2158 - 0.9(x + 120) - 9(x + 120)÷8
2770 - 3.45x = 2158 - 2.025(x + 120)
612 = 3.45x - 2.025x – 243
1.425x = 855
x = 600

1. Ans. (B)
Total Laptop Sold by Sony - Total Laptop Sold by HP = 2770 - 2168 = 602

2. Ans. ©
Sony (May + June):Dell(April + May) = (900 + 700):(700 + 600) = 16:13

3. Ans. (B)
June (Sony + Dell) ´100 (700 + 810) ´100 151 ´100
= = = 56.5%
Total in April 2670 267

4. Ans. (A)
Revenue earned by Sony in April + Revenue earned by Dell in June
1170 × 30000 + 810 × 35000 = 6,34,50,000 Rs.

5. Ans. (B)
Total Laptop sold by three companies, in three month = 2670 + 2220 + 2158 = 7048

(6 – 10)
Total printer till
Tuesday = 1080, Thursday = 1530, Friday = 1620, Monday = 810, Wednesday = 1215
Laser Printer till
Thursday = 900, Tuesday = 550, Monday = 430, Friday = 946, Wednesday = 630

PAGE h ps://t.me/studified h ps://instagram.com/aashisharorasocial?utm_medium=copy_link


Follow
186 Aashish Arora on: h ps://youtube.com/channel/UCYa4_JrOrf8R5Kz2uOtccXQ https://www.facebook.com/aashisharorasocial/
अ ास by Aashish Arora
(SBI/IBPS/RBI/LIC/All other banking and insurance Exams)

Day => Monday Tuesday Wednesday Thursday Friday


Total 810 270 135 315 90
Laser 430 120 80 270 46
Inkjet 380 150 55 45 44

6. Ans. (B)
Total Printer sold on Thursday = 315

7. Ans. ©
On Tuesday, Laser:Inkjet = 120:150 = 4:5

8. Ans. (B)
(Wed - Thus) ´100 (55 - 45) ´100
Inkjet printer = = = 22.22%
Thus 45
9. Ans. (A)
Minimum Inkjet printer sold on = Friday

10. Ans. (B)


55 + 45
Average Inkjet printer sold on Wed. & Thus. = = 50
2
55.55% of Total printer sold on Friday = 50
41.66% of laser printed sold on Tuesday = 50
Only (A), (C) & (D)

(11 – 15)
Nano, in Delhi = 300, Jaipur = 300 × (19/12) = 475
Datsun, In Delhi = 475 - 85 = 390, In Jaipur = 1066 - 390 = 676
Kwid, In Delhi = 676 + 39 = 715, In Jaipur = 550
Alto, in Delhi = 663, Jaipur = 585

11. Ans. (B)


J(N + K):D(N + K) = (475 + 550):(300 + 715) = 205:203

12. Ans. ©
Total Kwid Sold = 715 + 550 = 1265

13. Ans. (B)


Total cars sold in, Delhi = 2068, Jaipur = 2286
Difference = 2286 - 2068 = 218

14. Ans. (A)


Total Cars sold in Jaipur & Delhi together = 2286 + 2068 = 4354.

15. Ans. (A)


In Jaipur, Kwid:Alto = 550:585 = 110:117

PAGE h ps://t.me/studified h ps://instagram.com/aashisharorasocial?utm_medium=copy_link


Follow
187 Aashish Arora on: h ps://youtube.com/channel/UCYa4_JrOrf8R5Kz2uOtccXQ https://www.facebook.com/aashisharorasocial/
अ ास by Aashish Arora
(SBI/IBPS/RBI/LIC/All other banking and insurance Exams)

(16 – 20)
792 ´ 34 792 ´ 55 792 Ì43
Who went by Cycle = = 204, Cycle = = 330, Cycle = = 258
132 132 132
In morning, By Car = x, Bike = x - 30, Cycle = x - 60, x = 140
In Noon, By Cycle = 8y, Bike = 11y, Car = 6y, y=8
In Evening, By Cycle = 6z, Bike = 12z + 12, Car = 7z z = 10

Cycle Bike Car Total


Morning 80 110 140 330
Afternoon 64 88 48 200
Evening 60 132 70 262
Total 204 330 258 792

16. Ans. (B)


Total Number of People who went by Car = 258

17. Ans. ©
By Bike, Evening:Morning = 132:110 = 6:5

18. Ans. (B)


(88 - 80) ´100
= 9.09%
88

19. Ans. (A)


(330 + 262)
Average of people who go in Morning & Evening = = 296.
2
20. Ans. (B)
Total Number of People who go in evening = 262

(21 – 25)
To Vienna, Passengers = 256, Seats = 256/0.8 = 320
To Berlin, Passengers = 320×9/8 = 360, Seats = 360/0.9 = 400
To Paris, Seats = 400 + 50 = 450, Passengers = 450
To Copenhagen, Seats = 450 - 250 = 200, Passenger = 0.7 × 200 = 140

21. Ans. ©
Vacant seats in train to Berlin is less than vacant seats in train to Copenhagen by

PAGE h ps://t.me/studified h ps://instagram.com/aashisharorasocial?utm_medium=copy_link


Follow
188 Aashish Arora on: h ps://youtube.com/channel/UCYa4_JrOrf8R5Kz2uOtccXQ https://www.facebook.com/aashisharorasocial/
अ ास by Aashish Arora
(SBI/IBPS/RBI/LIC/All other banking and insurance Exams)

(60 - 40) ´100


= = 33.33% = 33%
60
22. Ans. (A)
Difference of seats in train to Paris and Vienna = 450 - 320 = 130

23. Ans. (B)


Passengers in train to Paris:Seats in train to Berlin = 450:400 = 9:8

24. Ans. (D)


(0 + 64 + 40 + 60)
Average of vacant seats per train = = 41
4
25. Ans. (A)
Percentage of vacant seats in train to Vienna and Berlin with Total seats in train to
(64 + 40) ´100
Copenhagen = = 52%
200

(26 – 30)

School A B C D Total
Student 845 802 780 818 3245
Teacher 260 402 338 250 1250
Total 1105 1204 1118 1068 4495

26. Ans. (D)


780 + 818
Average of student in School C & D = = 799
2

27. Ans. (A)


Student in A – Teachers in B = 845 – 402 = 443

28. Ans. (B)


Number of present student = 802 – 202 = 600
Number of present girls = 0.6 × (802 – 202) = 0.6 × 600 = 360

29. Ans. ©
845
In school A, Number of Teachers required = = 169
5
Number of Teachers, who can be weeded out = 260 – 169 = 91

30. Ans. (B)


260 + 402 + 250
Average of teachers in School A, B & D = = 304
3

PAGE h ps://t.me/studified h ps://instagram.com/aashisharorasocial?utm_medium=copy_link


Follow
189 Aashish Arora on: h ps://youtube.com/channel/UCYa4_JrOrf8R5Kz2uOtccXQ https://www.facebook.com/aashisharorasocial/
अ ास by Aashish Arora
(SBI/IBPS/RBI/LIC/All other banking and insurance Exams)

(31 – 35)

Club => A B C Total


Boys 90 80 186 356
Girls 100 124 155 379
Total 190 204 341 735

31. Ans. ©
Number of member in college A = 190

32. Ans. (A)


(100 - 80) ´100
Boys who joined B is less than Girls who joined A by = = 20%
100
33. Ans. (A)
In club B, Boys : Girls = 80 : 124 = 20:31

34. Ans. (B)


100 + 124
Average girls in club A & B = = 112
2
35. Ans. ©
Member in C – Member in B = 341 – 204 = 137

(36 – 40)

Hospital => P Q R S Total


Male 1300 800 1000 1400 4500
Female 900 1100 1200 1000 4200
Total 2200 1900 2200 2400 8700

36. Ans. (D)


Average of female nurses = 4200/4 = 1050

37. Ans. (A)


(1400 - 900) ´100
Male nurses in S is more than Female nurses in P by = = 55.55%
900
38. Ans. (B)
Male nurses in R : Female nurses in S = 1000:1000 = 1:1

39. Ans. ©
Total Nurses in S – Male Nurses in Q = 2400 - 800 = 1600

40. Ans. (D)


Highest number of nurses is in = Hospital S

PAGE h ps://t.me/studified h ps://instagram.com/aashisharorasocial?utm_medium=copy_link


Follow
190 Aashish Arora on: h ps://youtube.com/channel/UCYa4_JrOrf8R5Kz2uOtccXQ https://www.facebook.com/aashisharorasocial/
अ ास by Aashish Arora
(SBI/IBPS/RBI/LIC/All other banking and insurance Exams)

(41 – 45)

Day => Mon Tue Wed Thu Total


Boys 400 336 870 540 2146
Girls 520 464 580 620 2184
Total 920 800 1450 1160 4330

41. Ans. ©
On Thursday = Boys : Girls = 540 : 620 = 27:31

42. Ans. (B)


800 + 1450
Average of student on Tuesday & Wednesday = = 1125
2
43. Ans. (A)
Boys on Wednesday – Boys on Tuesday = 870 – 336 = 534

44. Ans. ©
(520 - 400) ´100
On Monday, Girls are more than by Boys by = = 30%
400
45. Ans. (A)
Total girls on Monday & Thursday = 520 + 620 = 1140

(46 – 50)

Year => 2017 2018 2019 Total


Park 200 280 220 700
Museums 150 140 250 540
Total 350 420 470 1240

46. Ans. ©
Average Number of Museum in 3 year = 540 / 3 = 180

47. Ans. (A)


(250 - 220) ´100
In 2019, Number of Park more than number of Museums by = = 12%
250
48. Ans. (A)
2018 (Museums) : 2019 (Park + Museums) = 140 : 470 = 14:47

49. Ans. (D)


2017 (Park + Museums) – 2019 (Park) = 350 – 220 = 130

50. Ans. (D)


In 2017, Park for Adult + Museums for other purpose = 200 × 0.6 + 150 × 0.7 = 225

PAGE h ps://t.me/studified h ps://instagram.com/aashisharorasocial?utm_medium=copy_link


Follow
191 Aashish Arora on: h ps://youtube.com/channel/UCYa4_JrOrf8R5Kz2uOtccXQ https://www.facebook.com/aashisharorasocial/
अ ास by Aashish Arora
(SBI/IBPS/RBI/LIC/All other banking and insurance Exams)

(51 – 55)

City => A B C D Total


Male 375 595 680 354 2004
Female 450 705 765 476 2396
Total 825 1300 1445 830 4400

51. Ans. (B)


Male Toilets in City A = 375

52. Ans. ©
Total Toilet in City C = 1445

53. Ans. (B)


Female Toilet in city B – Female Toilets in City D = 705 – 476 = 229

55. Ans. (A)


705 ´100
% of Female toilets in City B with all Toilets = = 16%
4400
55. Ans. (B)
Number of Male Toilets in all cities = 2004

(56 – 60)
Train A Train B Total
Male Female Male Female
Passenger 150 100 200 100
Ticket Cost (in 50 40 70 49
Rs)
Revenue (in Rs) 7500 4000 14000 4900 30400

56. Ans. (B)


In A & B, Male passenger together more than by female passenger by
= (150 + 200) – (100 + 100) = 150

57. Ans. (D)


In train A, Money collected from male passenger more than that of female passenger by
= 7500 – 4000 = 3500 Rs

58. Ans. (A)


Money collected from train B – Money collected from train A
(14000 + 4900) – (7500 + 4000) = 7400 Rs

59. Ans. (B)


Ticket price of Female in train B is more than that of female in train A by = 49 – 40 = 9 Rs

60. Ans. (A)

PAGE h ps://t.me/studified h ps://instagram.com/aashisharorasocial?utm_medium=copy_link


Follow
192 Aashish Arora on: h ps://youtube.com/channel/UCYa4_JrOrf8R5Kz2uOtccXQ https://www.facebook.com/aashisharorasocial/
अ ास by Aashish Arora
(SBI/IBPS/RBI/LIC/All other banking and insurance Exams)

% of money collected from males - % of money collected from females


(7500 + 14000) ´100 (4000 + 4900) ´100 21500 8900 12600
= - = - = = 41.44% = 41%
30400 30400 304 304 304

(61 – 65)

Monday Tuesday Wednesday Thursday Total


Balls A = 180 B = 225 E = 246 369 1020
Bats 230 C = 267 D = 369 F = 131 997
Total 410 492 615 500 2017

61. Ans. (A)


E : A = 246:180 = 41:30

62. Ans. (B)


B = 225

63. Ans. ©
C = 267

64. Ans. (D)


F = 131

65. Ans. ©
(369 - 230) ´100
D is more than Bats sold on Monday by = = 60.43% = 60%
230
(66 – 70)
Subject History Geography Zoology Total
2020 6000 10000 4500 20500
2021 5500 10500 8000 24000
Total 11500 20500 12500 44500

66. Ans. (A)


Total question in 2021 – Total question of History = 24000 – 11500 = 12500

67. Ans. (A)


Questions of History in 2020 is less than Questions of Zoology in 2021 by
(8000 - 6000) ´100
= = 25%
8000
68. Ans. (E)
20500
Average number of question of Geography = = 10250
2

PAGE h ps://t.me/studified h ps://instagram.com/aashisharorasocial?utm_medium=copy_link


Follow
193 Aashish Arora on: h ps://youtube.com/channel/UCYa4_JrOrf8R5Kz2uOtccXQ https://www.facebook.com/aashisharorasocial/
अ ास by Aashish Arora
(SBI/IBPS/RBI/LIC/All other banking and insurance Exams)

69. Ans. (D)


In 2020, Total Number of question of History & Geography = 6000 + 10000 = 16000
Question in Hindi language = 0.25 × 16000 = 4000
Geography question in Hindi language = 4000 – 1850 = 2150

70. Ans. (D)


In 2021, Question of Geography: Question of History = 10500:5500 = 21:11

(71 – 75)

In State X:-
Subject English Mathematics Physics Total
Male 180 220 150 550
Female 100 90 200 390
Total 280 310 350 940

In State Y:-

Subject English Mathematics Physics Total


Male 210 150 180 540
Female 170 200 180 550
Total 380 350 360 1090

71. Ans. (B)


English Teachers in State X : Physics Teachers in State Y = 280:360 = 7:9

72. Ans. (A)


Female mathematics teachers in State X is less than female physics teachers in state Y is
(180 - 90) ´100
= = 50%
180
73. Ans. ©
Male English teachers in both states – Mathematics teachers in State Y
= (180 + 210) – 350 = 40

74. Ans. (A)


Average of Male physics teachers in state Y & Male Mathematics teachers in state X
180 + 220
= = 200
2
75. Ans. (B)
Female English Teachers in State Z = 2 × 150 - 170 = 130
Female English Teachers in State Z – Female English Teachers in State X = 130 – 100 =
30

(76 – 80)

PAGE h ps://t.me/studified h ps://instagram.com/aashisharorasocial?utm_medium=copy_link


Follow
195 Aashish Arora on: h ps://youtube.com/channel/UCYa4_JrOrf8R5Kz2uOtccXQ https://www.facebook.com/aashisharorasocial/
अ ास by Aashish Arora
(SBI/IBPS/RBI/LIC/All other banking and insurance Exams)

Days Monday Tuesday Wednesday Total


Mangoes 540 510 450 1500
Oranges 400 432 378 1210
Total 940 942 828 2710
76. Ans. (D)
Orange sold on Wednesday is less than Mangoes sold on Monday by
(540 - 378) ´100
= = 30%
540

77. Ans. (B)


Average orange sold in 3 days = 1210/3 = 403

78. Ans. (A)


Mango sold on Monday:Orange sold on Tuesday & Wednesday = 540:(432 + 378) = 2:3

79. Ans. (A)


Mango sold on Thursday = 1.3 × 450 = 585
Total Fruits sold on Thursday = 432 + 668 = 1100
Orange sold on Thursday = 1100 - 585 = 515

80. Ans. (B)


Fruits sold on next Monday = 0.6 × 940 = 564
Orange sold on next Monday = 0.25 × 564 = 141

(81 – 85)

Shop => P Q R S Total


AC 49 60 66 36 211
Coolers 50 84 75 133 342
Total 99 144 141 169 553

81. Ans. (B)


Total Coolers sold – Total AC sold = 342 – 211 = 131

82. Ans. ©
Total AC & Coolers sold by shop T = 1.15 × 60 + 5/7 × 84 = 129

83. Ans. (B)


Total AC & Coolers sold in March by R = 141 × 4/3 = 188

84. Ans. (A)


(75 - 60) ´100
Coolers sold by R are more than AC sold by Q by = = 25%
60
85. Ans. (B)
Revenue Earned by Shop S = 50000 × 36 + 5000 × 133 = 24,65,000 Rs

PAGE h ps://t.me/studified h ps://instagram.com/aashisharorasocial?utm_medium=copy_link


Follow
195 Aashish Arora on: h ps://youtube.com/channel/UCYa4_JrOrf8R5Kz2uOtccXQ https://www.facebook.com/aashisharorasocial/
अ ास by Aashish Arora
(SBI/IBPS/RBI/LIC/All other banking and insurance Exams)

(86 – 90)

School => P Q R S Total


Boys 300 400 600 750 2050
Girls 450 550 750 1000 2750
Total 750 950 1350 1750 4800

86. Ans. (B)


Girls – Boys = 2750 – 2050 = 700

87. Ans. (B)


In School S, Boys:Girls = 750:1000 = 3:4

88. Ans. (A)


When 100 New boys admitted in R, Total Boys in School R = 600 + 100 = 700
(750 - 700) ´100
In School R, Boys are less than Girls by = = 6.66%
750
89. Ans. (D)
Girls in School R & S – (Girls in school Q + Boys in School R)
(750 + 1000) – (550 + 600) = 600

90. Ans. (B)


Student Present in School R = 1350 × 0.8 = 1080
Student Present in School Q = 950 × 0.96 = 912
912 ´100
% of present student in school Q compared to school R = = 84.44%
1080
(21 – 25)
Test Series provided by A, SSC = 4x, Banking = (67 - 5x)
Test Series provided by B, SSC = (56 - 4x), Banking = 5x
56 - 4x 6
=
67 - 5x 7
7(56 -4x) = 6(67 – 5x)
392 - 28x = 402 - 30x
2x = 10
x=5

91. Ans. ©
SSC by A:Banking by B = 20:25 = 4:5

92. Ans. (D)


Total Test Series Provided by A = 20 + 42 = 62

PAGE h ps://t.me/studified h ps://instagram.com/aashisharorasocial?utm_medium=copy_link


Follow
196 Aashish Arora on: h ps://youtube.com/channel/UCYa4_JrOrf8R5Kz2uOtccXQ https://www.facebook.com/aashisharorasocial/
अ ास by Aashish Arora
(SBI/IBPS/RBI/LIC/All other banking and insurance Exams)

93. Ans. (D)


Banking by A - SSC by B = 42 - 36 = 6

94. Ans. (B)


(42 – 20)×100/20 = 110%

95. Ans. (A)


SSC test series by B 36 12
= =
Total test series by both institute (56 + 67) 41

(96 – 100)

96. Ans. (E)


Maximum Employees in = Company C

97. Ans. ©
Employee in (C + D) – Employee in A = (8000 + 4000) – 6000 = 6000

98. Ans. (A)


Female in C:Female in B = (0.4 × 8000):(0.25 × 7200) = 3200:1800 = 16:9

99. Ans. ©
Employee in F = 0.6 × (8000 + 5400) = 8040
(6000 + 8040)
Average employee in Company A & F = = 7020
2
100. Ans. (B)
4000 ´ 7
Female in D = = 1750
16
% of Female employee in D is less than Total employee in E by
(5400 - 1750) ´100
= = 67.59% = 68%
5400
(101 – 105)

101. Ans. ©
In 1993, Profit Ratio = 5:3:4,
15000 ´ 3
Profit of Bikram = = 3750 Rs.
12

PAGE h ps://t.me/studified h ps://instagram.com/aashisharorasocial?utm_medium=copy_link


Follow
197 Aashish Arora on: h ps://youtube.com/channel/UCYa4_JrOrf8R5Kz2uOtccXQ https://www.facebook.com/aashisharorasocial/
अ ास by Aashish Arora
(SBI/IBPS/RBI/LIC/All other banking and insurance Exams)

102. Ans. (A)


In 1995, Profit Ratio = 126:112:168 = 9:8:12 = (A × 24):(B × 16):(64000 × x)
9:8 = (A × 24):(B × 16)
A:B = 3:4
A = 24000, B = 32000
(32000 × 16):(64000 × x) = 2:3
Chandan invest for x = 12 month

103. Ans. ©
7700 ´16
Arjun’s profit in, 1991 = = 11200 Rs,
11
8800 ´ 24
1992 = = 19200 Rs.
11

Ratio of profit = 11200:19200 = 7:12

104. Ans. (B)


4000 ´ 7 3500 ´ 3
In 1994, Profit of Chandan = = 3500, in 1995 = = 2625
8 4
105. Ans. (B)
In 1996,
Profit of Bikram = 0.08 × 30000 = 2400 Rs, Profit of Chandan = 0.1 × 22000 = 2200 Rs
Ratio (Chanda:Bikram) = 2200:2400 = 11:12

(106 – 110)
Bajra growen by Rajat = 91 kg, Rice = 91 + 57 = 148 kg
Rice growen by Rohit = 308 – 148 = 160 kg, Wheat = 160×3/4 = 120 kg
Bajra growen by Rohit = x kg, Wheat growen by Rajat = 2x kg
Sorghum growen by Rohit = Jawear growen by Rajat = 2x – 92 kg
120 + 160 + 92 + x + 2x – 92 = 535
3x = 255
x = 85 kg

106. 78 + 107 = 185 kg Ans. (B)

107. 170 kg Ans. (D)

108. 160 – 148 = 12 kg Ans. (A)

109. 170 + 148 + 78 + 91 + 107 = 594 kg Ans. ©

110. 92 + 78 = 170 kg Ans. (B)

(111 – 115)

PAGE h ps://t.me/studified h ps://instagram.com/aashisharorasocial?utm_medium=copy_link


Follow
198 Aashish Arora on: h ps://youtube.com/channel/UCYa4_JrOrf8R5Kz2uOtccXQ https://www.facebook.com/aashisharorasocial/
अ ास by Aashish Arora
(SBI/IBPS/RBI/LIC/All other banking and insurance Exams)

5x + (900 – 4x) + (1200 – 3x) = 1800


5x – 7x + 2100 = 1800
2x = 300
x = 150

111. Average of prtvate sector = 1800/3 = 600 Ans. (C)

112. Prtvate sector tn S = 300 + 600 = 900


900/600 = 3/2 = 3:2 Ans. (B)

113. (1050 + 450) - 1200 = 300 Ans. (A)

114. (750 – 600)×100/750 = 20% Ans. (D)

115. In T,
Government Sector = 0.8×1050 = 840,
Prtvate sector = x
(x + 840)×0.25 = 840
x = 2520 Ans. (D)

(116 – 120)

116. Ans. (E)


Boys in class A = 420

117. Ans. (A)


In class C, Boys:Girls = 360:240 = 3:2

118. Ans. (D)


(300 - 200) ´100
Girls in class B is less than Girls in class A by = = 33(1/3)%
300
119. Ans. ©
(720 + 600)
Average student in Class A & C = = 660
2
120 Ans. (B)
Total student in class D = 1.4 × 480 = 672,
360 ´19
Boys in class D = = 456,
15

PAGE h ps://t.me/studified h ps://instagram.com/aashisharorasocial?utm_medium=copy_link


Follow
199 Aashish Arora on: h ps://youtube.com/channel/UCYa4_JrOrf8R5Kz2uOtccXQ https://www.facebook.com/aashisharorasocial/
अ ास by Aashish Arora
(SBI/IBPS/RBI/LIC/All other banking and insurance Exams)

Girls = 672 – 456 = 216


Girls in class D – girls in Class B = 216 – 200 = 16

(121 – 125)

121. Ans. (B)


Male in NGO S:Female in NGO P = 540:660 = 9:11

122. Ans. (D)


425 ´100
% of Males in NGO R = = 65.38%
650
123. Ans. (A)
Male in NGO S – Female in NGO R = 540 – 225 = 315

124. Ans. ©
660 + 640 + 425
Average of Female (P + Q) & Male in R = = 575
3
125. Ans. (B)
Total Employees in NGO Q, S & T = 1060 + 990 + 1.35 × 1100 = 3535

(126 – 130)

126. Ans. (B)


3500 ´100
% of boys from Bangalore with students from Bellary = = 83.33 = 83%
4200
127. Ans. (D)
Students from Mysore:Students from Bangalore = 6000:6500 = 12:13

128. Ans. (A)


(420 + 740 + 2200 + 3360)
Average of Boys from Mysore, Tumkur, Bellary & Kolar = = 2625
4
129. Ans. ©
Number of Girls is maximum is = Bangalore

130. Ans. (D)


Girls (Bellary + Banglore) – Girls (Tumkur + Kolar) = (2000 + 3000) – (888 + 2312) = 1800

PAGE h ps://t.me/studified h ps://instagram.com/aashisharorasocial?utm_medium=copy_link


Follow
200 Aashish Arora on: h ps://youtube.com/channel/UCYa4_JrOrf8R5Kz2uOtccXQ https://www.facebook.com/aashisharorasocial/
अ ास by Aashish Arora
(SBI/IBPS/RBI/LIC/All other banking and insurance Exams)

(131 – 135)

131. Ans. (B)

Average of people who shop from Delhi sites = 10200


= 3400
132. Ans. (A) 3
Flipkart in Mumbai:Myntra in Kolkata = 3000:4000 = 3:4

133. Ans. (B)


Number of female, who shop from Amazon in Delhi = x
Number of female = 0.28x
x + 0.28x = 3200
1.28x = 3200
x = 2500
Number of male = 0.28 × 2500 = 700

134. Ans. ©
People who shop from Myntra in Mumbai are less the people who shop from Amazon in

Kolkata by = (4000 - 1000) ´100


= 75%
135. Ans. (D) 400
Kolkata (Flipkart + Amazon) – Delhi (Myntra + Amazon)
= (4000 + 4000) – (3200 + 2000) = 2800

136. Overall marks of Anish = 100×0.65 = 65


In English = 30×0.7 = 21
In English & Science = 65×0.8 = 52
In Science = 52 – 21 = 31 (Ans.)

137. Total marks of Ramesh = 100×0.75 = 75 (Ans.)

138. Total marks of Ramesh = 75


In Math = 35×0.6 = 21, In Science = 21×4/3 = 28,
In English = 75 – (21 + 28) = 26
Science – English = 28 – 26 = 2 (Ans.)

139. Total Marks of Anish = 65


Passing Marks = 40
Anish passed by = 65 – 40 = 25 Marks (Ans.)

140. Total Marks of Anish & Ramesh


In Math = 21 + 13 = 34
In English = 26 + 21 = 47
Difference = 47 – 34 = 13 (Ans.)

PAGE h ps://t.me/studified h ps://instagram.com/aashisharorasocial?utm_medium=copy_link


Follow
201 Aashish Arora on: h ps://youtube.com/channel/UCYa4_JrOrf8R5Kz2uOtccXQ https://www.facebook.com/aashisharorasocial/
अ ास by Aashish Arora
(SBI/IBPS/RBI/LIC/All other banking and insurance Exams)

7 Data Interpretation (Graph Based)

The radar graph given below shows total number of Apps (Gaming App and Utility App)
developed by five different companies and the ratio of number of Gaming App to Utility Apps
developed by the companies.
नीचे िदए गए रडार ाफ पां च अलग-अलग कंपिनयो ं ारा िवकिसत ऐप (गेिमंग ऐप और यूिटिलटी ऐप) की कुल
सं ा और कंपिनयो ं ारा िवकिसत गेिमंग ऐप और यूिटिलटी ऐप की सं ा के अनुपात को दशाता है ।

Total Apps Developed

Company P
1030
1020
1010
1000
990
980
Company T 970 Company Q
960
950
940

Company S Company R

Company Gaming App : U lity App

P 9:5

Q 11:6

R 16:9

S 3:2

T 7:3

PAGE h ps://t.me/studified h ps://instagram.com/aashisharorasocial?utm_medium=copy_link


Follow
202 Aashish Arora on: h ps://youtube.com/channel/UCYa4_JrOrf8R5Kz2uOtccXQ https://www.facebook.com/aashisharorasocial/
अ ास by Aashish Arora
(SBI/IBPS/RBI/LIC/All other banking and insurance Exams)

1. What is the ratio of number of Gaming Apps developed by Company P to number of Utility
Apps developed by Company Q?
कंपनी P ारा िवकिसत गेिमंग ए की सं ा और कंपनी Q ारा िवकिसत यूिटिलटी ए की सं ा का
अनुपात ा है ?
A. 5:4 B. 7:2 C. 6:5
D. 7:4 E. None of these

2. What is the average of number of Gaming Apps developed by Company Q and Company
R?
कंपनी Q और कंपनी R ारा िवकिसत गेिमंग ऐ की सं ा का औसत ा है ?
A. 640 B. 650 C. 660
D. 670 E. None of these

3. Number of Utility Apps developed by Company S is how much percent more/less than the
number of Utility Apps developed by Company R?
कंपनी एस ारा िवकिसत यूिटिलटी एप की सं ा कंपनी आर ारा िवकिसत यूिटिलटी एप की सं ा से
िकतने ितशत अिधक / कम है ?
A. 5.5 B. 6.6 C. 7.7
D. 8.8 E. 9.9

4. What is the difference between number of Gaming App and Utility Apps developed by
Company T?
कंपनी टी ारा िवकिसत गेिमंग ऐप और यूिटिलटी ऐप की सं ाम ा अंतर है ?
A. 412 B. 414 C. 416
D. 418 E. None of these

5. What is the total number Utility App developed by Company P , Q, R and Company S
together?
कंपनी पी, ू, आर और कंपनी एस ारा एक साथ िवकिसत कुल उपयोिगता ऐप ा है ?
A. 1548 B. 1424 C. 1458
D. 1546 E. None of these

Two version of a learning app is available- one is free and the other is premium. The graph/data
is about the number of downloads of these two versions of the learning app by students in five
months.
एक लिनग ऐप के दो सं रण उपल ह- एक मु है और दू सरा ीिमयम है । डे टा, छा ो ं ारा इन दो सं रणो ं के
डाउनलोड की सं ा के बारे म है ।

PAGE h ps://t.me/studified h ps://instagram.com/aashisharorasocial?utm_medium=copy_link


Follow
203 Aashish Arora on: h ps://youtube.com/channel/UCYa4_JrOrf8R5Kz2uOtccXQ https://www.facebook.com/aashisharorasocial/
अ ास by Aashish Arora
(SBI/IBPS/RBI/LIC/All other banking and insurance Exams)

Free Version Download Premium Version Download


5600 250
240 240
5200
230
220
4800
210
205
4400 200
190 190
4000
180 180
175
170
3600
160
3200 150
January February March April May

6. Total number of downloads in May is how much percent(approx.) more/less than April?
मई म डाउनलोड की कुल सं ा अ ैल से िकतना ितशत अिधक / कम है ?
A. 16 B. 22 C. 28
D. 35 E. 40

7. Total number of Premium downloads in January, March and May together is how much
percent (approx.) less than Free downloads in March?
जनवरी, माच और मई म ीिमयम डाउनलोड की कुल सं ा माच म ी डाउनलोड की तुलना म िकतने
ितशत कम है ?
A. 74 B. 78 C. 82
D. 86 E. 90

8. What is the ratio of total number of downloads in February and March, respectively?
फरवरी और माच म मशः डाउनलोड की कुल सं ा का अनुपात ा है ?
A. 995:772 B. 996:755 C. 994:767
D. 998:715 E. None of these

9. What is the average number of Premium Version downloads in January, May and March
together?
January, May और March म एक साथ ीिमयम सं रण डाउनलोड की औसत सं ा ा है ?
A. 206.6 B. 208.8 C. 302.2
D. 312.4 E. None of these

10. What is the difference between number of Free downloads in February and April and
Premium Downloads in February and April?

PAGE h ps://t.me/studified h ps://instagram.com/aashisharorasocial?utm_medium=copy_link


Follow
204 Aashish Arora on: h ps://youtube.com/channel/UCYa4_JrOrf8R5Kz2uOtccXQ https://www.facebook.com/aashisharorasocial/
अ ास by Aashish Arora
(SBI/IBPS/RBI/LIC/All other banking and insurance Exams)

फरवरी और अ ैल म मु डाउनलोड और फरवरी और अ ैल म ीिमयम डाउनलोड म ा अंतर है ?


A. 9230 B. 9430 C. 9530
D. 9630 E. None of these

In five exams different number of questions were asked in which few were moderate-level and
few were hard. The radar graph given below shows the number of Moderate Questions and Hard
Questions in five different exams.
पां च परी ाओं म िविभ पूछे गए थे िजनम कुछ म म र के थे और कुछ किठन थे। नीचे िदए गए रडार ाफ
पां च अलग-अलग परी ाओं म मॉडरे ट और किठन की सं ा दशाते ह।

11. What is the ratio of total number of questions in Exam P to the same in Exam S?
परी ा P और परी ा S म कुल ो ं का अनुपात ा है ?
A. 8:7 B. 9:8 C. 10:9
D. 11:10 E. None of these/इनम से कोई नही ं

12. Total number of questions in Exam Q are how much percent more/less than the same in
Exam R?
ए ाम ू म कुल ए ाम आर से िकतने ितशत अिधक / कम ह?
A. 3.125 B. 6.25 C. 9.75
D. 2.125 E. None of these/इनम से कोई नही ं

13. What is the average number of Moderate Question in Exam Q, R and Exam T together?
Exam Q, R and Exam T म एक साथ मॉडरे ट े न के ो ं की औसत सं ा ा है ?
A. 90 B. 100 C. 85

PAGE h ps://t.me/studified h ps://instagram.com/aashisharorasocial?utm_medium=copy_link


Follow
205 Aashish Arora on: h ps://youtube.com/channel/UCYa4_JrOrf8R5Kz2uOtccXQ https://www.facebook.com/aashisharorasocial/
अ ास by Aashish Arora
(SBI/IBPS/RBI/LIC/All other banking and insurance Exams)

D. 110 E. None of these/इनम से कोई नही ं

14. What is the total number of Hard Question in Exam P, Exam Q and S together?
ए ाम पी, ए ाम ू और एस म एक साथ कुल िकतने किठन ह?
A. 265 B. 270 C. 275
D. 280 E. None of these/इनम से कोई नही ं

15. Total number of questions in another Exam F is 80% more than total questions of Exam Q. If
the ratio of number of Moderate Question to Hard Question in Exam F is 5:4, then find the
number of Moderate Question in Exam F.
िकसी अ परी ा F म कुल ो ं की सं ा, परी ा Q के कुल ो ं से 80% अिधक है । यिद परी ा F म मॉडरे ट
और किठन की सं ा का अनुपात 5: 4 है , तो परी ा F म मॉडरे ट की सं ा ात कर।
A. 115 B. 165 C. 125
D. 130 E. None of these/इनम से कोई नही ं

Transport Department in Delhi have five offices which issue two types of license i.e. HMV (Heavy
Motor Vehicle) and LMV (Light Motor Vehicle) License. The line graph given below shows the
number of HMV license issued, and the percentage of LMV license issued by the respective
offices in the month of February.
िद ी म प रवहन िवभाग के पाँ च कायालय ह जो दो कार के लाइसस जारी करते ह अथात् HMV (है वी मोटर
ीकल) लाइसस और LMV (लाइट मोटर ीकल) लाइसस। नीचे िदए गए लाइन ाफ एचएमवी लाइसस जारी
करने की सं ा और संबंिधत कायालयो ं ारा जारी िकए गए एलएमवी लाइसस का ितशत दशाता है ।

85 1000
875 900
75 810 832 800
65 605 700
600
55 500
400
45
290 300
35 200
100
25 0
Office P Office Q Office R Office S Office T
% of LMV License issued Number of HMV License issued

PAGE h ps://t.me/studified h ps://instagram.com/aashisharorasocial?utm_medium=copy_link


Follow
206 Aashish Arora on: h ps://youtube.com/channel/UCYa4_JrOrf8R5Kz2uOtccXQ https://www.facebook.com/aashisharorasocial/
अ ास by Aashish Arora
(SBI/IBPS/RBI/LIC/All other banking and insurance Exams)

16. What is the number of LMV license issued by Office P?


Office P ारा जारी LMV लाइसस की सं ा ा है ?
A. 370 B. 375 C. 385
D. 295 E. None of these/इनम से कोई नही ं

17. Total number of license issued by Office S is how much percent more/less than the same
issued by Office Q?
Office S ारा जारी िकए गए लाइसस की कुल सं ा Office Q से िकतने ितशत अिधक / कम है ?
A. 7.4 B. 8.6 C. 9.2
D. 12/8 E. None of these/इनम से कोई नही ं

18. What is the difference between number of LMV license issued by Office R and that of Office
S?
Office R और Office S ारा जारी िकए गए LMV लाइसस की सं ा म ा अंतर है ?
A. 645 B. 635 C. 675
D. 665 E. None of these/इनम से कोई नही ं

19. What is the average number of HMV license issued by all five Offices together?
सभी पां च कायालयो ं ारा एक साथ जारी िकए गए एचएमवी लाइसस की औसत सं ा ा है ?
A. 681.4 B. 682.4 C. 692.4
D. 702.4 E. None of these/इनम से कोई नही ं

20. If Office T later rejects only 25% of LMV license issued by it, then find the number of total
license that are not rejected by Office T in the month of February?
यिद Office T बाद म उसके ारा जारी िकए गए LMV लाइसस के 25% को अ ीकार कर दे ता है , तो
लाइसस की सं ा ात कर जो February म Office T से अ ीकृत नही ं ह?
A. 1116 B. 1182 C. 1168
D. 1142 E. None of these/इनम से कोई नही ं

The given table shows the percentage of employees posted in rural area, number of employees
posted in urban area and total number of employees (urban + rural) , in five different banks. Find
the missing data and answer accordingly.
दी गई तािलका म ामीण े म तैनात कमचा रयो ं का ितशत, शहरी े म तैनात कमचा रयो ं की सं ा और
कमचा रयो ं की सं ा (शहरी + ामीण), पां च अलग-अलग बको ं म दशाया गया है । डे टा का पता लगाएं और
तदनुसार जवाब द।

PAGE h ps://t.me/studified h ps://instagram.com/aashisharorasocial?utm_medium=copy_link


Follow
207 Aashish Arora on: h ps://youtube.com/channel/UCYa4_JrOrf8R5Kz2uOtccXQ https://www.facebook.com/aashisharorasocial/
अ ास by Aashish Arora
(SBI/IBPS/RBI/LIC/All other banking and insurance Exams)

Total Employees Percentage of Employees Number of Employees


posted in Rural Area posted in Urban Area

Bank P 19500
Bank Q 44%
Bank R 16000 7680
Bank S 36% 15680
Bank T 18000

21. If the ratio of total number of employees in bank T and Q is 9:11, respectively then find the
number of employees posted in urban area in bank Q.
यिद बक T और Q म कमचा रयो ं की कुल सं ा का अनुपात मशः 9:11 है , उसके बाद बक Q म शहरी े
म तैनात कमचा रयो ं की सं ा ात कीिजए।
A. 12350 B. 12330 C. 12320
D. 12310 E. None of these

22. If the number of employees posted in rural area in bank T is 6260 less than that of same in R,
then find the number of employees posted in urban area in bank T.
यिद बक T म ामीण े म तैनात कमचा रयो ं की सं ा R की तुलना म 6260 कम है , तो बक T म शहरी े म
तैनात कमचा रयो ं की सं ा ात कीिजए।
A. 15940 B. 15950 C. 15960
D. 15970 E. None of these

23. If the ratio of the number of employees posted in urban area in banks S and P is 224:117,
respectively then find the percentage of employees posted in rural area out of total number
of employees in bank P.
यिद बक एस और पी म शहरी े म तैनात कमचा रयो ं की सं ा का अनुपात मशः 224: 117 है , तो बक पी
म कुल कमचा रयो ं की सं ा से बाहर ामीण े म तैनात कमचा रयो ं का ितशत ात कर।
A. 64 B. 67 C. 58
D. 73 E. None of these

24. Find the ratio of the number of employees posted in rural area in bank R to total number of
employees in bank T.
बक R म ामीण े म तैनात कमचा रयो ं की सं ा और बक T म कमचा रयो ं की कुल सं ा का अनुपात ात
कीिजए।
A. 102:223 B. 101:225 C. 104:225
D. 103:223 E. None of these

25. If the total number of employees in bank T is (100/11)% more than that in Q and the total
number of employees in bank S is 8500 more than that in R, then find the average of total
number of employees in all 5 banks.

PAGE h ps://t.me/studified h ps://instagram.com/aashisharorasocial?utm_medium=copy_link


Follow
208 Aashish Arora on: h ps://youtube.com/channel/UCYa4_JrOrf8R5Kz2uOtccXQ https://www.facebook.com/aashisharorasocial/
अ ास by Aashish Arora
(SBI/IBPS/RBI/LIC/All other banking and insurance Exams)

यिद बक T म कमचा रयो ं की कुल सं ा Q से 100/11 % अिधक है और बक S म कमचा रयो ं की कुल


सं ा R से 8500 अिधक है , तो सभी 5 बको ं म कमचा रयो ं की कुल सं ा का औसत ात कर।
A. 19000 B. 18900 C. 21000
D. 22000 E. None of these

A student filed an RTI to find out vacant seats in five government departments of a country. The
first pie chart graph shows the percent distribution of total number of seats and the second pie
chart shows the percent distribution of total number of vacant seats, in five different
departments. 200 seats are vacant in Department of Railway.
एक छा ने दे श के पां च सरकारी िवभागो ं म खाली सीटो ं का पता लगाने के िलए एक आरटीआई दायर की। पहला
पाई चाट ाफ कुल सीटो ं की सं ा का ितशत िवतरण दशाता है और दू सरा पाई चाट पां च अलग-अलग िवभागो ं
म र सीटो ं की कुल सं ा का ितशत िवतरण दशाता है । रे लवे िवभाग म 200 सीट खाली ह।

PAGE h ps://t.me/studified h ps://instagram.com/aashisharorasocial?utm_medium=copy_link


Follow
209 Aashish Arora on: h ps://youtube.com/channel/UCYa4_JrOrf8R5Kz2uOtccXQ https://www.facebook.com/aashisharorasocial/
अ ास by Aashish Arora
(SBI/IBPS/RBI/LIC/All other banking and insurance Exams)

26. The number of non-vacant seats in Department of Energy is how much percent more/less
than the total number of seats in Postal Department?
ऊजा िवभाग म गैर- र सीटो ं की सं ा डाक िवभाग की कुल सीटो ं की तुलना म िकतने ितशत अिधक /
कम है ?
A. 3.5 B. 6.66 C. 8
D. 12.5 E. None of these

27. Find the ratio of the number of non-vacant seats in Department of Railway to the number
of vacant seats in Department of Textiles.
रे ल िवभाग म गैर- र सीटो ं की सं ा और कपड़ा िवभाग म र सीटो ं की सं ा का अनुपात ात
कीिजए।
A. 91:128 B. 93:128 C. 94:127
D. 95:127 E. None of these

28. Find the average of number of vacant seats in Postal Department, Department of Railway
and Department of Energy.
डाक िवभाग, रे लवे िवभाग और ऊजा िवभाग म र सीटो ं की सं ा का औसत ात कीिजए।
A. 138.6 B. 140.5 C. 142.5
D. 148.2 E. None of these

29. Find the difference between number of non-vacant seats in Postal Department and
Department of Textiles.
डाक िवभाग और कपड़ा िवभाग म गैर- र सीटो ं की सं ा के बीच अंतर का पता लगाएं ।
A. 201 B. 202 C. 203
D. 204 E. None of these

30. 30% of seats in railways are exclusive for women. Find the number of seats which are
exclusive to women.
रे लवे म 30% सीट मिहलाओं के िलए ख़ास ह। उन सीटो ं की सं ा ात कीिजए जो मिहलाओं के िलए ह।
A. 113 B. 114 C. 117
D. 119 E. None of these

5 bookstores sell only two types of books- Educational Books and Literature Books. The pie chart
given below shows the total number of educational books, and the line graph shows the
percentage of literature books in the respective bookstores.
5 बुक ोर केवल दो कार की पु क बेचते ह- शैि क पु क और सािह पु क। नीचे िदया गया पाई चाट
शैि क पु को ं की कुल सं ा को दशाता है और लाइन ाफ संबंिधत िकताबो ं की दु कानो ं म सािह पु को ं का
ितशत दशाता है ।

PAGE h ps://t.me/studified h ps://instagram.com/aashisharorasocial?utm_medium=copy_link


Follow
210 Aashish Arora on: h ps://youtube.com/channel/UCYa4_JrOrf8R5Kz2uOtccXQ https://www.facebook.com/aashisharorasocial/
अ ास by Aashish Arora
(SBI/IBPS/RBI/LIC/All other banking and insurance Exams)

80
76
70
67
60

50

40
37
30
24
20
16
10
Bookstore A Bookstore B Bookstore C Bookstore D Bookstore E

% of Literature Books Linear (% of Literature Books)

Number of Educa onal Books

693 Bookstore A
Bookstore B
1176
Bookstore C
Bookstore D
Bookstore E
684

396 192

31. What is total number of books in Bookstore D?


बुक ोर डी म पु को ं की कुल सं ा ा है ?
A. 1100 B. 900 C. 800
D. 1200 E. None of these/इनम से कोई नही ं

32. Total number of books in Bookstore A is how much percent more/less than the same in
Bookstore C?
बुक ोर ए म पु को ं की कुल सं ा, बुक ोर सी से िकतने ितशत अिधक / कम है ?

PAGE h ps://t.me/studified h ps://instagram.com/aashisharorasocial?utm_medium=copy_link


Follow
211 Aashish Arora on: h ps://youtube.com/channel/UCYa4_JrOrf8R5Kz2uOtccXQ https://www.facebook.com/aashisharorasocial/
अ ास by Aashish Arora
(SBI/IBPS/RBI/LIC/All other banking and insurance Exams)

A. 30 B. 37.5 C. 40
D. 42.5 E. None of these/इनम से कोई नही ं

33. What is the ratio of literature books in Bookstore C to the same in Bookstore E?
बुक ोर सी और बुक ोर ई म सािह पु को ं की अनुपात ा है ?
A. 11:9 B. 12:7 C. 13:8
D. 19:7 E. None of these/इनम से कोई नही ं

34. What is the average number of educational books in all five stores together?
एक साथ सभी पाँ च दु कानो ं म शैि क पु को ं की औसत सं ा ा है ?
A. 622.6 B. 624.8 C. 636.2
D. 628.2 E. None of these/इनम से कोई नही ं

35. What is the difference between number of literature books in Bookstore B and that of
Bookstore D?
बुक ोर बी और बुक ोर डी के सािह पु को ं की सं ा म ा अंतर है ?
A. 584 B. 586 C. 588
D. 600 E. None of these/इनम से कोई नही ं

The graph given below shows the percent distribution of total number of Agro-based business
firms in 5 different states, and the ratio of number of business firm of less than 5 crore turnover to
number of business firm of more than 5 crore turnover.
Note: Total number of business firms having exact 5 crore turnover is zero. Total Number of
Business Firms = 17000

नीचे िदया गया ाफ 5 िविभ रा ो ं म कृिष-आधा रत ापार फम की कुल सं ा का ितशत िवतरण, और 5


करोड़ से कम की वसाय फम की सं ा और, 5 करोड़ से अिधक के कारोबार वाली फम की सं ा का अनुपात
दशाता है ।
नोट: सटीक 5 करोड़ टनओवर वाली वसाियक फम की कुल सं ा शू है । ावसाियक फम की कुल सं ा
= 17000

Total business firms

12%
22% Bihar
Jharkhand
2%
28% U ar Pradesh
Madhya Pradesh
36% Rajasthan

PAGE h ps://t.me/studified h ps://instagram.com/aashisharorasocial?utm_medium=copy_link


Follow
212 Aashish Arora on: h ps://youtube.com/channel/UCYa4_JrOrf8R5Kz2uOtccXQ https://www.facebook.com/aashisharorasocial/
अ ास by Aashish Arora
(SBI/IBPS/RBI/LIC/All other banking and insurance Exams)

State Less than 5 crore turnover : More than 5 crore turnover


Bihar 11:9

Jharkhand 12:5

U ar Pradesh 8:7

Madhya Pradesh 5:3

Rajasthan 13:11

36. What is average number of business firm of more than 5 crore turnover in MP and
Rajasthan together?
एमपी और राज थान म ५ करोड़ से अिधक टनओवर वाली िबजनेस फम की औसत सं ा ा है ?
A. 1345 B. 1450 C. 1455
D. 1360 E. None of these/इनम से कोई नही ं

37. What is the difference between business firms of less than 5 crore turnover in Bihar and
Jharkhand?
िबहार और झारखंड म ५ करोड़ से कम टनओवर वाली ावसाियक कंपिनयो ं के बीच ा अंतर है ?
A. 1814 B. 1817 C. 1822
D. 1824 E. None of these/इनम से कोई नही ं

38. What is the ratio of number of business firm of less than 5 crore turnover in UP to number of
business firm of more than 5 crore turnover in Jharkhand?
यूपी म ५ करोड़ से कम टनओवर वाली िबजनेस फम की सं ा और झारखंड म ५ करोड़ से अिधक टनओवर
वाली िबजनेस फम की सं ा का अनुपात ा है ?
A. 816:25 B. 817:28 C. 818:22
D. 819:20 E. None of these/इनम से कोई नही ं

39. Number of business firm of more than 5 crore turnover in UP is how much percent
more/less than number of business firm of less than 5 crore turnover in MP?
यूपी म ५ करोड़ से अिधक टनओवर वाली िबजनेस फम की सं ा म म ५ करोड़ से कम टनओवर वाली
िबजनेस फम की सं ा से िकतने ितशत अिधक / कम है ?
A. 11 B. 22 C. 8
D. 4 E. None of these/इनम से कोई नही ं

40. If 45%, 15% and 5% business firm of MP, Bihar and UP respectively use only renewable
source of energy, then find total number of business firms in these states that use non-
renewable source of energy.
यिद मशः एमपी, िबहार और यूपी की ४५%, १५% और ५% वसाियक फम ऊजा के केवल नवीकरणीय
ोत का उपयोग करती है , तो इन रा ो ं म कुल वसाय फम की सं ा ात कर जो ऊजा के गैर-
नवीकरणीय ोत का उपयोग करती ह।

PAGE h ps://t.me/studified h ps://instagram.com/aashisharorasocial?utm_medium=copy_link


Follow
213 Aashish Arora on: h ps://youtube.com/channel/UCYa4_JrOrf8R5Kz2uOtccXQ https://www.facebook.com/aashisharorasocial/
अ ास by Aashish Arora
(SBI/IBPS/RBI/LIC/All other banking and insurance Exams)

A. 11609 B. 11611 C. 11617


D. 11623 E. None of these/इनम से कोई नही ं

The bar graph given below shows the number of patient infected by newer strain of covid and
patient infected by older strain of covid in six different cities. Total number of patients = Number
of patient infected by newer strain of covid + Number of patient infected by older strain of covid.
नीचे िदया गया बार ाफ छह अलग-अलग शहरो ं म कोिवड के नए े न से सं िमत मरीजो ं और पुराने े न से
सं िमत मरीजो ं की सं ा को दशाता है । रोिगयो ं की कुल सं ा = कोिवड के नए े न से सं िमत रोिगयो ं की
सं ा + कोिवड के पुराने े न से सं िमत रोिगयो ं की सं ा।

7.4

6.4

5.4

s 4.4
d
n
as
u 3.4
o
h
T
2.4
City P City Q City R City S City T City U

Pa ent infected by Newer Strain Pa ent infected by older Strain

41. What is ratio of total number of patients in City P to the same in City U?
िसटी पी और िसटी यू म रोिगयो ं की कुल सं ा का अनुपात ा है ?
A. 51:53 B. 50:57 C. 50:59
D. 51:55 E. None of these

42. Number of patient infected by older strain of covid in City S and T together is how much
percent more/less than number of patient infected by newer strain of covid in City R?
िसटी एस और टी म कोिवद के पुराने Strain से सं िमत रोगी की सं ा िसटी आर म कोिवद के नए Strain से
सं िमत रोगी की सं ा से िकतने ितशत अिधक / कम है ?
A. 13.5 B. 14.5 C. 15.5
D. 16.5 E. 17.5

43. What is the average number of patient infected by newer strain of covid in City P, S and T
together?

PAGE h ps://t.me/studified h ps://instagram.com/aashisharorasocial?utm_medium=copy_link


Follow
214 Aashish Arora on: h ps://youtube.com/channel/UCYa4_JrOrf8R5Kz2uOtccXQ https://www.facebook.com/aashisharorasocial/
अ ास by Aashish Arora
(SBI/IBPS/RBI/LIC/All other banking and insurance Exams)

िसटी पी, एस और टी म एक साथ कोिवद के नए Strain से सं िमत रोगी की औसत सं ा ा है ?


A. 5864.66 B. 5866.66 C. 5843.33
D. 5853.33 E. None of these

44. What is the difference between number of patient infected by older strain of covid in City P
and U together and number of patient infected by older strain of covid in City R alone?
िसटी पी और यू म एक साथ कोिवड के पुराने Strain से सं िमत रोगी की सं ा और अकेले िसटी आर म
कोिवद के पुराने Strain से सं िमत रोगी की सं ा के बीच अंतर ा है ?
A. 3600 B. 3800 C. 3200
D. 3400 E. None of these

45. Out of total number of patients from City R and S together, 46% are regular smokers too.
Find the number of patients in City R and S together who are not smokers.
िसटी आर और एस के कुल रोिगयो ं म से, 46% िनयिमत धू पान करने वाले भी ह। िसटी आर और एस म एक
साथ ऐसे रोिगयो ं की सं ा ात कीिजए जो धू पान करने वाले नही ं ह
A. 10682 B. 10686 C. 10690
D. 10692 E. None of these

Five students sat for five different examinations. The line graph shows total number of questions
asked in the examination in which the respective student sat. Note that the total questions asked
in exam = Question Attempted + Questions not attempted. The bar graph shows the value of
(Attempted question – Unattempted question) by each of the student.
पां च छा पां च अलग-अलग परी ाओं म बैठे। लाइन ाफ परी ा म पूछे गए ो ं की कुल सं ा िदखाता है िजसम
संबंिधत छा बैठे थे। ान द िक परी ा म पूछे गए कुल = का यास िकया गया + यास नही ं िकया
गया। बार ाफ ेक छा ारा (अटे िकए गए - अनअटे ेड ) के मू को दशाता है ।

308

288

268

248

228
Student Q Student S Student T

PAGE h ps://t.me/studified h ps://instagram.com/aashisharorasocial?utm_medium=copy_link


Follow
215 Aashish Arora on: h ps://youtube.com/channel/UCYa4_JrOrf8R5Kz2uOtccXQ https://www.facebook.com/aashisharorasocial/
अ ास by Aashish Arora
(SBI/IBPS/RBI/LIC/All other banking and insurance Exams)

Student T

Student S

Student R

Student P

0 10 20 30 40 50 60

A empted Ques on : Una empted Ques on


P 16:9
Q 7:3
R 3:2

46. Find the total number of questions asked in the examination in which P appeared.
उस परी ा म पूछे गए ो ं की कुल सं ा ात कर िजसम P था।
A. 150 B. 200 C. 230
D. 250 E. None of these

47. Ratio of total attempted to unattempted question by S is x:7. Find the value of x.
S ारा कुल अटे और अनअटे ेड का अनुपात x: 7 है । ए का मान ात कर।
A. 9 B. 8 C. 11
D. 12 E. None of these

48. Find the total number of questions asked in the examination in which R appeared.
उस परी ा म पूछे गए ो ं की कुल सं ा ात कर िजसम R था।
A. 220 B. 240 C. 260
D. 280 E. None of these
49. Ratio of total attempted to unattempted question by T is 11:Y. Find the value of y.
T ारा कुल अटे और अनअटे ेड का अनुपात 11:y है । y का मान ात कर।
A. 9 B. 8 C. 7
D. 6 E. None of these

PAGE h ps://t.me/studified h ps://instagram.com/aashisharorasocial?utm_medium=copy_link


Follow
216 Aashish Arora on: h ps://youtube.com/channel/UCYa4_JrOrf8R5Kz2uOtccXQ https://www.facebook.com/aashisharorasocial/
अ ास by Aashish Arora
(SBI/IBPS/RBI/LIC/All other banking and insurance Exams)

50. Find the difference of total attempted and unattempted question by Q.


Q ारा कुल अटे और अनअटे ेड का अंतर ात कीिजए।
A. 80 B. 100 C. 120
D. 140 E. None of these

The given line graph shows the total number of Students who sat for Unacademy Combat Tests
for receiving scholarship. The radar graph shows the number of Students who got the
scholarship.
दी गई रे खा का ाफ उन छा ो ं की कुल सं ा दशाता है जो छा वृि ा करने के िलए Unacademy Combat
टे के िलए बैठे थे। रडार ाफ उन छा ो ं की सं ा दशाता है िज छा वृि िमली थी।

2000

1900

1800

1700

1600
Test I Test II Test III Test IV Test V
Students who appeared in Unacademy Combat

Test I
900

850

Test V 800 Test II


Students who got
750
Scholarship

Test IV Test III

51. Find the ratio of the number of Students who got the scholarship in Test V to number of
Students who did not get the scholarship in Test II.
टे V म छा वृि पाने वाले छा ो ं की सं ा और टे II म छा वृि नही ं पाने वाले छा ो ं की सं ा का
अनुपात ात कीिजए।
A. 37:47 B. 36:49 C. 38:43
D. 39:49 E. None of these

PAGE h ps://t.me/studified h ps://instagram.com/aashisharorasocial?utm_medium=copy_link


Follow
217 Aashish Arora on: h ps://youtube.com/channel/UCYa4_JrOrf8R5Kz2uOtccXQ https://www.facebook.com/aashisharorasocial/
अ ास by Aashish Arora
(SBI/IBPS/RBI/LIC/All other banking and insurance Exams)

52. Out of total Students who appeared in Test III, 35% attempted the test in Hindi . If number
of Students who got the scholarship by attempting in Hindi is 15% of total number of
Students who got the scholarship, then find the number of Students who attempted in
Hindi who did not get the scholarship.
टे III म उप थत होने वाले कुल छा ो ं म से 35% ने िहं दी भाषा म परी ा का यास िकया। यिद िहं दी म
यास करके छा वृि पाने वाले छा ो ं की सं ा कुल छा ो ं की कुल सं ा का 15% है , िज छा वृि िमली
है , तो िहं दी म यास करने वाले छा ो ं की सं ा का पता लगाएं , िज छा वृि नही ं िमली।
A. 570 B. 580 C. 590
D. 610 E. None of these

53. The number of Students who got the scholarship in Test IV is how much percent more/less
than the number of Students who did not get the scholarship in Test I?
टे IV म छा वृि पाने वाले छा ो ं की सं ा टे I म छा वृि नही ं पाने वाले छा ो ं की सं ा से िकतने
ितशत अिधक / कम है ?
A. 8.2 B. 5.3 C. 3.3
D. 2.5 E. None of These

54. Find the average number of Students who did not got the scholarship in Test II and V.
टे II और V म छा वृि नही ं पाने वाले छा ो ं की औसत सं ा ात कीिजए।
A. 1010 B. 1050 C. 1020
D. 1030 E. None of these

55. Out of total number of Students who did not get the scholarship in Test I, II and IV together,
65% were preparing for Banks and rest preparing for SSC. Find the number of students
preparing for SSC who did not get the scholarship.
टे I, II और IV म छा वृि नही ं पाने वाले कुल छा ो ं म से, 65% बको ं की तैयारी कर रहे थे और बाकी
एसएससी की तैयारी कर रहे थे। SSC की तैयारी करने वाले छा ो ं की सं ा ात कर िज छा वृि नही ं
िमली।
A. 951 B. 953 C. 957
D. 959 E. None of these

The given bar graph shows the total number of districts (districts where curfew due to covid is
not imposed + districts where curfew is imposed) in five different states of a State. The radar
graph shows the percentage of districts where curfew is imposed in those states.
िदए गए बार ाफ एक दे श के पां च अलग-अलग रा ो ं म िजलो ं की कुल सं ा को दशाता है , (िजन िजलो ं म कोिवद
के कारण क ू नही ं लगाया गया है + वे िजले जहां क ू लगाया गया है ). रडार ाफ उन िजलो ं का ितशत दशाता है
जहां उन रा ों म क ू लगाया गया है ।

PAGE h ps://t.me/studified h ps://instagram.com/aashisharorasocial?utm_medium=copy_link


Follow
218 Aashish Arora on: h ps://youtube.com/channel/UCYa4_JrOrf8R5Kz2uOtccXQ https://www.facebook.com/aashisharorasocial/
अ ास by Aashish Arora
(SBI/IBPS/RBI/LIC/All other banking and insurance Exams)

230

220

210

200

190

180

170
State A State B State C State D State E

Total Districts

% of districts where curfew is imposed

State A
60
50
40
50 30 25
State E State B
20
20
10

40

State D 60 State C

56. Find the ratio of districts where curfew is imposed in State 'B' to districts where curfew is
not imposed in State 'C'.
उन िजलो ं के अनुपात का पता लगाएं जहां े ट बी म क ू लगाया गया है और जहां रा 'सी' म क ू नही ं
लगाया गया है ।
A. 12:43 B. 11:45 C. 15:46
D. 14:45 E. None of these
57. Find the average number of districts where curfew is imposed in State A , B and E.
उन िजलो ं की औसत सं ा ात कर जहाँ रा A , B और E म क ू लगाया गया है ।
A. 60 B. 61 C. 62
D. 63 E. None of these
58. The number of districts where curfew is not imposed in State 'B' is how much percent
more/less than the districts where curfew is imposed in State 'C'?
उन िजलो ं की सं ा जहां B म क ू नही ं लगाया गया है , उन िजलो ं की तुलना म िकतना ितशत अिधक / कम
है जहां रा 'C' म क ू लगाया गया है ?

PAGE h ps://t.me/studified h ps://instagram.com/aashisharorasocial?utm_medium=copy_link


Follow
219 Aashish Arora on: h ps://youtube.com/channel/UCYa4_JrOrf8R5Kz2uOtccXQ https://www.facebook.com/aashisharorasocial/
अ ास by Aashish Arora
(SBI/IBPS/RBI/LIC/All other banking and insurance Exams)

A. 90.25 B. 92.35 C. 94.55


D. 95.65 E. None of these

59. Out of total districts where curfew is not imposed in states 'A' and 'E', together, only
500/11 % districts are 100 percent COVID-free. Find the district where curfew is not
imposed in states 'A' and 'E', that have a few covid cases.
कुल िजलो ं म से जहां रा ो ं ए और ई म क ू नही ं लगाया गया है , केवल 500/11% िजले 100 ितशत
COVID मु ह। उस िजले का पता लगाएं (जहां रा ो ं ए और ई म क ू नही ं लगाया गया है ), िजसम कुछ
कोिवद -19 मामले ह।
A. 130 B. 132 C. 134
D. 136 E. None of these

60. There is another state F where some districts are under curfew and some are not. The
districts where curfew is imposed in State 'F' is 20% more than the same in State B. If the
districts where curfew is not imposed in State 'F' is 45% more than the same in State B, then
find the total districts in State 'F'.
एक और रा एफ है जहां कुछ िजले क ू के अधीन ह और कुछ नही ं ह। िजन िजलो ं म रा F 'म क ू
लगाया गया है , वे रा B से 20% अिधक ह। यिद िजन िजलो ं म क ू नही ं लगाया गया है , वे े ट बी के से 45%
अिधक ह, तो रा 'एफ' म कुल िजलो ं को खोज।
A. 305 B. 310 C. 315
D. 320 E. None of these

The pie chart given below shows the percent distribution of total number of State Bank Of India
customers (illiterate + literate) in five different cities, and the bar graph shows the number of
illiterate customers out of them.
नीचे िदया गया पाई चाट पां च अलग-अलग शहरो ं म भारतीय े ट बक के ाहको ं (अनपढ़ + सा र) की कुल सं ा
का ितशत िवतरण िदखाता है , और बार ाफ अनपढ़ ाहको ं की सं ा को दशाता है ।

PAGE h ps://t.me/studified h ps://instagram.com/aashisharorasocial?utm_medium=copy_link


Follow
220 Aashish Arora on: h ps://youtube.com/channel/UCYa4_JrOrf8R5Kz2uOtccXQ https://www.facebook.com/aashisharorasocial/
अ ास by Aashish Arora
(SBI/IBPS/RBI/LIC/All other banking and insurance Exams)

Total Illiterate SBI Customers


Total Illiterate SBI Customers

City T

City S

City R

City Q

City P

5000 10000 15000 20000 25000 30000 35000 40000

61. Literate SBI customers in City R is how much percent (approx.) more/less as compared to
Literate customers in City Q?
िसटी आर म िलटरे ट SBI ाहक िसटी ू म िलटरे ट ाहको ं की तुलना म िकतना ितशत अिधक / कम है ?
A. 88.9 B. 89.5 C. 91.9
D. 89.9 E. None of these

62. Literate SBI customers in City S is what percentage of illiterate customers in City Q?
िसटी एस म िलटरे ट एसबीआई ाहक, िसटी ू म अनपढ़ ाहको ं का िकतना ितशत है ?
A. 82.15 B. 85.15 C. 87.15
D. 89.15 E. None of these

63. If the literate customers in City X was 25% more than the literate customers in City P and the
literate customers in City Y is 40% more than the literate customers in City T, then find the
sum of the literate customers in City X and City Y.
यिद िसटी ए म सा र ाहक िसटी पी म सा र ाहको ं से 25% अिधक था, और िसटी वाई म सा र ाहक
िसटी टी म सा र ाहको ं की तुलना म 40% अिधक है , तो िसटी ए और िसटी वाई म सा र ाहको ं का
योग?
A. 10639 B. 10682 C. 10732
D. 10734 E. None of these

64. In all 5 cities, what is the difference between the average literate customer, and average
illiterate customer?
सभी 5 शहरो ं म, औसत सा र ाहक और औसत िनर र ाहक के बीच अंतर ा है ?
A. 18200 B. 18400 C. 18600
D. 18800 E. None of these

PAGE h ps://t.me/studified h ps://instagram.com/aashisharorasocial?utm_medium=copy_link


Follow
221 Aashish Arora on: h ps://youtube.com/channel/UCYa4_JrOrf8R5Kz2uOtccXQ https://www.facebook.com/aashisharorasocial/
अ ास by Aashish Arora
(SBI/IBPS/RBI/LIC/All other banking and insurance Exams)

65. What is the ratio of illiterate customers and literate customers in City P?
िसटी पी म िनर र ाहको ं और सा र ाहको ं का अनुपात ा है ?
A. 105:16 B. 103:12 C. 100:17
D. 102:11 E. None of these

The given pie-chart shows the degree distribution of total scientists (female scientists + male
scientists), in four different countries. The given table shows the ratio of male scientists and
female scientists in those respective countries.
िदए गए पाई-चाट चार अलग-अलग दे शो ं म कुल वै ािनको ं (मिहला वै ािनको ं + पु ष वै ािनको)ं के िड ी िवतरण
को दशाता है । दी गई तािलका उन संबंिधत दे शो ं म पु ष वै ािनको ं और मिहला वै ािनको ं के अनुपात को दशाती है ।

Country Male Scien st : Female Scien st

P 11:4

Q 7:5

R 13:8

S 19:5

66. Female scientists in country S is how much percent more/less than the total scientists in
countries P and Q, together?
दे श S म मिहला वै ािनक P और Q दे शो ं के कुल वै ािनको ं की तुलना म िकतने ितशत अिधक/कम ह?
A. 81.52 B. 84.23 C. 86.17
D. 88.59 E. None of these

PAGE h ps://t.me/studified h ps://instagram.com/aashisharorasocial?utm_medium=copy_link


Follow
222 Aashish Arora on: h ps://youtube.com/channel/UCYa4_JrOrf8R5Kz2uOtccXQ https://www.facebook.com/aashisharorasocial/
अ ास by Aashish Arora
(SBI/IBPS/RBI/LIC/All other banking and insurance Exams)

67. Find the ratio of male scientists in country R to the female scientists in country Q.
दे श R म पु ष वै ािनको ं और दे श Q म मिहला वै ािनको ं का अनुपात पता लगाएं ।
A. 25:12 B. 27:14 C. 26:11
D. 23:15 E. None of these

68. Out of total male scientists in countries P and Q, 5/11th of them are Physics Graduates. Find
the male scientists that aren't Physics Graduates, in countries P and Q, together.
पी और ू दे शो ं के कुल पु ष वै ािनको ं म से 5/11 भौितकी ातक ह। उन पु ष वै ािनको ं को खोज, जो पी
और ू दे शो ं म एक साथ, भौितकी ातक नही ं ह।
A. 464 B. 466 C. 468
D. 470 E. None of these

69. Find the average female scientists in countries S, Q and R.


दे शो ं S, Q और R म औसत मिहला वै ािनको ं का पता लगाएं ।
A. 340 B. 350 C. 330
D. 320 E. None of these

70. The male scientists in country T is 80% more than in country R. If the female scientists in
country T is 25% more than country P, then find the total scientists in country T.
दे श T म पु ष वै ािनक दे श R से 80% अिधक ह। यिद दे श T म मिहला वै ािनक दे श P से 25% अिधक है , तो
दे श T म कुल वै ािनको ं को खोज।
A. 1581 B. 1582 C. 1583
D. 1584 E. None of these

The given bar graph shows the total number of journalists (Broadcast Media Journalist + Print
Media Journalist) working in five different cities.
िदए गए बार ाफ पां च अलग-अलग शहरो ं म काम कर रहे प कारो ं ( ॉडका मीिडया जनिल + ि ंट मीिडया
जनिल ) की कुल सं ा को दशाता है ।

City E

City D

City C

City B

City A

0 130 260 390 520 650 780 910

PAGE h ps://t.me/studified h ps://instagram.com/aashisharorasocial?utm_medium=copy_link


Follow
223 Aashish Arora on: h ps://youtube.com/channel/UCYa4_JrOrf8R5Kz2uOtccXQ https://www.facebook.com/aashisharorasocial/
अ ास by Aashish Arora
(SBI/IBPS/RBI/LIC/All other banking and insurance Exams)

71. Out of total number of journalists working in City A, 5/13th are Print Media Journalist. If
45% of Broadcast Media Journalist are working for Republic India, then find the ratio of
number of Broadcast Media Journalist working in City A that are not working for Republic
India and the number of journalists working in Print Media Journalist City A.
िसटी ए म काम करने वाले प कारो ं की कुल सं ा म से 5/13 वी ं ि ंट मीिडया प कार ह। अगर 45%
ॉडका मीिडया जनिल रप क इं िडया के िलए काम कर रहे ह िफर िसटी ए म ॉडका मीिडया
जनिल जो रप क इं िडया के िलए काम नही ं कर रहे ह और िसटी ए म ि ंट मीिडया जनिल की सं ा
का अनुपात खोज।
A. 21:29 B. 22:25 C. 23:26
D. 24:29 E. None of these

72. The number of Print Media Journalist working in City B is how much percent more/less than
the total number of journalists working in City D if out of total number of journalists
working in City B, 80% are Broadcast Media Journalist.
िसटी बी म काम करने वाले ि ंट मीिडया जनिल की सं ा िसटी डी म काम करने वाले प कारो ं की कुल
सं ा से िकतने ितशत अिधक / कम है , अगर िसटी बी म काम करने वाले कुल प कारो ं म से 80%
ॉडका मीिडया जनिल ह।
A. 75.25 B. 81.66 C. 88.88
D. 91.35 E. None of these

73. Find the average number of journalists working in all the five cities.
सभी पाँ च शहरो ं म काम करने वाले प कारो ं की औसत सं ा ात कीिजए।
A. 580 B. 585 C. 590
D. 595 E. None of these

74. Find the ratio of total number of journalists working in cities A and B together and number
of journalists working in City E.
शहरो ं A & B म काम करने वाले प कारो ं की कुल सं ा और िसटी ई म काम करने वाले प कारो ं की सं ा
के बीच का ratio ात कीिजए।
A. 17:13 B. 15:14 C. 18:13
D. 19:14 E. None of these

75. Out of total number of journalists working in cities B and E, together, 5/13 of them are
Broadcast Media Journalist. Find the number of Print Media Journalist working in City B if
there are 175 print media journalist in City E.
B और E शहरो ं म काम करने वाले प कारो ं की कुल सं ा म से, 5/13 ॉडका मीिडया जनिल ह। िसटी
बी म काम करने वाले ि ंट मीिडया जनिल की सं ा ात कर यिद िसटी ई म 175 ि ंट मीिडया जनिल
ह।
A. 535 B. 540 C. 545
D. 550 E. None

PAGE h ps://t.me/studified h ps://instagram.com/aashisharorasocial?utm_medium=copy_link


Follow
224 Aashish Arora on: h ps://youtube.com/channel/UCYa4_JrOrf8R5Kz2uOtccXQ https://www.facebook.com/aashisharorasocial/
अ ास by Aashish Arora
(SBI/IBPS/RBI/LIC/All other banking and insurance Exams)

Every year, a college in D.U invites applications for B.A and B.Sc. The given bar graph shows the
total applications received for B.A + B.Sc in five different years.
हर साल D.U म एक कॉलेज B.A और B.Sc. के िलए आवेदन आमंि त करता है । िदए गए बार ाफ पां च अलग-
अलग वष म B.A + B.Sc के िलए ा कुल आवेदनो ं को दशाता है ।

18600

17600

16600

15600

14600

13600
2011 2012 2013 2014 2015

Applica on for B.A. Applica on for B.Sc

76. Out of total applications received for B.Sc in 2012, 26% was rejected. Find the difference
between applications received in 2012 for B.Sc which was approved and applications
received in 2011, for B.A.
2012 म B.Sc के िलए ा कुल आवेदनो ं म से 26% को अ ीकार कर िदया गया था। B.Sc के िलए 2012 म
ा आवेदनो ं िजसे अनुमोिदत िकया गया था और 2011 म ा B.A. आवेदन प के बीच का अंतर ात कर
A. 4588 B. 4688 C. 4788
D. 4888 E. None of these

77. The applications received in 2015 for B.Sc is how much percent more/less than the total
applications received in 2014 ?
B.Sc के िलए 2015 म ा आवेदन प 2014 म ा कुल आवेदनो ं की तुलना म िकतने ितशत अिधक /
कम है ?
A. 49.35 B. 55.35 C. 59.38
D. 65.38 E. None of these

78. Find the average number of applications received for B.Sc from 2013 to 2015.
2013 से 2015 तक B.Sc के िलए ा आवेदनो ं की औसत सं ा ात कीिजए।
A. 15100 B. 15150 C. 15200
D. 15250 E. None of these

79. Out of total applications received in the year 2012 to 2014 for B.A, 10 percent got rejected
and out of remaining 20% actually got admission. Find how many students got admission
from 2012 to 2014.

PAGE h ps://t.me/studified h ps://instagram.com/aashisharorasocial?utm_medium=copy_link


Follow
225 Aashish Arora on: h ps://youtube.com/channel/UCYa4_JrOrf8R5Kz2uOtccXQ https://www.facebook.com/aashisharorasocial/
अ ास by Aashish Arora
(SBI/IBPS/RBI/LIC/All other banking and insurance Exams)

B.A के िलए वष 2012 से 2014 तक ा कुल आवेदनो ं म से 10 ितशत खा रज हो गए और शेष म से 20%


को वा व म वेश िमल गया। जान िक 2012 से 2014 तक िकतने छा ो ं को वेश िमला।
A. 9090 B. 9095 C. 9070
D. 9085 E. None of these

80. In which year, the average of application received for both courses is lowest?
िकस वष म, दोनो ं पा मो ं के िलए ा आवेदन का औसत सबसे कम है ?
A. 2011 B. 2012 C. 2013
D. 2014 E. 2015

The mass vaccination programme - a 4-day 'Tika Utsav' or vaccine festival was conducted from
11th till April 14th. The given table shows the total number of people (above 60 age + below 60
age) and ratio of the number of above 60 age people to below 60 age people, who got the
vaccine.
सामूिहक टीकाकरण काय म 11 अ ैल से 14 अ ैल तक 4 िदवसीय टीका उ व आयोिजत िकया गया था। दी गई
तािलका म कुल लोगो ं की सं ा (60 वष से कम आयु + 60 वष से अिधक) और 60 वष से अिधक आयु के लोगो ं की
सं ा और 60 वष से कम आयु के लोगो ं का अनुपात को िदखाया गया है , िज टीका िमला था।

City People who got vaccinated Ra o of Above 60 : Below 60

P 15500 16:9

Q 17800 31:9

R 19600 23:5

S 18700 17:5

T 17850 11:3

81. Find the difference between number of below 60 age people who got the vaccine in City T
and the number of above 60 age people who got the vaccine in City Q.
िसटी टी म वै ीन पाने वाले 60 से कम उ के लोगो ं और िसटी ू म वै ीन पाने वाले 60 से अिधक उ के
लोगो ं के बीच का अंतर ात कर.
A. 9920 B. 9950 C. 9960
D. 9970 E. None

82. Find the ratio of total number of above 60 age people who got the vaccine in City P and R
together to total number of people who got the vaccine in City R.
िसटी पी और आर म कुल िमलाकर वै ीन पाने वाले 60 से अिधक आयु वाले लोगो ं की कुल सं ा और िसटी
आर म वै ीन पाने वाले कुल लोगो ं की सं ा का अनुपात ात कीिजए ।
A. 1301:980 B. 1302:970 C. 1303:990
D. 1304:995 E. None

PAGE h ps://t.me/studified h ps://instagram.com/aashisharorasocial?utm_medium=copy_link


Follow
226 Aashish Arora on: h ps://youtube.com/channel/UCYa4_JrOrf8R5Kz2uOtccXQ https://www.facebook.com/aashisharorasocial/
अ ास by Aashish Arora
(SBI/IBPS/RBI/LIC/All other banking and insurance Exams)

83. Out of total number of below 60 age people who got the vaccine in City Q and R, together,
20% were earlier infected by Covid. Find the number of below 60 age people who got the
vaccine in City Q and R who were not infected by covid.
िसटी ू और आर म वै ीन पाने वाले 60 से कम आयु के कुल लोगो ं म से, 20% पहले कोिवद ारा सं िमत
थे। 60 वष से कम आयु के उन लोगो ं की सं ा ात कर, िज िसटी ू और आर म वै ीन िमली थी, जो
कोिवद से सं िमत नही ं थे।
A. 6004 B. 6008 C. 6010
D. 6024 E. None

84. Find the average of number of below 60 age people who got the vaccine in City P and S.
िसटी पी और एस म वै ीन पाने वाले 60 से कम उ के लोगो ं की सं ा का औसत ात कीिजए।
A. 4905 B. 4910 C. 4920
D. 4915 E. None

85. The number of above 60 age people who got the vaccine in another City U is 20% of the
total number of above 60 age people who got the vaccine in City S and T, together. Find the
total number of above 60 age people who got the vaccine in City U.
60 से अिधक उ के लोगो ं की सं ा, जो िकसी अ शहर यू म वै ीन ा करते ह, 60 से अिधक उ के
लोगो ं की कुल सं ा का 20% है , िज िसटी एस और टी म वै ीन िमला है । शहर यू म वै ीन पाने वाले 60
से अिधक उ के लोगो ं की कुल सं ा का पता लगाएं ।
A. 5580 B. 5685 C. 5695
D. 5690 E. None

A shop sold different number of refrigerators out of which some were Classic Refrigerator and
some were French-Door Refrigerator, of five different brands. The radar graph and bar graph
given below shows the total number of Classic Refrigerator and French-Door Refrigerator sold
by the shop of the respective brands.
एक दु कान ने पां च अलग-अलग ां डो ं के रे ि जरे टर की अलग-अलग सं ा बेची, िजनम से कुछ ािसक
रे ि जरे टर थे और कुछ च-डोर रे ि जरे टर थे। नीचे िदए गए राडार ाफ और बार ाफ संबंिधत ां डो ं की दु कान
ारा बेची जाने वाली कुल ािसक रे ि जरे टर और च-डोर रे ि जरे टर को दशाते ह।

Classic Refrigerator
Samsung
192
168
144
120
96
Bosch 72 LG
48
24

Hitachi Whirlpool

PAGE h ps://t.me/studified h ps://instagram.com/aashisharorasocial?utm_medium=copy_link


Follow
227 Aashish Arora on: h ps://youtube.com/channel/UCYa4_JrOrf8R5Kz2uOtccXQ https://www.facebook.com/aashisharorasocial/
अ ास by Aashish Arora
(SBI/IBPS/RBI/LIC/All other banking and insurance Exams)

Bosch 165

Hitachi 352

Whirlpool 154

LG 307

Samsung 187

88 132 176 220 264 308 352 396

French Door Refrigerator

86. What is the ratio of number of Classic Refrigerators sold of Samsung and Bosch together
to total number of refrigerators sold of Hitachi?
सैमसंग और बॉश के बेचे जाने वाले ािसक रे ि जरे टर की सं ा और िहटाची के बेचे जाने वाले कुल
रे ि जरे टर की सं ा का अनुपात ा है ?
A. 36:117 B. 39:118 C. 40:119
D. 41:120 E. None of these

87. Number of Classic Refrigerators sold of LG and Hitachi together is how much percent
more/less than total number of French-Door Refrigerator refrigerators sold of Samsung
and LG together.
एलजी और िहताची की एक साथ बेची जाने वाली ािसक रे ि जरे टर की सं ा, सैमसंग और एलजी के एक
साथ िबकने वाले च-डोर रे ीिजरे टर रे ि जरे टर की कुल सं ा से िकतने ितशत अिधक / कम है ।
A. 51.6 B. 47.4 C. 41.6
D. 53.8 E. None of these

88. What is the average number of refrigerators sold of LG, Whirlpool and Hitachi together?
ां ड एलजी, लपूल और िहताची के बेचे जाने वाले रे ि जरे टर की औसत सं ा ा है ?
A. 407 B. 409 C. 411
D. 413 E. None of these

89. If number of Classic Refrigerator sold of LG, Whirlpool, Hitachi and Bosch is represented
of a pie chart then the central angle (approx.) made by number of Classic Refrigerator sold
of Hitachi will be?
यिद एलजी, लपूल, िहताची और बॉश के िबकने वाले Classic रे ीिजरे टर की सं ा पाई चाट का
ितिनिध करती है तो िहताची के बेचे जाने वाले Classic रे ीिजरे टर की सं ा से बना क ीय कोण होगा?
A. 45 B. 68.5 C. 22.5
D. 75.25 E. 85.5

90. Number of Classic Refrigerator and French-Door Refrigerator sold of Sony is 45% more
than number of Classic Refrigerator sold by Hitachi and 40% less than French-Door
refrigerators by Bosch respectively. Find the Total number of refrigerators sold of Sony?
सोनी ारा बेचे जाने वाले ािसक रे ीिजरे टर और च-डोर रे ीिजरे टर रे ीिजरे टर की सं ा मशः

PAGE h ps://t.me/studified h ps://instagram.com/aashisharorasocial?utm_medium=copy_link


Follow
228 Aashish Arora on: h ps://youtube.com/channel/UCYa4_JrOrf8R5Kz2uOtccXQ https://www.facebook.com/aashisharorasocial/
अ ास by Aashish Arora
(SBI/IBPS/RBI/LIC/All other banking and insurance Exams)

िहताची और बॉश ारा बेचे जाने वाले मशः ािसक रे ीिजरे टर और च-डोर रे ीिजरे टर की कुल
सं ा से 40% अिधक है । सोनी को बेचे जाने वाले रे ि जरे टर की कुल सं ा ात कीिजये?
A. 271 B. 272 C. 273
D. 274 E. None of these

Students studying in 5 different class of various branches of DAV School throughout India were
surveyed. They were asked to choose one subject out of the four. The given table shows the
number of students who like four different subjects, in five different classes.
पूरे भारत म डीएवी ू ल की िविभ शाखाओं के 5 अलग-अलग क ाओं म पढ़ने वाले छा ो ं का सव ण िकया
गया। उ चार म से एक िवषय चुनने को कहा गया। दी गई तािलका म पाँ च अलग-अलग क ाओं म चार अलग-
अलग िवषयो ं को पसंद करने वाले छा ो ं की सं ा को दशाया गया है ।

Social
Language Maths Science TOTAL
Science

VI 7400 5200 5600 8200 26400

VII 6200 4800

VIII 4800 1800 16600

IX 1200 1800 2200 1400

X 2800 1700 12000

Total 21100 17100 14300 75500

91. Find the ratio of number of students who like Language in Class Xth to number of students
who like Maths in Class VIIIth.
दसवी ं क ा म भाषा पसंद करने वाले छा ो ं की सं ा और आठवी ं क ा म गिणत पसंद करने वाले छा ो ं की
सं ा का अनुपात जान।
A. 15:28 B. 17:26 C. 13:28
D. 14:27 E. None of these

92. Find the difference between total number of students who like Language in given five
classes and total number of students who like given four subjects in Class Ixth.
भाषा पसंद करने वाले छा ो ं की कुल सं ा और क ा IXth म चार िवषयो ं को पसंद करने वाले छा ो ं की कुल
सं ा के बीच अंतर ात कीिजए।
A. 16400 B. 16500 C. 16600
D. 16700 E. None of these

93. Out of total number of students who like Science in classes VIIth and VIIIth, together, 54%
belong to Tier-3 cities. Find the number of students who like Science in classes VIIth and
VIIIth who do not belong to Tier-3 cities.
VIIth और VIIIth की क ाओं म साइं स पसंद करने वाले कुल छा ो ं म से, 54% टीयर - 3 शहरो ं के ह। उन

PAGE h ps://t.me/studified h ps://instagram.com/aashisharorasocial?utm_medium=copy_link


Follow
229 Aashish Arora on: h ps://youtube.com/channel/UCYa4_JrOrf8R5Kz2uOtccXQ https://www.facebook.com/aashisharorasocial/
अ ास by Aashish Arora
(SBI/IBPS/RBI/LIC/All other banking and insurance Exams)

छा ो ं की सं ा ात कर जो िव ान को सातवी ं और आठवी ं क ाओं म पसंद करते ह जो िटयर - 3 शहरो ं से


नही ं ह।
A. 2970 B. 2860 C. 2760
D. 2990 E. None of these

94. The number of students who like Social Science in Class VIIth is how much percent
more/less than the number of students who like Maths in Class Xth?
क ा VII म सामािजक-िव ान पसंद करने वाले छा ो ं की सं ा, दसवी ं क ा म गिणत पसंद करने वाले छा ो ं
की सं ा से िकतने ितशत अिधक / कम है ?
A. 65.25 B. 70.73 C. 75.56
D. 73.33 E. None of these

95. The total number of students who like given four subjects in Class Vth is 3/5th of Class
IXth. The ratio of the number of students who like Language, Maths, Science and Social
Science in city F is 9:11:8:5, respectively. Find the total number of students who like
Language and Science, together in Class Vth?
क ा Vth म चार िवषयो ं को पसंद करने वाले छा ो ं की कुल सं ा IXth क ा की 3/5 वी ं है । शहर एफ म
भाषा, गिणत, िव ान और सामािजक िव ान पसंद करने वाले छा ो ं की सं ा का अनुपात मशः 9: 11: 8: 5
है । क ा Vth म एक साथ भाषा और िव ान पसंद करने वाले छा ो ं की कुल सं ा ात कीिजए?
A. 2018 B. 2022 C. 2036
D. 2040 E. None of these

The given pie chart shows the percent distribution of total number of films (Hindi Films + Non-
Hindi Films) released in 4 different years and the bar graph shows the number of Hindi Films
released and number of Non-Hindi Films released.
िदए गए पाई चाट म 4 अलग-अलग वष म रलीज़ ई कुल िफ ो ं (िहं दी िफ + गैर-िहं दी िफ ) का ितशत
िवतरण िदखाया गया है और बार ाफ िहं दी िफ ो ं की सं ा और गैर-िहं दी िफ की सं ा को दिशत करता
है ।

Total films released = 4800

14%
32%

42%
12%

2011 2012 2013 2014

PAGE h ps://t.me/studified h ps://instagram.com/aashisharorasocial?utm_medium=copy_link


Follow
230 Aashish Arora on: h ps://youtube.com/channel/UCYa4_JrOrf8R5Kz2uOtccXQ https://www.facebook.com/aashisharorasocial/
अ ास by Aashish Arora
(SBI/IBPS/RBI/LIC/All other banking and insurance Exams)

Hindi Films Non-Hindi Films

2014 7 17

2013 5 7

2012 8 13

2011 5 23

96. Find the difference between the number of Hindi Films released in 2012 and number of
Non-Hindi Films released in 2013.
2012 म रलीज़ ई िहं दी िफ़ ो ं की सं ा और 2013 म रलीज़ ई गैर-िहं दी िफ़ ो ं की सं ा के बीच
अंतर ात कीिजए।
A. 430 B. 432 C. 434
D. 436 E. None of these

97. Find the ratio of total number of Hindi Films released in 2011 and 2013, together to the
number of Non-Hindi Films released in 2012.
2011 और 2013 म जारी िहं दी िफ ो ं की कुल सं ा और 2012 म जारी गैर-िहं दी िफ ो ं की सं ा का
अनुपात ात कीिजए।
A. 13:51 B. 14:53 C. 15:53
D. 15:52 E. None of these

98. 25% of released Hindi Films in 2012 and 75% of Hindi Film released in 2011 were declared
flop and rest were hit . Find total Hindi films that were declared 'hit' in 2012 and 2011.
2012 म रलीज़ िहं दी िफ़ ो ं का 25% और 2011 म रलीज़ ई िहं दी िफ़ ो ं म से 75% ॉप घोिषत की गईं
और बाकी िहट रही।ं 2012 और 2011 म कुल िहं दी िफ ो ं की सं ा ात कर िज 'िहट' घोिषत िकया गया
था।
A. 602 B. 603 C. 606
D. 607 E. None of these

99. On an average each Hindi Film earned 27 crore in 2011 and 34 crore in 2013 Find the total
revenue earned (in crores) in 2011 and 2013 by the Hindi films.
औसतन ेक िहं दी िफ ने 2011 म 27 करोड़ और 2013 म 34 करोड़ कमाए। िहं दी िफ ो ं ारा 2011
और 2013 म अिजत कुल राज का पता लगाएं ।
A. 11400 B. 11600 C. 11200
D. 11800 E. None of these

100. Number of Hindi Films released in 2015 is 25% more than that in 2014. If the number of
Hindi Films released in 2015 is 28% of total number of films (Hindi Films + Non-Hindi Films)

PAGE h ps://t.me/studified h ps://instagram.com/aashisharorasocial?utm_medium=copy_link


Follow
231 Aashish Arora on: h ps://youtube.com/channel/UCYa4_JrOrf8R5Kz2uOtccXQ https://www.facebook.com/aashisharorasocial/
अ ास by Aashish Arora
(SBI/IBPS/RBI/LIC/All other banking and insurance Exams)

released, then find the number of Non-Hindi Films released in 2015.


2015 म रलीज़ ई िहं दी िफ़ ो ं की सं ा 2014 की तुलना म 25% अिधक है । यिद 2015 म रलीज़ होने
वाली िहं दी िफ़ ो ं की सं ा कुल िफ़ ो ं की 28% है (िहं दी िफ़ + गैर-िहं दी िफ़ ), तो 2015 म रलीज
ई गैर-िहं दी िफ की सं ा ात कर।
A. 1420 B. 1430 C. 1440
D. 1450 E. None of these

Directions : Read the following questions carefully and answer the questions that follow.
Below bar graph shows production of cars from three companies namely Sutuki, Tia and Ronda
in thousands during years 2015 to 2019.

दी गयी जानकारी को ान से पढ़ और िन िल खत ो ं के उ र द।
नीचे िदया गया बार ाफ वष 2015 से 2019 के दौरान तीन कंपिनयो ं अथात् सुतकी, िटया और रोड
ं ा ारा कारो ं के
उ ादन को दशाता है ।(हजारो ं म)

Car Produced
300

250

200

Suzuki
150
Tia

100 Ronda

50

0
2015 2016 2017 2018 2019

101. What is the average number of cars produced by Sutuki during all the years taken together
(in thousands)?
सभी वष म एक साथ िलए जाने के दौरान सुतकी ारा उ ािदत कारो ं की औसत सं ा ा है ? (हजारो ं म)
A. 180 B. 240 C. 190
D. 175 E. 158.5

102. What is the ratio of cars produced from all the three companies Sutuki, Tia and Ronda
respectively for 2018 and 2019 combined?
2018 और 2019 के िलए तीनो ं कंपिनयो ं मशः सुतुकी, िटया और रोड
ं ा से उ ािदत कारो ं का अनुपात ा
है ?
A. 15 : 23 : 20 B. 12 : 16 : 21 C. 15 : 18 : 17
D. 18 : 16 : 17 E. None of these

PAGE h ps://t.me/studified h ps://instagram.com/aashisharorasocial?utm_medium=copy_link


Follow
232 Aashish Arora on: h ps://youtube.com/channel/UCYa4_JrOrf8R5Kz2uOtccXQ https://www.facebook.com/aashisharorasocial/
अ ास by Aashish Arora
(SBI/IBPS/RBI/LIC/All other banking and insurance Exams)

103. What is the percentage change in production of cars produced by all three companies
together from 2016 to 2019?
2016 से 2019 तक सभी तीन कंपिनयो ं ारा एक साथ उ ािदत कारो ं के उ ादन म ितशत प रवतन
िकतना है ?
A. 75.35 B. 80.38 C. 77.72
D. 58.82 E. 85.93

104. If 80% of the cars produced during 2018 are sold in the ratio 5 : 4 : 6 from Sutuki, Tia and
Ronda respectively, then what is the difference of revenue between Tia and Ronda (in
thousand lacs) ? (Note:- Price per car for Sutuki, Tia and Ronda is Rs 8 lakhs, Rs 9 lakhs
and Rs 12 lakhs respectively)
यिद 2018 के दौरान उ ािदत 80% कारो ं को मशः सुतुकी, िटया और रोड ं ा ारा 5: 4: 6 के अनुपात म
बेचा जाता है , तो िटया और रोड
ं ा के बीच राज का अंतर ा है (हजार लाख म)? (नोट: - सुतकी, िटया और
रोडं ा के िलए ित कार मू मशः , 8 लाख पये, 9 लाख पये और 12 लाख पये है )
A. 1430 B. 1280 C. 1152
D. 1358 E. 1224

105. If cars produced by Sutuki and Ronda during 2020 increased by 20% and 25%
respectively, then what is the approximate percentage change in cars produced by Sutuki
and Ronda together from 2017 to 2020 ?
यिद 2020 के दौरान सुतकी और रोड ं ा ारा उ ािदत कारो ं म मशः 20% और 25% की वृ ई है , तो
2017 से 2020 म सुतकी और रोड ं ा ारा िनिमत कारो ं म अनुमािनत ितशत प रवतन ा है ?
A. 53.4 B. 47.6 C. 33.33
D. 57.1 E. 43.9

Directions :Following is the data regarding the revenue and expenditure of 4 companies in
2019 and 2020. All figures are in Rs Crores.
Profit = Revenue – Expenditure
Percentage Profit = (Profit/Expenditure) × 100

िन िल खत आं कड़े 2019 और 2020 म 4 कंपिनयो ं के राज और य के आं कड़े ह। सभी आं कड़े पये करोड़ो ं
म ह।
लाभ = राज – य
ितशत लाभ = (लाभ / य) × 100

2019 2020

Revenue Expenditure Revenue Expenditure

HDFB 7600 6800 8000 7000

IEFC 8200 7400 7800 6800

AXES 7000 6500 7600 6700

ICBI 6300 6300 6300 6300

PAGE h ps://t.me/studified h ps://instagram.com/aashisharorasocial?utm_medium=copy_link


Follow
233 Aashish Arora on: h ps://youtube.com/channel/UCYa4_JrOrf8R5Kz2uOtccXQ https://www.facebook.com/aashisharorasocial/
अ ास by Aashish Arora
(SBI/IBPS/RBI/LIC/All other banking and insurance Exams)

106. Which company in which year has seen the maximum percentage profit?
िकस कंपनी ने िकस वष म अिधकतम ितशत लाभ दे खा है ?
A. IEFC in 2020 B. HDFB in 2020 C. ICBI in 2019
D. AXES in 2020 E. None of these

107. Which company has seen the maximum percentage increase in profit from 2019 to 2020?
िकस कंपनी ने 2019 से 2020 म लाभ म अिधकतम ितशत वृ दे खी है ?
A. HDFB B. IEFC C. AXES
D. ICBI E. None of these

108. What is the approximate sum of numerical values of the percentage profit of HDFB in 2019
& AXES in 2020?
2019 म HDFB और 2020 म AXES के ितशत लाभ के सं ा क मानो ं का अनुमािनत योग ा है ?
A. 21.4 B. 22.3 C. 25.19
D. 27.3 E. 30.1

109. What is the ratio of the numerical values of the approximate percentage profit of HDFB in
2019 to HDFB in 2020?
2019 म एचडीएफबी और 2020 म एचडीएफबी के अनुमािनत ितशत लाभ के सं ा क मू ो ं का
अनुपात ा है ?
A. 14 : 17 B. 45 : 33 C. 41 : 51
D. 47 : 61 E. 38 : 65

110. For all 4 companies combined, (approximately) what is the percentage increase in the sum
of their profits from 2019 to 2020 ?
संयु 4 कंपिनयो ं के िलए, (लगभग) 2019 से 2020 तक उनके मुनाफे के योग म ितशत वृ ा है ?
A. 48 B. 44 C. 38
D. 31 E. 27

The given table shows the ratio of the number of Urban and rural branches and the difference
between the number of Urban branches and rural branches, in five different banks located in a
particular state.
दी गई तािलका म एक िवशेष रा म थत पां च अलग-अलग बको ं म शहरी और ामीण बक शाखाओं की सं ा
और शहरी शाखाओं और ामीण शाखाओं की सं ा के बीच का अंतर, िदखाया गया है ।

96

76

56

36
Bank P Bank Q Bank R Bank S Bank T

Difference between Urban Branches and Rural Branch

PAGE h ps://t.me/studified h ps://instagram.com/aashisharorasocial?utm_medium=copy_link


Follow
234 Aashish Arora on: h ps://youtube.com/channel/UCYa4_JrOrf8R5Kz2uOtccXQ https://www.facebook.com/aashisharorasocial/
अ ास by Aashish Arora
(SBI/IBPS/RBI/LIC/All other banking and insurance Exams)

Bank Urban Branches: Rural Branches

P 21:5

Q 14:9

R 17:7

S 13:5

T 15:7

111. The number of rural branches in Bank U is 45% more than that in Bank R. If total number of
branches (Urban + rural) in Bank U is 112 more than that in Bank R, then find the number of
Urban branches in Bank U.
बक U म ामीण शाखाओं की सं ा बक R से 45% अिधक है । यिद बक U म शाखाओं की कुल सं ा
(शहरी + ामीण) बक R की तुलना म 112 अिधक है , तो Bank U म शहरी शाखाओं की सं ा ात कीिजए।
A. 214 B. 215 C. 216
D. 217 E. None of these

112. Find the difference between the total number of branches in Bank T and number of rural
branches in Bank Q and S, together.
बक टी म शाखाओं की कुल सं ा और बक ू और एस म ामीण शाखाओं की सं ा के बीच का अंतर ात
कीिजए।
A. 5 B. 10 C. 15
D. 0 E. None of these

113. Find the average number of branches in Bank P, R and S.


बक पी, आर और एस म शाखाओं की औसत सं ा ात कीिजए।
A. 184 B. 188 C. 192
D. 196 E. None of these

114. 55% of rural branches in Bank S and 70% of rural branches in Bank P are scale-1
branches. Find total number of rural branches in Bank S and P which are scale-1
branches.
बक S म 55% ामीण शाखाएँ और Bank P म 70% ामीण शाखाएँ , े ल-1 शाखाएँ ह। बक एस और पी
म कुल ामीण शाखाएँ खोज जो े ल -1 शाखाएँ ह ।
A. 54 B. 55 C. 56
D. 57 E. None of these

115. The total number of branches in Bank Q is how much percent more/less than the number
of Urban branches in banks P and T, together?
बक Q की शाखाओं की कुल सं ा, बक P और T म शहरी शाखाओं की सं ा की तुलना म िकतने ितशत
अिधक / कम है ?

PAGE h ps://t.me/studified h ps://instagram.com/aashisharorasocial?utm_medium=copy_link


Follow
235 Aashish Arora on: h ps://youtube.com/channel/UCYa4_JrOrf8R5Kz2uOtccXQ https://www.facebook.com/aashisharorasocial/
अ ास by Aashish Arora
(SBI/IBPS/RBI/LIC/All other banking and insurance Exams)

A. 12.82 B. 10.56 C. 16.92


D. 14.81 E. None of these

A raid was conducted at four factories and different number of notes of two denominations were
seized. Out of the seized notes, some were genuine and some were fake. The given graph
shows the number of notes seized and percent of fake notes out of the total.
चार कारखानो ं म छापे मारे गए और दो मू वग के अलग-अलग नोट ज िकए गए। ज िकए गए नोटो ं म से कुछ
असली थे और कुछ नकली थे। िदए गए ाफ़ म ज िकए गए नोटो ं की सं ा और नकली नोटो ं के ितशत को
दशाया गया है ।

Number of notes seized


2550

2300

2050

1800

1550

1300
Factory P Factory Q Factory R Factory S

Notes of 2000 Notes of 500

% of fake notes
95
90
85
80
75
70
65
Factory P Factory Q Factory R Factory S

Notes of 2000 Notes of 500

116. Find the ratio of the genuine notes of 2000 denomination in Factory P and Factory Q,
together to the fake notes of 500 denomination in Factory S.
फ़ै री P और फ़ै री Q म 2000 मू वग के असली नोटो ं और फ़ै री S म 500 मू वग के नकली नोटो ं
के अनुपात को खोज।

PAGE h ps://t.me/studified h ps://instagram.com/aashisharorasocial?utm_medium=copy_link


Follow
236 Aashish Arora on: h ps://youtube.com/channel/UCYa4_JrOrf8R5Kz2uOtccXQ https://www.facebook.com/aashisharorasocial/
अ ास by Aashish Arora
(SBI/IBPS/RBI/LIC/All other banking and insurance Exams)

A. 49:102 B. 51:103 C. 52:103


D. 47:102 E. None of these

117. Find the difference between fake notes of 2000 denomination in Factory S and genuine
notes of 500 denomination in Factory P and Factory S, together.
फ़ै री S म 2000 मू वग के नकली नोट और फ़ै री P और फ़ै री S म 500 मू वग के असली नोटो ं
के अंतर को ात कीिजए।
A. 505 B. 515 C. 525
D. 535 E. None of these

118. The total monetary value of all genuine notes of 2000 denomination in Factory R is how
much (in crores) ?
फै ी आर म 2000 मू वग के सभी वा िवक नोटो ं का मौि क मू िकतना है ?
A. 5 cr B. 0.5 cr C. 50 cr
D. 0.05 cr E. None of these

119. Find the ratio of the total monetary value of all genuine notes of 2000 denomination in
Factory S and fake notes of 500 denomination in Factory S ?
फ़ै री S म 2000 मू वग के सभी वा िवक नोटो ं और फ़ै री S म 500 मू वग के नकली नोटो ं के कुल
मौि क मू का अनुपात ात कीिजए?
A. 1:2 B. 2:3 C. 3:4
D. 4:5 E. None of these

120. Find the average fake notes of 2000 denomination in Factory P, Q, R and S.
फ़ै री पी, ू, आर और एस म 2000 मू वग के औसत नकली नोट खोज।
A. 1550 B. 1555 C. 1560
D. 1565 E. None of these

Directions : Study the following information and answer the related questions.
The given line graph shows the number of employees who use Samsung and Apple
smartphones in five different countries. (Both these brand provide a triple camera and a dual
camera model only)

िन िल खत जानकारी का अ यन कर और संबंिधत ो ं के उ र द।
दी गई रे खा का ाफ़ उन कमचा रयो ं की सं ा दशाता है जो पाँ च अलग-अलग कंपिनयो ं म सैमसंग और ऐ ल
ाटफोन का उपयोग करते ह। (ये दोनो ं ां ड केवल एक िटपल कैमरा और एक डु अल कैमरा मॉडल दान करते
है )

PAGE h ps://t.me/studified h ps://instagram.com/aashisharorasocial?utm_medium=copy_link


Follow
237 Aashish Arora on: h ps://youtube.com/channel/UCYa4_JrOrf8R5Kz2uOtccXQ https://www.facebook.com/aashisharorasocial/
अ ास by Aashish Arora
(SBI/IBPS/RBI/LIC/All other banking and insurance Exams)

70

60

50

40

30

20

10

0
P Q R S T

Samsung Apple

121. In company P, total number of employees who use triple camera phones is 48 and ratio of
total number of employees, who use Samsung triple camera phones to Apple triple
camera phones is 7 : 5, then what is the ratio of employees who use Apple dual camera
phones to Samsung dual camera phones is ?
कंपनी P म, िटपल कैमरा फोन का उपयोग करने वाले कमचा रयो ं की कुल सं ा 48 है और कमचा रयो ं की
कुल सं ा का अनुपात, जो ए ल िटपल कैमरा फोन और सैमसंग िटपल कैमरा फोन का उपयोग करते ह
मशः 5 : 7 है , ए ल डु अल कैमरा फोन और सैमसंग डु अल कैमरा फोन का उपयोग करने वाले
कमचा रयो ं का अनुपात ा है ?
A. 17 : 19 B. 15 : 17 C. 7 : 11
D. 17 : 23 E. None of these

122. In company Q, the ratio of total number of employees who use triple camera phones to
dual camera phones is 14 : 9 respectively, and the ratio of total number of employees who
use Samsung triple camera to dual camera phones is 16 : 13 respectively. Find the
difference between total number of employees who use Apple triple camera phones and
dual camera phones in company B ?
कंपनी Q म, िटपल कैमरा फोन और डु अल कैमरा फोन का उपयोग करने वाले कमचा रयो ं की कुल सं ा
का अनुपात मशः 14: 9 है , और सैमसंग िटपल कैमरा और डु अल कैमरा फोन का उपयोग करने वाले
कमचा रयो ं की कुल सं ा का अनुपात मशः 16: 13 है । कंपनी Q म ए ल िटपल कैमरा फोन और डु अल
कैमरा फोन का उपयोग करने वाले कमचा रयो ं की कुल सं ा के बीच अंतर ात कीिजए?
A. 17 B. 24 C. 14
D. 8 E. None of these

123. In company R, total number of employees who use Apple triple camera phones is 20 which
is 12 more than the number people who use Samsung dual camera phones, then find the
difference between total number of employees who use triple and dual camera phones.
कंपनी R म, ऐ ल िटपल कैमरा फोन का उपयोग करने वाले कुल कमचा रयो ं की सं ा 20 है जो सैमसंग के
डु अल कैमरा फोन का उपयोग करने वाले लोगो ं की सं ा से 12 अिधक है , िफर िटपल और डु अल कैमरा
फोन का उपयोग करने वाले कुल कमचा रयो ं के बीच के अंतर का पता लगाएं ।

PAGE h ps://t.me/studified h ps://instagram.com/aashisharorasocial?utm_medium=copy_link


Follow
238 Aashish Arora on: h ps://youtube.com/channel/UCYa4_JrOrf8R5Kz2uOtccXQ https://www.facebook.com/aashisharorasocial/
अ ास by Aashish Arora
(SBI/IBPS/RBI/LIC/All other banking and insurance Exams)

A. 6 B. 2 C. 4
D. 0 E. None of these

124. If in company S, out of employees who use Samsung and Apple phones 25 % and 40%
respectively use dual camera phones, then the number of employees who use Samsung
triple camera phones is approximately what percent more/less than the total employees
who use triple camera phones?
यिद कंपनी S म, सैमसंग और ऐ ल फोन का उपयोग करने वाले कमचा रयो ं म से 25% और 40% मशः
डु अल कैमरा फोन का उपयोग करते ह, तो सैमसंग िटपल कैमरा फोन का उपयोग करने वाले कमचा रयो ं की
सं ा िटपल कैमरा फोन का उपयोग करने वाले कुल कमचा रयो ं की तुलना म लगभग िकतने ितशत
अिधक / कम है ?
A. 27.5 % B. 200 % C. 75 %
D. 50 % E. None of these

125. In company T, out of total employees who use phones 20% use Samsung triple camera
phones while one third use Apple dual camera phones, then what is the ratio of total
number of employees who use Samsung triple camera phones to Apple triple camera
phones respectively.
कंपनी T म, फोन का उपयोग करने वाले कुल कमचा रयो ं म से 20%, सैमसंग िटपल कैमरा फोन का उपयोग
करते ह, जबिक एक ितहाई ऐ ल डु अल कैमरा फोन का उपयोग करते ह, िफर मशः सैमसंग िटपल कैमरा
फोन और ए ल िटपल कैमरा फोन का उपयोग करने वाले कुल कमचा रयो ं की सं ा का अनुपात ा है ।
A. 5 : 7 B. 14 : 9 C. 7 : 5
D. 13 : 8 E. None of these

Directions : Read the following questions carefully and answer the questions that follow.
The bar graph given shows the consumption (in liter) of water, alcohol and cold drinks inn 4
different restaurants on a day..
दी गयी जानकारी को ान से पढ़ और िन िल खत ो ं के उ र द।
िदए गए बार ाफ म एक िवशेष िदन पर 4 अलग-अलग रे रां म पानी, मिदरा और को िडं की खपत को
(लीटर म) िदखाया गया है ।

Hoberoi

YTC

The Dheela

Raj

0 10 20 30 40 50 60 70 80 90

Cold Drink Alcohol Water

PAGE h ps://t.me/studified h ps://instagram.com/aashisharorasocial?utm_medium=copy_link


Follow
239 Aashish Arora on: h ps://youtube.com/channel/UCYa4_JrOrf8R5Kz2uOtccXQ https://www.facebook.com/aashisharorasocial/
अ ास by Aashish Arora
(SBI/IBPS/RBI/LIC/All other banking and insurance Exams)

126. The total consumption of water, alcohol and cold drinks in The Dheela is what percentage
of that of consumption of water, alcohol and cold drink in Raj.
दहे ला म पानी, मिदरा और को िडं क की कुल खपत राज म पानी, मिदरा और को िडं क की खपत का
िकतने ितशत है ।
A. 111 % B. 95 % C. 90 %
D. 108 % E. None of these

127. If YTC bought water, alcohol and cold drink at Rs 20, Rs 100 and Rs 50 (per liter)
respectively, them find out the total expenditure of YTC on these items.
यिद वाईटीसी ने पानी, मिदरा और को िडं क मशः 20 पये, 100 पये और 50 पये ( ित लीटर) पर
खरीदे , तो उ इन व ुओं पर वाईटीसी का कुल खच िकतना है ।
A. Rs 12100 B. Rs 11200 C. Rs 10350
D. Rs 12750 E. None of these

128. Find the ratio between the consumption of alcohol and cold drink together in Hoberoi to the
consumption of alcohol and cold drink together in The Dheela respectively.
होबेरोई म मिदरा और को िडं क की खपत और द ढे ला म एक साथ मिदरा और को िडं क की खपत के
बीच अनुपात ात कर।
A. 5 : 6 B. 9:7 C. 14 : 17
D. 10 : 11 E. None of these

129. If water was bought at an average of Rs 40/lt in all four restaurants and alcohol was bought
at an average of Rs 700/lt in all four restaurants, then find out the difference between
expenditure on alcohol and water by all four restaurants combined.
यिद सभी चार रे रां म पानी 40 पये / लीटर के औसत से खरीदा गया था और सभी चार रे रां म मिदरा
700 पये / लीटर के औसत से खरीदी गई थी, तो संयु प से सभी चार रे रां ारा मिदरा और पानी पर
खच के बीच अंतर का पता लगाएं ।
A. Rs 201000 B. Rs 190100 C. Rs 191000
D. Rs 185600 E. None of these

130. If the Dheela bought its water at Rs 25/lt and sold it with a profit of 250%, then find out the
profit made by The Dheela on water.
यिद ढे ला ने अपना पानी 25 पये / लीटर म खरीदा और उसे 250% के लाभ के साथ बेचा, तो पानी पर दी
ढे ला ारा िकए गए लाभ का पता लगाएं ।
A. Rs 6125 B. Rs 4375 C. Rs 4735
D. Rs 5125 E. None of these

The pie chart given below shows the % distribution of the number of seats in five different
theatres. The table given below shows the percentage of tickets sold by each theatre out of the
total number of seats. It also shows the price per ticket in each theatre.
नीचे िदया गया पाई चाट पां च अलग-अलग िथएटरो ं म सीटो ं की सं ा का % िवतरण दशाता है । नीचे दी गई तािलका
ेक िथएटर ारा कुल सीटो ं की सं ा म से बेची गई िटकटो ं का ितशत िदखाती है । यह ेक िथएटर म ित
िटकट की कीमत भी दशाता है ।

PAGE h ps://t.me/studified h ps://instagram.com/aashisharorasocial?utm_medium=copy_link


Follow
240 Aashish Arora on: h ps://youtube.com/channel/UCYa4_JrOrf8R5Kz2uOtccXQ https://www.facebook.com/aashisharorasocial/
अ ास by Aashish Arora
(SBI/IBPS/RBI/LIC/All other banking and insurance Exams)

Total Seats in Theatre

24%
408

18%
10%
14%

Theatre A Theatre B Theatre C Theatre D Theatre E

Theatre % of Tickets Sold Price of one cket (In Rs)

A 25 220

B 50 140

C 75 170

D 45 250

E 25 180

131. Find the total revenue generated by Theatre D.


िथएटर डी ारा उ कुल राज का पता लगाएं ।
A. 12500 B. 13500 C. 14500
D. 15500 E. None of these

132. Find the difference between the number of tickets sold by Theatre A and B.
A और B ारा बेचे गए िटकटो ं की सं ा के बीच अंतर ात कीिजए।
A. 32 B. 33 C. 34
D. 36 E. None of these

133. Revenue collected by Theatre C is how much percent less/more than the revenue
collected by Theatre D?
Theatre C ारा एक िकया गया राज Theatre D ारा एकि त राज से िकतने ितशत कम / अिधक
है ?
A. 54.33 B. 58.66 C. 55.25
D. 60.25 E. None of these

134. Find the average number of tickets sold by each of the five theatres.

PAGE h ps://t.me/studified h ps://instagram.com/aashisharorasocial?utm_medium=copy_link


Follow
241 Aashish Arora on: h ps://youtube.com/channel/UCYa4_JrOrf8R5Kz2uOtccXQ https://www.facebook.com/aashisharorasocial/
अ ास by Aashish Arora
(SBI/IBPS/RBI/LIC/All other banking and insurance Exams)

ेक पाँ च िसनेमाघरो ं ारा बेचे जाने वाले िटकटो ं की औसत सं ा ात कीिजए।


A. 92.4 B. 94.5 C. 95.8
D. 96.5 E. None of these

135. Find the difference between the revenues generated by Theatre A and E.
Theatre A और E ारा उ राज के बीच का अंतर ात कीिजये।
A. 2510 B. 2515 C. 2520
D. 2540 E. None of these

The radar graph given below shows total number of trees (mango + guava) in five different farms
and the number of mango trees in the farm.
नीचे िदए गए रडार ाफ पां च अलग-अलग खेतो ं म पेड़ो ं (आम + अम द) की कुल सं ा और खेत म आम के पेड़ो ं
की सं ा को दशाता है ।

Total Trees
Farm P
64
60
56
Farm U 52 Farm Q
48
44
40

Farm T Farm R

Farm S

Total Mango Trees


Farm P
40
36
32
Farm U Farm Q
28
24
20

Farm T Farm R

Farm S

PAGE h ps://t.me/studified h ps://instagram.com/aashisharorasocial?utm_medium=copy_link


Follow
242 Aashish Arora on: h ps://youtube.com/channel/UCYa4_JrOrf8R5Kz2uOtccXQ https://www.facebook.com/aashisharorasocial/
अ ास by Aashish Arora
(SBI/IBPS/RBI/LIC/All other banking and insurance Exams)

136. What is the ratio of total number of mango trees in Farm P and Farm R together to total
number of trees in Farm S?
फाम P और फाम R म आम के पेड़ो ं की कुल सं ा और Farm S के पेड़ो ं की कुल सं ा का अनुपात ा
है ?
A. 1:3 B. 3:5 C. 5:6
D. 7:6 E. None of these

137. On an average there are 47 mango fruits on each mango tree and 53 guavas on each
guava tree. Find the total number of fruits in Farm T.
ेक आम के पेड़ पर औसतन 47 आम और ेक अम द के पेड़ पर 53 अम द होते ह। फाम टी म
फलो ं की कुल सं ा ात कीिजए।
A. 3148 B. 3150 C. 3152
D. 3154 E. None of these

138. The number of trees in Farm R is what percent of the total number of trees in Farm P, Farm
Q, Farm T and Farm S?
फाम आर म पेड़ो ं की सं ा फाम पी, फाम ू, फाम टी और फाम एस म पेड़ो ं की कुल सं ा का िकतना
ितशत है ?
A. 12.8 B. 14.8 C. 16.8
D. 20.8 E. None of these

139. Number of guava trees in Farm S is how much percent more/less than the same in Farm
R?
फाम एस म अम द के पेड़ो ं की सं ा फाम आर से िकतने ितशत अिधक / कम है ?
A. 61.4 B. 71.4 C. 74.8
D. 68.8 E. None of these

140. What is the difference between number of mango trees in Farm P and Farm Q together
and number of guava trees in Farm P and Farm Q together?
फ़ाम पी और फ़ाम ू म आम के पेड़ो ं की सं ा और फ़ाम पी और फ़ाम ू म अम द के पेड़ो ं की सं ा म
ा अंतर है ?
A. 20 B. 25 C. 35
D. 30 E. None of these

The pie chart given below shows the degree distribution of total number teachers in five cities.
The second pie chart given below shows the degree distribution of number of graduate teachers
in all five cities.

नीचे िदया गया पाई चाट पां च शहरो ं म कुल िश को ं की िड ी िवतरण को दशाता है । नीचे िदया गया दू सरा पाई चाट
सभी पां च शहरो ं म ातक िश को ं की सं ा के िवतरण को दशाता है ।

PAGE h ps://t.me/studified h ps://instagram.com/aashisharorasocial?utm_medium=copy_link


Follow
243 Aashish Arora on: h ps://youtube.com/channel/UCYa4_JrOrf8R5Kz2uOtccXQ https://www.facebook.com/aashisharorasocial/
अ ास by Aashish Arora
(SBI/IBPS/RBI/LIC/All other banking and insurance Exams)

Total teachers = 1500

72°
90°

36°
36°

126°

City P City Q City R City S City T

Total graduate teachers = 700

43.2° 36°

54° 64.8°

162°

City P City Q City R City S City T

141. What is the ratio of number of post-graduate to graduate teachers in City P?


िसटी पी म ातको र और ातक िश को ं की सं ा का अनुपात ा है ?
A. 21:5 B. 22:8 C. 23:7
D. 24:5 E. None of these

142. What is the difference between number of post-graduate and graduate teachers in City Q?
िसटी ू म ातको र और ातक िश को ं की सं ा म ा अंतर है ?
A. 100 B. 101 C. 102
D. 103 E. None of these

143. Number of graduate teachers in City R is how much percent more/less than number of
post-graduate teachers in same City?

PAGE h ps://t.me/studified h ps://instagram.com/aashisharorasocial?utm_medium=copy_link


Follow
244 Aashish Arora on: h ps://youtube.com/channel/UCYa4_JrOrf8R5Kz2uOtccXQ https://www.facebook.com/aashisharorasocial/
अ ास by Aashish Arora
(SBI/IBPS/RBI/LIC/All other banking and insurance Exams)

िसटी आर म ातक िश को ं की सं ा ातको र िश को ं की सं ा से िकतने ितशत अिधक / कम है ?


A. 50 B. 55 C. 60
D. 65 E. None of these

144. Number of post-graduate teachers in City S is 25% of total number of teachers in City U. If
the ratio of number of post-graduate to graduate teachers in City U is 3:2, then find the
number of post-graduate teachers in City U.
िसटी एस म पो - ेजुएट िश को ं की सं ा, िसटी यू म कुल िश को ं की सं ा का 25% है । यिद िसटी यू म
पो - ेजुएट और ेजुएट िश को ं की सं ा का अनुपात 3: 2 है , तो शहर U म ातको र िश को ं की
सं ा का पता लगाएं ।
A. 100 B. 108 C. 110
D. 120 E. None of these

145. What is the number of post-graduate teachers in City T?


िसटी टी म ातको र िश को ं की सं ा िकतनी है ?
A. 279 B. 281 C. 289
D. 291 E. None of these

The given bar graph shows the average number of employees (Officer + clerks) who applied for
promotion and the line graph shows the percentage of employees who got the promotion out of
total number of employees who applied, in five different banks.

िदए गए बार ाफ म कमचा रयो ं की औसत सं ा (अिधकारी + क) दशाई गई है , िज ोन


ं े पदो ित के िलए
आवेदन िकया था और लाइन ाफ उन कमचा रयो ं का ितशत िदखाता है , िज ोन
ं े पां च अलग-अलग बको ं से, लागू
िकए गए कुल कमचा रयो ं म से पदो ित ा की थी।

8500
7995
8000
7500 7375
7175
6900
7000
6500
6040
6000
5500
5000
PNB SBI Canara Bank Bank Of Union Bank
Baroda

Average number of Officer and Clerk

PAGE h ps://t.me/studified h ps://instagram.com/aashisharorasocial?utm_medium=copy_link


Follow
245 Aashish Arora on: h ps://youtube.com/channel/UCYa4_JrOrf8R5Kz2uOtccXQ https://www.facebook.com/aashisharorasocial/
अ ास by Aashish Arora
(SBI/IBPS/RBI/LIC/All other banking and insurance Exams)

46

41

36

31

26

21

16
PNB SBI Canara Bank Bank Of Union Bank
Baroda

% of employees who got promo on

146. 40% of the employees who got the promotion in PNB and 50% of the employees who got
the promotion in SBI, were Officers. Find the number of clerks who got the promotion in
PNB and SBI, together.
PNB म मोशन पाने वाले कमचा रयो ं म से 40% और SBI म मोशन पाने वाले 50% कमचारी, अिधकारी
थे। PNB और SBI म मोशन पाने वाले क की सं ा एक साथ ात कीिजए।
A. 4768 B. 4824 C. 4968
D. 5014 E. None of these

147. Find the ratio of the number of employees who got the promotion in Canara Bank to the
number of employees who did not get the promotion in PNB.
कैनरा बक म पदो ित पाने वाले कमचा रयो ं की सं ा और PNB म पदो ित न पाने वाले कमचा रयो ं की
सं ा का अनुपात ात कीिजए।
A. 150:577 B. 151:590 C. 152:592
D. 150:563 E. None of these

148. Find the difference between the number of employees who got the promotion in Canara
Bank and Union Bank, together and number of employees who did not get the promotion in
SBI.
केनरा बक और यूिनयन बक म पदो ित पाने वाले कमचा रयो ं की सं ा और SBI म पदो ित नही ं पाने वाले
कमचा रयो ं की सं ा के बीच अंतर ात कीिजए।
A. 3701 B. 3702 C. 3703
D. 3704 E. None of these

149. Out of the total number of employees who did not get the promotion in Canara Bank, 6660
were clerks. Find the total number of officers who did not get the promotion in Canara
Bank.
केनरा बक म िजन कमचा रयो ं को पदो ित नही ं िमली, उनम से 6660 क थे। उन अिधका रयो ं की कुल
सं ा ात कीिजए िज केनरा बक म पदो ित नही ं िमली।

PAGE h ps://t.me/studified h ps://instagram.com/aashisharorasocial?utm_medium=copy_link


Follow
246 Aashish Arora on: h ps://youtube.com/channel/UCYa4_JrOrf8R5Kz2uOtccXQ https://www.facebook.com/aashisharorasocial/
अ ास by Aashish Arora
(SBI/IBPS/RBI/LIC/All other banking and insurance Exams)

A. 2200 B. 2300 C. 2400


D. 2500 E. None of these

150. The total number of employees who applied for promotion in Axis Bank is 530 more than
the number of employees who did not get the promotion in Bank Of Baroda. If 4715
employees who applied for promotion in Axis Bank were Officers, then find the number of
clerks who applied for promotion in Axis Bank.
ए स बक म पदो ित के िलए आवेदन करने वाले कमचा रयो ं की कुल सं ा बक ऑफ बड़ौदा म पदो ित
नही ं पाने वाले कमचा रयो ं की सं ा से 530 अिधक है । यिद ए स बक म पदो ित के िलए आवेदन करने
वाले 4715 कमचारी अिधकारी थे, तो उन क की सं ा ात कर, िज ोन
ं ेए स बक म पदो ित के िलए
आवेदन िकया था।
A. 4775 B. 4785 C. 4795
D. 4805 E. None of these

Directions : Study the information carefully and answer the following questions :

The pie charts show the percentage distribution of Doctors in six cities.
The line graph shows the number of males doctors in six cities.
The total number of Doctors in sic cities are 4500

िनदश: जानकारी का ानपूवक अ यन कर और िन िल खत ो ं के उ र द:

पाई चाट छह शहरो ं म डॉ रो ं के ितशत िवतरण को दशाता है ।


रे खा ाफ छह शहरो ं म पु ष डॉ रो ं की सं ा को दशाता है ।
इन शहरो ं म डॉ रो ं की कुल सं ा 4500 है ।

Percentage wise distribu on of Doctors in


six ci es

6%
14% City A
21% City B
16% City C
City D
15% City E
28% City F

PAGE h ps://t.me/studified h ps://instagram.com/aashisharorasocial?utm_medium=copy_link


Follow
247 Aashish Arora on: h ps://youtube.com/channel/UCYa4_JrOrf8R5Kz2uOtccXQ https://www.facebook.com/aashisharorasocial/
अ ास by Aashish Arora
(SBI/IBPS/RBI/LIC/All other banking and insurance Exams)

700

600 600
500 500
400 400
300

200 200
100 100 100
0
City A City B City C City D City E City F
Male Doctor in each city

151. What is total number of male Doctors in city F, Female doctors in city C and female doctors
in city B together?
शहर F म पु ष डॉ रो,ं शहर C म मिहला डॉ रो ं और शहर B म मिहला डॉ रो ं की कुल सं ा िकतनी
है ?
A. 1080 B. 1800 C. 1200
D. 1260 E. None of these

152. The number of female doctors in city D is approximately what percent of the total number of
doctors in city A?
शहर D म मिहला डॉ रो ं की सं ा शहर A म डॉ रो ं की कुल सं ा का लगभग िकतना ितशत है ?
A. 84% B. 91% C. 64%
D. 72% E. None of these

153. In which city is the number of male doctors more than the number of female doctors ?
िकस शहर म पु ष डॉ रो ं की सं ा मिहला डॉ रो ं की सं ा से अिधक है ?
A. City B only B. City E only C. Both City B and City E
D. City C only E. None of these

154. What is the difference between the number of female doctors in city F and the total number
of doctors in city E?
शहर F म मिहला डॉ रो ं की सं ा और शहर E म डॉ रो ं की कुल सं ा के बीच का अंतर िकतना है ?
A. 800 B. 650 C. 625
D. 775 E. None of these

155. What is the ratio of the number of male doctors in city C to the number of female doctors in
City B?
शहर C म पु ष डॉ रो ं की सं ा का शहर B म मिहला डॉ रो ं की सं ा से अनुपात िकतना है ?
A. 11:12 B. 15:13 C. 8:11
D. 15:8 E. None of these

PAGE h ps://t.me/studified h ps://instagram.com/aashisharorasocial?utm_medium=copy_link


Follow
248 Aashish Arora on: h ps://youtube.com/channel/UCYa4_JrOrf8R5Kz2uOtccXQ https://www.facebook.com/aashisharorasocial/
अ ास by Aashish Arora
(SBI/IBPS/RBI/LIC/All other banking and insurance Exams)

Directions : Study the information carefully and answer the following questions :
The bar graph shows the percentage of plastic tables manufactured by companies A, B and C
during different years.
The total number of tables = total number of plastic tables + total number of wooden tables.

िनदश: बार ाफ िविभ वष के दौरान कंपिनयो ं ए, बी और सी ारा िनिमत ा क टे बल का ितशत


दशाता है ।
टे बल की कुल सं ा= ा क टे बल की कुल सं ा लकड़ी की टे बल की कुल सं ा।

100
90
80%
80
70
60
50%
50
40% 40%
40
30 25%
20% 20%
20
10% 10%
10
0
2018 2019 2020
Company A Compnay B Company C

156. If the total number of table manufactured by Company A, B and C in 2019 was
14000,10000 and 2000 respectively, then what was the difference between the total
number of plastic and wooden table manufactured by 3 companies together?
यिद 2019 म कंपनी A, B और C ारा िनिमत टे बल की कुल सं ा मशः 14000,10000 और 2000 थी,
तो 3 कंपिनयो ं ारा एकसाथ िनिमत ा क और लकड़ी की टे बल की कुल सं ा के बीच का अंतर िकतना
था?
A. 2400 B. 2800 C. 3200
D. 3600 E. None of these

157. The total number of table manufactured by company A in 2018 and company C in 2020
was equal. If the total number of plastic table manufactured by Company A in 2018 and
Company C in 2020 together was 2800, then what was the total number of table
manufactured by A in 2018 ?
2018 म कंपनी A और 2020 म कंपनी C ारा िनिमत टे बल की कुल सं ा बराबर थी। यिद 2018 म कंपनी
A ारा और 2020 म कंपनी C ारा िनिमत ा क टे बल की कुल सं ा 2800 थी, तो 2018 म A ारा
िनिमत टे बल की कुल सं ा िकतनी थी?
A. 7000 B. 8000 C. 9000
D. 6000 E. None of these

158. The total number of tables manufactured by company B in 2018 and 2019 were 6000 and

PAGE h ps://t.me/studified h ps://instagram.com/aashisharorasocial?utm_medium=copy_link


Follow
249 Aashish Arora on: h ps://youtube.com/channel/UCYa4_JrOrf8R5Kz2uOtccXQ https://www.facebook.com/aashisharorasocial/
अ ास by Aashish Arora
(SBI/IBPS/RBI/LIC/All other banking and insurance Exams)

10000 respectively. The number of wooden tables manufactured by company B in 2019


was what percent (approximately) of wooden tables manufactured by company B in 2018?
2018 और 2019 म कंपनी B ारा िनिमत टे बल की कुल सं ा मशः 6000 और 10000 थी। 2019 म
कंपनी B ारा िनिमत लकड़ी की मेज की सं ा 2018 म कंपनी B ारा िनिमत लकड़ी की मेज का ितशत
(लगभग) ा थी?
A. 37% B. 35% C. 33%
D. 29% E. None of these

159. The total number of table manufactured by companies A ,B and C in 2020 was 31000,
23500 and 20000 respectively. If 14%, 26% and 11% of plastic table manufactured by
companies A, B and C in 2020 were defective and none of the wooden table were
defective, then what was the total number of defective tables manufactured by 3
companies together in 2020?
2020 म कंपिनयो ं A,B और C ारा िनिमत टे बल की कुल सं ा मशः 31000,23500 और 20000 थी।
यिद 2020 म कंपिनयो ं A, B और C ारा िनिमत ा क टे बल का 14%, 26% और 11% खराब था और
लकड़ी की कोई भी टे बल ख़राब नही ं थी, तो 2020 म 3 कंपिनयो ं ारा िनिमत दोषपूण टे बल की कुल सं ा
िकतनी थी?
A. 3325 B. 5323 C. 3523
D. 3532 E. None of these

160. In 2020, the total number of tables manufactured by companies A, B and C was 9000, 7500
and 8000 respectively. What was the ratio of the total number of plastic table manufactured
by Company B to the total number of plastic tables manufactured by companies A and C
together?
2020 म, कंपिनयो ं A, B और C ारा िनिमत टे बलो ं की कुल सं ा मशः 9000, 7500 और 8000 थी।
कंपनी B ारा िनिमत ा क टे बल की कुल सं ा का कंपनी A और C ारा िमलकर िनिमत ा क
टे बल की कुल सं ा से अनुपात िकतना था?
A. 19:15 B. 19:13 C. 15:13
D. 16:19 E. None of these

Directions : Study the information carefully and answer the following questions :
The table given below shows the distribution of the number of bag sold, price of each bag and
the total revenue generated by beach of five seller in 2020. Total number of bags is 680.
िनदश: जानकारी का ानपूवक अ यन कर और िन िल खत ो ं के उ र द:
नीचे दी गई तािलका म बेचे गए बैग की सं ा, ेक बैग की कीमत और 2020 म पां च िव े ताओं के बीच ारा
उ कुल राज का िवतरण िदखाया गया है । बैग की कुल सं ा 680 है ।

Seller Number of Bags sold Price of one Bag Total revenue

A 4x+5 500 2000x+2500

B 3x 480 1440x

C 5x-15 360 1800x-5400

D 4x+10 300 1200x+3000

E 3x+15 400 1200x+6000

PAGE h ps://t.me/studified h ps://instagram.com/aashisharorasocial?utm_medium=copy_link


Follow
250 Aashish Arora on: h ps://youtube.com/channel/UCYa4_JrOrf8R5Kz2uOtccXQ https://www.facebook.com/aashisharorasocial/
अ ास by Aashish Arora
(SBI/IBPS/RBI/LIC/All other banking and insurance Exams)

161. What is ratio of the number of bags sold by C in 2020 to that by E in 2020?
2020 म C ारा बेचे गए बैगो ं की सं ा का 2020 म E ारा बेचे गए बैगो ं की सं ा से अनुपात िकतना है ?
A. 4:3 B. 1:2 C. 5:4
D. 7:5 E. None of these

162. What is the average number of bag sold by A, B, C and D in 2020?


2020 म A, B, C और D ारा बेचे गए बैग की औसत सं ा ा है ?
A. 150 B. 145 C. 140
D. 135 E. None of these

163. Find the difference between the total revenue generated by B in 2020 and the total
revenue generated by D in 2020.
2020 म B ारा उ कुल राज और 2020 म D ारा उ कुल राज के बीच का अंतर ात कीिजए।
A. 4400 B. 4800 C. 5800
D. 5400 E. None of these

164. Total revenue generated by A in 2020 is how much percent more/less than the total number
of revenue generated by E in 2020.
2020 म A ारा उ कुल राज , 2020 म E ारा उ कुल राज से िकतने ितशत अिधक/कम है ?
A. 41.04% B. 51.04% C. 57.04%
D. 67.04% E. None of these

165. What is average revenue generated by C and D?


C और D ारा उ औसत राज ा है ?
A. 51000 B. 53000 C. 53100
D. 51300 E. None of these

Direction : Study the data carefully and answer the following questions :
The Line graph shows the total number of earphone(wired + bluetooth) manufactured and the
number of wired earphone manufactured by five different company. Total number of earphone
manufactured is 5000. Total number of wired earphone manufactured is 2500
िनदश : आं कड़ो ं का ानपूवक अ यन कर और िन िल खत ो ं के उ र द :
लाइन ाफ़ िनिमत इयरफ़ोन (वायड + ूटूथ) की कुल सं ा और पाँ च अलग-अलग कंपनी ारा िनिमत वायड
इयरफ़ोन की सं ा को दशाता है । िनिमत इयरफ़ोन की कुल सं ा 5000 है । िनिमत वायड इयरफ़ोन की कुल
सं ा 2500 है

28x
24x

18x
16x
14x 28y
24y
16y 18y
14y

Boat Mivi Skullcandy JBL Realme

Total number of earphone manufacture Number of wired earphone manufactured

PAGE h ps://t.me/studified h ps://instagram.com/aashisharorasocial?utm_medium=copy_link


Follow
251 Aashish Arora on: h ps://youtube.com/channel/UCYa4_JrOrf8R5Kz2uOtccXQ https://www.facebook.com/aashisharorasocial/
अ ास by Aashish Arora
(SBI/IBPS/RBI/LIC/All other banking and insurance Exams)

166. What is the difference between the number of Wired earphone manufactured by Mivi and
Skullcandy together and the number of Bluetooth earphone manufactured by JBL and
realme.
Mivi और Skullcandy ारा एक साथ िनिमत वायड ईयरफोन की सं ा और JBL और realme ारा
िनिमत ूटूथ ईयरफोन की सं ा के बीच िकतना अंतर है ।
A. 100 B. 120 C. 130
D. 110 E. None of these

167. Total number of Bluetooth earphone manufactured by Skullcandy is how much percent of
the total earphone manufactured by Boat?
Skullcandy ारा िनिमत ूटूथ इयरफ़ोन की कुल सं ा, Boat ारा िनिमत कुल इयरफ़ोन का िकतना
ितशत है ?
A. 50% B. 45% C. 30%
D. 25% E. None of these

168. Total number of wired earphone manufactured by MI is 60% of total number of earphone
manufactured by it, which is equal to average number of earphone manufactured by five
companies. Find the number of Bluetooth earphone manufactured by MI.
MI ारा िनिमत वायड ईयरफोन की कुल सं ा इसके ारा िनिमत ईयरफोन की कुल सं ा का 60% है ,
और यह पां च कंपिनयो ं ारा िनिमत ईयरफोन की औसत सं ा के बराबर है । MI ारा िनिमत ूटूथ
इयरफ़ोन की सं ा ात कीिजए।
A. 350 B. 400 C. 450
D. 500 E. None of these

169. What is the ratio of the number of Bluetooth manufactured by Boat and Mivi together and
Wired earphone manufactured by JBL and Realme together.
Boat और Mivi ारा एकसाथ िनिमत ूटूथ और JBL और Realme ारा एक साथ िनिमत वायड ईयरफोन
की सं ा का अनुपात िकतना है ?
A. 13:11 B. 9:8 C. 23:19
D. 26:23 E. None of these

170. If 80% earphone manufactured by Boat is non defective and ratio of defective wired and
bluetooth earphone is 9:5, then find number of non defective Bluetooth earphone.
यिद बोट ारा िनिमत 80% ईयरफोन खराब नही ं है और खराब वायड और ूटूथ ईयरफोन का अनुपात 9:5
है , तो नॉन िडफे व ूटूथ ईयरफोन की सं ा ात कीिजए।
A. 720 B. 800 C. 700
D. 820 E. None of these

Directions -: Study the following bar chart carefully and answer the questions given
beside
The bar chart shows the gross state income of 3 months before the GST (in billion rupees) of five
different states of India

िनदश : िन िल खत बार चाट का ानपूवक पिढ़ए और नीचे िदए गए ो ं के उ र दीिजए।


बार चाट भारत के पां च अलग-अलग रा ो ं के जीएसटी (अरब पये म) से 3 महीने पहले की सकल रा आय को
दशाता है

PAGE h ps://t.me/studified h ps://instagram.com/aashisharorasocial?utm_medium=copy_link


Follow
252 Aashish Arora on: h ps://youtube.com/channel/UCYa4_JrOrf8R5Kz2uOtccXQ https://www.facebook.com/aashisharorasocial/
अ ास by Aashish Arora
(SBI/IBPS/RBI/LIC/All other banking and insurance Exams)

171. What is the difference between gross state income of UP, HR and MH together in May and
that of UK, MP and MH together in April? (In billion rupees)
मई म यूपी, एचआर और एमएच की िमलाकर सकल रा आय और अ ैल म यूके, एमपी और एमएच की
सकल रा आय के बीच िकतना अंतर है ? (अरब पये म)
A. 595 B. 527 C.589
D. 570 E. None of these

172. The gross state income of UP in June is what per cent of total gross state income of UP in
all the given months together?
जून म यूपी की सकल रा आय सभी िदए गए महीनो ं म यूपी की कुल सकल रा आय का िकतना ितशत
है ?
A. 28 % B. 35 % C. 42 %
D. 26 % E. None of these

173. What is the ratio of the gross state income of MP, UK and HR together in April to the gross
state income of UK, HR and MH together in June?
अ ैल म एमपी, यूके और एचआर की सकल रा आय का जून म यूके, एचआर और एमएच की सकल रा
आय से अनुपात िकतना है ?
A. 289 : 251 B. 271 : 257 C. 259 : 268
D. 251 : 276 E. None of these

174. What is the average gross state income of all the states in May? (in billion rupees)
मई म सभी रा ो ं की औसत सकल रा आय ा है ? (अरब पये म)
A. 1109.4 B. 1119.5 C. 1524.4
D. 1247.8 E. None of these

175. The average gross state income of UP is how much per cent less than that of MP?
उ र दे श की औसत सकल रा आय म दे श की तुलना म िकतने ितशत कम है ?
A. 12.65 % B. 17.68 % C. 14.72 %
D. 15.75 % E. None of these

PAGE h ps://t.me/studified h ps://instagram.com/aashisharorasocial?utm_medium=copy_link


Follow
253 Aashish Arora on: h ps://youtube.com/channel/UCYa4_JrOrf8R5Kz2uOtccXQ https://www.facebook.com/aashisharorasocial/
अ ास by Aashish Arora
(SBI/IBPS/RBI/LIC/All other banking and insurance Exams)

Direction: The line graph given below gives the information about the percentage increase in
the length of a python over the previous year in the first five consecutive years of its life. Assume
that every python is born 200/3 cm long.

िनदश: नीचे िदया गया लाइन ाफ एक अजगर की लंबाई म उसके जीवन के पहले पां च लगातार वष म िपछले वष
की तुलना म ितशत वृ के बारे म जानकारी दे ता है । मान ल िक ेक अजगर का ज 200/3 सेमी लंबा होता है ।

60%
50%
50%
40%
40%
30% 25%
20%
20%
10%
10%
0%
1st year 2nd year 3rd year 4th year 5th year

176. In the fourth year of its life, what was the increment in its length over the previous year?
अपने जीवन के चौथे वष म, िपछले वष की तुलना म इसकी लंबाई म िकतनी वृ ई?
A. 35 cm B. 40 cm C. 50 cm
D. 60 cm E. None of these

177. The length of a 5-year-old python will be how many cm more than that of a 3-year-old
python?
5 साल के अजगर की लंबाई 3 साल के अजगर से िकतने सटीमीटर ादा होगी?
A. 45 B. 81 C. 71
D. 91 E. None of these

178. The weight of a python of length greater than 175 cm but less than 180 cm is 200 kg. the
weight of how many years old python will be 200 kg?
175 सेमी से अिधक लंबाई के अजगर लेिकन 180 सेमी से कम लंबाई के अजगर का बजन 200 िकलो ाम
है । िकतना साल पुराना अजगर 200 िकलो का होगा?
A. 4 years B. 5 years C. 6 years
D. 7 years E. Can't be determined

179. In the sixth year of its life, its length is increased by 50% over the previous year then what
was the increase in length (in cm) in the sixth year of its life?
अपने जीवन के छठे वष म, िपछले वष की तुलना म इसकी लंबाई 50% बढ़ी है , तो उसके जीवन के छठे वष म
लंबाई (सेमी म) म वृ ा थी?
A. 118.5 cm B. 125.5 cm C. 115.5 cm
D. 116.5 cm E. None of these

180. In a forest, there are 6 four year old, 5 three year old, 4 five year old and 10 two year old
pythons , then what is the average length of all the pythons of the forest?

PAGE h ps://t.me/studified h ps://instagram.com/aashisharorasocial?utm_medium=copy_link


Follow
254 Aashish Arora on: h ps://youtube.com/channel/UCYa4_JrOrf8R5Kz2uOtccXQ https://www.facebook.com/aashisharorasocial/
अ ास by Aashish Arora
(SBI/IBPS/RBI/LIC/All other banking and insurance Exams)

जंगल म, 6 चार वष य, 5 तीन वष य, 4 पां च वष य और 10 दो वष य अजगर ह, तो जंगल के सभी अजगरो ं की


औसत लंबाई ा है ?
A. 167.36 cm B. 192.48 cm C. 154.35 cm
D. 160.25 cm E. None of these

Direction : Study the data carefully and answer the following questions :
The bar diagram given below shows the percentage distribution of total expenditure of a
company under various expense during a year.

िनदश : आं कड़ो ं का ानपूवक अ यन कर और िन िल खत ो ं के उ र द :


नीचे िदया गया दं ड आरे ख एक वष के दौरान िविभ यो ं के अंतगत एक कंपनी के कुल य का ितशत िवतरण
दशाता है ।

181. The expenditure on Expense G is what percent more /less than the expenditure on
Expense B?
य G पर य, य B पर य से िकतने ितशत अिधक/कम है ?
A. 30% B. 50% C. 40%
D. 20% E. None of these

182. What is the ratio of the total expenditure on Expense A and Expense B to the total
expenditure on Expense D and Expense G?
य A और य B पर कुल य का य D और य G पर कुल य का अनुपात िकतना है ?
A. 3:4 B. 5:12 C. 13:11
D. 9:11 E. None of these

183. If the expenditure on Expense C is Rs.2.10 crores, then the difference between the
expenditure on Expense B and Expense D?
यिद य C पर य 2.10 करोड़ पये है , तो य B और य D पर य के बीच का अंतर ा है ?
A. 25 lakhs B. 30 lakhs C. 35 lakhs
D. 40 lakhs E. None of these

PAGE h ps://t.me/studified h ps://instagram.com/aashisharorasocial?utm_medium=copy_link


Follow
255 Aashish Arora on: h ps://youtube.com/channel/UCYa4_JrOrf8R5Kz2uOtccXQ https://www.facebook.com/aashisharorasocial/
अ ास by Aashish Arora
(SBI/IBPS/RBI/LIC/All other banking and insurance Exams)

184. The total expenditure of the company is how many times the expenditure on Expense E?
कंपनी का कुल य, य E पर य का िकतना गुना है ?
A. 10 B. 5 C. 15
D. 20 E. None of these

185. If the Expenditure on Expense G is Rs.2.45 crores, then the total expenditure on Expense
C, Expense D and Expense E is?
यिद य G पर य 2.45 करोड़ पये है , तो य C, य D और य E पर कुल य है ?
A. 5.2 crores B. 4.2 crores C. 6.2 crores
D. 7.2 crores E. None of these

Direction : Study the data carefully and answer the following questions :
The Line graph below represents the production (in tonnes) and sales (in tonnes) of Company A
from 2015-2020.

The table given below represents the ratio of the production (in tonnes) of Company A to
production (in tonnes ) of company B, An d the ratio of sales (in tonnes) of Company A to the
sales (in tonnes) of Company B.

िनदश : आं कड़ो ं का ानपूवक अ यन कर और िन िल खत ो ं के उ र द :


नीचे िदया गया लाइन ाफ 2016-2020 तक कंपनी ए के उ ादन (टन म) और िब ी (टन म) का ितिनिध
करता है

नीचे दी गई तािलका कंपनी ए के उ ादन (टन म) का कंपनी बी के उ ादन (टन म) के अनुपात को दशाती है ,
कंपनी ए की िब ी (टन म) का अनुपात कंपनी बी की िब ी (टन म) से है ।

900
800
800 750
700
700 650
600
600 550 550 550
500
500 450
400
400
300
300

200
2015 2016 2017 2018 2019 2020
Produc on Sales

PAGE h ps://t.me/studified h ps://instagram.com/aashisharorasocial?utm_medium=copy_link


Follow
256 Aashish Arora on: h ps://youtube.com/channel/UCYa4_JrOrf8R5Kz2uOtccXQ https://www.facebook.com/aashisharorasocial/
अ ास by Aashish Arora
(SBI/IBPS/RBI/LIC/All other banking and insurance Exams)

YEAR Produc on (A:B) Sales (A:B)


2015 5:4 2:3
2016 8:7 11:12
2017 3:4 9:14
2018 11:12 4:5
2019 14:13 10:9
2020 13:14 1:1

186. What is the approximate percentage increase in the production of company A from the
year 2018 to the production of company A in the year 2019?
वष 2018 से कंपनी A के उ ादन म वष 2019 म कंपनी A के उ ादन म लगभग ितशत वृ िकतनी है ?
A. 23% B. 32% C. 27%
D. 17% E. None of these

187. The sales of Company A in the year 2018 was approximately what percent of the
production of Company A in the same year?
वष 2018 म कंपनी A की िब ी उसी वष कंपनी A के उ ादन का लगभग िकतना ितशत थी?
A. 73% B. 79% C. 57%
D. 61% E. None of these

188. What is the average production of Company B from the year 2015 to the year 2020?
वष 2015 से वष 2020 तक कंपनी B का औसत उ ादन िकतना है ?
A. 723 B. 548 C. 675
D. 754 E. None of these

189. What is the ratio of the total production of Company A to the total sales of Company A?
कंपनी A के कुल उ ादन का कंपनी A की कुल िब ी का अनुपात िकतना है ?
A. 72:83 B. 81:55 C. 71:55
D. 64:81 E. None of these
190. What is the ratio of production of Company B in the year 2015 to production of company B
in the years 2017?
वष 2015 म कंपनी B के उ ादन का वष 2017 म कंपनी B के उ ादन का अनुपात िकतना है ?
A. 2:3 B. 3:4 C. 5:7
D. 6:1 E. None of these

Direction: Study the given pie chart carefully to answer the questions that follow:
Percentage of students in a different class of class IV, V, VI, VII, VIII and IX.

िनदश: िदए गए पाई चाट का ानपूवक अ यन कर और नीचे िदए गए ो ं के उ र द:


क ा IV, V, VI, VII, VIII और IX की िविभ क ाओं म छा ो ं का ितशत।

PAGE h ps://t.me/studified h ps://instagram.com/aashisharorasocial?utm_medium=copy_link


Follow
257 Aashish Arora on: h ps://youtube.com/channel/UCYa4_JrOrf8R5Kz2uOtccXQ https://www.facebook.com/aashisharorasocial/
अ ास by Aashish Arora
(SBI/IBPS/RBI/LIC/All other banking and insurance Exams)

Percentage of students in different a class

CLASS IV CLASS V CLASS VI CLASS VII CLASS VIII CLASS IX

12%
8% 28%

25%
20%
7%

Percentage of boys in different a class


CLASS IV CLASS V CLASS VI CLASS VII CLASS VIII CLASS IX

12% 10%
5%

30%
35%

8%

191. How many girls are there in school in the class IV and VII together?
क ा IV और VII म िमलाकर ू ल म िकतनी लड़िकयां ह?
A. 798 B. 995 C. 879
D. 786 E. None of these

192. What will be the ratio of the number of girls in the class V, VIII and IX to the number of boys
in the class V, VI and VIII?
क ा V, VIII और IX म लड़िकयो ं की सं ा का क ा VI, VII और VIII म लड़को ं की सं ा से अनुपात ा
होगा?
A. 11:12 B. 11:47 C. 17:48
D. 47:13 E. None of these

193. The total number of boys is how much percent more/less than the total number of girls in
the school?
लड़को ं की कुल सं ा, ू ल म लड़िकयो ं की कुल सं ा से िकतने ितशत अिधक/कम है ?
A. 30.65% B. 84% C. 45.97%
D. 66.66% E. None of these

PAGE h ps://t.me/studified h ps://instagram.com/aashisharorasocial?utm_medium=copy_link


Follow
258 Aashish Arora on: h ps://youtube.com/channel/UCYa4_JrOrf8R5Kz2uOtccXQ https://www.facebook.com/aashisharorasocial/
अ ास by Aashish Arora
(SBI/IBPS/RBI/LIC/All other banking and insurance Exams)

194. The number of girls in class X is 40% more than total number of boys in class VIII. If the
total number of students in class X is 850, then find the number of boys in the class.
क ा X म लड़िकयो ं की सं ा, क ा VIII म लड़को ं की कुल सं ा से 40% अिधक है । यिद क ा X म
िव ािथयो ं की कुल सं ा 850 है , तो क ा म लड़को ं की सं ा ात कीिजए।
A. 540 B. 479 C. 577
D. 675 E. None of these

195. The ratio of the number of girls in the class IX and XI, 2:7 respectively. If the number boys in
the XI is 30% less than the number of girls, then find the number of boys in the class XI.
क ा IX और XI म लड़िकयो ं की सं ा का अनुपात मशः 2:7 है । यिद XI म लड़को ं की सं ा लड़िकयो ं
की सं ा से 30% कम है , तो क ा XI म लड़को ं की सं ा ात कीिजए।
A. 540 B. 735 C. 441
D. 280 E. None of these

Direction: Study the given bar graph and table carefully to answer the questions that
follow:
The following data interpretation shows the percentage-wise break up of 'P', 'Q', 'R' and 'S'
University students in three different categories (I, II and III)

िनदश: नीचे िदए गए ो ं के उ र दे ने के िलए िदए गए बार ाफ और तािलका का ानपूवक अ यन कर:


िन िल खत डे टा ा ा तीन अलग-अलग ेिणयो ं (I, II और III) म 'पी', ' ू', 'आर' और 'एस' िव िव ालय के
छा ो ं के ितशत-वार ेक अप को दशाती है ।
Universi es Total Students

P 18000

Q 27000

R 36000

S 32000

Categoty of Students in University


Category I Category II Category III

46%
44%
40%
40%
36% 37%
33% 32%
28%
24% 23%
17%

P Q R S

PAGE h ps://t.me/studified h ps://instagram.com/aashisharorasocial?utm_medium=copy_link


Follow
259 Aashish Arora on: h ps://youtube.com/channel/UCYa4_JrOrf8R5Kz2uOtccXQ https://www.facebook.com/aashisharorasocial/
अ ास by Aashish Arora
(SBI/IBPS/RBI/LIC/All other banking and insurance Exams)

196. What will be the average of the total number of categories (II and III) of universities 'P' and
'S' and all categories of university 'R'?
िव िव ालयो ं 'पी' और 'एस' की ेिणयो ं (II और III) की कुल सं ा और िव िव ालय 'आर' की सभी ेिणयो ं
की कुल सं ा का औसत ा होगा?'
A. 64090 B. 22320 C. 53306
D. 57063 E. None of these

197. If universities 'P' and 'S' interchange their values, then find the ratio of the total number of
categories (I and III) of universities 'Q' and 'S' (together) to the total number of all
categories of university 'P'
यिद िव िव ालय 'P' और 'S' अपने मू ो ं का आदान- दान करते ह, तो िव िव ालयो ं 'Q' और 'S' (एक
साथ) की ेिणयो ं की कुल सं ा (I और III) का िव िव ालय की सभी ेिणयो ं की कुल सं ा से अनुपात ात
कीिजए। 'P'
A. 89:101 B. 101:81 C. 81:89
D. 81:100 E. None of these

198. What is the total number in two universities together having the least students in category
II.
दो िव िव ालयो ं म ेणी II म सबसे कम छा ो ं की कुल सं ा िकतनी है
A. 13505 B. 12640 C. 13555
D. 12500 E. None of these

199. If university 'T' increased 25% of university 'S', then find the ratio of university 'T' to the
university 'R'.
यिद िव िव ालय 'T' ने िव िव ालय 'S' से 25% की वृ की, तो िव िव ालय 'T' का िव िव ालय 'R' से
अनुपात ात कीिजए।
A. 8:9 B. 3:10 C. 10:9
D. 3:8 E. None of these

200. Find the average of all universities of category II.


सभी िव िव ालयो ं का ेणी II का औसत ात कीिजए।
A. 1009 B. 9010 C. 1100
D. 1090 E. None of these

Directions -: Study the information carefully and answer the following questions :

The given line graph shows the total number of Watches (Analog + Digital) manufactured and
the number of Analog watch manufactured by five different companies

िनदश: जानकारी का ानपूवक अ यन कर और िन िल खत ो ं के उ र द:

िदया गया लाइन ाफ िनिमत घिड़यो ं (एनालॉग + िडिजटल) की कुल सं ा और पां च अलग-अलग कंपिनयो ं ारा
िनिमत एनालॉग घड़ी की सं ा को दशाता है

PAGE h ps://t.me/studified h ps://instagram.com/aashisharorasocial?utm_medium=copy_link


Follow
260 Aashish Arora on: h ps://youtube.com/channel/UCYa4_JrOrf8R5Kz2uOtccXQ https://www.facebook.com/aashisharorasocial/
अ ास by Aashish Arora
(SBI/IBPS/RBI/LIC/All other banking and insurance Exams)

1000
900
800 720
700 600
600 540
480 480
500
400 360 360
300 240
200
200 120
100
0
Company A Company B Company C Company D Company E

Total Number of Watches Analog Watches

201. Find the ratio of total number of watches manufactured in company A and C together to the
number of Digital watches manufactured in company B?
कंपनी A और C म िनिमत घिड़यो ं की कुल सं ा का कंपनी B म िनिमत िडिजटल घिड़यो ं की सं ा से
अनुपात ात कीिजए?
A. 45:11 B. 57:12 C. 64:17
D. 72:19 E. None of these

202. The total number of watches manufactured in company B and D, together is how much
percent more/less than the number of Digital watches manufactured in Company E.
कंपनी B और D म िनिमत घिड़यो ं की कुल सं ा, कंपनी E म िनिमत िडिजटल घिड़यो ं की सं ा से िकतने
ितशत अिधक/कम है ।
A. 245(7/12)% B. 285(5/7)% C. 214(11/12)%
D. 310(8/13)% E. None of these

203. Out of total number of watches in company C, 20% are defective . If 1/9th of digital watches
are defective of company C, then find the number of Analog which are non defective.
कंपनी C म घिड़यो ं की कुल सं ा म से 20% खराब ह। यिद कंपनी C की 1/9वी ं िडिजटल घिड़याँ ख़राब ह,
तो उन एनालॉग की सं ा ात कीिजए जो गैर दोषपूण ह।
A. 235 B. 242 C. 272
D. 265 E. None of these

204. Find the difference between number of analog watches of company B and number of
Digital watches of company D and E together.
कंपनी B की एनालॉग घिड़यो ं की सं ा और कंपनी D और E की िडिजटल घिड़यो ं की सं ा के बीच का
अंतर ात कीिजए।
A. 40 B. 50 C. 60
D. 70 E. None of these

205. The number of digital watches of Company F is 120 more than that of company C. If the
number of Analog watches of company F is 20% of the total number of watches

PAGE h ps://t.me/studified h ps://instagram.com/aashisharorasocial?utm_medium=copy_link


Follow
261 Aashish Arora on: h ps://youtube.com/channel/UCYa4_JrOrf8R5Kz2uOtccXQ https://www.facebook.com/aashisharorasocial/
अ ास by Aashish Arora
(SBI/IBPS/RBI/LIC/All other banking and insurance Exams)

manufactured in all five companies , then find the total number of watches manufactured
by Company F.
कंपनी F की िडिजटल घिड़यो ं की सं ा कंपनी C की तुलना म 120 अिधक है । यिद कंपनी F की एनालॉग
घिड़यो ं की सं ा सभी पां च कंपिनयो ं म िनिमत घिड़यो ं की कुल सं ा का 20% है , तो कंपनी F म िनिमत
घिड़यो ं की कुल सं ा ात कीिजए।
A. 900 B. 640 C. 720
D. 840 E. None of these

Directions : Study the information carefully and answer the following questions :
The given table show the total number of pencil (Apsara pencil + Nataraj pencil ) sold by a
Shopkeeper on five different days and the ratio of Apsara pencil and Nataraj pencil sold by the
shopkeeper.

िनदश -: िनदश: जानकारी का ानपूवक अ यन कर और िन िल खत ो ं के उ र द:


दी गई तािलका पां च अलग-अलग िदनो ं म एक दु कानदार ारा बेची गई पिसल (अ रा पिसल + नटराज पिसल) की
कुल सं ा और अ रा पिसल और नटराज पिसल का अनुपात िदखाती है

Ra o of the number of sold


Days Total Pencil Sold
(Apsara pencil : Nataraj pencil )

Monday 540 5:4


Tuesday 420 11:10
Wednesday 520 6:7
Thursday 480 13:11
Friday 525 3:4
Saturday 380 9:10
206. What is the number of Apsara pencil sold by the Shopkeeper on Tuesday?
मंगलवार को दु कानदार ारा बेची गई अ रा पिसल की सं ा िकतनी है ?
A. 300 B. 340 C. 220
D. 240 E. None of these

207. Number of Apsara pencil sold on Thursday is how much percent more than the number of
Nataraj Pencil sold on Monday?
गु वार को बेची गई अ रा पिसल की सं ा सोमवार को बेची गई नटराज पिसल की सं ा से िकतने
ितशत अिधक है ?
A. 8(1/3)% B. 6(1/3)% C. 4(1/3)%
D. 5(1/3)% E. None of these

208. What is the ratio of Natraj pencil sold on Tuesday to number of Natraj pencil sold on
Saturday.
मंगलवार को बेची गई नटराज पिसल का शिनवार को बेची गई नटराज पिसल की सं ा से अनुपात िकतना
है ?
A. 2:3 B. 3:1 C. 1:2
D. 1:1 E. None of these

PAGE h ps://t.me/studified h ps://instagram.com/aashisharorasocial?utm_medium=copy_link


Follow
262 Aashish Arora on: h ps://youtube.com/channel/UCYa4_JrOrf8R5Kz2uOtccXQ https://www.facebook.com/aashisharorasocial/
अ ास by Aashish Arora
(SBI/IBPS/RBI/LIC/All other banking and insurance Exams)

209. What is the average number of Apsara pencil sold on Wednesday and Saturday?
बुधवार और शिनवार को बेची गई अफसरा पिसल की औसत सं ा िकतनी है ?
A. 200 B. 240 C. 210
D. 220 E. None of these

210. What is difference between number of Apsara pencil and Nataraj pencil sold on Tuesday
and Thursday?
मंगलवार और गु वार को बेची गई अ रा पिसल और नटराज पिसल की सं ा के बीच का अंतर िकतना है ?
A. 90 B. 60 C. 72
D. 84 E. None of these

Directions -: In the following pie chart shows the degree distribution of laptops production in 5
states. Total investment for laptops production by director is 6000 crore rupees.

िनदश -: िन िल खत पाई चाट म 5 रा ो ं म लैपटॉप उ ादन का िड ी िवतरण िदखाया गया है । िनदे शक ारा
लैपटॉप उ ादन के िलए कुल िनवेश 6000 करोड़ पये है ।
Investment in laptops produc on (in degree)
in different states

45 Delhi
105
Banglore
81 Chennai
Kolkata
54 Mumbai
75

211. Investment in Delhi is what percentage more or less than that invest in kolkata?
िद ी म िनवेश, कोलकाता म िनवेश से िकतने ितशत अिधक या कम है ?
A. 33.97% B. 34.09% C. 29.62%
D. 13.25% E. None of these

212. What is the ratio of investment in Delhi and Chennai together to that in Bangalore and
Mumbai together?
िद ी और चे ई म एक साथ िनवेश का बंगलौर और मुंबई म िमलाकर िनवेश का अनुपात िकतना है ?
A. 20:11 B. 11:9 C. 7:9
D. 3:11 E. None of these

213. If 50% of Delhi's investment was reallocated to Kolkata and Mumbai states in the ratio 2:3
respectively, then, what is the new ratio of investment in Delhi to that in Mumbai?
यिद िद ी के िनवेश का 50% मश: 2:3 के अनुपात म कोलकाता और मुंबई रा ो ं को पुनः आवंिटत िकया
गया था, तो िद ी म िनवेश का मुंबई म िनवेश का नया अनुपात ा है ?

PAGE h ps://t.me/studified h ps://instagram.com/aashisharorasocial?utm_medium=copy_link


Follow
263 Aashish Arora on: h ps://youtube.com/channel/UCYa4_JrOrf8R5Kz2uOtccXQ https://www.facebook.com/aashisharorasocial/
अ ास by Aashish Arora
(SBI/IBPS/RBI/LIC/All other banking and insurance Exams)

A. 51:42 B. 43:29 C. 38:39


D. 35:51 E. None of these

214. In Kolkata's 40% investment goes to Tamil Nadu and 30% of the remaining goes to
Telangana and the rest goes to Assam. What is the investment of Assam (in crore rupees)?
कोलकाता म 40% िनवेश तिमलनाडु म जाता है और शेष का 30% तेलंगाना म जाता है और शेष असम म
जाता है । असम का िनवेश (करोड़ पये म) ा है ?
A. 698 B. 567 C. 45
D. 710 E. None of these

215. What percentage of total investment in Mumbai?


मुंबई म कुल िनवेश का िकतना ितशत है ?
A. 9.05% B. 14.79% C. 11%
D. 12.5% E. None of these

Direction: Study the Following information carefully and answer the related questions.
Following bar graph represent the data regarding quantity of different types of fruits (in kg) sold
by two store in a week

िनदश: िन िल खत जानकारी का ानपूवक अ यन कर और संबंिधत ो ं के उ र द।


िन िल खत बार ाफ एक स ाह म दो ोर ारा बेचे जाने वाले िविभ कार के फलो ं (िकलो म) की मा ा के संबंध
म डे टा का ितिनिध करता है
Quan ty of Fruits sold (in kg)
Store A Store B
360
320
280
260
240
190
160
140
120
80 90
60

Apple Mango Orange Guava Lichi Watermelon

216. Quantity of apple and mango sold by store A is approximately what percentage more/less
than quantity of lichi and watermelon sold by store B?
ोर A ारा बेचे गए सेब और आम की मा ा, ोर B ारा बेची गई लीची और तरबूज की मा ा से लगभग
िकतने ितशत अिधक/कम है ?
A. 66.25% B. 78.57% C. 72.5%
D. 79.07% E. None of these

217. If orange sold by store C is 55% more than that of sold by store A, then, what is the average
quantity of orange sold by three store taken together?
यिद ोर C ारा बेचा गया संतरा ोर A ारा बेचे गए संतरा से 55% अिधक है , तो तीन ोर ारा बेचे गए
संतरे की औसत मा ा िकतनी है ?

PAGE h ps://t.me/studified h ps://instagram.com/aashisharorasocial?utm_medium=copy_link


Follow
264 Aashish Arora on: h ps://youtube.com/channel/UCYa4_JrOrf8R5Kz2uOtccXQ https://www.facebook.com/aashisharorasocial/
अ ास by Aashish Arora
(SBI/IBPS/RBI/LIC/All other banking and insurance Exams)

A. 187 B. 255 C. 261


D. 193 E. None of these

218. What is the respective ratio of quantity of orange and guava together sold by store B to
quantity of lichi sold by store A?
ोर B ारा बेची गई संतरे और अम द की मा ा का ोर A ारा बेची गई लीची की मा ा से संबंिधत
अनुपात िकतना है ?
A. 9:4 B. 5:9 C. 7:9
D. 2:7 E. None of these

219. What is the difference between quantity of apple, orange and watermelon sold by store A
and quantity of other types of fruits sold by store B?
ोर A ारा बेचे गए सेब, संतरा और तरबूज की मा ा और ोर B ारा बेचे गए अ कार के फलो ं की मा ा
के बीच का अंतर िकतना है ?
A. 185 B. 150 C. 320
D. 250 E. None of these

220. If average quantity of mango sold by store B and C taken together is 380 and quantity of
watermelon sold by store A and C are in the ratio 4:5 respectively, then, find the quantity of
mango and watermelon together sold by store C?
यिद ोर B और C ारा िमलाकर बेचे गए आम की औसत मा ा 380 है और ोर A और C ारा बेचे गए
तरबूज की मा ा मशः 4:5 के अनुपात म है , तो ोर C ारा बेचे गए आम और तरबूज की कुल मा ा ात
कीिजए?
A. 680 B. 630 C. 450
D. 790 E. None of these

Directions -: Read the data carefully and answer the following questions.

An investment-related seminar is conducted on five different months Jan, Feb, Mar, Apr and
May. There are only types of investment in the seminar ie; Mutual Fund and Fixed Deposit.
Every month different number of persons attended the seminar out of which some persons
invested in Mutual Fund while the rest invested in Fixed Deposit.

िनदश -: डे टा को ान से पढ़ और िन िल खत ो ं के उ र द।

पां च अलग-अलग महीनो ं जनवरी, फरवरी, माच, अ ैल और मई म िनवेश संबंधी सेिमनार आयोिजत िकया जाता है ।
सेिमनार म केवल कार के िनवेश होते ह i,e; ूचुअल फंड और िफ िडपॉिजट। हर महीने अलग-अलग
यो ं ने सेिमनार म भाग िलया, िजनम से कुछ यो ं ने ूचुअल फंड म िनवेश िकया जबिक बाकी ने साविध
जमा म िनवेश िकया।

PAGE h ps://t.me/studified h ps://instagram.com/aashisharorasocial?utm_medium=copy_link


Follow
265 Aashish Arora on: h ps://youtube.com/channel/UCYa4_JrOrf8R5Kz2uOtccXQ https://www.facebook.com/aashisharorasocial/
अ ास by Aashish Arora
(SBI/IBPS/RBI/LIC/All other banking and insurance Exams)

Total number of persons who a ended the


seminer (In thousands)

Feb, 350 Apr, 340


Jan, 280
May, 260

Mar, 180

Jan Feb Mar Apr May

Number of persons who investment in Mutual funds as


percentage of number who a ended the seminer

Apr, 90%
May, 85%
Feb, 74%

Jan, 65%
Mar, 55%

Jan Feb Mar Apr May

221. The number of persons who invested in Fixed Deposit in Mar is what percentage of that on
Jan?
माच म साविध जमा म िनवेश करने वाले यो ं की सं ा, जनवरी की तुलना म िकतने ितशत है ?
A. 93.97% B. 74.09% C. 79.62%
D. 82.65% E. None of these

222. Find the ratio of the number of persons who invested in Fixed Deposit on Jan and Apr
(together) to that on Feb and Mar (together)?
जनवरी और अ ैल (एक साथ) म साविध जमा म िनवेश करने वाले यो ं की सं ा का फरवरी और माच
(एक साथ) से अनुपात ात कीिजए?
A. 33:43 B. 11:9 C. 39:43

PAGE h ps://t.me/studified h ps://instagram.com/aashisharorasocial?utm_medium=copy_link


Follow
266 Aashish Arora on: h ps://youtube.com/channel/UCYa4_JrOrf8R5Kz2uOtccXQ https://www.facebook.com/aashisharorasocial/
अ ास by Aashish Arora
(SBI/IBPS/RBI/LIC/All other banking and insurance Exams)

D. 19:9 E. None of these

223. Find the average number of persons who invested in Mutual Funds in Feb, Mar, and that
May?
फरवरी, माच और मई म ूचुअल फंड म िनवेश करने वाले यो ं की औसत सं ा ात कीिजए?
A. 193 B. 350 C. 303
D. 101 E. None of these

224. Find the ratio of the number of person who invested in Fixed Deposit on Jan, Feb, and Mar
to that of who invested in Mutual Fund on Apr and on Mar?
जनवरी, फरवरी और माच म साविध जमा म िनवेश करने वाले यो ं की सं ा का अ ैल और माच म
ूचुअल फंड म िनवेश करने वालो ं से अनुपात ात कीिजए?
A. 4:5 B. 2:3 C. 5:9
D. 3:7 E. None of these

225. What percentage of total invested in Fixed Deposit?


साविध जमा म कुल िनवेश का िकतना ितशत?
A. 22.98% B. 16.45% C. 31.09%
D. 24.32% E. None of these

Direction: Study the Following table chart carefully and answer the questions.
The data given below the information about various types of daily uses produced(in thousand
tons) in our country, 'India' for two years. It also show the percentage contribution of Rajasthan,
one of the states of our country, to the total production of the daily uses.

िनदश: िन िल खत तािलका चाट का ानपूवक अ यन कर और ो ं के उ र द।


हमारे दे श 'भारत' म दो वष से उ ािदत (हजार टन म) िविभ कार के दै िनक उपयोगो ं के बारे म सूचना के नीचे
िदया गया डे टा। यह हमारे दे श के रा ो ं म से एक राज थान के दै िनक उपयोग के कुल उ ादन म ितशत योगदान
को भी दशाता है ।

Produc on in 2014 Produc on in 2015

400

280
280
220 270
150 200
160

125
80

CLOTHS BAGS SHOES SANDALS OTHERS

PAGE h ps://t.me/studified h ps://instagram.com/aashisharorasocial?utm_medium=copy_link


Follow
267 Aashish Arora on: h ps://youtube.com/channel/UCYa4_JrOrf8R5Kz2uOtccXQ https://www.facebook.com/aashisharorasocial/
अ ास by Aashish Arora
(SBI/IBPS/RBI/LIC/All other banking and insurance Exams)

Percentage contribu on of rajashthan in2014


Percentage contribu on of rajashthan in2015

50
45 50
40

30 35 32 34
30
20

0 1 2 3 4 5 6

226. What was the percentage of daily uses produced in the states Rajasthan in the 2014?
2014 म राज थान रा ो ं म उ ािदत दै िनक उपयोग का ितशत िकतना था?
A. 66.25% B. 38.75% C. 52.5%
D. 49.07% E. None of these

227. If production of bags in 2016 is 30% more than production in 2014, then what is the total
bags produced in 3 years(together)?
यिद 2016 म बैग का उ ादन 2014 म उ ादन से 30% अिधक है , तो 3 वष म (एक साथ) कुल बैग का
उ ादन िकतना है ?
A. 605 B. 380 C. 470
D. 590 E. None of these

228. In the state Rajasthan, in 2014, the price of bags was 60% higher than that of sandals, then
total amount received from the sandals was how much percentage more than total amount
received from bags ?
राज थान रा म, 2014 म, बैग की कीमत सडल की तुलना म 60% अिधक थी, तो सडल से ा कुल रािश
बैग से ा कुल रािश से िकतने ितशत अिधक थी?
A. 87.5 % B. 79.25% C. 92.67%
D. 74.97% E. None of these

229. What was the ratio of the total quantity of daily uses produced in the year 2014 in India to
that in the year 2015 in India?
भारत म वष 2014 म उ ािदत दै िनक उपयोग की कुल मा ा का भारत म वष 2015 म उ ादन की कुल
मा ा से अनुपात िकतना था?
A. 2013:245 B. 231:224 C. 312:211
D. 224:209 E. None of these

230. In the year 2014, India exports 20% of total shoes produced in that year, and in the year

PAGE h ps://t.me/studified h ps://instagram.com/aashisharorasocial?utm_medium=copy_link


Follow
268 Aashish Arora on: h ps://youtube.com/channel/UCYa4_JrOrf8R5Kz2uOtccXQ https://www.facebook.com/aashisharorasocial/
अ ास by Aashish Arora
(SBI/IBPS/RBI/LIC/All other banking and insurance Exams)

2015, it exports 30% of total shoes produced in that year then total what quantity of shoes
did India exports in the year 2014 and 2015 together?
वष 2014 म, भारत उस वष म उ ािदत कुल जूतो ं का 20% िनयात करता है , और वष 2015 म, यह उस वष
म उ ािदत कुल जूतो ं का 30% िनयात करता है , तो वष 2014 और 2015 म भारत ने कुल िकतने जूतो ं का
िनयात िकया? ?
A. 112 k pairs B. 78 k pairs C. 250 k pairs
D. 104 k pairs E. None of these

Directions : Study the information carefully and answer the following questions :
The given bar graph shows the number of Laptops manufactured in last five years and the
percentage of number of laptops sold.

िनदश: जानकारी का ानपूवक अ यन कर और िन िल खत ो ं के उ र द:


िदया गया बार ाफ िपछले पां च वष म िनिमत लैपटॉप की सं ा और बेचे गए लैपटॉप की सं ा का ितशत
दशाता है ।

800 720
700 640
600 540
480
500
400 360
300 37.5%
43.75%
200 30% 26.25%
22.5%
100
0
2016 2017 2018 2019 2020

Total number of Laptops manufactured Percentage of number of Laptops sold

231. Find the average of the number of unsold Laptops in 2017 and 2019?
2017 और 2019 म िबना िबके लैपटॉप की औसत सं ा ात कीिजये?
A. 432 B. 324 C. 342
D. 243 E. None of these

232. Find the ratio of the number of Laptops sold in 2016 to that the number of sold Laptops in
2020?
2016 म बेचे गए लैपटॉप की सं ा का 2020 म बेचे गए लैपटॉप की सं ा से अनुपात ात कीिजये?
A. 53:31 B. 35:51 C. 45:28
D. 32:27
E. None of these

233. What is the total number of unsold laptops in 2017 to 2019?


2017 से 2019 म िबना िबके लैपटॉप की औसत सं ा ा है ?
A. 927 B. 972 C. 792
D. 729 E. None of these

PAGE h ps://t.me/studified h ps://instagram.com/aashisharorasocial?utm_medium=copy_link


Follow
269 Aashish Arora on: h ps://youtube.com/channel/UCYa4_JrOrf8R5Kz2uOtccXQ https://www.facebook.com/aashisharorasocial/
अ ास by Aashish Arora
(SBI/IBPS/RBI/LIC/All other banking and insurance Exams)

234. Find the difference between the number of unsold laptops in 2018 and the number of
laptops sold in 2020?
2018 म बेची गई लैपटॉप की सं ा और 2020 म बेची गई लैपटॉप की सं ा के बीच का अंतर ात
कीिजए?
A. 222 B. 111 C. 121
D. 212 E. None of these

235. Out of the number of laptops sold in 2016, 30% are defective. The number of defective
laptops sold in 2016 is what percent of the number of laptops sold in 2018?
2016 म बेचे गए लैपटॉप की सं ा म से 30% खराब ह। 2016 म बेचे गए खराब लैपटॉप की सं ा 2018 म
बेचे गए लैपटॉप की सं ा का िकतना ितशत है ?
A. 50% B. 60% C.80%
D. 100% E. None of these

Directions : Study the information carefully and answer the following questions :
The pie Chart shows the average number of people who travelled by Ola and Uber on Five
different days and the number of people who travelled by Uber on each five days.

िनदश: जानकारी का ानपूवक अ यन कर और िन िल खत ो ं के उ र द:


पाई चाट पां च अलग-अलग िदनो ं म ओला और उबर ारा या ा करने वाले लोगो ं की औसत सं ा और ेक पां च
िदनो ं म उबर ारा या ा करने वाले लोगो ं की सं ा को दशाता है ।

1400

1300 1260 1260


1225
1200
1190
1100 1050 1155
1120
1015
1000

900 945 945

800
Monday Tuesday Wednesday Thursday Friday
Average number of people travelled by Ola and Uber Number of people tavelled by Uber

236. What is total number of people who travelled by Ola on Tuesday and Friday together?
मंगलवार और शु वार को िमलाकर ओला से या ा करने वाले यो ं की कुल सं ा िकतनी है ?
A. 2030 B. 2240 C. 2450
D. 1940 E. None of these

237. What is the ratio of the number of people who travelled by Ola to that Uber on Wednesday?
बुधवार को ओला से या ा करने वाले यो ं की सं ा का उस उबर से िकतना अनुपात है ?
A. 7:11 B. 9:13 C. 7:9
D. 13:11 E. None of these

238. If the average number of people who travelled by Ola and Rapido together on Monday is

PAGE h ps://t.me/studified h ps://instagram.com/aashisharorasocial?utm_medium=copy_link


Follow
270 Aashish Arora on: h ps://youtube.com/channel/UCYa4_JrOrf8R5Kz2uOtccXQ https://www.facebook.com/aashisharorasocial/
अ ास by Aashish Arora
(SBI/IBPS/RBI/LIC/All other banking and insurance Exams)

1035, then find the number of people who travelled by Rapido on Monday?
यिद सोमवार को ओला और रै िपडो से या ा करने वाले यो ं की औसत सं ा 1035 है , तो सोमवार को
रै िपडो से या ा करने वाले यो ं की सं ा ात कीिजए?
A. 900 B. 915 C. 905
D. 925 E. None of these

239. If the ratio of the number of male and female who travelled by Uber on Thursday is 5:4, Find
the number of male who travelled by Uber on Thursday?
यिद गु वार को Uber से या ा करने वाले पु षो ं और मिहलाओं की सं ा का अनुपात 5:4 है , तो गु वार को
Uber से या ा करने वाले पु षो ं की सं ा ात कीिजए?
A. 500 B. 600 C. 700
D. 800 E. None of these

240. The number of people who travelled by Uber on Monday and Thursday together is how
much percent more/less of the total number of people who travelled by Ola and Uber on
Tuesday ?
सोमवार और गु वार को एक साथ उबर से या ा करने वाले लोगो ं की सं ा मंगलवार को ओला और उबर से
या ा करने वाले लोगो ं की कुल सं ा का िकतना ितशत अिधक/कम है ?
A. 18.33% B. 17.33% C. 13.33%
D. 24.33% E. None of these

Directions : Study the information carefully and answer the following questions :
The table below shows the number of students who passed the class 10th examinations and the
number of students who failed the class 12th examination in five different schools. It also shows
the percentage of students who failed the class 10th examination and the percentage of
students who passed the class 12th examination out of the total number of students in that
class in five different schools.

िनदश: जानकारी का ानपूवक अ यन कर और िन िल खत ो ं के उ र द:


नीचे दी गई तािलका म पां च अलग-अलग ू लो ं म क ा 10 वी ं की परी ा उ ीण करने वाले छा ो ं की सं ा और
क ा 12 वी ं की परी ा म अनु ीण होने वाले छा ो ं की सं ा को दशाया गया है । यह उन छा ो ं के ितशत को भी
दशाता है जो क ा 10 वी ं की परी ा म अनु ीण ए और पां च अलग-अलग ू लो ं म उस क ा के छा ो ं की कुल
सं ा म से 12 वी ं क ा उ ीण करने वाले छा ो ं का ितशत।

Number of students Number of students Percentage of Percentage of


who passed the class who failed the class students who failed students who passed
Schools
10 th examina on 12 th examina on the class 10 th the class 12 h
examina on examina on
A 124 143 20% 48%
B 90 30 25% 60%
C 136 75 32% 50%
D 70 45 50% 64%
E 105 45 40% 75%

PAGE h ps://t.me/studified h ps://instagram.com/aashisharorasocial?utm_medium=copy_link


Follow
271 Aashish Arora on: h ps://youtube.com/channel/UCYa4_JrOrf8R5Kz2uOtccXQ https://www.facebook.com/aashisharorasocial/
अ ास by Aashish Arora
(SBI/IBPS/RBI/LIC/All other banking and insurance Exams)

241. In which school the number of students who failed the class 10th examinations is second
lowest?
िकस ू ल म क ा 10वी ं की परी ा म अनु ीण होने वाले छा ो ं की सं ा दू सरा सबसे कम है ?
A. School A B. School D C. School C
D. School B E. None of these

242. Number of students studying in Class 12th of schools A and C together is what percentage
more/less that the number of students studying in Class 10th of school B and D together?
ू ल A और C म िमलाकर क ा 12वी ं म पढ़ने वाले छा ो ं की सं ा, ू ल B और D म िमलाकर क ा 10वी ं
म पढ़ने वाले छा ो ं की सं ा से िकतने ितशत अिधक/कम है ?
A. 63(3/13)% B. 61(4/13)% C. 61(5/13)%
D. 63(6/13)% E. None of these

243. What are the average number of students who passed the 12th class examination in
School A, B and C?
ू ल ए, बी और सी म 12 वी ं क ा की परी ा उ ीण करने वाले छा ो ं की औसत सं ा ा है ?
A. 54 B. 84 C. 64
D. 72 E. None of these

244. What is the ratio of the number of students studying in class 10th of schools C and E
together to the number of students studying in class 12th of schools A and D together?
ू ल C और E म िमलाकर क ा 10वी ं म पढ़ने वाले िव ािथयो ं की सं ा का ू ल A और D म िमलाकर
क ा 12वी ं म पढ़ने वाले िव ािथयो ं की सं ा का अनुपात िकतना है ?
A. 11:12 B. 15:11 C. 15:16
D. 11:16 E. None of these

245. Number of students studying in 9th class of school C are 30% more than the number of
students studying in 10th class of the same school. Number of students studying in 11th
class of school C are 40% less than the number of students studying in 12th class of school
C. Find the difference between the number of students studying in 11th class of school C
and that in class 9th the same school.
ू ल C की 9वी ं क ा म पढ़ने वाले छा ो ं की सं ा उसी ू ल के 10वी ं क ा म पढ़ने वाले छा ो ं की सं ा से
30% अिधक है । ू ल C की 11वी ं क ा म पढ़ने वाले छा ो ं की सं ा, ू ल C की 12वी ं क ा म पढ़ने वाले
छा ो ं की सं ा से 40% कम है । ू ल C की 11वी ं क ा म पढ़ने वाले छा ो ं की सं ा और उसी ू ल म 9वी ं
क ा म पढ़ने वाले छा ो ं की सं ा के बीच का अंतर ात कीिजए। .
A. 160 B. 170 C. 190
D. 180 E. None of these

Directions : Study the information carefully and answer the following questions :
The given bar graph shows total number of mobile (Realme + Mi) sold by a Shop and
percentage of Mi mobile sol by the shop in five different years.

िनदश: जानकारी का ानपूवक अ यन कर और िन िल खत ो ं के उ र द:


िदया गया बार ाफ एक दु कान ारा बेचे गए मोबाइल (Realme + Mi) की कुल सं ा और पां च अलग-अलग वष
म दु कान ारा बेचे गए Mi मोबाइल का ितशत दशाता है ।

PAGE h ps://t.me/studified h ps://instagram.com/aashisharorasocial?utm_medium=copy_link


Follow
272 Aashish Arora on: h ps://youtube.com/channel/UCYa4_JrOrf8R5Kz2uOtccXQ https://www.facebook.com/aashisharorasocial/
अ ास by Aashish Arora
(SBI/IBPS/RBI/LIC/All other banking and insurance Exams)

900 840
800
800
720
700 630
71.42%
600 560
500 55%
400 62.5% 57.14%
300 37.5%

200
100
2016 2017 2018 2019 2020

Total number of mobile sold Percentage of MI mobile sold

246. Find the total number of Realme mobile sold in 2016.


2016 म बेचे गए Realme मोबाइल की कुल सं ा ात कीिजए।
A. 200 B. 250 C. 230
D. 210 E. None of these

247. Find the average number of Mi mobile sold in 2017 to 2019?


2017 से 2019 म बेचे गए Mi मोबाइल की औसत सं ा ात कीिजए?
A. 310 B. 350 C. 410
D. 450 E. None of these

248. The number of Mi mobile sold in 2020 is how much more/less than the number of Realme
mobile sold in 2017?
2020 म बेचे गए Mi मोबाइल की सं ा 2017 म बेचे गए Realme मोबाइल की सं ा से िकतनी
अिधक/कम है ?
A. 83.33% B. 71.42% C. 87.33%
D. 56.33% E. None of these

249. What is the difference between the total number of Realme mobile sold and the total
number of Mi mobile sold.
बेचे गए Realme मोबाइल की कुल सं ा और बेचे गए Mi मोबाइल की कुल सं ा के बीच का अंतर िकतना
है ?
A. 300 B. 390 C. 400
D. 490 E. None of these

250. What is the ratio between Realme mobile sold by the shop in 2018 to that Realme mobile
sold in 2019?
2018 म दु कान ारा बेचे गए Realme मोबाइल और 2019 म बेचे गए Realme मोबाइल का अनुपात
िकतना है ?
A. 3:5 B. 3:4 C. 4:5
D. 2:3 E. None of these

PAGE h ps://t.me/studified h ps://instagram.com/aashisharorasocial?utm_medium=copy_link


Follow
273 Aashish Arora on: h ps://youtube.com/channel/UCYa4_JrOrf8R5Kz2uOtccXQ https://www.facebook.com/aashisharorasocial/
अ ास by Aashish Arora
(SBI/IBPS/RBI/LIC/All other banking and insurance Exams)

The given bar graph shows the total number of Antivirus Software (Internet Security Antivirus +
Computer Security Antivirus) sold and number of Internet Security Antivirus sold, by five
different shops.

िदए गए बार ाफ पां च अलग-अलग दु कानो ं ारा कुल एं टीवायरस सॉ टवेयर (इं टरनेट सुर ा एं टीवायरस +
कं ूटर सुर ा एं टीवायरस) की कुल सं ा और बेची गई इं टरनेट सुर ा एं टीवायरस की सं ा को दशाता है ।

510

460

410

360

310

260

210

160
Shop P Shop Q Shop R Shop S Shop T
Total An virus So ware Sold Internet Security An virus Sold

251. The number of Internet Security Antivirus sold by Shops P and Q, together is how much
percent more/less than the number of Computer Security Antivirus sold by Shop T?
Shop P and Q ारा बेची गई Internet Security Antivirus की सं ा, Shop T ारा बेची गई
Computer Security Antivirus की सं ा से िकतने ितशत अिधक / कम है ?
A. 566.66 B. 540.25 C. 533.33
D. 520.25 E. None

252. Find the average number of Computer Security Antivirus sold by Shops Q and S.
दु कान Q और S ारा बेची गई कं ूटर सुर ा एं टीवायरस की औसत सं ा ात कर।
A. 175 B. 180 C. 185
D. 190 E. None

253. Out of total number of Computer Security Antivirus sold by Shop R and T, 60% were 10.22
version and rest were other versions. Find the number of 10.22 version Computer Security
Antivirus sold by Shop R and T.
शॉप आर और टी ारा बेचे गए कं ूटर सुर ा एं टीवायरस की कुल सं ा म से 60% 10.22 सं रण थे और
बाकी अ सं रण थे। शॉप आर और टी ारा बेचे गए 10.22 सं रण कं ूटर सुर ा एं टीवायरस की
सं ा ात कर।
A. 112 B. 114 C. 116
D. 118 E. None

PAGE h ps://t.me/studified h ps://instagram.com/aashisharorasocial?utm_medium=copy_link


Follow
274 Aashish Arora on: h ps://youtube.com/channel/UCYa4_JrOrf8R5Kz2uOtccXQ https://www.facebook.com/aashisharorasocial/
अ ास by Aashish Arora
(SBI/IBPS/RBI/LIC/All other banking and insurance Exams)

254. The number of Internet Security Antivirus sold by Shop U is 40% more than that by Shops
S and T, together. If total number of Antivirus Software (Internet Security Antivirus +
Computer Security Antivirus) sold by Shop U is 20% more than that by Shops P and Q,
together, then find the number of Computer Security Antivirus sold by Shop U.
शॉप 'यू' ारा बेची जाने वाली इं टरनेट िस ो रटी एं टीवायरस की सं ा दु कान एस और टी से 40% अिधक
है । यिद Shop U ारा बेचा गया एं टीवायरस सॉ टवेयर (इं टरनेट िस ो रटी एं टीवायरस + कं ूटर
िस ो रटी एं टीवायरस) की कुल सं ा, दु कान P और Q से 20% अिधक है , तो Shop U ारा बेचे गए
कं ूटर सुर ा एं टीवायरस की सं ा ात कर।
A. 55 B. 65 C. 70
D. 75 E. None

255. Find the ratio of the number of Internet Security Antivirus sold by Shop R to number of
Computer Security Antivirus sold by Shop Q.
शॉप R ारा बेचे गए इं टरनेट िस ो रटी एं टीवायरस की सं ा और शॉप Q ारा बेचे गए कं ूटर िस ो रटी
एं टीवायरस की सं ा का अनुपात ात कीिजए।
A. 7:5 B. 8:5 C. 9:4
D. 9:5 E. None

The given bar graph shows the percentage by which number of Laptops sold is more than that of
the number of Desktops sold. The given line graph shows the difference between the number of
Laptops and Desktops sold, in five different months.

िदए गए बार ाफ ितशत को दशाता है िक बेची जाने वाली लैपटॉप की सं ा डे टॉप की िब ी की सं ा से


िकतना ितशत अिधक है । िदए गए लाइन ाफ पां च अलग-अलग महीनो ं म बेचे गए लैपटॉप और डे टॉप की
सं ा के बीच अंतर को दशाता है ।

% in which Laptops sold is more than Desktop Sold


65

55

45

35

25
February March April May June

PAGE h ps://t.me/studified h ps://instagram.com/aashisharorasocial?utm_medium=copy_link


Follow
275 Aashish Arora on: h ps://youtube.com/channel/UCYa4_JrOrf8R5Kz2uOtccXQ https://www.facebook.com/aashisharorasocial/
अ ास by Aashish Arora
(SBI/IBPS/RBI/LIC/All other banking and insurance Exams)

Difference between number of Laptop and Desktop


Sold
200
174
150
128
100

57 60
50 48

0
February March April May June

256. Find the sum of number of Laptops sold in February to the total number of computer device
(Laptop + Desktop) sold in April.
फरवरी म बेचे गए लैपटॉप की सं ा और अ ैल म बेचे गए कं ूटर िडवाइस (लैपटॉप + डे टॉप) की कुल
सं ा का योग ात कीिजए।
A. 560 B. 565 C. 570
D. 575 E. None

257. If average cost of each desktop is $300 and each laptop is $400, then find the total revenue
earnt by the shop in February.
यिद ेक डे टॉप की औसत लागत $ 300 है और ेक लैपटॉप की औसत लागत $ 400 है , तो फरवरी
म दु कान ारा कुल राज ात कीिजए।
A. $ 71200 B. $ 73400 C. $ 75200
D. $ 75800 E. None

258. Find the average of the number of computer device (Laptop + Desktop) sold in April and
May.
अ ैल और मई म बेचे जाने वाले कं ूटर िडवाइस (लैपटॉप + डे टॉप) की सं ा का औसत ात कर।
A. 199.25 B. 200.5 C. 201.25
D. 202.5 E. None

259. Find the difference between total number of Desktops sold in May and June, together and
total number of computer device (Laptop + Desktop) sold in March.
मई और जून म बेचे गए डे टॉप की कुल सं ा और माच म बेचे गए कं ूटर िडवाइस (लैपटॉप +
डे टॉप) की कुल सं ा के बीच अंतर ात कीिजए।
A. 72 B. 74 C. 76
D. 78 E. None

260. The total number of computer device (Laptop + Desktop) sold in January is 45% more than
that in May out of which 218 are Laptops. Find the number of Desktops sold in January.

PAGE h ps://t.me/studified h ps://instagram.com/aashisharorasocial?utm_medium=copy_link


Follow
276 Aashish Arora on: h ps://youtube.com/channel/UCYa4_JrOrf8R5Kz2uOtccXQ https://www.facebook.com/aashisharorasocial/
अ ास by Aashish Arora
(SBI/IBPS/RBI/LIC/All other banking and insurance Exams)

जनवरी म बेचे गए कं ूटर िडवाइस (लैपटॉप + डे टॉप) की कुल सं ा मई की तुलना म 45% अिधक है ,
िजसम से 218 लैपटॉप ह। माह जनवरी म बेचे गए डे टॉप की सं ा ात कीिजए।
A. 302 B. 304 C. 306
D. 308 E. None

The pie chart given below shows the percentage distribution of total number of students (2700)
selected from a coaching in the given exams. The line graph given below shows the number of
girls selected in the respective exams.

नीचे िदया गया पाई चाट 6 अलग-अलग परी ाओं म एक कोिचंग से चयिनत कुल छा ो ं (2700) का ितशत िवतरण
दशाता है । नीचे िदया गया लाइन ाफ संबंिधत परी ा म चयिनत लड़िकयो ं की सं ा दशाता है ।

Total Girls Selected


600
500 517
400
300 283 283
200
100 114 112 106
0
SBI PO SBI IBPS PO IBPS RRB PO RRB
CLERK CLERK CLERK

261. What is the difference between the number of girls to number of boys selected in IBPS
Clerk from the coaching ?
कोिचंग से IBPS क म चयिनत लड़िकयो ं की सं ा और लड़को ं की सं ा का अंतर ा है ?
A. 180 B. 185 C. 190
D. 195 E. None

PAGE h ps://t.me/studified h ps://instagram.com/aashisharorasocial?utm_medium=copy_link


Follow
277 Aashish Arora on: h ps://youtube.com/channel/UCYa4_JrOrf8R5Kz2uOtccXQ https://www.facebook.com/aashisharorasocial/
अ ास by Aashish Arora
(SBI/IBPS/RBI/LIC/All other banking and insurance Exams)

262. 500/11% of the students of the coaching who were selected in SBI Clerk, IBPS Clerk and
RRB Clerk together are of Hindi Medium. Find total number of students who were of
English Medium in these three exams.
कोिचंग के 500/11% छा जो SBI क, IBPS क और RRB क म एक साथ चुने गए थे वे िहं दी
मीिडयम के ह। उन छा ो ं की कुल सं ा ात कर जो इन तीन परी ाओं म अं ेजी मा म के थे।
A. 805 B. 810 C. 815
D. 820 E. None

263. Number of girls selected in RRB Clerk is how much percent more/less than the number of
boys selected in the same exam?
RRB क म चयिनत लड़िकयो ं की सं ा समान परी ा म चयिनत लड़को ं की सं ा से िकतने ितशत
अिधक / कम है ?
A. 82.45 B. 83.92 C. 89.28
D. 92.41 E. None

264. Number of boys selected in RBI Assistant is 50% less than same in RRB PO. Find total
number of students selected in RBI Assistant if ratio of number of girls to number of boys
selected in RBI Assistant is 5:2
RBI अिस ट म चुने गए लड़को ं की सं ा RRB PO से 50% कम है । RBI अिस ट म चुने गए छा ो ं की
कुल सं ा ात कर, यिद RBI अिस ट म चयिनत लड़िकयो ं और लड़को ं की सं ा का अनुपात 5:2 है ।
A. 182 B. 184 C. 186
D. 188 E. None

265. Find the number of boys selected in SBI Clerk.


SBI क म चुने गए लड़को ं की सं ा ात कीिजए।
A. 283 B. 284 C. 285
D. 286 E. None

266. What is the average number of boys selected in SBI PO and IBPS PO together?
SBI PO और IBPS PO म चयिनत लड़को ं की औसत सं ा ा है ?
A. 181 B. 182 C. 183
D. 184 E. None

The line graph given below shows total number of students who attempted the mock test and the
bar graph shows the number of students who cleared the cut-off marks.

नीचे िदया गया लाइन ाफ कुल छा ो ं की सं ा दशाता है िज ोन


ं े मॉक टे का यास िकया और बार ाफ कट-
ऑफ अंको ं से उ ीण छा ो ं की सं ा को दशाता है ।

PAGE h ps://t.me/studified h ps://instagram.com/aashisharorasocial?utm_medium=copy_link


Follow
278 Aashish Arora on: h ps://youtube.com/channel/UCYa4_JrOrf8R5Kz2uOtccXQ https://www.facebook.com/aashisharorasocial/
अ ास by Aashish Arora
(SBI/IBPS/RBI/LIC/All other banking and insurance Exams)

Number of students who a empted the Mock Test


432
408
384
360
336
312
288
264
Mock I Mock II Mock III Mock IV Mock V

Number of students who cleared the Cut-off marks


Mock V

Mock IV

Mock III

Mock II

Mock I

0 52 104 156 208 260 312 364

267. Ratio of number of English Medium Student to Hindi Medium Student who cleared the cut-
off in Mock Test 1 is 4:3 respectively. Ratio of number of English Medium Student to Hindi
Medium Student who could not clear the cut-off in Mock Test 1 is 1:1 respectively. Find total
number of English Medium students who attempted Mock Test 1.
मॉक टे 1 म कट-ऑफ यर करने वाले इं श मीिडयम ू डट और िहं दी मीिडयम ू डट की सं ा
का अनुपात 4: 3 है । मॉक टे 1 म कट-ऑफ नही ं यर करने वाले इं श मीिडयम ू डट और िहं दी
मीिडयम ू डट की सं ा का अनुपात 1:1 है । मॉक टे 1 का यास करने वाले अं ेजी मा म के छा ो ं की
कुल सं ा ात कर।
A. 154 B. 155 C. 156
D. 157 E. None

268. What is the ratio of number of students who cleared the cut-off to number of students who
could not clear the cut-off in Mock Test 2?
मॉक टे 2 म कट-ऑफ ीयर कर पाने वाले छा ो ं की सं ा और कट-ऑफ यर नही ं करने वाले छा ो ं
की सं ा का अनुपात ा है ?
A. 39:89 B. 37:87 C. 38:85
D. 36:83 E. None

PAGE h ps://t.me/studified h ps://instagram.com/aashisharorasocial?utm_medium=copy_link


Follow
279 Aashish Arora on: h ps://youtube.com/channel/UCYa4_JrOrf8R5Kz2uOtccXQ https://www.facebook.com/aashisharorasocial/
अ ास by Aashish Arora
(SBI/IBPS/RBI/LIC/All other banking and insurance Exams)

269. Number of students who cleared the cut-off in Mock Test 4 is how much percent of total
number of students who attempted the mock test?
मॉक टे 4 म कट-ऑफ यर करने वाले छा ो ं की सं ा मॉक टे का यास करने वाले छा ो ं की कुल
सं ा का िकतना ितशत है ?
A. 67.33 B. 70.33 C. 72.33
D. 69.33 E. None

270. What is the difference between numbers of students who cleared the cut-off and who could
not clear the cut-off in Mock Test 5?
मॉक टे 5 म िजन छा ो ं ने कट-ऑफ यर िकया था और जो कट-ऑफ ीयर नही ं कर पाए थे, उनकी
सं ा म ा अंतर है ?'
A. 180 B. 185 C. 190
D. 195 E. None

271. Find the average number of students who cleared the cutoff in Test I, II , III and IV.
टे I, II, III और IV म कटऑफ यर करने वाले छा ो ं की औसत सं ा ात कीिजए।
A. 143 B. 144 C. 145
D. 145 E. None

The car dealer sold three types of cars, i.e., Petrol, Diesel, and Electric in six different months of
a particular year. The line graph given below shows the percentage of petrol and percentage of
diesel cars sold in the respective months out of the total number of cars sold by the shopkeeper
in that month. The table graph shows the number of electric cars sold in different months.

कार डीलर ने एक िवशेष वष के छह अलग-अलग महीनो ं म तीन तरह की कार, यानी, पेटोल, डीजल और इले क
बेची।ं नीचे िदए गए लाइन ाफ म उस महीने म दु कानदार ारा बेची गई कारो ं की सं ा से संबंिधत महीनो ं म िबकने
वाले पेटोल और डीजल कारो ं के ितशत को िदखाया गया है । टे बल ाफ िविभ महीनो ं म बेची गई इले क कारो ं
की सं ा को दशाता है ।

70%

60%

50%

40%

30%

20%

10%

0%
Jan Feb Mar Apr May June

%age of Petrol %age of Diesel

PAGE h ps://t.me/studified h ps://instagram.com/aashisharorasocial?utm_medium=copy_link


Follow
280 Aashish Arora on: h ps://youtube.com/channel/UCYa4_JrOrf8R5Kz2uOtccXQ https://www.facebook.com/aashisharorasocial/
अ ास by Aashish Arora
(SBI/IBPS/RBI/LIC/All other banking and insurance Exams)

Months Number of electric cars sold

Jan 132

Feb 108

Mar 160

Apr 84

May 96

June 112

272. Find the ratio of total number of cars sold in February and May together to the total number
of cars sold in April?
फरवरी और मई म बेची गई कारो ं की कुल सं ा और अ ैल म बेची गई कारो ं की कुल सं ा का अनुपात
ात कीिजए?
A. 62 : 49 B. 31 : 28 C. 62 : 35
D. 31 : 21 E. None of these

273. The number of petrol cars sold in April is what percent more/less than the number of petrol
and diesel cars sold in February together?
अ ैल म िबकने वाली पेटोल कारो ं की सं ा फरवरी म बेची गई पेटोल और डीजल कारो ं की सं ा से िकतने
ितशत अिधक / कम है ?
A. 20% B. 16.66% C. 25%
D. 18.18% E. None of these

274. If the number of petrol cars sold in January, March, May, and June is represented in a pie
chart, then find the central angle made by petrol cars sold in January?
यिद जनवरी, माच, मई और जून म िबकने वाली पेटोल कारो ं की सं ा को पाई चाट म दशाया जाए, तो
जनवरी म बेची गई पेटोल कारो ं ारा बनाए गए क ीय कोण का पता लगाएं ?
A. 47.5 B. 36 C. 40
D. 45 E. None of these

275. Find the average number of diesel cars sold in February, March, May, and June?
फरवरी, माच, मई और जून म िबकने वाली डीजल कारो ं की औसत सं ा ात कीिजये?
A. 119 B. 117 C. 124
D. 121 E. None of these

276. The diesel cars sold in April is 7.5% of the total number of cars (petrol, diesel, and electric)
manufactured in that month. If the ratio of petrol, diesel, and electric cars manufactured in
April is 6:4:5 respectively, then find the difference between the number of petrol and diesel
cars that are unsold in April?
अ ैल म िबकने वाली डीजल कार उसी महीने म िनिमत कारो ं (पेटोल, डीजल और इले क) की कुल सं ा

PAGE h ps://t.me/studified h ps://instagram.com/aashisharorasocial?utm_medium=copy_link


Follow
281 Aashish Arora on: h ps://youtube.com/channel/UCYa4_JrOrf8R5Kz2uOtccXQ https://www.facebook.com/aashisharorasocial/
अ ास by Aashish Arora
(SBI/IBPS/RBI/LIC/All other banking and insurance Exams)

का 7.5% है । यिद अ ैल म िनिमत पेटोल, डीजल और इले क कारो ं का अनुपात मशः 6:4:5 है , तो
अ ैल म िबना िबके पेटोल और डीजल कारो ं की सं ा के बीच अंतर ात कीिजए?
A. 140 B. 160 C. 120
D. 150 E. None of these

The given table graph shows the total number of donuts (chocolate and strawberry) sold and the
percentage of chocolate donuts sold as compared to the strawberry donuts sold by five different
sellers.

दी गई तािलका का ाफ पां च अलग-अलग िव े ताओं ारा बेचे गए डोनट् स (चॉकलेट और ॉबेरी) की कुल सं ा
और ॉबेरी डोनट् स की तुलना म बेचे गए चॉकलेट डोनट् स के ितशत को िदखाता है ।

%age of chocolate donuts as compared to


Sellers Total Donuts sold
strawberry donuts sold

A 1500 140%

B 1750 100%

C 2250 80%

D 1800 125%

E 1650 120%

277. Find the ratio of number of chocolate donuts sold by C and the number of strawberry
donuts sold by E?
C ारा बेचे जाने वाले चॉकलेट डोनट् स की सं ा और E ारा बेचे जाने वाले ॉबेरी डोनट् स के अनुपात का
पता लगाएं ?
A. 5:6 B. 4:3 C. 3:4
D. 6:5 E. None of these

278. If seller A sold each chocolate donuts for Rs 12 and each strawberry donuts for Rs 18, then
find the average price of each donut?
यिद िव े ता A ने ेक चॉकलेट डोनट् स को 12 पये म बेचा है और ेक ॉबेरी डोनट् स को 18 पये म
बेचा है , तो ेक डोनट की औसत कीमत ात कर?
A. 15.5 B. 12.5 C. 14.5
D. 16.5 E. None of these

279. The total number of Strawberry donuts sold by B and D together is how much percent
more/less than the total donuts sold by C?
B और D ारा एक साथ बेचे गए ॉबेरी डोनट् स की कुल सं ा C ारा बेचे गए कुल डोनट् स से िकतने
ितशत अिधक / कम है ?

PAGE h ps://t.me/studified h ps://instagram.com/aashisharorasocial?utm_medium=copy_link


Follow
282 Aashish Arora on: h ps://youtube.com/channel/UCYa4_JrOrf8R5Kz2uOtccXQ https://www.facebook.com/aashisharorasocial/
अ ास by Aashish Arora
(SBI/IBPS/RBI/LIC/All other banking and insurance Exams)

A. 25.56% B. 24.33% C. 27.5%


D. 26.83% E. None of these

280. Find the difference between total chocolate donuts sold by A and E together and total
strawberry donuts sold by C and E?
A और E ारा बेचे गए कुल चॉकलेट डोनट् स और C और E ारा बेचे गए कुल ॉबेरी डोनट् स के बीच अंतर
ात कीिजए?
A. 275 B. 250 C. 200
D. 225 E. None of these

281. The number of strawberry donuts sold by F is 67.5% more than chocolate donuts sold by
C. The chocolate donuts sold by F is 64% of the strawberry donuts sold by him. Find the
total number of donuts sold by F?
F ारा बेचे जाने वाले ॉबेरी डोनट् स की सं ा C ारा बेचे जाने वाले चॉकलेट डोनट् स से 67.5% अिधक
है । F ारा बेचे जाने वाले चॉकलेट डोनट् स उनके ारा बेचे गए ॉबेरी डोनट् स का 64% है । F ारा बेचे गए
डोनट् स की कुल सं ा ात कीिजये?
A. 2647 B. 2747 C. 2587
D. 2687 E. None of these

The bar graph given below shows the total number of boys from five different schools and the
line graph shows the percentage of girls out of total strength of each school.

नीचे िदया गया बार ाफ पां च अलग-अलग ू लो ं के लड़को ं की कुल सं ा िदखाता है और लाइन ाफ ेक
ू ल की कुल लड़िकयो ं का ितशत िदखाता है ।

600 540
500 480
450 420
400 360

300

200

100

0
DAV SD IPS BPS DPS

Number of Boys

PAGE h ps://t.me/studified h ps://instagram.com/aashisharorasocial?utm_medium=copy_link


Follow
283 Aashish Arora on: h ps://youtube.com/channel/UCYa4_JrOrf8R5Kz2uOtccXQ https://www.facebook.com/aashisharorasocial/
अ ास by Aashish Arora
(SBI/IBPS/RBI/LIC/All other banking and insurance Exams)

70%
64%
55%
60%
50%
36% 40% 44%
40%
30%
20%
10%
0%
DAV SD IPS BPS DPS

%age of Girls

282. The number of girls from SD is how much percent more/less than the number of girls from
BPS?
एसडी की लड़िकयो ं की सं ा बीपीएस की लड़िकयो ं की सं ा से िकतने ितशत अिधक / कम है ?
A. 54.54% B. 40% C. 45%
D. 44.44% E. None of these

283. Out of the total students from DAV and DPS, 46.66% students like cricket. Find the
difference between the number of students from DAV and DPS those who not like cricket
and the number of girls from IPS?
डीएवी और डीपीएस के कुल छा ो ं म से 46.66% छा ि केट पसंद करते ह। ि केट को पसंद नही ं करने
वाले डीएवी और डीपीएस के छा ो ं की सं ा और आईपीएस से लड़िकयो ं की सं ा के बीच अंतर ात
कीिजए?
A. 440 B. 560 C. 480
D. 540 E. None of these

284. Find the ratio between the number of boys from SD, IPS and DPS together and the number
of girls from DAV, BPS, and DPS?
एसडी, आईपीएस और डीपीएस के लड़को ं की सं ा और डीएवी, बीपीएस और डीपीएस की लड़िकयो ं की
सं ा के बीच का अनुपात ात कीिजए?
A. 33:47 B. 66:87 C. 33:35
D. 66:77 E. None of these

285. Find the average number of girls from DAV, SD, IPS and DPS?
DAV, SD, IPS और DPS से लड़िकयो ं की औसत सं ा ात कीिजये?
A. 350 B. 450 C. 480
D. 360 E. None of these

286. If the students from all the schools combined to sit in a another school and each room
consist of only 50 students, then how many rooms will be used?
यिद सभी ू लो ं के छा ो ं को एक दू सरे ू ल म बैठने के िलए और ेक कमरे म केवल 50 छा ह, तो
िकतने कमरो ं का उपयोग िकया जाएगा?

PAGE h ps://t.me/studified h ps://instagram.com/aashisharorasocial?utm_medium=copy_link


Follow
284 Aashish Arora on: h ps://youtube.com/channel/UCYa4_JrOrf8R5Kz2uOtccXQ https://www.facebook.com/aashisharorasocial/
अ ास by Aashish Arora
(SBI/IBPS/RBI/LIC/All other banking and insurance Exams)

A. 87 B. 92 C. 89
D. 91 E. None of these

The table given below shows the number of clerk and percentage of manager out of total
employees in five different banks. The employees are clerk and manager only. The sum of the
number of clerks in five different banks is 11000.

नीचे दी गई तािलका म पाँ च अलग-अलग बको ं म कुल कमचा रयो ं म से क की सं ा और बंधक का ितशत
िदखाया गया है । कमचारी केवल क और बंधक ह। पां च अलग-अलग बको ं म क की सं ा का योग 11000
है ।

Percentage of Manager out of total


Banks Number of Clerks
employees

SBI 25x 37.5%

UCO 16x 20%

BOB 20x 44.44%

AXIS 12x 25%

ICICI 15x 37.5%

287. Find the ratio of number of clerk from BOB and the number of manager from SBI?
बीओबी से क की सं ा और एसबीआई से बंधक की सं ा का अनुपात ात कीिजए?
A. 5:4 B. 4:3 C. 7:5
D. 6:5 E. None of these

288. Find the average number of manager from UCO, BOB and AXIS?
UCO, BOB और AXIS से बंधक की औसत सं ा ात कीिजये?
A. 1000 B. 1200 C. 900
D. 1100 E. None of these

289. The number of clerks in UBI is 13.33% more than the number of clerks in AXIS. If the total
employees in UBI is 22.22% less than that of BOB, then find the number of managers in
UBI?
यूबीआई म क की सं ा ए स म क की सं ा से 13.33% अिधक है । यिद UBI म कुल कमचारी
BOB की तुलना म 22.22% कम ह, तो UBI म बंधको ं की सं ा ात कर?
A. 1700 B. 1900 C. 1800
D. 2000 E. None of these

290. The number of manager from SBI and ICICI together is how much percent more/less than
the number of clerks from AXIS?

PAGE h ps://t.me/studified h ps://instagram.com/aashisharorasocial?utm_medium=copy_link


Follow
285 Aashish Arora on: h ps://youtube.com/channel/UCYa4_JrOrf8R5Kz2uOtccXQ https://www.facebook.com/aashisharorasocial/
अ ास by Aashish Arora
(SBI/IBPS/RBI/LIC/All other banking and insurance Exams)

SBI और ICICI के एक साथ बंधक की सं ा AXIS से क की सं ा से िकतने ितशत अिधक / कम


है ?
A. 200% B. 100% C. 150%
D. 175% E. None of these

291. The ratio of number of managers from KOTAK and SBI is 14:25, respectively. If the number
of manager constitutes 43.75% of the total employees in KOTAK bank, then find the
difference between managers and clerk?
कोटक और एसबीआई से बंधको ं की सं ा का अनुपात मशः 14:25 है । यिद KOTAK बक म बंधक
की सं ा 43.75% है , तो बंधक और क के बीच अंतर ात कीिजए?
A. 300 B. 250 C. 360
D. 320 E. None of these

A dealer sold bikes of only 3 companies namely Honda, Bajaj, and TVS. The table given below
shows the percentage of Honda bikes sold out of total bikes sold, the ratio of Bajaj and TVS
bikes sold, and the number of Honda bikes sold in different months.

एक डीलर ने केवल 3 कंपिनयो ं जैसे होड


ं ा, बजाज और टीवीएस की बाइक बेची।ं नीचे दी गई तािलका म कुल िबकने
वाली बाइको ं म िबकने वाली होड
ं ा बाइ का ितशत, बजाज और टीवीएस की िबकने वाली बाइ का अनुपात
और िविभ महीनो ं म िबकने वाली होड
ं ा बाइ की सं ा को िदखाया गया है ।

Number of Bajaj : Number of


Months %age of Honda
Number of TVS Honda bikes sold

January 20% 9:7 256

February 30% 9:5 216

March 25% 13:12 275

April 40% 11:7 360

May 35% 15:11 336

292. Find the ratio of number of Bajaj bikes sold in January and May?
जनवरी और मई म िबकने वाली बजाज बाइक की सं ा का अनुपात ात कीिजये?
A. 7:5 B. 9:5 C. 8:5
D. 6:5 E. None of these

293. The number of TVS bikes sold in march is how much percent more/less than the number of
Honda bikes sold in February?
माच म बेची गई टीवीएस बाइक की सं ा फरवरी म बेची गई होड ं ा बाइक की सं ा से िकतने ितशत
अिधक / कम है ?

PAGE h ps://t.me/studified h ps://instagram.com/aashisharorasocial?utm_medium=copy_link


Follow
286 Aashish Arora on: h ps://youtube.com/channel/UCYa4_JrOrf8R5Kz2uOtccXQ https://www.facebook.com/aashisharorasocial/
अ ास by Aashish Arora
(SBI/IBPS/RBI/LIC/All other banking and insurance Exams)

A. 83.33% B. 81.66% C. 87.5%


D. 77.5% E. None of these

294. If the bikes sold in April month is represented in pie chart, then find the central angle made
by number of TVS bikes sold?
यिद अ ैल महीने म बेची गई बाइको ं को पाई चाट म दशाया गया है , तो बेची गई टीवीएस बाइक की सं ा से
बना क ीय कोण ात कर?
A. 76 B. 84 C. 96
D. 72 E. None of these

295. Find the average number of bikes sold in all the given months?
सभी िदए गए महीनो ं म िबकने वाली बाइक की औसत सं ा ात कीिजये?
A. 1016 B. 992 C. 996
D. 1008 E. None of these

296. If the number of bikes sold in June is 41.66% more than that of sold in may month and the
ratio of bikes sold of Honda, Bajaj, and TVS is 5:7:4, then find the difference between Bajaj
and TVS bikes sold in June?
यिद जून म िबकने वाली बाइक की सं ा मई महीने म िबकने वाली बाइक की सं ा से 41.66% अिधक है
और होड
ं ा, बजाज और टीवीएस की िबकने वाली बाइक का अनुपात 5: 7: 4 है , तो जून म बजाज और टीवीएस
की िबकने वाली बाइक के बीच का अंतर खोज?
A. 265 B. 275 C. 295
D. 255 E. None of these

Different number of people went through Bird-flu test in five different hospitals and some of them
are tested positive while rest of them are tested negative. The pie chart given below shows the
distribution of number of people who were tested in the respective hospitals. Total number of
people who were tested in all five hospital together is 9600. The line graph shows the number of
people who were tested positive out of total number of people who were tested.

पां च अलग-अलग अ तालो ं म बड ू परी ण के मा म से अलग-अलग लोग गए और उनम से कुछ का परी ण


सकारा क है जबिक बाकी का परी ण नकारा क है । नीचे िदया गया पाई चाट उन लोगो ं की सं ा के िवतरण को
दशाता है िज संबंिधत अ तालो ं म परी ण िकया गया था। एक साथ सभी पां च अ ताल म परी ण िकए गए
लोगो ं की कुल सं ा 9600 है । लाइन ाफ उन लोगो ं की सं ा दशाता है िज परी ण िकए गए कुल लोगो ं म से
सकारा क परी ण िकया गया था।

PAGE h ps://t.me/studified h ps://instagram.com/aashisharorasocial?utm_medium=copy_link


Follow
287 Aashish Arora on: h ps://youtube.com/channel/UCYa4_JrOrf8R5Kz2uOtccXQ https://www.facebook.com/aashisharorasocial/
अ ास by Aashish Arora
(SBI/IBPS/RBI/LIC/All other banking and insurance Exams)

1100 1060
1050
1000 980
950 920
900 880
850
800 760
750
700
A B C D E

Number of people who tested posi ve

297. Find the ratio of number of people who tested negative in hospital C and the number of
people who tested positive in hospital E?
अ ताल C म नकारा क परी ण करने वाले लोगो ं की सं ा और अ ताल E म सकारा क परी ण
करने वाले लोगो ं की सं ा का अनुपात ात कीिजए?
A. 19:27 B. 38:53 C. 19:29
D. 38:57 E. None of these

298. The number of people who tested positive in hospital D is what percent more/less than the
number of people who tested negative from same hospital?
अ ताल डी म सकारा क परी ण करने वाले लोगो ं की सं ा समान अ ताल से नकारा क परी ण
करने वाले लोगो ं की सं ा से िकतने ितशत अिधक / कम है ?
A. 15.38% B. 14.88% C. 13.38%
D. 14.23% E. None of these

PAGE h ps://t.me/studified h ps://instagram.com/aashisharorasocial?utm_medium=copy_link


Follow
288 Aashish Arora on: h ps://youtube.com/channel/UCYa4_JrOrf8R5Kz2uOtccXQ https://www.facebook.com/aashisharorasocial/
अ ास by Aashish Arora
(SBI/IBPS/RBI/LIC/All other banking and insurance Exams)

299. The difference between the number of positive and negative people from hospital E is what
percent of the total people tested from same hospital?
अ ताल ई से सकारा क और नकारा क लोगो ं की सं ा के बीच का अंतर उसी अ ताल से परी ण
िकए गए कुल लोगो ं का िकतना ितशत है ?
A. 13.33% B. 12.5% C. 13.75%
D. 11.67% E. None of these

300. Find the average number of people from hospital B, C, D, and E who are tested negative?
अ ताल के लोगो ं की औसत सं ा B, C, D, और E से ात कर जो नकारा क परी ण कर रहे ह?
A. 965 B. 955 C. 975
D. 945 E. None of these

301. 60% of the number of people who are tested positive in hospital A and 40% of the number
of people who are tested negative in hospital A, are males. Find the total number of
females in hospital A?
अ ताल ए म सकारा क परी ण िकए गए लोगो ं की सं ा का 60% और अ ताल ए म नकारा क
परी ण िकए जाने वाले लोगो ं की सं ा का 40% पु ष ह। अ ताल A म मिहलाओं की कुल सं ा ात
कीिजये?
A. 1300 B. 1100 C. 1200
D. 1050 E. None of these

Join
Joinme
meonon

Join me on
Join me on

Join me on
Join me on
Join me on
BY: AASHISH ARORA

PAGE h ps://t.me/studified h ps://instagram.com/aashisharorasocial?utm_medium=copy_link


Follow
289 Aashish Arora on: h ps://youtube.com/channel/UCYa4_JrOrf8R5Kz2uOtccXQ https://www.facebook.com/aashisharorasocial/
अ ास by Aashish Arora
(SBI/IBPS/RBI/LIC/All other banking and insurance Exams)

SOLUTION

(1 – 5)

Company P Q R S T Total
=>
Total App 980 1020 1000 970 1030 5000
Gaming 630 660 640 582 721 3233
App
Utility App 350 360 360 388 309 1767

1. Ans. (D)
Gaming App by P : Utility App by Q = 630:360 = 7:4

2. Ans. (B)
660 + 640
Average of Gaming App developed by Q & R = = 650
2
3. Ans. ©
(388 - 360) ´100
Utility App developed by S is more than R by = = 7.7%
360
4. Ans. (A)
Gaming App by T – Utility App by T = 721 – 309 = 412

5. Ans. ©
Total Utility App developed by P, Q, R & S = 350 + 360 + 360 + 388 = 1458

(6 – 10)

6. Ans. (B)
Total download in May = 205 + 4000 = 4205
Total download in April = 190 + 5200 = 5390
(5390 - 4205) ´100
Total download in May is less than of April by = = 21.98 = 22%
5390
7. Ans. ©
Total premium download in January, March & May = 240 + 175 + 205 = 620
Free download in March = 3600
Total premium download in3 month is less than free download in March by
(3600 - 620) ´100
= = 82.77% = 82%
3600
8. Ans. (B)

PAGE h ps://t.me/studified h ps://instagram.com/aashisharorasocial?utm_medium=copy_link


Follow
290 Aashish Arora on: h ps://youtube.com/channel/UCYa4_JrOrf8R5Kz2uOtccXQ https://www.facebook.com/aashisharorasocial/
अ ास by Aashish Arora
(SBI/IBPS/RBI/LIC/All other banking and insurance Exams)

Total Download in Feb:Total Download in Mar. = (4800 + 180):(3600 + 175) = 996:755

9. Ans. (A)
240 + 175 + 205
Average premium download in January, March & May = = 206.6
3
10. Ans. (D)
Free download (Feb + Apr) – Premium download (Feb + Apr)
= (4800 + 5200) – (180 + 190) = 9630

(11 – 15)
Exam => P Q R S T Total
Moderate 80 95 75 105 85 440
Hard 120 70 85 75 115 465
Total 200 165 160 180 200 905

11. Ans. ©
Total Question in Exam P : Total Question in Exam S = 200:180 = 10:9

12. Ans. (A)


(165 - 160) ´100
Total Que. in Exam Q are more than Total Que. In Exam R by = = 3.125%
160
13. Ans. ©
95 + 75 + 85
Average of Moderate Question in Exam Q, R & S = = 85
3
14. Ans. (A)
Total Hard Question in Exam P, Q & S = 120 + 70 + 75 = 265

15. Ans. (B)


1.8 ´165 ´ 5
In Exam F, Number of Moderate Question = = 165
9
(16 – 20)
Office => P Q R S T Total
HMV 875 810 605 290 832 3412
LMV 375 540 495 1160 448 3018
Total 1250 1350 1100 1450 1280 6430

16. Ans. (B)


LMV License issued by office P = 375

17. Ans. (A)


(1450 - 1350) ´100
Total license issued by S is more than total license issued by Q = = 7.4%
1350

PAGE h ps://t.me/studified h ps://instagram.com/aashisharorasocial?utm_medium=copy_link


Follow
291 Aashish Arora on: h ps://youtube.com/channel/UCYa4_JrOrf8R5Kz2uOtccXQ https://www.facebook.com/aashisharorasocial/
अ ास by Aashish Arora
(SBI/IBPS/RBI/LIC/All other banking and insurance Exams)

18. Ans. (D)


LMV license by office S – LMV license by office R = 1160 – 495 = 665

19. Ans. (B)


3412
Average of HMV license issued = = 682.4
5
20. Ans. ©
Total license not rejected by office T = 832 + 0.75 × 448 = 1168

(21 – 25)

21. Ans. ©
18000 ´11
Employee in Bank Q = = 22000
9
Employee posted in urban area in bank Q = 0.56 × 22000 = 12320 Rs

22. Ans. (A)


In Bank R, Employee posted in rural area = 16000 – 7680 = 8320
In Bank T, Employee posted in rural area = 8320 – 6260 = 2060
In Bank T, Employee posted in urban area = 18000 – 2060 = 15940

23. Ans. ©
15680 ´117
In Bank P, Employee posted in urban area = = 8190
224
(19500 - 8190) ´100
% of Employee posted in Rural area in Bank P = = 58%
19500
24. Ans. ©
Employee posted in rural area in bank R : Total employee in bank T
= (16000 – 7680):18000 = 104:225

25. Ans. (B)


18000 ´11
Employee in Bank Q = = 16500
12
Employee in Bank S = 16000 + 8500 = 24500
(19500 + 16500 + 16000 + 24500 + 18000)
Average employee in all banks = = 18900
5
(26 – 30)
Department Telecom Postal Railway Textiles Energy Total
=>
Total 270 120 390 480 240 1500
Vacant 128 100 204 256 112 800
Non-Vacant 142 20 186 224 128 700

26. Ans. (B)


Non-Vacant seats in Energy dep. is more than Total seats in Postal department by

PAGE h ps://t.me/studified h ps://instagram.com/aashisharorasocial?utm_medium=copy_link


Follow
292 Aashish Arora on: h ps://youtube.com/channel/UCYa4_JrOrf8R5Kz2uOtccXQ https://www.facebook.com/aashisharorasocial/
अ ास by Aashish Arora
(SBI/IBPS/RBI/LIC/All other banking and insurance Exams)

(128 - 120) ´100


= = 6.66%
120
27. Ans. (B)
Non-Vacant seats in Railway : Vacant seats in Textiles = 186:256 = 93:128

28. Ans. (A)


100 + 204 + 112
Average of vacant seat in Postal, Railway & Energy dept = = 138.66
3
29. Ans. (D)
Non-Vacant seats Textiles – Non-Vacant seats in Postal = 224 – 20 = 204

30. Ans. ©
Number of seats exclusive for woman = 390 × 0.3 = 117

(31 – 35)
Bookstore => A B C D E Total
Educational 693 684 192 396 1176 3141
Literature 407 216 608 804 224 2259
Total 1100 900 800 1200 1400 5400

31. Ans. (D)


Total books in Bookstore D = 1200

32. Ans. (B)


(1100 - 800) ´100
Books in store A is more than Books in store C by = = 37.5
800
33. Ans. (D)
Literature book in store C:Literature book in store E = 608:224 = 19:7

34. Ans. (D)


Average number of educational books = 3141/5=628.2

35. Ans. ©
Literature books in store D – Literature books in store B = 804 – 216 = 588

(36 – 40)
State => Bihar Jh. UP MP Raj . Total
Total 3740 340 6120 4760 2040 17000
Less than 5 Crore 2057 240 3264 2975 1105 9641
More than 5 Crore 1683 100 2856 1785 935 7359

36. Ans. (D)


1785 + 935
Average number of firm in MP & Rajasthan of more than 5 crore turnover = = 1360
2

PAGE h ps://t.me/studified h ps://instagram.com/aashisharorasocial?utm_medium=copy_link


Follow
293 Aashish Arora on: h ps://youtube.com/channel/UCYa4_JrOrf8R5Kz2uOtccXQ https://www.facebook.com/aashisharorasocial/
अ ास by Aashish Arora
(SBI/IBPS/RBI/LIC/All other banking and insurance Exams)

37. Ans. (B)


Less than 5 crore turnover
Business firms in Bihar – Business firms in Jharkhand = 2057 – 240 = 1817

38. Ans. (A)


Firm of less than 5 crore turnover in UP:Firm of more than 5 crore in Jharkhand
= 3264:100 = 816:25

39. Ans. (D)


Business firms in UP of more than 5 crore turnover is less than Business firms in MP of
(2975 - 2856) ´100
less than 5 crore turnover by = = 4%
2975
40. Ans. (B)
Number of total firms in MP, Bihar & UP, who use non renewable source of energy
= 4760 × 0.55 + 3740 × 0.85 + 6120 × 0.95 = 11611

(41 – 45)

41. Ans. ©
Total patients in P : Total patients in U = (6.8 + 3.2):(7 + 4.8) = 50:59

42. Ans. (A)


Number of Patients in S & T infected by older strain = 2.8 + 3.6 = 6.4
Number of Patients in R infected by newer strain = 7.4
(7.4 - 6.4) ´100
= = 13.5
7.4
43. Ans. (B)
Average number of patient infected by newer strain in city P, S & T
6800 + 5200 + 5600
= = 5866.66
3
44. Ans. (A)
Number of patient infected by older strain in P & U = 3200 + 4800 = 8000
Number of patient infected by older strain in R = 4400
Difference = 8000 – 4400 = 3600

45. Ans. (D)


Non-smokers patients in R & S = 0.54(11800 + 8000) = 10692

(46 – 50)
ĜŔMÖ => Ĩ Ï Ī Ĭ İ İ ŎPMÕ
İ ŎPMÕ ČĆĆ ČDĆ ČÐĆ ĊĆĆ ČÇĆ ĈČĐĆ
ĖPPÑÖŐPÑŇ ĈČÐ ĈĐD ĈĎÐ ĈĎĆ ĈĊČ ĐĎĊ
Í ŎPĖPPÑÖŐPÑŇ ĐČ ĐD ĈĈČ ĈÇĆ ĈĆÐ DĆĐ

46. Ans. (B)


Total question in exam of P = 200

PAGE h ps://t.me/studified h ps://instagram.com/aashisharorasocial?utm_medium=copy_link


Follow
294 Aashish Arora on: h ps://youtube.com/channel/UCYa4_JrOrf8R5Kz2uOtccXQ https://www.facebook.com/aashisharorasocial/
अ ास by Aashish Arora
(SBI/IBPS/RBI/LIC/All other banking and insurance Exams)

47. Ans. (B)


For S, Attempted Question : UnAttempted Question = 160:140 = x:7
x=8

48. Ans. (D)


Total Number of Question in Exam of R = 280

49. Ans. (A)


For T, Attempted Question : UnAttempted Question = 132:108 = 11:y
y=9

50. Ans. (B)


For Q, Attempted Question – Un Attempted Question = 175 – 75 = 100

(51– 55)

51. Ans. (D)


Who get scholarship in test 5 : Who did not get scholarship in test 2
= 780:(1860 – 880) = 39:49

52. Ans. (A)


In Test 3, Number of student who attempt in Hindi = 1980 × 0.35 = 693
Who get scholarship = 0.15 × 820 = 123
Who did not get scholarship = 693 – 123 = 570

53. Ans. (D)


Who did not get scholarship 1 = 1650 – 860 = 790
Who get scholarship 4 = 810
(810 - 790) ´100
= = 2.5%
790
54. Ans. (A)
Who did not get scholarship 2 = 1860 – 880 = 980
Who did not get scholarship 5 = 1820 – 780 = 1040
980 + 1040
Average = = 1010
2
55. Ans. (D)
Who did not get scholarship 1 = 1650 – 860 = 790
Who did not get scholarship 2 = 1860 – 880 = 980
Who did not get scholarship 4 = 1780 – 810 = 970
Student preparing for SSC, who didn't get scholarship = 0.35(790 + 980 + 970) = 959

(56 – 60)

56. Ans. ©
In state B, district where curfew imposed = 225 × 0.2 = 45
In state C, district where curfew not imposed = 230 × 0.6 = 138
Ratio = 45:138 = 15:46

57. Ans. (B)


Average number of district where curfew is imposed in state A, B & E

PAGE h ps://t.me/studified h ps://instagram.com/aashisharorasocial?utm_medium=copy_link


Follow
295 Aashish Arora on: h ps://youtube.com/channel/UCYa4_JrOrf8R5Kz2uOtccXQ https://www.facebook.com/aashisharorasocial/
अ ास by Aashish Arora
(SBI/IBPS/RBI/LIC/All other banking and insurance Exams)

208 ´ 0.25 + 225 ´ 0.2 + 172 ´ 0.5 52 + 45 + 86


= = = 61
3 3
58. Ans. (D)
In state B, district where curfew is not imposed = 0.8 × 225 = 180
In state C, district where curfew imposed = 230 × 0.4 = 92
(180 - 92) ´100
= = 95.65%
92
59. Ans. (B)
In state A & state E, number of district where curfew is not imposed
= 208 × 0.75 + 172 × 0.5 = 242
Number of curfew free district where had covid cases = (1 – 5/11) × 242 = 132

60. Ans. ©
Total district in state F = 225(0.2 × 1.2 + 0.8 × 1.45) = 225 × 1.4 = 315

(61 – 65)
City => P Q R S T Total
Total 37440 25740 8580 53430 30810 156000
Illiterate 32000 20000 8000 36000 28000 124000
Literate 5440 5740 580 17430 2810 32000

61. Ans. (D)


(5740 - 580) ´100
Literate customers in City R is less than that of Q by = = 89.9%
5740
62. Ans. ©
17430 ´100
% of literate customers in City S with Illiterate customers in City Q = = 87.15%
20000
63. Ans. (D)
Literate customers in city X + Literate customers in city Y
= 1.25 × 5440 + 1.4 × 2810 = 10734

64. Ans. (B)


124000 - 32000
Average illiterate customers – Average literate customers = = 18400
5
65. Ans. ©
In city P, Illiterate customers : Literate customers = 32000:5440 = 100:17

(66 – 70)
Country => P Q R S Total
Total 540 792 1260 1008 3600
Male 396 462 780 798 2436
Female 144 330 480 210 1164

PAGE h ps://t.me/studified h ps://instagram.com/aashisharorasocial?utm_medium=copy_link


Follow
296 Aashish Arora on: h ps://youtube.com/channel/UCYa4_JrOrf8R5Kz2uOtccXQ https://www.facebook.com/aashisharorasocial/
अ ास by Aashish Arora
(SBI/IBPS/RBI/LIC/All other banking and insurance Exams)

66. Ans. (B)


Total scientists in P & Q = 540 + 792 = 1332
(1332 - 210) ´100
Female scientist in S is less than total scientist of P & Q by = = 84.23%
1332
67. Ans. ©
Male scientist in R : Female scientist in Q = 780:330 = 26:11

68. Ans. ©
(396 + 462) ´ 6
In country P & Q, Male scientist who are not physics graduate = = 468
11
69. Ans. ©
300 + 480 + 210
Average of female scientist in S, Q & R = = 330
3
70. Ans. (D)
Total scientist in country T = 1.8 × 780 + 1.25 × 144 = 1584

(71 – 75)

71. Ans. (B)


5
In City A, Print Media Journalists = 780 ´= 300
13
Broadcast Media Journalists = 780 – 300 = 480
Broadcast Media Journalists who do not work for Republic India = 0.55 × 480 = 264
Ratio, 264:300 = 22:25

72. Ans. ©
In City B, Print Media Journalists = 0.2 × 325 = 65
Total Journalist in City D = 585
Print Media Journalist in City B is less than Total Journalist in City D by
(585 - 65) ´100
= = 88.88%
585
73. Ans. (B)
780 + 325 + 390 + 585 + 845
Average Number of Journalist in All city = = 585
5
74. Ans. (A)
Journalist (City A + City B) : Journalist (City E) = (780 + 325):845 = 17:13

75. Ans. ©
8
Print Media Journalists in City B & City E = (325 + 845) ´ = 720
13
Print Media Journalist in City B = 720 – 175 = 545

(76 – 80)

76. Ans. ©

PAGE h ps://t.me/studified h ps://instagram.com/aashisharorasocial?utm_medium=copy_link


Follow
297 Aashish Arora on: h ps://youtube.com/channel/UCYa4_JrOrf8R5Kz2uOtccXQ https://www.facebook.com/aashisharorasocial/
अ ास by Aashish Arora
(SBI/IBPS/RBI/LIC/All other banking and insurance Exams)

Approved application of BSc in 2012 = 0.74 × 13800 = 10212


Difference between Total BA applications in 2011 & Approved application in 2012
= 15000 – 10212 = 4788

77. Ans. (A)


BSc application in 2015 is less than total application in 2014 by
(30800 - 15600) ´100
= = 49.35%
30800
78. Ans. ©
15400 + 14600 + 15600
Average of BSc application from 2013 to 2015 = = 15200
3
79. Ans. (A)
Total BA Application from 2012 to 2014 = 17500 + 16800 + 16200 = 50400
Number of student who get admission = 50400 × 0.9 × 0.2 = 9090

80. Ans. (A)


Average application is lowest in year 2011 = 15100

(81 – 85)
81. Ans. (D)
Below age 60 in city T = 17850 × 3/14 = 3825, Above age 60 in city Q = 17800 × 31/40 =
13795
Difference = 13795 – 3825 = 9970

82. Ans. (A)


Above age 60 in city P & R = 15500 × 16/25 + 19600 × 23/28 = 26020
Above age 60 in City P & R : Total in City R = 26020:19600 = 1301:980

83. Ans. (A)


Below age 60 in City Q & R = 17800 × 9/40 + 19600 × 5/28 = 7505
Who were not infected by covid = 0.8 × 7505 = 6004

84. Ans. (D)

Below age 60 in City P & S = 15500 × 9/25 + 18700 × 5/22 = 9830


Average = 9830/2 = 4915

85. Ans. ©
æ 17 11 ö
Above age 60 in city U = 0.2 ´ ç18700 ´ + 17850 ´ ÷ = 0.2 ´ 28475 = 5695
è 22 14 ø

(86 – 90)

86. Ans. (B)


Classic Refrigerators (Samsung + Bosch) : Total Refrigerators of Hitachi
= (60 + 96) : (120 + 352) = 39:118

87. Ans. (D)


Classic Refrigerators sold of LG & Hitachi = 108 + 120 = 228

PAGE h ps://t.me/studified h ps://instagram.com/aashisharorasocial?utm_medium=copy_link


Follow
298 Aashish Arora on: h ps://youtube.com/channel/UCYa4_JrOrf8R5Kz2uOtccXQ https://www.facebook.com/aashisharorasocial/
अ ास by Aashish Arora
(SBI/IBPS/RBI/LIC/All other banking and insurance Exams)

French Door Refrigerators sold by LG & Samsung = 307 + 187 = 494


(494 - 228) ´100
= = 53.8%
494
88. Ans. (A)
Refrigerators sold by
LG = 307 + 108 = 415,
Whirlpool = 154 + 180 = 334
Hitachi = 120 + 352 = 472
415 + 334 + 472
Average Refrigerators sold by 3 of them = = 407
3
89. Ans. (E)
In pie chart of classic refrigerators of LG, Whirlpool, Hitachi & Bosch, Central angle of
120 5
classic refrigerators sold by Hitachi = 360 ´ = 360 ´ = 85.5
(180 + 180 + 120 + 96) 21
90. Ans. ©
Total Refrigerators sold by Sony = 1.45 × 120 + 0.6 × 16 = 273

(91 – 95)
Class Language Maths Science Social Sc. Total
VI 7400 5200 5600 8200 26400
VII 6200 4800 1700 1200 13900
VIII 4800 5200 4800 1800 16600
IX 1200 1800 2200 1400 6600
X 3400 4100 2800 1700 12000
Total 23000 21100 17100 14300 75500

91. Ans. (B)


Language in X : Maths in VIII = 3400 : 5200 = 17:26

92. Ans. (A)


Who like Language in all class – Who like four subject in class IX = 23000 – 6600 = 16400

93. Ans. (D)


Student in class VII & VIII, who like science but not belong to Tier – 3 cities
= 0.46(1700 + 4800) = 2990

94. Ans. (B)


Who like Social Science in class VII is less than who like Math in class X by
(4100 - 1200) ´100
= = 70.73%
4100
95. Ans. (D)
In class V, who like all subject = 6600 × 3/5 = 3960
9x + 11x + 8x + 5x = 3960
33x = 3960
x = 120

PAGE h ps://t.me/studified h ps://instagram.com/aashisharorasocial?utm_medium=copy_link


Follow
299 Aashish Arora on: h ps://youtube.com/channel/UCYa4_JrOrf8R5Kz2uOtccXQ https://www.facebook.com/aashisharorasocial/
अ ास by Aashish Arora
(SBI/IBPS/RBI/LIC/All other banking and insurance Exams)

Who like Language & Science = 120(9 + 8) = 2040

(96 – 100)
Year => 2011 2012 2013 2014 Total
Hindi 120 768 240 448 1576
Non – Hindi 552 1248 336 1088 3224
Total 672 2016 576 1536 4800

96. Ans. (B)


Hindi films in 2012 – Non-Hindi films in 2013 = 768 – 336 = 432

97. Ans. (D)


Hindi films (2011 + 2013):Non-Hindi in 2012 = (120 + 240):1248 = 15:52

98. Ans. ©
Hit Hindi films in 2011 & 2012 = 0.25 × 120 + 0.75 × 768 = 606

99. Ans. (A)


Revenue Earned by Hindi movies in 2011 & 2013 = 27 × 120 + 34 × 240 = 11400

100. Ans. ©
Total Hindi Films in 2015 = 448 × 1.25 = 560
Total Films in = 560 / 0.28 = 2000
Non-Hindi films in 2015 = 2000 – 560 = 1440

(101 - 105)

101. Ans. (A)


125 + 150 + 175 + 200 + 250
Average of cars produced by Suzuki in all year = = 180
2
102. Ans. (D)
In 2018 & 2019, Suzuki:Tia:Ronda = (200 + 250):(175 + 225):(225 + 200) = 18:16:17

103. Ans. (D)


Total cars, in 2016 = 150 + 125 + 150 = 425, in 2019 = 250 + 225 + 200 = 675
(675 - 425) ´100
% change in production of total cars from 2016 to 2019 = = 58.82%
425
104. Ans. ©
Cars sold in 2018 = 0.8(200 + 175 + 225) = 0.8 × 600 = 480
Cars sold by Tia = 480 × 4/15 = 128. by Ronda = 480 × 6/15 = 192
Difference of revenue generated between Ronda & Tia = 12 × 192 – 9 × 128 = 1152

105. Ans. (D)


In 2017, Total cars of Suzuki & Ronda = 175 + 175 = 350
In 2020, Total cars of Suzuki & Ronda = 1.2 × 250 + 1.25 × 200 = 550
(550 - 350) ´100
% change in production of cars from 2017 to 2020 = = 57.1%
350

PAGE h ps://t.me/studified h ps://instagram.com/aashisharorasocial?utm_medium=copy_link


Follow
300 Aashish Arora on: h ps://youtube.com/channel/UCYa4_JrOrf8R5Kz2uOtccXQ https://www.facebook.com/aashisharorasocial/
अ ास by Aashish Arora
(SBI/IBPS/RBI/LIC/All other banking and insurance Exams)

(106 - 110)
Year => 2019 2020 HŌŃǾÑMŒÑ ÒŌ
Company Profit Profit% Profit Ĩ ǾŎŅÒPÃ Profit
HDFB 800 11.76% 1000 ĈÇBČÐÃ ČDÃ
IEFC 800 10.81% 1000 ĈÇBĐĆÃ ČDÃ
AXES 500 7.69% 900 ĈĊBÇĊÃ ÐĆÃ
ICBI 0 0% 0 ĆÃ ĆÃ
Total 2100 2900

106. Ans. (A)


Maximum profit % = IEFC in 2020

107. Ans. ©
Maximum % increase in profit is seen by = AXES

108. Ans. ©
Profit % of HDFB in 2019 + Profit % of AXES in 2020 = 11.76 + 13.43 = 25.19

109. Ans. (A)


In HDFB, P% in 2019 : P% in 2020 = 11.76 : 14.28 = 14:17

110. Ans. ©
(2900 - 2100) ´100
% increase in profit of all the companies = = 38%
2100
(111 – 115)

Bank => P Q R S T Total


Urban 126 112 136 156 90 620
Rural 30 72 56 60 42 260
Total 156 184 192 216 132 880

111. Ans. (D)


Rural Branches in Bank U = 1.45 × 60 = 87
Total Branches in Bank U = 192 + 112 = 304
Urban Branches in Bank U = 304 – 87 = 217

112. Ans. (D)


Total Branches in Bank T – Rural Branches in Bank Q & S = 132 – (72 + 60) = 0

113. Ans. (B)


156 + 192 + 216
Average Branches in Bank P, R & S = = 188
3
114. Ans. (A)
Scale – 1 Rural branches in Bank S & P = 0.55 × 60 + 0.7 × 30 = 54

PAGE h ps://t.me/studified h ps://instagram.com/aashisharorasocial?utm_medium=copy_link


Follow
301 Aashish Arora on: h ps://youtube.com/channel/UCYa4_JrOrf8R5Kz2uOtccXQ https://www.facebook.com/aashisharorasocial/
अ ास by Aashish Arora
(SBI/IBPS/RBI/LIC/All other banking and insurance Exams)

115. Ans. (D)


Urban Branches in P & T = 126 + 90 = 216
Branches in Bank Q is less than Total Urban Branches of Bank P & T by
(216 - 184) ´100
= = 14.81%
216
(16 – 20)

116. Ans. (A)


Genuine Notes of 2000 denomination in Factory P & Q = 0.25 × 1500 + 0.2 × 1800 = 735
Fake Notes on 500 denomination in Factory S = 1800 × 0.85 = 1530
Ratio = 735 : 1530 = 49 : 102

117. Ans. (B)


Fake notes of 2000 denomination in Factory S – Genuine notes of 500 Denomination in
Factory P & S = 1700 × 0.85 – (2200 × 0.3 + 1800 × 0.15) = 1445 – (660 + 270) = 515

118. Ans. (B)


In factory R, Total Monetary Value of All genuine notes of 2000 denomination
= 2000 × 2500 × 0.1 = 50000 = 0.5 Cr.

119. Ans. (B)


In Factory S,
Monetary value of genuine notes of 2000 : Monetary value of fake notes on 500
= (2000 × 1700 × 0.15):(500 × 1800 × 0.85) = 2:3

120. Ans. (D)


Fake notes of 2000 denomination in
Factory P = 1500 × 0.75 = 1125
Factory Q = 1800 × 0.8 = 1440
Factory R = 2500 × 0.9 = 2250
Factory S = 1700 × 0.85 = 1445
1125 + 1440 + 2250 + 1445
Average = = 1565
4
(121 – 125)

121. Ans. (A)


In company P, Who use Samsung Triple Camera Phone = 48 × 7/12 = 28
Who use Apple Triple Camera Phone = 48 – 28 = 20
Apple Dual Camera Phone:Samsung Dual Camera Phone = (54 – 20):(66 – 28) = 17:19

122. Ans. ©
In company Q, Total Mobile Users = 58 + 34 = 92
Who use Triple Camera Phone = 92 × 14/23= 56, use Dual Camera Phone = 92 – 56 = 36
Who use Samsung Triple Camera Phone = 58 × 16/29 = 32
Who use Samsung Dual Camera Phone = 58 – 32 = 26
Apple Triple Camera Phone – Apple Dual Camera Phone = (56 – 32) – (36 – 26) = 14

123. Ans. (B)


In Company R,
Who use Apple Triple Camera Phone = 20, Who use Samsung Dual Camera Phone = 8

PAGE h ps://t.me/studified h ps://instagram.com/aashisharorasocial?utm_medium=copy_link


Follow
302 Aashish Arora on: h ps://youtube.com/channel/UCYa4_JrOrf8R5Kz2uOtccXQ https://www.facebook.com/aashisharorasocial/
अ ास by Aashish Arora
(SBI/IBPS/RBI/LIC/All other banking and insurance Exams)

Who use Apple Dual Camera Phone = 36 – 20 = 16,


Who use Samsung Triple Camera Phone = 14 – 8 = 6
Who use Triple Camera Phone – Who use Dual Camera Phone = (20 + 6) – (16 + 8) = 2

124. Ans. (D)


In company S, Samsung Triple Phone Camera Users = 0.75 × 40 = 30,
Apple Triple Phone Camera Users = 0.6 × 50 = 30
Total Triple Phone Camera Users = 30 + 30 = 60
Samsung Triple Phone Camera Users are less than Total Triple Phone Camera users by
(60 - 30) ´100
= = 50%
60
125. Ans. ©
In company T,
Samsung Triple Camera Phone Users = 0.2(50 + 55) = 21
Apple Triple Camera Phone User = 55 – (50 + 55) × 1/3 = 15
Samsung Triple Camera Phone Users: Apple Triple Camera Phone Users = 21:15 = 7:5

(126 – 130)

126. Ans. ©
Total Consumption of Water, Alcohol & Cold Drink in
The Dheela = 50 + 60 + 70 = 180, Raj = 60 + 75 + 65 = 200
180 ´100
% of consumption of The Dheela with Raj = = 90%
200
127. Ans. (A)
Total expenditure of YTC = 20 × 55 + 100 × 85 + 50 × 50 = 12100 Rs

128. Ans. (D)


Hoberoi(Alcohol + Cold Drink):The Dheela(Alcohol + Cold Drink)
= (35 + 65):(50 + 60) = 10:11

129. Ans. (B)


Expenditure on Water = 40(65 + 70 + 55 + 45) = 9400
Expenditure on Alcohol = 700(75 + 60 + 85 + 65) = 199500
Difference = 199500 – 9400 = 190100 Rs

130. Ans. (A)


Profit on Water of The Dheela = 25 × 70 × 3.5 = 6125 Rs

(131 – 135)

Total Seats = 408 / 0.34 = 1200

131. Ans. (B)


Revenue Generated by D = 1200 × 0.1 × 0.45 × 250 = 13500 Rs

132. Ans. (D)


Ticket Sold by A = 1200 × 0.24 × 0.25 = 72, Ticket Sold by B = 1200 × 0.18 × 0.5 = 108
Difference = 108 – 72 = 36

PAGE h ps://t.me/studified h ps://instagram.com/aashisharorasocial?utm_medium=copy_link


Follow
303 Aashish Arora on: h ps://youtube.com/channel/UCYa4_JrOrf8R5Kz2uOtccXQ https://www.facebook.com/aashisharorasocial/
अ ास by Aashish Arora
(SBI/IBPS/RBI/LIC/All other banking and insurance Exams)

133. Ans. (B)


Revenue collected by C = 1200 × 0.14 × 0.75 × 170 = 1200 × 17.85
Revenue collected by D = 1200 × 0.1 × 0.45 × 250 = 1200 × 11.25
Revenue collected by C is more than Revenue collected by D by
(1200 ´17.85 - 1200 ´11.25) ´100 (17.85 - 11.25) ´100
= = = 58.66%
1200 ´11.25 11.25
134. Ans. (A)
Average of Ticket sold by all theaters
1200(0.24 ´ 0.25 + 0.18 ´ 0.5 + 0.14 ´ 0.75 + 0.1´ 0.45 + 0.34 Ì0.25)
=
5
= 240(0.06 + 0.09 + 0.105 + 0.045 + 0.085) = 240 × 0.385 = 92.4

135. Ans. ©
Revenue collected by E - Revenue collected by A
= 408 × 0.25 × 180 – 1200 × 0.24 × 0.25 × 220 = 18360 – 15840 = 2520 Rs

(136 – 140)

136. Ans. ©
Mango Tree(P + R) : Total Tree in S = (36 + 34):60 = 7:6

137. Ans. ©
Total Fruits in Farm T = 47 × 40 + 53 × (64 – 40) = 1880 + 1272 = 3152

138. Ans. (D)


Total Tree in Farm R = 48, Total Tree in Farm (P, Q, T, S) = 50 + 56 + 64 + 60 = 230
48 ´100
= = 20.8%
230
139. Ans. (B)
Guava tree in Farm S = 60 – 36 = 24, Guava tree in Farm R = 48 – 34 = 14
(24 - 14) ´100
Guava tree in Farm S is more than Farm R by = = 71.42%
14
140. Ans. (D)
Mango Tree in Farm P & Q = 36 + 32 = 68
Guava Tree in Farm P & Q = 50 + 56 – 68 = 38
Difference = 68 – 38 = 30

(141 – 145)

City => P Q R S T Total


Total 300 150 525 150 375 1500
Graduate 70 126 315 105 84 700
Post Graduate 230 24 210 45 291 800

141. Ans. ©
In City P, Post Graduate:Graduate = 230:70 = 23:7

PAGE h ps://t.me/studified h ps://instagram.com/aashisharorasocial?utm_medium=copy_link


Follow
304 Aashish Arora on: h ps://youtube.com/channel/UCYa4_JrOrf8R5Kz2uOtccXQ https://www.facebook.com/aashisharorasocial/
अ ास by Aashish Arora
(SBI/IBPS/RBI/LIC/All other banking and insurance Exams)

142. Ans. (D)


In City Q, Graduate – Post Graduate = 126 – 24 = 102

143. Ans. (B)


In City R,
(315 - 210) ´100
Graduate teachers more than Post-Graduate Teachers by = = 50%
210
144. Ans. (B)
Total teachers in City U = 48 / 0.25 = 180,
Post Graduate teachers = 180 × 3 / 5 = 108

145. Ans. (D)


Post Graduate Teachers in City T = 291

(146 – 150)

146. Ans. ©
Promoted clerk in PNB + Promoted clerk in SBI
= 2 × 7375 × 0.2 × 0.6 + 2 × 7995 × 0.4 × 0.5 = 1770 + 3198 = 4968

147. Ans. (B)


Employee Promoted in Canara Bank : Employee Not Promoted in PNB Bank
= (2 × 6040 × 0.25) : (2 × 7375 × 0.8) = 151:590

148. Ans. (D)


Employee not promoted in SBI – Employee promoted in Canara & Union Bank
= 2 × 7995 × 0.6 – (2 × 6040 × 0.25 + 2 × 7175 × 0.2) = 9594 – (3020 + 2870) = 3704

149. Ans. ©
Not Promoted Officers in Canara Bank = 2 × 6040 × 0.75 – 6660 = 2400

150. Ans. (B)


In Axis Bank, Who applied for promotion = 2 × 6900 × 0.65 + 530 = 9500
Clerk who applied for promotion = 9500 - 4715 = 4785

(151 – 155)

151. Ans. (A)


Male in F + Female in C + Female in B = 100 + 320 + 660 = 1080

152. Ans. (B)


Female in D ´100 575 ´100
= = 91% (Approx)
Total in A 630

PAGE h ps://t.me/studified h ps://instagram.com/aashisharorasocial?utm_medium=copy_link


Follow
305 Aashish Arora on: h ps://youtube.com/channel/UCYa4_JrOrf8R5Kz2uOtccXQ https://www.facebook.com/aashisharorasocial/
अ ास by Aashish Arora
(SBI/IBPS/RBI/LIC/All other banking and insurance Exams)

153. Ans. ©
Both City B & City E

154. Ans. (D)


Total in E - Female in F = 945 - 170 = 775

155. Ans. (D)


Male in C:Female in B = 600:320 = 15:8

(156 – 160)

156. Ans. ©
In 2019, Plastic tables manufactured by A, B & C
14000×0.4 + 10000×0.8 + 2000×0.5 = 14600
Wooden table manufactured by all three companies = (14000 + 10000 + 2000) - 14600 =
11400
Difference of Plastic tables & Wooden tables = 14600 - 11400 = 3200

157. Ans. (B)


Total tables by A in 2018 = Total tables by C in 2020 = x
0.25x + 0.1x = 2800
x = 2800/0.35 = 8000

158. Ans. (A)


Wooden tables by B,
In 2018 = 6000×0.9 = 5400
In 2019 = 10000×0.2 = 2000
2000 ´100
= 37.03% = 37%
5400
159. Ans. (D)
Defective tables manufactured by
A = 31000×0.2×0.14 = 868
B = 23500×0.4×0.26 = 2444
C = 20000×0.1×0.11 = 220
Total Defective table = 868 + 2444 + 220 = 3532

160. Ans. ©
Plastic tables by B:Total Plastic tables by A & C together
= (7500×0.4):(9000×0.2 + 8000×0.1) = 3000:(1800 + 800) = 3000/2600 = 15:13

(161 – 165)
Total No. of Bags
(4x + 5) + 3x + (5x – 15) + (4x + 10) + (3x + 15) = 680
19x + 15 = 680
x = 35

PAGE h ps://t.me/studified h ps://instagram.com/aashisharorasocial?utm_medium=copy_link


Follow
306 Aashish Arora on: h ps://youtube.com/channel/UCYa4_JrOrf8R5Kz2uOtccXQ https://www.facebook.com/aashisharorasocial/
अ ास by Aashish Arora
(SBI/IBPS/RBI/LIC/All other banking and insurance Exams)

161. Ans. (A)


Bag sold by C:Bag sold by E = 160:120 = 4:3

162. Ans. ©
(145 + 105 + 160 + 150)
Average No. of Bag sold by A, B, C & D = = 140
4
163. Ans. (D)
Revenue of B – Revenue of D = 50400 – 45000 = 5400 Rs

164. Ans. (B)


(72500 - 48000) ´100
Revenue of A is more than Revenue of E by = = 51.04%
48000
165. Ans. (D)
(57600 + 45000)
Average Revenue of C & D = = 51300
2
(166 – 170)

166. Ans. (A)


Bluetooth (JBL + Realme) – Wired (Mivi + Skullcandy) = (500 + 350) – (400 + 350) = 100

167. Ans. (D)


350 ´100
% of Bluetooth earphone of Skullcandy with Total earphone of Boat = = 25%
1400
168. Ans. (B)
Total Earphone manufacture by MI = 5000 / 5 = 1000
Bluetooth Earphone manufacture by MI = 0.4 × 1000 = 400

169. Ans. (D)


Bluetooth (Boat + Mivi):Wired (JBL + Realme) = (800 + 500):(700 + 450) = 26:23

170. Ans. ©
Defective earphone manufactured by Boat = 0.2 × 1400 = 280
280 ´ 5
Defective Bluetooth earphone = = 100
14
Non Defective Bluetooth earphone = 800 – 100 = 700

(171 – 175)

171. Ans. ©
May(UP + HR + MH) – April(UK + MP + MH)
= (1051 + 1291 + 1318) – (643 + 1180 + 1248) = 589 Billion

PAGE h ps://t.me/studified h ps://instagram.com/aashisharorasocial?utm_medium=copy_link


Follow
307 Aashish Arora on: h ps://youtube.com/channel/UCYa4_JrOrf8R5Kz2uOtccXQ https://www.facebook.com/aashisharorasocial/
अ ास by Aashish Arora
(SBI/IBPS/RBI/LIC/All other banking and insurance Exams)

172. Ans. (B)


% of UP’s Gross state income in June with Gross state income of all three Month
1100 ´100
= = 34.98 = 35%
3144
173. Ans. (D)
April(MP + UK + HR):June(UK + HR + MH)
= (1180 + 643 + 1189):(654 + 1376 + 1282) = 251:276

174. Ans. (A)


1051 + 1230 + 657 + 1291 + 1318
Average income of all states in May = = 1109.4
5
175. Ans. ©
993 + 1051 + 1100 1180 + 1230 + 1277
Average of, UP = = 1048, MP = = 1229
3 3
(1229 - 1048) ´100
Income of UP is less then Income of MP by = = 14.72%
1229
(176 – 180)

176. Ans. (D)


Increase in Length in 4th year from 3rd year = 210 – 150 = 60 cm

177. Ans. (B)


Length of 5 year old – Length of 3 year old = 231 – 150 = 81

178. Ans. (E)


Can’t be determined

179. Ans. ©
Increase in length in 6th year = 0.5 × 231 = 115.5 cm

180. Ans. (A)


(6 ´ 210 + 5 ´150 + 4 ´ 231 + 10 ´125)
Average Length of all pythons = = 167.36 cm
6 + 5 + 4 + 10
181. (17.5 – 12.5)×100/12.5 = 40% Ans. (C)

182. (20 + 12.5)/(10 + 17.5) = 13:11 Ans. (C)

183. 210×(12.5 – 10)/15 = 35 Lakhs Ans. (C)

184. 100/5 = 20 Ans. (D)

185. 2.45×(15 + 10 + 5)/17.5 = 4.2 crores Ans. (B)

PAGE h ps://t.me/studified h ps://instagram.com/aashisharorasocial?utm_medium=copy_link


Follow
308 Aashish Arora on: h ps://youtube.com/channel/UCYa4_JrOrf8R5Kz2uOtccXQ https://www.facebook.com/aashisharorasocial/
अ ास by Aashish Arora
(SBI/IBPS/RBI/LIC/All other banking and insurance Exams)

186. (700 – 550)×100/550 = 27.27 = 27% Ans. (C)

187. 400×100/550 = 72.72 = 73% Ans. (A)

188. (600 + 700 + 800 + 600 + 650 + 700)/6 = 675 Ans. (C)

189. Total Proyuction = 750 + 800 + 600 + 550 + 700 + 650 = 4050
Total Sell = 300 + 550 + 450 + 400 + 500 + 550 = 2750
Proyuction/Sale = 4050/2750 = 81:55 Ans. (B)

190. 600/800 = 3/4 Ans. (B)

(191 – 195)

191. 870 + 125 = 995 Ans. (B)

192. (50 + 195 + 180)/(200 + 875 + 125) = 17:48 Ans. (C)

193. (2500 – 1500)×100/1500 = 66.66% Ans. (D)

194. Gtrls tn class X = 125×1.4 = 175


Boys tn class X = 850 – 175 = 675 Ans. (D)

195. Gtrls tn class XI = 180×7/2 = 630


Boys tn class XI = 630×0.7 = 441 Ans. (C)

(196 – 200)

196. [(7200 + 6480) + (5440 + 11840) + 36000]/3 = 22320 Ans. (B)

197. In category (I & III) of P & S


In P, Total = 32000
In Q (I & III) = 8910 + 6210 = 15120
In S (I & III) = 4320 + 6480 = 10800
Ratto = (15120 + 10800)/32000 = 81/100 Ans. (D)

198. 7200 + 5440 = 12640 Ans. (B)

199. T/R = (1.25×32000)/36000 = 10/9 Ans. (C)

200. (7200 + 11880 + 11520 + 5440)/4 = 9010 Ans. (B)

PAGE h ps://t.me/studified h ps://instagram.com/aashisharorasocial?utm_medium=copy_link


Follow
309 Aashish Arora on: h ps://youtube.com/channel/UCYa4_JrOrf8R5Kz2uOtccXQ https://www.facebook.com/aashisharorasocial/
अ ास by Aashish Arora
(SBI/IBPS/RBI/LIC/All other banking and insurance Exams)

(201 – 205)

201. Ans. (B)


Total (A + C):Digital B = (600 + 540):(720 – 480) = 57:12

202. Ans. (B)


Total Watches manufactured in B & D = 720 + 360 = 1080
Digital Watches by E = 480 – 200 = 280
(1080 - 280) ´100
= 285 (5/7)%
280

203. Ans. ©
Defective watches in C = 0.2 × 540 = 108,
540 - 360
Defective digital watches = = 20
9
Defective Analog watches = 108 – 20 = 88
Non Defective Analog watches = 360 – 88 = 272

204. Ans. (A)


Analog watches by B = 480
Digital watches by D & E = (360 – 120) + (480 – 200) = 520
Difference = 520 – 480 = 40

205. Ans. (D)


Digital watches in F = 120 + (540 – 360) = 300
Analog watches in F = 0.2(600 + 720 + 540 + 360 + 480) = 540
Total watches manufactured by F = 300 + 540 = 840

(206 – 210)

206. Ans. ©
420 ´11
Apsara pencil sold on Tuesday = = 220
21
207. Ans. (A)
540 ´ 4
Natraj pencil sold on Monday = = 240
9
480 ´13
Apsara pencil sold on Thursday = = 260
24
(260 - 240) ´100
= 8 (1/3)%
240

208. Ans. (D)


420 ´10
Natraj pencil sold on Tuesday = = 200
21
380 ´10
Natraj pencil sold on Saturday = = 200
19
Ratio = 200:200 = 1:1

PAGE h ps://t.me/studified h ps://instagram.com/aashisharorasocial?utm_medium=copy_link


Follow
310 Aashish Arora on: h ps://youtube.com/channel/UCYa4_JrOrf8R5Kz2uOtccXQ https://www.facebook.com/aashisharorasocial/
अ ास by Aashish Arora
(SBI/IBPS/RBI/LIC/All other banking and insurance Exams)

209. Ans. ©
520 ´ 6
Apsara pencil sold on Wednesday = = 240
13
380 ´ 9
Apsara pencil sold on Saturday = = 180
19
(240 + 180)
Average = = 210
2
210. Ans. (B)
Apsara (Tue + Thur) = 480, Natraj(Tue + Thur) = 420
Difference = 480 – 420 = 60

(211 – 215)

211. Ans. ©
(105 - 81) ´100
Investment in Delhi is more than Investment in Kolkata by = = 29.62%
81
212. Ans. (A)
Investment in (Delhi + Chennai):Investment in (Banglore + Mumbai)
= (105 + 75):(54 + 45) = 20:11

213. Ans. (D)


(50% Investment of Delhi):(New Investment of Mumbai)
(0.5 × 105):(45 + 0.5 × 105 × 3/5) = 52.5:76.5 = 35:51

214. Ans. (B)


æ 81 ö
Investment to Assam = ç 6000 ´ ÷ ´ 0.6 ´ 0.7 = 567
è 360 ø
215. Ans. (D)
45 ´100
% of Investment in Mumbai = = 12.5%
360
(216 – 220)

216. Ans. (B)


Apple & Mango, sold by A = 360 + 140 = 500
Lichi & Watermelon, sold by B = 190 + 90 = 280
(500 - 280) ´100
= 78.57%
280
217. Ans. ©
Orange sold by A = 260, B = 120, C = 1.55 × 260 = 403
260 + 120 + 403
Average = = 261
3
218. Ans. (A)
By B(Orange + Guava):By A(Lichi) = (120 + 240):160 = 9:4

219. Ans. (B)


By A, Apple + Orange + Watermelon = 360 + 260 + 280 = 900

PAGE h ps://t.me/studified h ps://instagram.com/aashisharorasocial?utm_medium=copy_link


Follow
311 Aashish Arora on: h ps://youtube.com/channel/UCYa4_JrOrf8R5Kz2uOtccXQ https://www.facebook.com/aashisharorasocial/
अ ास by Aashish Arora
(SBI/IBPS/RBI/LIC/All other banking and insurance Exams)

By B, Mango + Guava + Lichi = 320 + 240 + 190 = 750


Difference = 900 – 750 = 150

220. Ans. (D)


Mango sold by C = 2 × 380 – 320 = 440
5
Watermelon sold by A = 280 ´ = 350
4
Sum = 440 + 350 = 790

(221 – 225)

221. Ans. (D)


81´100
% of person who invest in March with January in FD = = 82.65%
98
222. Ans. (A)
In FD, (Jan + Apr):(Feb + Mar) = (98 + 34):(91 + 81) = 132:172 = 33:43

223. Ans. (A)


(259 + 99 + 221)
In MF, Average of Feb, March & May = = 193
3
224. Ans. (B)
FD(Jan + Feb + Mar):MF(Apr + Mar) = (98 + 91 + 81):(306 + 99) = 270:405 = 2:3

225. Ans. (D)


343 ´100
% of FD with Total Investment = = 24.32%
1410
(226 – 230)

226. Ans. (B)


In 2014, Total daily uses produced = 1120
Daily uses produced by Rajasthan = 434
434 ´100
% of daily uses produced by Rajasthan = = 38.75%
1120

PAGE h ps://t.me/studified h ps://instagram.com/aashisharorasocial?utm_medium=copy_link


Follow
312 Aashish Arora on: h ps://youtube.com/channel/UCYa4_JrOrf8R5Kz2uOtccXQ https://www.facebook.com/aashisharorasocial/
अ ास by Aashish Arora
(SBI/IBPS/RBI/LIC/All other banking and insurance Exams)

227. Ans. ©
Total Bags produced in 3 year = 150 + 125 + 1.3 × 150 = 470

228. Ans. (A)


Let, in 2014, Price of Sandals = 100 Rs, Price of Bags = 160 Rs
Amount received from Sandals = 100 × 135 = 13500 Rs
Amount received from Bags = 160 × 45 = 7200 Rs
(13500 - 7200) ´100
= 87.5%
7200
229. Ans. (D)
Total production in 2014:Total production in 2015 = 1120:1045 = 224:209

230. Ans. (D)


Total Shoes Export in 2014 & 2015 = 280 × 0.2 + 160 × 0.3 = 104 k pairs

(231 – 235)

231. Ans. (B)


Unsold laptop in 2017 = 540 × 0.7 = 378, in 2019 = 480 × 0.5625 = 270
378 + 270
Average = = 324
2
232. Ans. ©
Sold laptop in 2016:Sold laptop in 2020 = (720 × 0.375):(640 × 0.2625) = 45:28

233. Ans. (E)


B Unsold laptop in 2017 = 540 × 0.7 = 378, in 2019 = 480 × 0.5625 = 270
Total = 378 + 270 = 648

234. Ans. (B)


Unsold laptop in 2018 – Sold laptop in 2020 = 360 × 0.775 - 640 × 0.2625 = 111

235. Ans. (D)


Defective laptops in 2016 = 720 × 0.375 × 0.3 = 81
81´100
% of defective laptops in 2016 with sold laptops in 2018 = = 100%
360 ´ 0.225
(236 – 240)

236. Ans. (A)


Total number of people who travelled by Ola on Tuesday & Friday
= (2 × 1050 – 945) + (2 × 945 – 1015) = 2030

237. Ans. (D)


On Wednesday,
People who travelled by Uder = 1155, Ola = 2 × 1260 – 1155 = 1365
Ratio = Ola:Uber = 1365:1155 = 13:11

238. Ans. (B)


On Monday, Who travelled by Ola = 2 × 1190 – 1225 = 1155
Who travelled by Rapido = 2 × 1035 – 1155 = 915

PAGE h ps://t.me/studified h ps://instagram.com/aashisharorasocial?utm_medium=copy_link


Follow
313 Aashish Arora on: h ps://youtube.com/channel/UCYa4_JrOrf8R5Kz2uOtccXQ https://www.facebook.com/aashisharorasocial/
अ ास by Aashish Arora
(SBI/IBPS/RBI/LIC/All other banking and insurance Exams)

239. Ans. ©
1260 ´ 5
On Thursday, Number of Male who travelled by Uber = = 700
9
240. Ans. (A)
Who travelled by Uber on Monday & Thursday = 1225 + 1260 = 2485
Who travelled on Tuesday = 2 × 1050 = 2100
(2485 - 2100) ´100
= 18.33%
2100
241. In 10th class, Number of failed student in
Class A = 124×0.2/0.8 = 31
Class B = 90×0.25/0.75 = 30
Class C = 136×0.32/0.68 = 64
Class D = 70×0.5/0.5 = 70
Class E = 105×0.4/0.6 = 70
Second Lowest = Class A (Ans.)

242. In 12th class = Student in A + Student in C = 143/0.52 + 75/0.5 = 425


In 10th class = Student in B + Student in C = 90/0.75 + 70/0.5 = 260
(425 – 260)×100/260 = 63 6/13% (Ans.)

243. Average of Student who passed in class 12, in school A, B & C


(143×0.48/0.52 + 30×0.6/0.4 + 75×0.5/0.5)/3 = 84 (Ans.)

244. Student of class 10th in school C & E = 136/0.68 + 105/0.6 = 375


Student of class 12th in school A & D = 143/0.52 + 45/0.36 = 400
375/400 = 15/16 (Ans.)

245. In school C,
Student in class 9th = 1.3×136/0.68 = 260
Student in class 11th = (75/0.5)0.6 = 90
260 – 90 = 170 (Ans.)

246. 560×0.375 = 210 (Ans.)

247. (840×5/7 + 630×4/7 + 720×0.375)/3 = 410 (Ans.)

248. Mi mobile in 2020 = 800×0.55 = 440


Realme mobile in 2017 = 840×2/7 = 240
(440 – 240)×100/240 = 83.33% (Ans.)

249. Realme = 560×0.375 + 840×2/7 + 630×3/7 + 720×0.625 + 800×0.45 = 1530


MI = (560 + 840 + 630 + 720 + 800) – 1530 = 2020
2020 – 1530 = 490 (Ans.)

250. (630×3/7)/(720×0.625) = 3/5

251. Internet Security Antivirus sold by P & Q = 190 + 210 = 400


Computer Security Antivirus sold by T = 380 – 320 = 60
Internet security antivirus sold by P & Q more than Computer security antivirus sold by T

PAGE h ps://t.me/studified h ps://instagram.com/aashisharorasocial?utm_medium=copy_link


Follow
314 Aashish Arora on: h ps://youtube.com/channel/UCYa4_JrOrf8R5Kz2uOtccXQ https://www.facebook.com/aashisharorasocial/
अ ास by Aashish Arora
(SBI/IBPS/RBI/LIC/All other banking and insurance Exams)

(400 - 60) ´100


by = = 566.66%
60
252. Computer security antivirus sold,
by Q = 410 – 210 = 200, by S = 460 – 290 = 170
200 + 170
Average = = 185
2
253. Computer Security antivirus sold,
by R = 490 – 360 = 130, by T = 380 – 320 = 60
10.22 version sold = 0.6(130 + 60) = 114

254. Internet security Antivirus sold by U = 1.4(290 + 320) = 854


Total antivirus sold by U = 1.2(360 + 410) = 924
Computer Security Antivirus = 924 – 854 = 70

255. Internet security antivirus by R : Computer security antivirus by Q = 360:(410 – 210) = 9:5
Month Feb Mar Apr May June Total
Laptop 128 328 247 210 474 1387
Desktop 80 200 190 150 300 920
Total 208 528 437 360 774 2307
256. Laptops sold in Feb + Total device sold in Apr = 128 + 437 = 565

257. Total Revenue in Feb = 300 × 80 + 400 × 128 = $75200


437 + 360
258. Average of device sold in April & May = = 398.5
2
259. Computer device sold in March - Desktop sold (May + June) = 528 – (150 + 300) = 78

260. Desktop sold in January = 1.45 × 360 – 218 = 304

Exam => SBI PO SBI CL IBPS PO IBPS CL RRB PO RRB CL Total


Total 378 567 621 756 216 162 2700
Girls 114 283 517 283 112 106 1415
Boys 264 284 104 473 104 56 1285

261. In IBPS Clerk, Boys – Girls = 473 – 283 = 190

262. English Medium Student in SBI Cl, IBPS Cl, RRB Cl = 6/11 × (567+756+162) = 810
(106 - 56) ´100
263. In RRB CL exam, Girls selected are more than Boys by = = 89.28%
56
264. Boys selected in RBI Asst. = 0.5 × 104 = 52
Total student selected in RBI Asst. = 52 × 7/2 = 182

265. Boys Selected in SBI CL = 284


264 + 104
266. Average of Boys selected in SBI PO & IBPS PO = = 184
2

PAGE h ps://t.me/studified h ps://instagram.com/aashisharorasocial?utm_medium=copy_link


Follow
315 Aashish Arora on: h ps://youtube.com/channel/UCYa4_JrOrf8R5Kz2uOtccXQ https://www.facebook.com/aashisharorasocial/
अ ास by Aashish Arora
(SBI/IBPS/RBI/LIC/All other banking and insurance Exams)

267. English medium student who attempt Mock 1 = 182 × 4/7 + (288 - 182) × 1/2 = 104 + 53 =
157

268. In Mock 2, Who clear cut-off : Who do not clear cut-off = 117:(384 – 117) = 39:89
208 ´100
269. In Mock 4, % of student who clear cut-off to Total student = = 69.33
300
270. In Mock 5, Who clear cut-off – Who do not clear cut off = 299 – (408 – 299) = 190

271. Average of Student who clear cut-off in Mock 1, 2, 3, 4 = (182 + 117 + 65 + 208)/4 = 143
Month => Jan Feb Mar Apr May June Total
Petrol 66 144 96 210 96 336 948
Diesel 132 108 64 126 192 112 734
Electric 132 108 160 84 96 112 692
Total 330 360 320 420 384 560 2374

272. Total cars sold in February & May: Total cars sold in April
= (360 + 384):420 = 62:35

273. Petrol cars sold in April = 210


Petrol & Diesel cars sold in February = 144 + 108 = 252
Petrol cars sold in April are less than Petrol & Diesel cars sold in February by
(252 - 210) ´100
= = 16.66%
252
274. Total Petrol Cars sold in 4 given month = 66 + 96 + 96 + 336 = 594
Central angle made by Petrol cars sold in January in pie chart of Petrol cars sold in
o 66
January, March, May & June = 360 ´ = 40o
594
275. Average of Diesel cars sold in February, March, May, June
108 + 64 + 192 + 112
= = 119
4
276. Total cars manufactured in April = 126 / 0.075 = 1680
Manufactured Petrol cars = 1680 × 6/15 = 672
Manufactured Diesel cars = 1680 × 4/15 = 448
Unsold Petrol Cars = 672 – 210 = 462
Unsold Diesel Cars = 448 – 126 = 322
Difference = 462 – 322 = 140
Sellers => A B C D E Total
Chocolate Donuts 875 875 1000 1000 900 4650
Strawberry Donuts 625 875 1250 800 750 4300
Total 1500 1750 2250 1800 1650 8950

277. Chocolate Donuts by C: Strawberry Donuts by E = 1000:750 = 4:3


12 ´ 875 + 18 ´ 625
278. Average price of each donut in Shop A = = 14.5
1500

PAGE h ps://t.me/studified h ps://instagram.com/aashisharorasocial?utm_medium=copy_link


Follow
316 Aashish Arora on: h ps://youtube.com/channel/UCYa4_JrOrf8R5Kz2uOtccXQ https://www.facebook.com/aashisharorasocial/
अ ास by Aashish Arora
(SBI/IBPS/RBI/LIC/All other banking and insurance Exams)

279. Strawberry donuts = 875 + 800 = 1675


Strawberry donuts sold by B & D are less than total donuts sold by C
(2250 - 1675) ´100
= = 25.55%
2250
280. Strawberry donuts (C + E) – Chocolate donuts (A + E)
= (1250 + 750) – (875 + 900) = 2000 – 1775 = 225

281. Total Donuts sold by F = 1.675 × 1000 × 1.64 = 2747

School => DAV SD IPS BPS DPS Total


Boys 480 360 540 450 420 2250
Girls 270 440 360 800 330 2200
Total 750 800 900 1250 750 4450

(800 - 440) ´100


282. Girls from SD is less than Girls from BPS by = = 45%
800
283. In DAV & DPS, who don't like cricket = (750 + 750) × 8/15 = 800
Who don't like cricket in DAV & DPS – Girls in IPS = 800 – 360 = 440

284. Boys (SD + IPS + DPS): Girls (DAV + BPS + DPS)


= (360 + 540 + 420):(270 + 800 + 330) = 1320:1400 = 33:35
270 + 440 + 360 + 330
285. Average of Girls in DAV, SD, IPS, & DPS = = 350
4
286. Number of rooms, that will be used = 4450 / 50 = 89
Banks => SBI UCO BOB AXIS ICICI Total
Clerk 3125 2000 2500 1500 1875 11000
Manager 1875 500 2000 500 1125 6000
Total 5000 2500 4500 2000 3000 17000

287. Clerks in BOB: Manager in SBI = 2500:1875 = 4:3


500 + 2000 + 500
288. Average of Managers in UCO, BOB & AXIS = = 1000
3
289. Managers in UBI = 4500 × 7/9 – 1500 × 17/15 = 3500 – 1700 = 1800

290. Manager from SBI & ICICI = 1875 + 1125 = 3000


Manager from SBI & ICICI more than Clerks from AXIS by
(3000 - 1500) ´100
= = 100%
1500
291. In Kotak, Managers = 1875 × 14/25 = 1050
Clerks in Kotak = 1050 / 0.4375 – 1050 = 1350
Difference = 1350 – 10520 = 300

PAGE h ps://t.me/studified h ps://instagram.com/aashisharorasocial?utm_medium=copy_link


Follow
317 Aashish Arora on: h ps://youtube.com/channel/UCYa4_JrOrf8R5Kz2uOtccXQ https://www.facebook.com/aashisharorasocial/
अ ास by Aashish Arora
(SBI/IBPS/RBI/LIC/All other banking and insurance Exams)

Month => Jan Feb Mar Apr May Total


Honda 256 216 275 360 336 1443
Bajaj 576 324 429 330 360 2019
TVS 448 180 396 210 264 1498
Total 1280 720 1100 900 960 4960

292. Bajaj Bikes, January: May = 576: 360 = 8:5

293. TVS Bikes sold in March are more than Honda Bikes sold in February by
(396 - 216) ´100
= = 83.33%
216
294. Central angle made by TVS bikes sold in April in pie chart of bikes sold in
210
April = 360o ´ = 84o
900
295. Average number of Bikes sold = 4960 / 5 = 992

296. Total Bikes sold in June = 960 × 17/12 = 1360


(7 - 4)
Difference of Bikes sold by Bajaj & TVS = 1360 ´ = 255
16

Hospital => A B C D E Total


Total 2160 1440 1680 1920 2400 9600
Positive 980 760 920 880 1060 4600
Negative 1180 680 760 1040 1340 5000

297. Negative in Hospital C: Positive in Hospital E = 760:1060 = 38:53

298. In Hospital D, who are tested Positive are less than who are Negative by
(1040 - 880) ´100
= = 15.38%
1040
299. In Hospital E, % of Difference of Negative & Positive with total people
(1340 - 1060) ´100
= = 11.66%
2400
300. Average of tested Negative in Hospital B, C, D & E
680 + 760 + 1040 + 1340
= = 955
4
301. Females in Hospital A = 0.4 × 980 + 0.6 × 1180 = 1100

PAGE h ps://t.me/studified h ps://instagram.com/aashisharorasocial?utm_medium=copy_link


Follow
318 Aashish Arora on: h ps://youtube.com/channel/UCYa4_JrOrf8R5Kz2uOtccXQ https://www.facebook.com/aashisharorasocial/
अ ास by Aashish Arora
(SBI/IBPS/RBI/LIC/All other banking and insurance Exams)

8 Memory Based Paper

Directions (1-5): What value should come in the place of (?) in the following number series?

1. 49, 58, 33, 82, 1, ?


A. 17 B. 89
C. 122 D. 101
E. 65

2. 2, 7, ?, 146, 877
A. 32 B. 29
C. 27 D. 23
E. 39

3. 19, 21, 24, 29, ?, 47


A. 36 B. 38
C. 40 D. 41
E. 32

4. 12, 48, 168, ?, 1260, 2520


A. 496 B. 498
C. 502 D. 504
E. 508

5. 11, 26, 46, 71, ?, 136


A. 91 B. 97
C. 101 D. 119
E. 111

Directions (6-10): What value should come in the place of (?) in the following questions.

6. 560 ÷ 14 + 65 % of 80 = ? * 23
A. 4 B. 5
C. 3 D. 6
E. 7

7. 16.5 * 6 + 15 * 18 – 22 * 9= ?
A. 179 B. 175
C. 171 D. 168
E. 163

8. 12 * ? = √1296 * 9 + 29 * 12
A. 63 B. 56
C. 52 D. 58
E. 65

9. 414 ÷ √529 + 418 ÷ √361+ ? = √3721

PAGE h ps://t.me/studified h ps://instagram.com/aashisharorasocial?utm_medium=copy_link


Follow
319 Aashish Arora on: h ps://youtube.com/channel/UCYa4_JrOrf8R5Kz2uOtccXQ https://www.facebook.com/aashisharorasocial/
अ ास by Aashish Arora
(SBI/IBPS/RBI/LIC/All other banking and insurance Exams)

A. 19 B. 21
C. 31 D. 24
E. 25

10 (3 ÷ 7) * (91 ÷ √144) * 48= ?


A. 138 B. 152
C. 148 D. 156
E. 142

11. 40 % of 125 + 70 % of 110 = ?


A. 119 B. 121
C. 123 D. 125
E. 127

12. 57% of (8/19) * (5/13) * 650 = ?


A. 45 B. 50
C. 60 D. 55
E. 65

13. (? ÷ 18) * (13 ÷ 221) = √1156 ÷ ?


A. 102 B. 104
C. 105 D. 101
E. 107

14. (216)13 * (36)5 = 6? * 69


A. 30 B. 35
C. 40 D. 45
E. 25

15. 15 * 35 + 55% of 120 – 12 * 18 = ?


A. 345 B. 355
C. 365 D. 325
E. 375

Directions (16-20): Study the following information carefully and answer the questions given
below.
The given bar graph shows the total number of flats and the number of unoccupied flats in five
different
buildings.

PAGE h ps://t.me/studified h ps://instagram.com/aashisharorasocial?utm_medium=copy_link


Follow
320 Aashish Arora on: h ps://youtube.com/channel/UCYa4_JrOrf8R5Kz2uOtccXQ https://www.facebook.com/aashisharorasocial/
अ ास by Aashish Arora
(SBI/IBPS/RBI/LIC/All other banking and insurance Exams)

16. If the number of person in each occupied flat in building A and B is 3 and 2 respectively,
then find the total number of person in buildingA and B?
A. 1020 B. 1040
C. 1060 D. 1080
E. 1100

17. What is the difference between the number of occupied flats in building C and D together
and the total number of flats in building B?
A. 520 B. 540
C. 560 D. 500
E. 580

18. What is the ratio of the number of unoccupied flat in building A to the total number of
occupied flat in building B and E together?
A. 40:23 B. 39:23
C. 39:22 D. 20:11
E. None of these

19. The number of unoccupied flat inbuilding D is what percent of the total number of flats in
building A and E together?
A. 58.81% B. 60.60%
C. 61.16% D. 64.44%
E. 66.67%

20. The number of occupied flats in building F is 40% more than that of A and the total number
flats in building F is 120% more than the number of occupied flat in building C. Find the
number of unoccupied flat inbuilding F?
A. 294 B. 292
C. 296 D. 290
E. 298

PAGE h ps://t.me/studified h ps://instagram.com/aashisharorasocial?utm_medium=copy_link


Follow
321 Aashish Arora on: h ps://youtube.com/channel/UCYa4_JrOrf8R5Kz2uOtccXQ https://www.facebook.com/aashisharorasocial/
अ ास by Aashish Arora
(SBI/IBPS/RBI/LIC/All other banking and insurance Exams)

Directions (21-25): Study the following information carefully and answer the questions given
below.

There are three classes A, B and C and each class has three different section x, y and z.

Class A: Total number of students is 360 and the number of students in section x is one-third of
the total number of students. The ratio of the number of students in y and z is 2:1.

Class B: The ratio of the number of students in x and y is 8:5. The number of students in z is 50%
of the total number of students in x and y. The total number of students is 390.

Class C: Total number of students in B and C is 3:2. The number of students in y in C is 20%
more than that of the number of students in y i B. Total number of students in x is 30% of the total
number of students in C.

21. What is the ratio of the number of students in section X in B to C?


A. 80:39 B. 27:13
C. 40:19 D. 54:35
E. None of these

22. Find the total number of students in Y in all the classes together?
A. 360 B. 370
C. 380 D. 390
E. 350

23. What is the average number of students in section z in A and C?


A. 71 B. 73
C. 69 D. 67
E. 70

24. Find the difference between the total number of students in section Yin B and C together
and the total number students in A?
A. 120 B. 130
C. 150 D. 140
E. 160

25. The number of students in section Z in B is what percent of the total number of students in
C?
A. 45% B. 50%
C. 55% D. 40%
E. 60%

26. 5 years ago, the age of A and B is in the ratio of 6:7. The average present age of A, B, C and
D is 29 years. C is 15 years elder than D. Find the present age of D, if the age of B 8 years
hence is 48 years?
A. 17 years B. 19 years
C. 15 years D. 13 years
E. None of these

27. Ratio of the speed of the boat in still water to speed of stream is 5: 1. If the time taken by

PAGE h ps://t.me/studified h ps://instagram.com/aashisharorasocial?utm_medium=copy_link


Follow
322 Aashish Arora on: h ps://youtube.com/channel/UCYa4_JrOrf8R5Kz2uOtccXQ https://www.facebook.com/aashisharorasocial/
अ ास by Aashish Arora
(SBI/IBPS/RBI/LIC/All other banking and insurance Exams)

boat traveling 150 km along the stream and against the stream is 12.5 hours, what is the
difference between the downstream speed and upstream speed?
A. 5 kmph B. 8 kmph
C. 10 kmph D. 12 kmph
E. None of these

28. Pipe A alone filled the tank in 20 hours and Pipe B and C together can fill the tank in 10
hours. If Pipe A is opened and after 8 hours it is closed and then pipe B filled the remaining
tank in 18 hours. In how many hours pipe C alone fill the tank?
A. 12 hours B. 15 hours
C. 18 hours D. 21 hours
E. None of these

29. A and B started a business by investing Rs.12000 and Rs.10000 respectively. After x
months, C joined with them and invested Rs.15000. Find the value of x if the profit share of
A is Rs.1560 out of total profit at the end of the year of Rs.3835?
A. 4 months B. 6 months
C. 8 months D. 10 months
E. None of these
2
30. The area of square is 484 cm and the side of the square is equal to the height of a triangle.
2
Find the breadth of the triangle, if the area of the triangle is 176 cm ?
A. 14 cm B. 12 cm
C. 15 cm D. 16 cm
E. 18 cm

31. Soni spends 20% of his salary to house rent, 30% of the salary to education fees and 80%
of the remaining salary to shopping. If she now left with her is Rs.800, then what is Soni’s
salary?
A. Rs.6000 B. Rs.7800
C. Rs.7200 D. Rs.8000
E. Rs.8400

32. A book is marked 20% above its cost price and sold after giving a discount of Rs.100 such
that there is a profit of 10%. What is its cost price?
A. Rs.1000 B. Rs.1500
C. Rs.2000 D. Rs.2500
E. None of these

33. Sahul invests Rs.x in a simple interest scheme at the rate of R% per annum for 4 years. If
the ratio of the interest received by Sahul to x is 3:5, then find the value of R?
A. 12% B. 10%
C. 15% D. 18%
E. 20%

34. A vessel has the mixture of milk and water in the ratio of 5:3. When 20 liters of the water is
added to the mixture, then the ratio becomes 1:1. What is the initial quantity of the mixture?
A. 60 liters B. 70 liters
C. 80 liters D. 90 liters
E. None of these

PAGE h ps://t.me/studified h ps://instagram.com/aashisharorasocial?utm_medium=copy_link


Follow
323 Aashish Arora on: h ps://youtube.com/channel/UCYa4_JrOrf8R5Kz2uOtccXQ https://www.facebook.com/aashisharorasocial/
अ ास by Aashish Arora
(SBI/IBPS/RBI/LIC/All other banking and insurance Exams)

35. The ratio of the number of boys to girls in the school is 4:5. After few months 12 boys and 8
girls joined the class. After few more months 30 boys and 26 girls left the class, then the
ratio becomes 1:2. What is the initial number of boys in the class?
A. 20 B. 24
C. 28 D. 30
E. None of these

Join
Joinme
meonon

Join me on
Join me on

Join me on
Join me on
Join me on
BY: AASHISH ARORA

PAGE h ps://t.me/studified h ps://instagram.com/aashisharorasocial?utm_medium=copy_link


Follow
324 Aashish Arora on: h ps://youtube.com/channel/UCYa4_JrOrf8R5Kz2uOtccXQ https://www.facebook.com/aashisharorasocial/
अ ास by Aashish Arora
(SBI/IBPS/RBI/LIC/All other banking and insurance Exams)

1. Answer: C 18 + 22 + ? = 61
2
49 + 3 = 58 ?
2
58 – 5 = 33 = 21
33 + 72 = 82
82 – 92 = 1 10. Answer: D
1 + 112 = 122 (3 ÷ 7) * (91 ÷ √144) * 48 = ?
3/7 * 91/12 * 48 = ?
2. Answer: B ? = 156
2*3+1=7
7 * 4 + 1 = 29 11. Answer: E
29 * 5 + 1 = 146 40 % of 125 + 70 % of 110 = ?
146 * 6 + 1 = 877 50 + 77 = ?
? = 127
3. Answer: A
19 + 2 = 21 12. Answer: C
21 + 3 = 24 57% of (8/19) * (5/13) * 650 = ?
24 + 5 = 29 ? = 60
29 + 7 = 36
36 + 11 = 47 13. Answer: A
(? ÷ 18) * (13 ÷ 221) = √1156 ÷ ?
4. Answer: D ? = 102
12 * 4 = 48
48 * 3.5 = 168 14. Answer: C
168 * 3 = 504 (216)13 * (36)5 = 6? * 69
504 * 2.5 = 1260 39 + 10 – 9 = ?
1260 * 2 = 2520 ? = 40

5. Answer: C 15. Answer: E


11 + 15 = 26 15 * 35 + 55% of 120 – 12 * 18 = ?
26 + 20 = 46 525 + 66 – 216 = ?
46 + 25 = 71 ? = 375
71 + 30 = 101
101 + 35 = 136 16. Answer: D
Required total = (3 * (1000 – 800)) + (2 *
6. Answer: A (1200 – 960))
560 ÷ 14 + 65 % of 80 = ? * 23 = 600 + 480
40 + 52 = ? * 23 = 1080
?=4
17. Answer: B
7. Answer: C Occupied flats in C = 1400 – 1140 = 260
16.5 * 6 + 15 * 18 – 22 * 9 = ? Number of occupied flats in D = 1600 –
99 + 270 – 198 = ? 1200 = 400
? = 171 Required difference = 1200 – (260 + 400)
= 540
8. Answer: B
12 * ? = √1296 * 9 + 29 * 12 18. Answer: A
? * 12 = 324 + 348 Required ratio = 800:((800 – 580) + (1200
? = 56
– 960))
= 800:460
9. Answer: B
= 40:23
414 ÷ √529 + 418 ÷ √361 + ? = √3721

PAGE h ps://t.me/studified h ps://instagram.com/aashisharorasocial?utm_medium=copy_link


Follow
325 Aashish Arora on: h ps://youtube.com/channel/UCYa4_JrOrf8R5Kz2uOtccXQ https://www.facebook.com/aashisharorasocial/
अ ास by Aashish Arora
(SBI/IBPS/RBI/LIC/All other banking and insurance Exams)

19. Answer: E The present age of A = [(35/7) * 6] + 5 = 35


Required percentage = 1200/(800 + years
1000) * 100 Given,
= 66.67% Let the present age of D be x,
A + B + C + D = 29 * 4
20. Answer: B 35 + 40 + x + 15 + x = 116
Total number of flats in F = 220/100 * 2x + 90 = 116
(1400 – 1140) = 572 2x = 26
Number of occupied flats in F = 140/100 x = 13
* (1000 – 800) = 280 The present age of D = 13 years
Required total = 572 – 280 = 292
27. Answer: C
21. Answer: A 150/6x + 150/4x = 12.5
Class A: 25+37.5 = 12.5x
Total = 360 x =5
X = 1/3 * 360 = 120 Downstream speed = 6x = 6 * 5 = 30 kmph
Total number of students in y and z = Upstream speed = 4 * 5= 20 kmph
360 – 120 Difference = 30 - 20 = 10 kmph
= 240
Y = 240 * 2/3 = 160 28. Answer: B
Z = 240 * 1/3 = 80 A = 1/20
Class B: B + C = 1/10
Total = 390 A fills in 8 hours = 8/20 = 2/5
z = 50/100 * (8x + 5x) = 6.5x Remaining work = 1 – 2/5 = 3/5
x = 8/(8 + 5 + 6.5) * 390 = 160 B filled 3/5 of the tank in 18 hours.
y = 5/8 * 160 = 100 B alone fills the whole tank in = 5/3 * 18 =
z = 6.5/8 * 160 = 130 30
Class C: hours
Total = 2/3 * 390 = 260 C alone= 1/10 – 1/30 = 1/15
x = 260 * 30/100 = 78
y = 120/100 * 100 = 120 29. Answer: B
z = 260 – 78 – 120 = 62 Profit ratio of A, B and C = 12000 *
Required ratio = 160:78 12:10000 *
= 80:39 12:15000 * (12 – x)
= 144:120:(180 – 15x)
22. Answer: C 144/(444 – 15x) = 1560/3835
Required total = (120 + 100 + 160) = 380 5772 – 195x = 4602
x=6
23. Answer: A
Required average = 62 + 80/2 = 71 30. Answer: D
2 2
The area of square (a ) = 484 cm
24. Answer: D Side of the square = 22 cm
Difference = 360 – (100 + 120) = 140 Height of a triangle = 22 cm
The area of triangle = (1/2) * b * h = 176
25. Answer: B cm2
Required % = 130/260 * 100 = 50% (1/2) * b * 22 = 176
Breadth of triangle = 176/11 = 16 cm
26. Answer: D
5 years ago, the age of A and B is in the 31. Answer: D
ratio = 6:7 Soni’s salary=x
The age of B, 8 years hence = 48 years House rent=x * 20/100=x/5
The present age of B = 40 years Education fee=30/100 * x=3x/10

PAGE h ps://t.me/studified h ps://instagram.com/aashisharorasocial?utm_medium=copy_link


Follow
326 Aashish Arora on: h ps://youtube.com/channel/UCYa4_JrOrf8R5Kz2uOtccXQ https://www.facebook.com/aashisharorasocial/
अ ास by Aashish Arora
(SBI/IBPS/RBI/LIC/All other banking and insurance Exams)

Remaining=x – x/5 – 3x/10=x/2 34. Answer: C


Shopping=x/2 * 80/100 5x / 3x + 20 = 1/1
Remaining=x/2 * 20/100 3x + 20 = 5x
x/2 * 20/100=800 2x = 20
x=Rs.8000 x = 10 liters
Quantity of the mixture = 8 * 10 = 80 liters
32. Answer: A
CP = 100x 35. Answer: B
MP = 100x * 120/100 = 120x 4x + 12 – 30 / 5x + 8 – 26 = 1/2
120x – 100 = 100x * 110/100 5x – 18 = 8x – 36
x = 10 3x = 18
CP = 100 * 10 = Rs.1000 x=6
Initial number of boys = 4 * 6 = 24
33. Answer: C
3a = 5a * R * 4/100
R = 15%

Join
Joinme
meonon

Join me on
Join me on

Join me on
Join me on
Join me on
BY: AASHISH ARORA

PAGE h ps://t.me/studified h ps://instagram.com/aashisharorasocial?utm_medium=copy_link


Follow
327 Aashish Arora on: h ps://youtube.com/channel/UCYa4_JrOrf8R5Kz2uOtccXQ https://www.facebook.com/aashisharorasocial/
Unacademy Plus Subscription Features
1. Access to 40k hours of Recorded Content.
2. Access to more than 150 High Quality Test Series.
3. Access to 20k Practice Questions.
4. 70+ Top Educators of the Country.
5. Ask-A-Doubt feature by Unacademy.
6. Live Tests and Quizes.
7. Dedicated Doubt Clearing Sessions.

SUBSCRIBE

Subscribe to Unacademy Plus & Get Unlimited Access of all


Live and Recorded Classes of All Bank Exams Educators Use
Code 'TT10' for 10% Discount in Subscription Fees.
Get Plus Subscription
Use Code TT10 for 10% Discount

STEP-1 STEP-2

TT10

You might also like